You are on page 1of 728

PREP

Chuẩn bị kiến thức cho kì thi


HSGQG Hóa học
Lời mở đầu
Mến chào quý độc giả,
vậy là sau một năm gián đoạn vì ảnh hưởng của dịch COVID, tạp chí KEM
lại có cơ hội ra mắt ấn phẩm mới thuộc series PREPxx. Nếu như
PREP20 chủ yếu tập trung vào những chủ đề thú vị liên quan đến Hóa
Vô cơ - Hóa Lí - Hóa Phân tích, còn PREP21 khai thác các lĩnh vực mới
xuất hiện trong chương trình thi Olympiad Hóa học những năm gần đây;
thì với ấn phẩm lần này, KEM quay trở lại với những thứ giản đơn và thiết
thực nhất. Thay vì cung cấp quá nhiều bài tập khó và mới, KEM tập
trung vào các kiến thức nền tảng để phục vụ cho việc học chuyên sâu
về sau. Mục đích của quyển sách này hướng tới nhóm độc giả chính là
"những học sinh đang muốn tự đánh giá trình độ bản thân để chuẩn bị
cho kì thi chọn đội tuyển HSGQG và kì thi chính thức". Tuy không có
phần lí thuyết riêng biệt, nhưng KEM đã cố gắng lồng ghép các trọng
điểm quan trọng vào phần hướng dẫn giải, nhằm giúp độc giả hiểu kĩ
hơn cách vận dụng.
Về mặt nội dung, KEM tập trung vào ba phần chính: Phần đầu tiên là
Cấu tạo chất, đặc biệt là về cấu tạo nguyên tử và liên kết hóa học - để
làm cơ sở cho phần thứ hai: Hóa học Hữu cơ! Với các tài liệu thông
thường thì Hóa học Hữu cơ thường được giảng dạy sau cùng, nhưng với
ấn phẩm này, KEM khuyến khích độc giả thử đi theo một hướng mới -
tức là học Hữu cơ sau khi đã vững vàng kiến thức về Cấu tạo chất. Phần
cuối cùng là Hóa Lí và Hóa Phân tích - với trọng điểm chính là các cân
bằng trong phase khí và dung dịch. Để cân bằng được độ khó và giữ
được tính cập nhật của tài liệu, KEM cũng có bổ sung thêm một số
chương ở dạng phụ lục - với một số phần được trích dẫn từ sách PREP21
(hiện đã ngừng xuất bản). Ngoài ra, cũng để tránh trùng lặp nội dung
nên những dạng bài đã được khai thác chi tiết trong series TOP100 của
KEM (cụ thể là một số chuyên đề thuộc Nhiệt động học, Tốc độ phản
ứng và trạng thái dừng hay Chuyển hóa của các hợp chất Vô cơ) sẽ
không được đề cập nhiều trong tuyển tập này. Nhiều nội dung trong ấn
phẩm này có tham khảo từ các tài liệu đào tạo Kĩ sư Chất lượng cao
MPSI-PTSI của Pháp và giáo trình Organic Chemistry (John McMurry).

1 | Chuẩn bị kiến thức cho kì thi HSGQG Hóa học


Chính bởi sự thay đổi về định hướng nội dung, từ năm nay, series này
sẽ chỉ có tên đơn giản là "PREP - Chuẩn bị kiến thức cho kì thi HSGQG"
và không còn đánh theo số năm (20, 21) như trước nữa và sẽ chỉ cập
nhật hàng năm theo dạng chỉnh sửa lỗi sai, bổ sung một số dạng bài
chứ không còn thay mới nội dung toàn bộ như trước đây.
Chân thành cảm ơn độc giả đã ủng hộ KEM.
BBT Tạp chí Olympiad Hóa học

2 | Chuẩn bị kiến thức cho kì thi HSGQG Hóa học


Mục lục
Phần A: Cấu tạo chất ................................................................................................ 7
Chương 1: Nguyên tử ........................................................................................... 7
Chủ đề 1: Đồng vị ............................................................................................. 7
Chủ đề 2: Phản ứng hạt nhân ........................................................................ 11
Chủ đề 3: Năng lượng .................................................................................... 13
Chủ đề 4: Nguyên tử nickel ........................................................................... 16
Chủ đề 5: Orbital nguyên tử........................................................................... 20
Chủ đề 6: Bảng tuần hoàn nguyên tố............................................................ 25
Chương 2: Phân tử ............................................................................................. 30
Chủ đề: Các bước thiết lập công thức Lewis ............................................... 30
Chủ đề 1: Công thức Lewis ........................................................................... 33
Đọc thêm: Cấu trúc cộng hưởng .................................................................... 39
Đọc thêm: Độ bền của các cấu trúc cộng hưởng .......................................... 42
Chủ đề 1: Sự phân cực của phân tử HCl ...................................................... 44
Chủ đề 2: Các hợp chất thơm........................................................................ 48
Chủ đề 3: Hình học phân tử........................................................................... 55
Đọc thêm: Thuyết VSEPR ............................................................................... 61
Chủ đề 4: Số oxid hóa .................................................................................... 63
Chương 3: Tinh thể học ...................................................................................... 67
Chủ đề 1: Mạng tinh thể bạc ......................................................................... 67
Chủ đề 2: Barium titanate .............................................................................. 75
Chủ đề 3: Các hốc tinh thể ............................................................................ 81
Chủ đề 4: Ammonium chloride ..................................................................... 88
Chủ đề 5: Germanium .................................................................................... 94
Phụ lục b: Phức chất và ứng dụng của thuyết trường tinh thể ........................ 97
Chủ đề 1: Đồng phân vô cơ ........................................................................... 97
Chủ đề 1: Thuyết trường tinh thể ................................................................ 106
Chủ đề 3: Tính chất từ ................................................................................. 112
Chủ đề 4: Các ion kim loại trong phức chất ............................................... 118
Chủ đề 5: Phức bis(salicylidene)ethylenediamine ..................................... 121
Chủ đề 6: Xác định phức chất ..................................................................... 124
Chủ đề 7: Liên kết ngược............................................................................. 127
Chủ đề 8: Hydrogen hóa xúc tác ................................................................. 129
Phần B: Hóa Hữu cơ ............................................................................................. 135
Chương 4: Cấu trúc và hoạt tính ...................................................................... 135
Chủ đề 1: Công thức phân tử - Cấu tạo khai triển ...................................... 135
Chủ đề 2: Sự lai hóa ..................................................................................... 141
Chủ đề 3: Một số tiểu phân đặc biệt ........................................................... 146
Chủ đề 4: Sự phân cực liên kết ................................................................... 150
Chủ đề 5: Điện tích hình thức ...................................................................... 153
Chủ đề 6: Sự cộng hưởng ............................................................................ 158
Chủ đề 7: Tổng quan về tính acid-base ...................................................... 164
Chủ đề 8: Acid-base theo Lewis.................................................................. 166
Chủ đề 9: Nhóm chức .................................................................................. 170
Chủ đề 10: Độ bất bão hòa .......................................................................... 174

3 | Chuẩn bị kiến thức cho kì thi HSGQG Hóa học


Chủ đề 11: Đồng phân cấu tạo.................................................................... 176
Chủ đề 12: Tính electrophile và nucleophile .............................................. 180
Chủ đề 13: Sử dụng mũi tên cơ chế phản ứng ........................................... 183
Chủ đề 14: Dự đoán sản phẩm phản ứng................................................... 189
Chủ đề 15: Giản đồ năng lượng phản ứng.................................................. 191
Chương 5: Hydrocarbon và dẫn xuất halide ................................................... 196
Chủ đề 1: Alkane .......................................................................................... 196
Chủ đề 2: Cấu dạng alkane ......................................................................... 202
Chủ đề 3: Cycloalkane ................................................................................. 205
Chủ đề 4: Sức căng vòng ............................................................................ 209
Chủ đề 5: Độ bền cấu dạng ......................................................................... 211
Chủ đề 6: Độ hơn cấp và tâm thủ tính ........................................................ 218
Chủ đề 7: Đồng phân meso ......................................................................... 225
Chủ đề 8: Hóa lập thể của sản phẩm phản ứng ......................................... 230
Chủ đề 9: Tính chất tiền-thủ tính ................................................................. 232
Chủ đề 10: Các hợp chất không có tâm thủ tính........................................ 238
Chủ đề 11: Danh pháp alkene và cycloalkene ............................................ 241
Chủ đề 12: Độ bền của alkene .................................................................... 246
Chủ đề 13: Phản ứng cộng electrophile vào alkene .................................. 249
Chủ đề 14: Phản ứng cộng electrophile ..................................................... 259
Chủ đề 16: Phản ứng hydroboron hóa ........................................................ 272
Chủ đề 17: Phản ứng thế allyl ..................................................................... 275
Chủ đề 18: Phản ứng oxid hóa .................................................................... 279
Chủ đề 19: Phản ứng cộng carbene vào alkene ........................................ 283
Chủ đề 20: Tổng hợp từ alkene ................................................................... 288
Chủ đề 21: Danh pháp alkyne ..................................................................... 290
Chủ đề 22: Phản ứng hóa học ..................................................................... 293
Chủ đề 23: Tổng hợp từ alkyne ................................................................... 298
Chủ đề 24: Tính nucleophile ........................................................................ 305
Chủ đề 25: Phản ứng thế ............................................................................. 308
Chủ đề 26: Ảnh hưởng của dung môi ......................................................... 312
Chủ đề 27: Hóa lập thể phản ứng thế ......................................................... 314
Chủ đề 28: Phản ứng tách ........................................................................... 319
Chủ đề 29: Phản ứng ghép cặp ................................................................... 328
Chủ đề 30: Rượu lá ...................................................................................... 332
Chủ đề 31: Hợp chất cơ lithium .................................................................. 337
Chủ đề 32: Phản ứng cộng electrophile vào hệ liên hợp mở..................... 341
Chủ đề 33: Phản ứng Diels-Alder ................................................................ 345
Chủ đề 34: Hợp chất thơm .......................................................................... 353
Chủ đề 35: Dị vòng thơm ............................................................................. 360
Chủ đề 36: Phản ứng thế electrophile nhân thơm ..................................... 363
Chủ đề 37: Phản ứng Friedel-Crafts ........................................................... 370
Chủ đề 38: Phản ứng thế nucleophile ......................................................... 376
Chủ đề 39: Phản ứng oxid hóa .................................................................... 377
Chủ đề 40: Tổng hợp hữu cơ ....................................................................... 380
Chủ đề 41: Cơ chế phản ứng ....................................................................... 383
Chủ đề 42*: Chuyển hóa của alkene ........................................................... 385

4 | Chuẩn bị kiến thức cho kì thi HSGQG Hóa học


Chủ đề 43*: Chuyển hóa của cycloalkene .................................................. 390
Chủ đề 44*: Hydrocarbon khung ................................................................. 394
Chủ đề 45*: Chuyển hóa của hydrocarbon vòng........................................ 399
Chủ đề 46*: Chuyển hóa của terpene ......................................................... 403
Chủ đề 47*: Phản ứng hoán vị .................................................................... 408
Chủ đề 48*: Phản ứng hoán vị .................................................................... 412
Chương 6: Các nhóm chức chứa oxygen ........................................................ 415
Chủ đề 1: Alcohol - Phenol .......................................................................... 415
Chủ đề 2: Tổng hợp alcohol ........................................................................ 419
Chủ đề 3: Chuyển hóa của alcohol ............................................................. 426
Chủ đề 4*: Lasiol .......................................................................................... 433
Chủ đề 5: Phenol - Ether .............................................................................. 436
Chủ đề 6: Aldehyde-Ketone ......................................................................... 445
Chủ đề 7: Phản ứng cộng nucleophile ........................................................ 448
Chủ đề 8: Phản ứng Wittig........................................................................... 455
Chủ đề 9: Hợp chất carbonyl không bão hòa ............................................. 461
Chủ đề 10: Cơ chế phản ứng ....................................................................... 464
Chủ đề 11: Tổng hợp hữu cơ ....................................................................... 468
Chủ đề 12: Carboxylic acid - Nitrile ............................................................. 475
Chủ đề 13: Dẫn xuất carboxylic acid .......................................................... 485
Chủ đề 14*: Phổ cộng hưởng từ ................................................................. 495
Chủ đề 15*: Bất đối xứng hóa các diol đối xứng ....................................... 498
Chủ đề 16: Keto-enol ................................................................................... 501
Chủ đề 17: Cơ chế phản ứng ....................................................................... 511
Chủ đề 18: Phản ứng ngưng tụ ................................................................... 516
Chủ đề 19: Các phản ứng ngưng tụ quan trọng ......................................... 524
Chủ đề 20: Ngưng tụ các hợp chất carbonyl .............................................. 529
Chủ đề 21: Ngưng tụ các hợp chất carbonyl .............................................. 531
Chủ đề 22: Cơ chế phản ứng ....................................................................... 534
Chủ đề 23*: Chuyển vị Schmidt................................................................... 543
Chủ đề 24*: Chuyển vị Schmidt................................................................... 546
Chủ đề 25*: Chuyển vị Claisen .................................................................... 549
Chủ đề 26*: Chuyển vị Schmidt................................................................... 552
Phụ lục: Một số bài tập giải thích cơ chế phản ứng ....................................... 555
Chủ đề 1: Chuyển hóa liên kế (tandem) tạo thành naphthalene thế tetra-
1,2,3,4 ........................................................................................................... 559
Chủ đề 2: Vòng hóa gốc tạo ra các hydroazulene ..................................... 562
Chủ đề 3: Chuyển hóa cinnoline thành indole ............................................ 564
Chủ đề 4: Chuyển vị trong quá trình đo NMR ............................................ 567
Chủ đề 5: Tổng hợp các dị vòng trifluoromethyl ........................................ 570
Chủ đề 6: Tổng hợp các chromone thế vị trí 3 từ các pyranobenzopyran 574
Chủ đề 7: Chuyển vị xúc tác acid của các 1-arylindole .............................. 576
Chủ đề 8: Loại nhóm bảo vệ bất thường của aryl ester ............................. 579
Chủ đề 9: Sự hình thành 3,1-benzoxathiin .................................................. 582
Chủ đề 10: Tổng hợp Arylacetone bởi chuyển vị kiểu-Carroll.................... 585
Chủ đề 11: Thử nghiệm phản ứng Knoevenagel lại tạo thành các sản phẩm
kiểu Mannich! ............................................................................................... 588

5 | Chuẩn bị kiến thức cho kì thi HSGQG Hóa học


Chủ đề 12: Các chuyển vị của muối pyridinium ......................................... 591
Phần C: Hóa Lí – Hóa Phân tích........................................................................... 594
Chương 7. Động hóa học ................................................................................. 594
Chủ đề 1: Bậc phản ứng .............................................................................. 594
Chủ đề 2: Chu kì bán hủy ............................................................................. 599
Chủ đề 3: Phản ứng bậc 2 ........................................................................... 604
Chủ đề 4: Loại bỏ các ion hypochlorite ...................................................... 610
Chủ đề 5: Phương pháp tốc độ đầu ............................................................ 615
Chương 8. Cân bằng hóa học .......................................................................... 621
Chủ đề 1: Nguyên lí Le Chatelier ................................................................. 621
Chủ đề 2: Hằng số cân bằng ....................................................................... 628
Chủ đề 3: Thành phần cân bằng ................................................................. 633
Chủ đề 4: Sự phụ thuộc của hằng số cân bằng vào nhiệt độ.................... 635
Chương 9. Cân bằng acid-base ....................................................................... 639
Chủ đề 1: pH và lực acid.............................................................................. 639
Chủ đề 2: Định lượng acid yếu bởi base mạnh........................................... 648
Chủ đề 3: Phosphoric acid .......................................................................... 655
Chủ đề 4: Sự diễn tiến của một hệ acid-base............................................. 662
Chương 10. Cân bằng tạo phức ...................................................................... 667
Chủ đề 1: Các hằng số cân bằng ................................................................ 667
Chủ đề 2: Phức amine của copper(II) ......................................................... 673
Chủ đề 3: Cân bằng cạnh tranh giữa hai cation ......................................... 681
Chương 11. Cân bằng kết tủa .......................................................................... 687
Chủ đề 1: Các hằng số cân bằng ................................................................ 687
Chủ đề 2: Hợp chất tan nhiều nhất ............................................................. 693
Chủ đề 3: Khối lượng kết tủa tạo thành ...................................................... 698
Chủ đề 4: Sự kết tủa cạnh tranh.................................................................. 701
Chương 12. Cân bằng oxid hóa-khử ................................................................ 705
Chủ đề 1: Phương trình Nernst ................................................................... 705
Chủ đề 2: Thế oxid hóa-khử chuẩn ............................................................. 711
Chủ đề 3: Pin chì-acid .................................................................................. 717
Chủ đề 4: Ảnh hưởng của acid .................................................................... 722

6 | Chuẩn bị kiến thức cho kì thi HSGQG Hóa học


Phần A: Cấu tạo chất
Chương 1: Nguyên tử
Chủ đề 1: Đồng vị
1) Kí hiệu 168 O biểu diễn một hạt nhân cụ thể. Liệu các kí hiệu đơn giản
hóa 16O và 8O có chứa cùng thông tin không? Nếu không, thì trong
mỗi trường hợp đã thiếu mất thông tin gì?
2) Có bao nhiêu neutron, proton và electron trong mỗi nguyên tử
hoặc ion sau:
55
25
Mn 40
18
Ar 96
Mo
42
48
Ti
22 82
Pb2+
207 80
35
Br − 51
Sb3+
122 31 3−
P
15

3) Bổ sung các thông tin còn thiếu trong bảng sau:


Số Số Số
Vi hạt Kí hiệu Z A
proton neutron electron
Nguyên tử

Sodium - - 12 - - -

- K
40
- - - - -

Silicon - - 14 - - 28

Rubidium - 37 - - - 85

Arsenic - - 42 33 - -

- - - - - 47 108

- - 53 74 - - -

- Au 79 - - - 197

- - - 138 - 88 -

Ion

Cadmium - - 64 46 - -

- - 26 - 23 - -

7 | Chuẩn bị kiến thức cho kì thi HSGQG Hóa học


- Se2- - - - 34 79

Chlorine - - - 18 - 35
4) Khi nói rằng “ion Bi3+ có 127 neutron, 83 proton và 81 electron và 210
nucleon” thì thông tin nào đúng, thông tin nào sai?
5) Lithium trong tự nhiên là hỗn hợp của hai đồng vị 6Li và 7Li với khối
lượng nguyên tử lần lượt là 6.017 và 7.018. Khối lượng nguyên tử của
lithium tự nhiên là 6.943. Hãy xác định thành phần đồng vị, tức tỉ lệ
phần trăm mỗi đồng vị.

8 | Chuẩn bị kiến thức cho kì thi HSGQG Hóa học


Hướng dẫn
1) Trong 16O, kí hiệu O chỉ ra rằng đó là nguyên tố oxygen và 16 chỉ ra
rằng nó là đồng vị với số khối 16 (còn có những đồng vị khác, ví dụ như
18
O). Với kiểu kí hiệu này, thông tin hạt nhân được xác định đầy đủ.
Trong 8O, thông tin về bản chất hạt nhân được lặp lại (hạt nhân có số
hiệu nguyên tử “8” thì chắc chắn là “O”), nhưng thông tin về đồng vị nào
thì không được nói rõ. Với kiểu kí hiệu này, không xác định được cụ thể
là hạt nhân nào.
2) Số lượng các hạt cơ bản trong mỗi vi hạt:
55
25
Mn 40
18
Ar 96
42
Mo 48
22
Ti 207
82
Pb2+ 80
35
Br − 122
51
Sb3+ 31
15
P 3−
n 30 22 54 26 125 45 71 16
p 25 18 42 22 82 35 51 15
e 25 18 42 22 80 36 48 18
3)
Số Số Số
Vi hạt Kí hiệu Z A
proton neutron electron
Nguyên tử

Sodium Na 11 12 11 11 23

Potassium 40
K 19 21 19 19 40

Silicon Si 14 14 14 14 28

Rubidium Rb 37 48 37 37 85

Arsenic As 33 42 33 33 37

Bạc Ag 47 61 47 47 108

Iodine I 53 74 53 53 127

Vàng Au 79 118 79 79 197

Radium Ra 88 138 88 88 226

Ion

Cadmium Cd2+ 48 64 46 48 112

Sắt Fe3+ 26 30 23 26 56

9 | Chuẩn bị kiến thức cho kì thi HSGQG Hóa học


Selenium Se2- 34 45 36 34 79

Chlorine Cl- 17 18 18 17 35
4) Nguyên tố bismuth có số hiệu nguyên tử là 83, do đó số proton chắc
chắn là 83 (nếu không thì nó sẽ là nguyên tố khác!). Ion Bi3+ có ít hơn 3
electron so với nguyên tử trung hòa, do đó thông tin về số electron như
trên là sai (phải là 80 chứ không phải 81). Số neutron, cũng như tổng số
nucleon, có thể chính xác, nếu nó chỉ đồng vị bismuth có số khối 210
( = 127 + 83). Thực tế thì khối lượng nguyên tử của nguyên tố tự nhiên
(209.0) là giá trị trung bình tất cả các khối lượng của các đồng vị
bismuth, và trong số đó có thể có một đồng vị là 210.
5) Đặt x và y là phần trăm đồng vị của 6Li và 7Li, vậy x + y = 1 (100%) và
6.017x + 7.018y = 6.943, vậy ta tìm được: 6Li: 7.4%; 7Li: 92.6%.

10 | Chuẩn bị kiến thức cho kì thi HSGQG Hóa học


Chủ đề 2: Phản ứng hạt nhân
Hoàn thành các phương trình với tỉ lệ hợp thức sau:

a) 58
26
Fe + 201 n →60
27
Co + ..........
40
b) 20Ca +21 d → ........... +11 p
c) 60
28
Ni + ......... →60
27
Co +11 p
242
d) 96
Cm +24  → ......... +01 n
e) 1530P → .......... +0+1 e
235
f) 92
U +01 n →56
142
Ba + .......... + 201 n
g) 35Cl + n →35 S + ..........
209
h) Bi + .......... →210 Bi + p
i) 58Fe + n →59 Co + ..........
j) 59Co + .......... →56 Mn + 
214
k) Pb →214 Bi + ..........

11 | Chuẩn bị kiến thức cho kì thi HSGQG Hóa học


Hướng dẫn
Cần chú ý đến các cách kí hiệu khác nhau của cùng một vi hạt, phổ biến
nhất là

▪ Neutron: 01 n, n
▪ Proton: 11p, p, 11H
▪ Deuterium: 21 d, d, 12H
▪ Hạt alpha (hay helium): 24 , , 24He
▪ Electron: 0−1 e, e− , −
▪ Positron: 0+1 e, e+ , +

Hoàn thành các phương trình phản ứng:

a) 58
26
Fe + 201 n →27
60
Co + e −
40
b) 20Ca +21 d →41 Ca +11 p
c) 60
28
Ni + n →60
27
Co +11 p
242
d) 96
Cm +24  →245 Cf +01 n
e) 1530P →30 Si +0+1 e
235
f) 92
U +01 n →56
142
Ba + 92 Kr + 201 n
g) 35Cl + n →35 S + p
209
h) Bi + d →210 Bi + p
i) 58Fe + n →59 Co + e -
j) 59Co + n →56 Mn + 
214
k) Pb →214 Bi + e -

12 | Chuẩn bị kiến thức cho kì thi HSGQG Hóa học


Chủ đề 3: Năng lượng
1) Một photon tương ứng với sóng điện từ có bước sóng  . Biểu thức
năng lượng E của photon này ở dạng hàm của  , hằng số Planck h
và tốc độ ánh sáng c là
h h hc c
A. E = B. E = C. E = D. E =
c c  h

2) Cho biết h = 6.63  1034 J  s, c = 3  108 m  s −1. Hãy tính năng lượng của
photon tương ứng với bức xạ hồng ngoại có bước sóng  = 0.1 mm.

A. E = 2.21 10−46 J B. E = 2.21 10−38 J

C. E = 1.38  10−27 eV D. E = 1.24  10−2 eV


3) Ở mức n, năng lượng thích hợp của nguyên tử hydrogen là
13.6
E = − 2 ( eV ) . Trong dãy phổ phát xạ của nguyên tử hydrogen,
n
vạch tương ứng với bước chuyển 3 → 2 thuộc dãy nào?
A. Dãy Brackett. B. Dãy Paschen.
C. Dãy Balmer. D. Dãy Lyman.
4) Tính bước sóng tương ứng với bước chuyển ở ý 3.
A. 3→2 = 103 nm B. 3→2 = 548 nm

C. 3→2 = 658 nm D. 3→2 = 823 nm

5) Tính năng lượng ion hóa Ei của nguyên tử hydrogen ở trạng thái
kích thích 3d.
A. Ei = 0.661 eV. B. Ei = -4.53 eV.
C. Ei = 13.6 eV. D. Ei = 1.51 eV.

13 | Chuẩn bị kiến thức cho kì thi HSGQG Hóa học


Hướng dẫn
1) Ngoài cách ghi nhớ thì biểu thức này có thể được xác định dựa vào
cách phân tích đơn vị (với đơn vị của hằng số Planck và tốc độ ánh sáng
hc J s  m  s 
−1

đã được cho ở ý 2): = = J (phương án C).


 m

2) Tính được năng lượng photon theo biểu thức ở ý 1:

hc 6.63  10−34  3  108


E= = = 1.99  10−21 J
 10 −4

Năng lượng này cũng có thể được chuyển thành đơn vị electron-volt
(eV), là đơn vị phù hợp hơn với độ lớn kết quả nhận được. Giá trị tuyệt
đối của điện tích một electron là e = 1.6  10−19 C

Khi đó: 1 eV = 1.6  10−19 C  1 V = 1.6  10−19 C  V = 1.6  10−19 J

1.99  10−21
Vậy: E = −19
= 1.24  10−2 eV (phương án D).
1.6  10
3) Tất cả các dãy được đề xuất đều thuộc vùng phổ phát xạ của nguyên
tử hydrogen, và tương ứng với bước chuyển kiểu n' → n với n’ > n, nghĩa
là sự bền hóa (ổn định) của nguyen tử đi từ trạng thái năng lượng cao
về trạng thái năng lượng thấp hơn.
Bước chuyển Vùng phổ Tên dãy
n' → 1 Tử ngoại Lyman
n' → 2 Khả kiến Balmer
n' → 3 Hồng ngoại Paschen
n' → 4 Hồng ngoại Brackett
n' → 5 Hồng ngoại Pfund
Dãy tạo thành bởi bước chuyển 3 → 2 thuộc dãy Balmer (phương án C).
4) Biến thiên năng lượng ∆E của nguyên tử hydrogen trong bước
chuyển 3 → 2:

 1 1 
E = E2 − E3 = −13.6   2 − 2  = −1.89 eV = −3.02  10−19 J
2 3 

14 | Chuẩn bị kiến thức cho kì thi HSGQG Hóa học


Vậy năng lượng mà nguyên tử mất đi là: −E = 3.02  10−19 J

Năng lượng này cho phép phát ra một photon có bước sóng  3→2 sao
hc
cho: −E = . Giá trị này là:
 3 →2

hc 6.63  10−34  3  108


3→2 = − = = 6.58  10−7 m = 658 nm (phương án C)
E 3.02  10 −19

5) Năng lượng ion hóa Ei của nguyên tử hydrogen đại diện cho năng
lượng phải được cung cấp cho nguyên tử này để tách electron độc thân
của nó, nghĩa là đưa electron này đến mức năng lượng đặc trưng bởi giá
trị số lượng tử chính n = .
Nguyên tử hydrogen ban đầu ở trạng thái kích thích 3d, giá trị Ei đại
diện cho biến thiên năng lượng:

 1 1 
Ei = E − E3 = −13.6  2 − 2  = 1.51 eV (phương án D)
 3 

15 | Chuẩn bị kiến thức cho kì thi HSGQG Hóa học


Chủ đề 4: Nguyên tử nickel
Các số lượng tử của một electron trong một nguyên tử nhiều electron
Kí Tập xác Quy tắc lượng Số electron
Tên gọi Mô tả
hiệu định tử hóa cực đại
Số lượng n = 1, 2, 3, 4, 5,
tử chính
Lớp n * 6, 7
2n2
Số lượng
tử phụ
Phân lớp l 0 l n−1 2(2l + 1)
Số lượng Orbital
tử từ nguyên tử
m −l  m  +l 2

Số lượng 1
tử spin
Electron s s= 1
2

Sự phân bố các electron trong nguyên tử tuân theo ba quy tắc:


1. Nguyên lí loại trừ Pauli: Hai electron của cùng một nguyên tử hoặc
của cùng một ion đơn nguyên tử không thể có bốn giá trị số lượng
giống nhau – có nghĩa rằng hai “bộ tứ” giống hệt nhau không thể
cùng tồn tại.
2. Quy tắc Klechkowski: Sự phân bố electron trên các phân lớp được
thực hiện theo trình tự (n + l) tăng dần, n tăng với (n + l) không đổi.
Do đó, các phân lớp được phân bố theo trình tự như dưới đây và việc
điền electron vào một phân lớp xác định chỉ bắt đầu khi phân lớp
trước đó đã bão hòa (đầy).
n + l = 1: 1s
n + l = 2: 2e
n + l = 3: 2p,3s
n + l = 4: 3p, 4s
n + l = 5: 3d,4p, 5s
n + l = 6: 4d, 5p, 6s
n + l = 7: 4f, 5d, 6p, 7s
n + l = 8: 5f, 6d, 7p

3. Quy tắc Hund: Khi một phân lớp chỉ bị chiếm một phần thì cấu hình
năng lượng thấp nhất, nghĩa là bền nhất, sẽ tương ứng với sự chiếm
nhiều orbital nguyên tử nhất.

16 | Chuẩn bị kiến thức cho kì thi HSGQG Hóa học


Dưới đây là các cấu hình electron được đề xuất cho nguyên tử nickel
với số hiệu nguyên tử Z = 28.
A. 1s22s22p63s23p63d104s0
B. 1s22s22p63s23p83d64s2
C. 1s22s22p63s23p63d84s2
D. 1s22s22p63s23p63d64s24p2
1) Những cấu hình nào không tuân theo nguyên lí loại trừ Pauli?
2) Cấu hình nào là của nguyên tử nickel ở trạng thái cơ bản.
3) Cấu hình nào sau đây không có các electron độc thân?
4) Cấu hình nào sau đây kém bền nhất?
5) Thành phần của ion 58Ni2+ gồm
A. 30 proton, 28 electron, 30 neutron.
B. 30 proton, 28 electron, 28 neutron.
C. 28 proton, 28 electron, 30 neutron.
D. 28 proton, 26 electron, 30 neutron.

17 | Chuẩn bị kiến thức cho kì thi HSGQG Hóa học


Hướng dẫn
1) Trong cấu hình 1s22s22p63s23p83d64s2 có 8 electron chiếm phân lớp
3p. Tuy nhiên, chỉ có 6 bộ tứ số lượng tử khả dĩ cho phân lớp này:
 1  1  1  1  1  1
 3,1, −1,  , 3,1, −1, −  , 3,1,0,  , 3,1,0, −  , 3,1,1,  , 3,1,1, − 
 2  2  2  2  2  2

Điều này có nghĩa là có một vài electron phải có cùng bộ tứ số lượng


tử, điều này rõ ràng vi phạm nguyên lí Pauli. Chọn phương án B.
2) Cấu hình A (1s22s22p63s23p63d104s0) vi phạm quy tắc Klechkowski bởi
phân lớp 3d bị chiếm trước phân lớp 4s.
Cấu hình D (1s22s22p63s23p63d64s24p2) cũng sai bởi phân lớp 4p đã bị
chiếm trong khi phân lớp 3d chưa bão hòa 10 electron.
Cấu hình duy nhất thỏa mãn quy tắc Klechkowski là
1s22s22p63s23p63d84s2 (phương án C).
3) Cấu hình electron B có phân lớp 3d6, vậy nên có 4 electron độc thân:

Cấu hình electron C có phân lớp 3d8, vậy nên có 2 electron độc thân:

Cấu hình electron của D bao gồm phân lớp 3d6 và 4p2, do đó có 6
electron độc thân:

Chỉ có cấu hình A, trong đó tất cả các phân lớp đều bão hòa, là không
chứa bất kì electron độc thân nào. (Phương án A)
4) Cấu hình kém bền nhất là: 1s22s22p63s23p63d64s24p2
Nguyên nhân là bởi:

18 | Chuẩn bị kiến thức cho kì thi HSGQG Hóa học


- Sự bão hòa electron trên các phân lớp có xu hướng làm bền hóa
nguyên tử, do đó cực tiểu hóa năng lượng của chúng. Tuy nhiên,
cấu hình này lại gồm hai phân lớp 3d và 4p chưa bão hòa
electron.
- Đấy là cấu hình duy nhất trong các cấu hình được đề xuất có sự
phân bố electron ở phân lớp 4p, có năng lượng cao hơn tất cả
các phân lớp bị chiếm khác.
5) Trong trường hợp ion 58Ni2+ với số hiệu nguyên tử Z = 28 thì:
- Có 28 proton.
- Số neutron là N = A – Z = 58 – 28 = 30.
- Ion mang điện tích 2+, có nghĩa là có ít hơn 2 electron so với số
proton, vậy có 26 electron.
Phương án D.

19 | Chuẩn bị kiến thức cho kì thi HSGQG Hóa học


Chủ đề 5: Orbital nguyên tử
Electron hóa trị của một nguyên tử là những electron có số lượng tử
chính n lớn nhất hoặc thuộc phân lớp đang được lấp đầy: chúng là
những electron ở xa nhất và ít gắn kết với hạt nhân nhất, do đó, chúng
dễ tham gia vào các liên kết cộng hóa trị (điều này được thể hiện qua
chính tên gọi của chúng). Các electron lõi là những electron khác,
phân bố ở các lớp bên trong, với giá trị n nhỏ hơn và do đó có năng
lượng thấp hơn: chúng là những electron liên kết chặt chẽ với hạt nhân
nhất.
Nguyên tố Kim loại Lanthanide
Kiểu nguyên Hydrogen và
nhóm chính chuyển tiếp và actinide
tố helium
(nhóm p) (nhóm d) (nhóm f)
ns
ns
ns (n – 2)f
Orbital hóa trị 1s (n – 1)d
np (n – 1)d
np
Np
Quy tắc đếm Quy tắc nhị tử Quy tắc bát tử Quy tắc thập
Quy tắc 32-e
số electron (duet/duplet) (Octet) bát tử (18-e)

1) Oxygen có số hiệu nguyên tử Z = 8.


A. Nguyên tử oxygen có 2 electron lõi và 6 electron hóa trị. Hóa trị
của nó bằng 6.
B. Nguyên tử oxygen có 2 electron lõi và 6 electron hóa trị. Hóa trị
của nó bằng 2.
C. Nguyên tử oxygen có 4 electron lõi và 4 electron hóa trị. Hóa trị
của nó bằng 2.
D. Nguyên tử oxygen có 4 electron lõi và 4 electron hóa trị. Hóa trị
của nó bằng 4.
2) Với một electron của nguyên tử nhiều electron thì
A. Có thể tồn tại 3 mức năng lượng riêng biệt có cùng giá trị số
lượng tử chính n = 4.
B. Có thể tồn tại 16 orbital nguyên tử có cùng giá trị số lượng tử
chính n = 4.
C. Có thể tồn tại 16 electron có cùng giá trị số lượng tử chính n = 4.

20 | Chuẩn bị kiến thức cho kì thi HSGQG Hóa học


D. Có thể tồn tại 3 mức năng lượng suy biến riêng biệt có cùng giá
trị số lượng tử chính n = 4.
3) Xác định các bộ tứ số lượng tử của 4 electron trong nguyên tử
beryllium (Z = 4).
 1  1  1  1
A.  1,0,0,  , 1,0,0, −  , 2,0,0,  , 2,0,0, − 
 2  2  2  2

 1  1  1  1
B.  1,0,0,  , 1,1,0,  , 2,0,0,  , 2,1,0, 
 2  2  2  2

 1  1  1  1
C.  1,0,0,  , 1,0,0, −  , 2,1,1,  , 2,1,1, − 
 2  2  2  2

 1  1  1  1
D.  1,0,0,  , 2,1,0,  , 2,2,1,  , 2,1, −1, − 
 2  2  2  2

4) Xác định các phương án phân bố electron khả dĩ đối với cấu hình
electron dạng nd6:

21 | Chuẩn bị kiến thức cho kì thi HSGQG Hóa học


Hướng dẫn
1) Cấu hình electron của nguyên tử oxygen là 1s22s22p4. Các electron
được phân bố trên các phân lớp như sau:

Oxygen có 2 electron lõi và 6 electron hóa trị. Giản đồ Lewis1 của nó


là:

Do đó, hóa trị2 của oxygen bằng 2, như trong trường hợp của phân tử
nước (H2O), trong đó nguyên tử oxygen liên kết với hai nguyên tử
hydrogen bởi hai liên kết đơn cộng hóa trị. Chọn phương án B.
2) Có thể tồn tại 4 mức năng lượng riêng biệt có cùng giá trị số lượng
tử chính n = 4: đó là các phân lớp 4s, 4p, 4d và 4f – lần lượt tương ứng
với các giá trị số lượng tử phụ l = 0, 1, 2, 3. Do đó phương án A sai.
Thực tế, có thể tồn tại 16 orbital nguyên tử có cùng giá trị số lượng tử
chính n = 4:
- 1 trong phân lớp 4s, tương ứng với bộ ba số lượng tử:
(n, l, m) = (4, 0, 0)
- 3 trong phân lớp 4p, tương ứng với bộ ba số lượng tử:
(n, l, m) = (4, 1, -1) / (4, 1, 0) / (4, 1, 1)
- 5 trong phân lớp 4d, tương ứng với bộ ba số lượng tử:
(n, l, m) = (4, 2, -2) / (4, 2, -1) / (4, 2, 0) / (4, 2, 1) / (4, 2, 2)
- 7 trong phân lớp 4f, tương ứng với bộ ba số lượng tử:
(n, l, m) = (4, 3, -3) / (4, 3, -2) / (4, 3, -1) / (4, 3, 0) / (4, 3, 1) / (4, 3, 2) / (4, 3,3)

1
Công thức Lewis (hay giản đồ Lewis) của một nguyên tử chỉ biểu diễn các electron
hóa trị xung quanh nguyên tử đó.
2
Hóa trị của một nguyên tố là số liên kết đơn cộng hóa trị mà nguyên tử của nguyên
tố đó có thể tham gia vào.

22 | Chuẩn bị kiến thức cho kì thi HSGQG Hóa học


Vậy phương án B đúng.
Mỗi orbital nguyên tử có thể chứa 2 electron, do đó có thể tồn tại 16 ×
2 = 32 electron có cùng giá trị số lượng tử chính n = 4. Do đó phương án
C sai.
Lớp n = 4 có 4 mức năng lượng riêng biệt:
- Mức 4s, không suy biến bởi nó chỉ tương ứng với 1 orbital nguyên
tử.
- Mức 4p suy biến bởi nó tương ứng với 3 orbital nguyên tử riêng
biệt.
- Mức 4d suy biến bởi nó tương ứng với 5 orbital nguyên tử riêng
biệt.
- Mức 4f suy biến bởi nó tương ứng với 7 orbital nguyên tử riêng
biệt.
Do đó, có thể tồn tại 3 mức năng lượng suy biến riêng biệt có cùng giá
trị số lượng tử chính n = 4: đó là các mức 4p, 4d, 4f. Vậy phương án D
cũng đúng.

 1
3) Bộ tứ  1,1,0,  của phương án B không thỏa mãn quy tắc lượng tử
 2
hóa của 0  l  n − 1 do n = l = 1.

 1  1  1  1
Các bộ tứ  2,1,0,  , 2,1,1,  , 2,1,1, −  , 2,1, −1, −  của các phương án
 2  2  2  2
B, C hoặc D thỏa mãn các quy tắc lượng tử hóa, nhưng không thỏa mãn
quy tắc Klechkowski: beryllium chỉ có 2 electron trong vỏ n = 2 nên
phân lớp 2p của nó không thể bị chiếm.
Chỉ có phương án A là thỏa mãn tất cả các quy tắc.
4) Các cách phân bố A và B không thỏa mãn quy tắc Hund (“khi một
phân lớp chỉ bị chiếm một phần thì cấu hình năng lượng thấp nhất sẽ
tương ứng với sự chiếm nhiều orbital nguyên tử nhất”).
Cách phân bố D không thỏa mãn nguyên lí loại trừ Pauli, do có 2
electron được xác định bởi bộ bốn số lượng tử giống nhau, dạng

23 | Chuẩn bị kiến thức cho kì thi HSGQG Hóa học


 1
 n,2, −2,  . Hãy nhớ rằng, theo quy tắc này, các electron trên cùng
 2
orbital nguyên tử phải có spin ngược chiều (đối song) nhau.
Vậy, chỉ có phương án C là thỏa mãn cả nguyên lí loại trừ Pauli lẫn quy
tắc Hund.

24 | Chuẩn bị kiến thức cho kì thi HSGQG Hóa học


Chủ đề 6: Bảng tuần hoàn nguyên tố
Dưới đây là 5 chu kì đầu tiên trong bảng tuần hoàn nguyên tố hóa học.

1) Nguyên tử của nguyên tố nào sau đây có 2 electron độc thân?


A. Helium He. B. Beryllium Be. C. Boron B. D. Oxygen O.
2) Xác định cấu hình electron của ion đơn nguyên tử được tạo thành
bởi lưu huỳnh:
A. 1s22s22p63s23p6 B. 1s22s22p63s2
C. 1s22s22p6 D. 1s22s22p63s23p5
3) Sử dụng bảng tuần hoàn ở trên, hãy dự đoán công thức hóa học của
oxide dễ được tạo thành bởi sodium Na nhất:
A. Na2O B. Na2O3 C. NaO D. NaO2
4) Sắp xếp các nguyên tố beryllium Be, oxygen O, fluorine F và
strontium Sr theo trình tự tăng dần độ âm điện.
A. F – O – Be – Sr B. Sr – F – O – Be
C. Be – O – F – Sr D. Sr – Be – O – F
5) Sắp xếp các nguyên tố beryllium Be, oxygen O, fluorine F và
strontium Sr theo trình tự giảm dần bán kính nguyên tử.
A. F – O – Be – Sr B. Sr – F – O – Be
C. Be – O – F – Sr D. Sr – Be – O – F

25 | Chuẩn bị kiến thức cho kì thi HSGQG Hóa học


Hướng dẫn
1) Các kết quả có thể được ghi vào bảng:
Số
Nguyên tử Z Cấu hình electron electron
độc thân
1s2
He 2 0

1s22s2
Be 4 0

1s22s22p1
B 5 1

1s22s22p4
O 8 2

Chọn phương án D.
2) Cấu hình electron của nguyên tử lưu huỳnh là
1s22s22p63s23p6
Ngoài những trường hợp ngoại lệ, có thể được giải thích bởi các yếu tố
động học, thì quy luật chi phối đa số các phản ứng hóa học đều giống
nhau: Các tiểu phân tồn tại, hoặc có xu hướng hình thành tự phát, là
các tiểu phân bền nhất, do đó, có năng lượng thấp nhất.
Thường thì, độ bền của nguyên tử các nguyên tố có liên hệ với mức độ
bị chiếm của lớp vỏ electron ngoài cùng, và các nguyên tố bền nhất
thì có lớp vỏ bão hòa electron. Các nguyên tố này, được đặt ở cột cuối
của bảng tuần hoàn và được gọi là các khí trơ, đôi khi gọi là các khí quý,
bởi tính chất trơ về mặt hóa học, bền và không bắt cháy. Quy tắc này
được áp dụng cho cả sự tạo thành các ion. Nếu một ion tồn tại thì nó
bền. Trong trường hợp của các nguyên tử thuộc ba chu kì đầu tiên, có
thể nói rằng trong quá trình tạo thành ion, đa số các nguyên tố đều có
xu hướng tạo thành cấu hình electron của khí quý và chúng chọn con
đường biến đổi để cực tiểu hóa điện tích.
- Các nguyên tố thuộc cột 1, 2 và 13 có xu hướng nhường [mất] 1
hoặc nhiều electron để tạo thành các cation.

26 | Chuẩn bị kiến thức cho kì thi HSGQG Hóa học


- Các nguyên tố thuộc cột 15, 16, 17 có xu hướng nhận thêm 1 hoặc
nhiều electron để tạo thành các anion.
Lưu huỳnh thuộc cột 16 nên có xu hướng tạo thành anion S2-, có cấu
hình electron của argon: 1s22s22p63s23p6 (phương án A).
3) Áp dụng quy tắc tạo thành các ion ở ý 2 để trả lời câu hỏi này:
- Sodium thuộc cột 1 nên có xu hướng nhường 1 electron để tạo
thành cation Na+ có cấu hình electron của neon (1s22s22p6).
- Oxygen thuộc cột 16 nên có xu hướng nhận thêm 2 electron để
tạo thành anion O2- có cấu hình electron của neon (1s22s22p6).
Công thức oxide tạo thành là Na2O (phương án A). Lưu ý rằng đây là hợp
chất dễ tạo thành nhất nhưng không phải hợp chất duy nhất giữa
sodium và oxygen, bởi còn có các hợp chất khác như sodium peroxide
Na2O2, sodium superoxide NaO2, sodium ozonide NaO3.
4) Độ âm điện (kí hiệu là: E hoặc ) là đại lượng đặc trưng cho khả năng
hút electron của nguyên tử một nguyên tố khi tạo thành liên kết hoá
học. Hạt nhân càng gần (R càng bé) và hút electron càng mạnh đồng
nghĩa với độ âm điện càng lớn (E càng lớn).
▪ Trong cùng một nhóm, theo chiều từ trên xuống thì số lớp electron
càng tăng  R càng tăng, khoảng cách giữa hạt nhân – electron
càng lớn  E càng giảm.
▪ Trong cùng một chu kì (có số lớp electron bằng nhau), theo chiều
từ trái sang phải thì điện tích hạt nhân Z tăng dần  Bán kính R
càng giảm, hạt nhân hút electron càng mạnh  E càng tăng.

27 | Chuẩn bị kiến thức cho kì thi HSGQG Hóa học


Với các nguyên tố đã cho thì độ âm điện Sr < Be < O < F (phương án D).
5) Bán kính nguyên tử (kí hiệu: R) có thể được xem là khoảng cách từ
hạt nhân tới các electron lớp ngoài cùng. Bán kính nguyên tử chịu ảnh
hưởng từ số lượng lớp electron và điện tích hạt nhân:
- Nếu số lớp electron càng lớn thì khoảng cách giữa hạt nhân và
lớp electron ngoài cùng càng lớn, do đó bán kính nguyên tử
càng lớn. Mà như ta đã biết, số lớp electron chính là số chu kì.
 Các nguyên tố ở chu kì lớn hơn sẽ có bán kính lớn hơn. Vậy
theo chiều từ trên xuống của cùng một nhóm, R tăng dần.
- Trong cùng một chu kì, nguyên tử của các nguyên tố đều có số
lớp electron giống nhau. Do đó điện tích hạt nhân Z càng lớn thì
lực hút giữa hạt nhân – electron càng lớn  Electron bị hút lại
gần hạt nhân hơn  Bán kính giảm dần. Theo chiều từ trái sang
phải của cùng một chu kì, R giảm dần.

28 | Chuẩn bị kiến thức cho kì thi HSGQG Hóa học


Với các nguyên tố đã cho thì bán kính Sr > Be > O > F (phương án D).

29 | Chuẩn bị kiến thức cho kì thi HSGQG Hóa học


Chương 2: Phân tử
Công thức Lewis (công thức điện tử) biểu diễn sự phân bố của tất cả
các electron hóa trị trong một vi hạt. Các cặp electron liên kết thường
được biểu diễn bởi các đường kẻ liền, còn các cặp electron chưa liên
kết thì bởi các dấu chấm.

Chủ đề: Các bước thiết lập công thức Lewis


1. Đếm tổng số electron hóa trị (a).
2. Đếm tổng số electron bị thiếu (b) (còn gọi là “độ hụt”), nghĩa là tổng
số electron cần thêm để đạt tới cấu hình electron của khí hiếm
quanh mỗi nguyên tử. Độ hụt này (b) chính là số electron trong các
cặp electron liên kết.
3. Chênh lệch giữa (a) và (b) là số electron trong các cặp electron chưa
liên kết.
4. Tính điện tích hình thức trên mỗi nguyên tử. Bạn có thể tính điện
tích hình thức bằng cách đếm số electron hóa trị thực trên mỗi
nguyên tử (= số cặp electron liên kết × 1 + số cặp electron chưa liên
kết × 2). Điện tích hình thức bằng: Số electron hóa trị trong nguyên
tử chưa tạo liên kết trừ đi số electron hóa trị thực này.
Hãy cố giữ cho các điện tích hình thức nhỏ nhất có thể bằng cách
dịch chuyển (tái sắp xếp) một hoặc nhiều cặp electron. Hãy nhớ
rằng các nguyên tử từ chu kì 3 trở đi trong bảng Tuần hoàn (đặc biệt
là P và S) có thể có nhiều hơn bốn cặp electron xung quanh chúng.
B thường chỉ có ba cặp electron.
5. Nguyên tử trung tâm thường có độ âm điện nhỏ nhất; các nguyên
tử âm điện nhất (và các nguyên tử chỉ tạo duy nhất một liên kết) thì
ở phía bên ngoài.

30 | Chuẩn bị kiến thức cho kì thi HSGQG Hóa học


Ví dụ 1: H2SO4
Tổng số electron hóa trị 2×1+5×6= 32

Số electron còn thiếu: 2×1+5×2= 12 – ứng với 6 cặp liên kết

20 - ứng với 10 cặp chưa liên kết

Ví dụ 2: NO3-
Tổng số electron hóa trị 5+3×6+1= 24
Số electron còn thiếu: 3+3×2–1= 8 – ứng với 4 cặp liên kết

16 - ứng với 8 cặp chưa liên kết

Với ion nitrate, không có sự tái sắp xếp khả dĩ các cặp electron để tạo
thành những điện tích hình thức nhỏ hơn. Tuy nhiên, bạn có thể biểu
diễn được hai công thức điện tử tương đương khác và cấu trúc thực tế
“nằm giữa” những công thức tới hạn này – hiện tượng này gọi là “sự cộng
hưởng”. Ba cặp electron không có vị trí cố định trong các công thức tới
hạn mà chúng được giải tỏa. Toàn bộ các liên kết N-O đều tương đương
nhau với bậc liên kết3 là 4/3.

3
Bậc liên kết là số cặp electron liên kết của một nguyên tử trong vi hạt.

31 | Chuẩn bị kiến thức cho kì thi HSGQG Hóa học


Ví dụ 3: BF3
Tổng số electron hóa trị 3+3×7= 24

Số electron còn thiếu: 5+3×1= 8 – ứng với 4 cặp liên kết

16 - ứng với 8 cặp chưa liên kết

Tái sắp xếp các cặp electron theo quy tắc ở bước 4 sẽ tạo thành công
thức với nguyên tử trung tâm B chỉ có ba liên kết.

32 | Chuẩn bị kiến thức cho kì thi HSGQG Hóa học


Chủ đề 1: Công thức Lewis
1) Biểu diễn công thức Lewis của phân tử hydroxylamine NH2OH:

2) Biểu diễn công thức Lewis của phân tử iodine trichloride ICl3:

3) Chỉ rõ điện tích hình thức trên mỗi nguyên tử của ion sulfite SO32-.

33 | Chuẩn bị kiến thức cho kì thi HSGQG Hóa học


4) Trong một cấu tạo đa nguyên tử, để tính điện tích hình thức của
một nguyên tử thì
A. Các electron của mỗi cặp tạo liên kết được gán 100% vào nguyên
tử âm điện hơn.
B. Các electron của mỗi cặp tạo liên kết được chia đều 50%-50%
giữa hai nguyên tử tham gia tạo liên kết.
C. Các electron của mỗi cặp tạo liên kết được gán 100% vào hai
nguyên tử tham gia tạo liên kết.
D. Các electron của các cặp tạo liên kết không được xét đến.
5) Xác định dạng cộng hưởng nào sau đây của ion chlorate ClO3- là bền
nhất:

A. Dạng 1, bởi tính đối xứng của nó.


B. Dạng 2, bởi nó thỏa mãn quy tắc bát tử (octet).
C. Dạng 3, bởi nó có ba liên kết đôi.
D. Dạng 4.

34 | Chuẩn bị kiến thức cho kì thi HSGQG Hóa học


Hướng dẫn
1) Công thức Lewis (hay “giản đồ Lewis”) của một phân tử thường phải
thỏa mãn ba điều kiện sau:
(1) Số electron xuất hiện trên công thức Lewis của phân tử phải
bằng tổng số electron hóa trị xuất hiện trên công thức Lewis
của mỗi nguyên tử;
(2) Phải thỏa mãn quy tắc nhị tử (duet) cho hydrogen và bát tử
(octet) cho nguyên tử các nguyên tố thuộc chu kì 2 và 3.
(3) Với các nguyên tố thuộc ba chu kì đầu tiên, phải tuân theo các
hóa trị phổ biến.
Hãy đánh giá các phân tử đã cho theo mỗi điều kiện trên.
Điều kiện (1): Các nguyên tử hydrogen (Z = 1), nitrogen (Z = 7) và oxygen
(Z = 8) tham gia tạo thành phân tử hydroxylamine có công thức Lewis
như sau:

Phân tử hydroxylamine NH2OH gồm 3 nguyên tử hydrogen, 1 nguyên tử


nitrogen và 1 nguyên tử oxygen. Do đó, tổng số electron phải xuất hiện
trong công thức Lewis của phân tử là:
3  1 + 1 5 + 1 6 = 14 electron
Các công thức Lewis B và C đều có 14 electron nên được giữ lại. Còn
các công thức A và D, chỉ có 12 electron, thì không hợp lí và bị loại bỏ.
Điều kiện (2): Chúng ta sẽ đếm số electron xung quanh mỗi nguyên tử,
xét đến tất cả các electron hóa trị, bất kể chúng thuộc cặp electron
chưa liên kết hay đã tạo liên kết. Trong các công thức đã cho thì công
thức A không thỏa mãn quy tắc octet do xung quanh nitrogen chỉ có 6
electron.
Điều kiện (3): Hóa trị của một nguyên tố là số liên kết đơn cộng hóa trị
mà nguyên tử của nguyên tố đó có thể tham gia vào. Do đó, nó cũng
thường là số electron độc thân xuất hiện trên công thức Lewis của
nguyên tử.
Hóa trị của hydrogen là 1 – được thỏa mãn ở cả bốn công thức Lewis đã
đề xuất.

35 | Chuẩn bị kiến thức cho kì thi HSGQG Hóa học


Hóa trị của nitrogen bằng 3 – được thỏa mãn ở các công thức A và C.
Nhưng không hợp lí (đều bằng 4) ở các công thức B và D.
Hóa trị của oxygen bằng 2 – được thỏa mãn ở các công thức A và C.
Nhưng không hợp lí ở các công thức B (bằng 1) và D (bằng 3).
Tổng kết lại: Chọn phương án C.
2) Hãy phân tích cả ba điều kiện như đã trình bày ở ý 1 ở trên.
Điều kiện (1): Các nguyên tử chlorine (Z = 17) và iodine (Z = 53) tham gia
tạo thành phân tử iodine trichloride có công thức Lewis tương tự nhau.

Phân tử iodine trichloride ICl3 có 3 nguyên tử chlorine và 1 nguyên tử


iodine. Do đó, tổng số electron phải xuất hiện trong công thức Lewis
của phân tử là: 3  7 + 1 7 = 28 electron
Công thức Lewis A có tổng cộng 28 electron được giữ lại. Công thức B
chỉ có 24 electron, còn các công thức C và D đều chỉ có 26 electron, thì
không phù hợp.
Có vẻ như phương án chính xác là A. Nhưng để chắc chắn, bạn có thể
xét thêm các điều kiện còn lại.
Điều kiện (2): Trong cả bốn công thức đã đề xuất thì quy tắc octet được
thỏa mãn với trường hợp chlorine. Tuy nhiên, trong công thức C thì
xung quanh iodine có 8 electron nhưng với B thì chỉ có 6 electron và với
A và D thì đều là 10 electron. Có vẻ như, nếu chỉ xét đến điều kiện (2) thì
công thức C là một lựa chọn tốt – tuy nhiên, điều đó lại không đúng
trong trường hợp này. Từ chu kì 4, quy tắc bát tử (octet/8e) và thập bát
tử (18e) thường không còn đóng vai trò quyết định.
Điều kiện (3): Chlorine và iodine đều có hóa trị 1. Hóa trị của chlorine
được thỏa mãn trong các công thức A, B, C. Nhưng hóa trị của iodine
thì không bằng 1 trong bất kì trường hợp nào cả: nó bằng 3 trong các
công thức A, B, C và bằng 4 trong công thức D. Điều này cũng không
có gì bất ngờ, với một nguyên tố thuộc chu kì 5 như iodine thì các điều
kiện đã cho ở trên không thể áp dụng máy móc được.

36 | Chuẩn bị kiến thức cho kì thi HSGQG Hóa học


Liệu chăng có một công thức nào khác bốn công thức đã đề xuất mà
các điều kiện (1) – (3) đều được thỏa mãn với iodine không? Đáng tiếc
là không.
Tổng hợp lại, phương án phù hợp nhất là công thức A.
Bài học rút ra ở đây: Hóa học là khoa học thực nghiệm, những quy tắc
hay điều kiện chỉ mang tính tương đối, chứ không được áp dụng máy
móc!
3) Các nguyên tử oxygen (Z = 8) và lưu huỳnh (Z = 16) tham gia tạo thành
ion sulfite có công thức Lewis tương tự nhau.

Ion sulfite SO32- gồm 3 nguyên tử oxygen, 1 nguyên tử lưu huỳnh và


mang điện tích toàn phần là -2. Do đó, tổng số electron phải xuất hiện
trong công thức Lewis của ion là:

( 3 6 + 1 6 ) + 2 = 26 electron
Trong mỗi công thức đã đề xuất đều có 26 electron.
Hãy xác định điện tích hình thức của mỗi nguyên tử.
Điện tích hình thức được xác định bằng cách so sánh số electron hóa
trị và số electron phân bố xung quanh sau khi phân chia 50%-50% từ
các cặp electron tham gia tạo liên kết (kể cả liên kết cho-nhận; chính
bởi vậy nên ta mới gọi là “điện tích hình thức”). Điện tích toàn phần của
cấu trúc đa nguyên tử (phân tử hoặc ion) bằng tổng đại số các điện tích
hình thức của tất cả các nguyên tử thành phần.
Theo đó, ta xác định được:
- Với nguyên tử oxygen liên kết với lưu huỳnh qua liên kết đôi: có
6 elecron, tương đương với số electron hóa trị của nó, vậy nên
điện tích hình thức bằng 0.
- Với hai nguyên tử oxygen liên kết với lưu huỳnh qua liên kết đơn:
có 7 electron, dư 1 electron, do đó điện tích hình thức là (1-).
- Với nguyên tử lưu huỳnh: có 6 elecron, tương đương với số
electron hóa trị của nó, vậy nên điện tích hình thức bằng 0.
Vậy phương án đúng là C.

37 | Chuẩn bị kiến thức cho kì thi HSGQG Hóa học


4) Như đã chỉ ra ở ý trước: điện tích hình thức của một nguyên tử bằng
số electron hóa trị trừ đi số electron phân bố xung quanh (được xác
định sau khi phân chia 50%-50% từ các cặp electron tham gia tạo liên
kết; bất kể có phải liên kết cho-nhận hay không).
Chọn phương án B.
5) Thường thì dạng cộng hưởng bền nhất (có khả năng tồn tại cao nhất)
là dạng:
(1) Phù hợp với bản chất độ âm điện của các nguyên tố;
(2) Có số lượng và giá trị các điện tích hình thức nhỏ nhất;
Hãy xét các điều kiện này cho trường hợp ion chlorate.
Điều kiện (1): Oxygen âm điện hơn chlorine. Tuy nhiên, dạng 3 có điện
tích âm trên chlorine mà lại không có điện tích trên oxygen – bởi lí do
này nên phải loại dạng 3.
Điều kiện (2): Các điện tích hình thức xuất hiện trên ba nguyên tử ở
dạng 2, và trên bốn nguyên tử ở dạng 1, nhưng chỉ có duy nhất trên một
nguyên tử ở dạng 4. Bên cạnh đó, ở dạng 1 còn tồn tại điện tích hình
thức (2+). Trên phương diện này thì dạng 4 có ưu thế nhất.
Chọn phương án D.

38 | Chuẩn bị kiến thức cho kì thi HSGQG Hóa học


Đọc thêm: Cấu trúc cộng hưởng
Đôi khi, liên kết trong một số phân tử hoặc ion không thể được mô tả
bởi chỉ một công thức Lewis duy nhất. Trong những trường hợp như
vậy, sự cộng hưởng được sử dụng để mô tả sự giải tỏa các electron.
Phân tử hoặc ion với các electron được giải tỏa được biểu diễn bởi một
số cấu trúc cộng hưởng thành phần (đôi khi được gọi là các dạng chính
tắc). Các cấu trúc cộng hưởng thành phần liên quan đến “sự dịch
chuyển tưởng tượng” của các electron tạo liên kết pi hoặc các electron
chưa tham gia tạo liên kết. Chú ý rằng trong quá trình cộng hưởng, các
liên kết sigma hoặc pi không thể được phá vỡ hoặc tạo thành. Tương
tự, vị trí của các nguyên tử cũng không thể thay đổi giữa các cấu trúc
cộng hưởng thành phần.
1) Mỗi phân tử hoặc ion chỉ có duy nhất một cấu trúc thực. Cấu trúc
thực này (gọi là “lai hóa cộng hưởng”) có những đặc trưng trung bình
của tất các cấu trúc cộng hưởng thành phần. Khi xét các cấu trúc cộng
hưởng thành phần thì chúng ta thấy cùng một phân tử hoặc ion nhưng
được biểu diễn theo những cách khác nhau.

Các cấu trúc cộng hưởng thành phần chính của ion formate.

Các biểu diễn lai hóa cộng hưởng của formate.


2) Lai hóa cộng hưởng bền hơn mọi cấu trúc cộng hưởng riêng lẻ.
Thường thì các cấu trúc cộng hưởng đại diện cho sự dịch chuyển điện
tích giữa hai hoặc nhiều nguyên tử. Điện tích được phân tán giữa các
nguyên tử này – do đó, được bền hóa thêm. Khi nhìn vào hình ảnh về
các cấu trúc cộng hưởng thành phần ở trên của ion formate thì điện
tích âm nằm ở một trong hai nguyên tử oxygen. Còn lai hóa cộng hưởng
biểu diễn điện tích âm được chia đều giữa hai oxygen. Trong lai hóa
cộng hưởng, điện tích âm được phân tán trên một hợp phần lớn hơn
của phân tử, do đó bền hơn.

39 | Chuẩn bị kiến thức cho kì thi HSGQG Hóa học


3) Các cấu trúc cộng hưởng thành phần không nhất thiết phải tương
đương nhau. Hai cấu trúc cộng hưởng dưới đây không tương đương bởi
một cấu trúc biểu diễn điện tích âm trên oxygen, trong khi cấu trúc kia
thì biểu diễn nó trên carbon. Cấu trúc thành phần với điện tích âm trên
oxygen (âm điện hơn) thì bền hơn – điều này có nghĩa rằng lai hóa cộng
hưởng không phải là hỗn hợp của hai cấu trúc thành phần với sự đóng
góp tương đương nhau.

4) Tất cả các cấu trúc cộng hưởng thành phần đều phải là những công
thức Lewis chính xác. Mỗi nguyên tử phải có lớp vỏ hóa trị hoàn chỉnh
(thường là thỏa mãn quy tắc duet hoặc octet) và biểu diễn điện tích
hình thức chính xác. Riêng trong các phân tử hữu cơ, carbocation (tức
tiểu phân chứa carbon với chỉ 6 electron hóa trị) là ngoại lệ duy nhất
được cho phép. Dưới đây là một cấu trúc cộng hưởng không hợp lệ, kể
cả khi nó chỉ biểu diễn sự dịch chuyển của một liên kết pi. Cấu trúc tạo
thành chứa một carbon với 10 electron, vi phạm quy tắc octet.

5) Tất các các cấu trúc cộng hưởng thành phần phải có cùng công thức
phân tử, cùng số electron và cùng điện tích toàn phần. Các phân tử
trong hình dưới đây không phải là các cấu trúc cộng hưởng bởi chúng
có công thức phân tử khác nhau (C2H5NO với C2H6NO). Ngoài ra, hai cấu
trúc cũng có điện tích toàn phần khác nhau (trung hòa với dương điện).

6) Các cấu trúc cộng hưởng thành phần chỉ khác nhau về vị trí của liên
kết pi và các cặp electron chưa liên kết. Các liên kết sigma không bao
giờ bị phá vỡ hoặc tạo thành, bởi các nguyên tử phải được bảo toàn vị

40 | Chuẩn bị kiến thức cho kì thi HSGQG Hóa học


trí. Các phân tử trong hình dưới đây không phải các cấu trúc cộng
hưởng dù rằng chúng có cùng công thức phân tử (C3H6O). Các phân tử
này được xem là các đồng phân cấu tạo bởi sự khác biệt giữa chúng
liên quan đến sự phá vỡ một liên kết sigma và dịch chuyển một nguyên
tử hydrogen.

41 | Chuẩn bị kiến thức cho kì thi HSGQG Hóa học


Đọc thêm: Độ bền của các cấu trúc cộng hưởng
Như đã nói ở ở phần trước,cấu trúc thực (tức “lai hóa cộng hưởng”) là sự
kết hợp của tất cả các cấu trúc cộng hưởng khả dĩ. Nếu các cấu trúc
cộng hưởng tương đương nhau về độ bền thì chúng đóng góp như nhau
vào cấu trúc dạng lai hóa. Tuy nhiên, nếu các cấu trúc cộng hưởng
khác nhau về độ bền thì chúng đóng góp vào cấu trúc lai hóa theo các
tỉ lệ liên quan đến độ bền tương đối. Có thể nói rằng lai hóa cộng hưởng
giống với cấu trúc cộng hưởng thành phần bền nhất. Bởi vậy, việc so
sánh độ bền của các cấu trúc cộng hưởng thành phần là điều rất quan
trọng. Trong ví dụ dưới đây, cấu trúc B kém quan trọng hơn về phương
diện đóng góp vào lai hóa cộng hưởng, bởi nó chứa một carbocation vi
phạm quy tắc octet. Độ bền tương đối của hai cấu trúc này có sự khác
biệt rất lớn, đến nỗi các cấu trúc chứa liên kết C=O hầu như chỉ được
viết ở dạng tương tự cấu trúc A.

Các quy tắc để đánh giá độ bền của cấu trúc cộng
hưởng thành phần
1) Các cấu trúc cộng hưởng trong đó tất cả các nguyên tử có vỏ hóa trị
hoàn chỉnh thì bền hơn. Điều này có nghĩa là đa số các nguyên tử có
lớp vỏ 8e (riêng với hydrogen là 2e). Trong ví dụ dưới đây, cấu trúc A có
nguyên tử carbon với điện tích
dương và do đó có vỏ octet không
hoàn chỉnh. Theo luận điểm này,
cấu trúc A kém bền hơn và đóng
góp ít hơn vào lai hóa cộng hưởng.

2) Các cấu trúc với số lượng nguyên tử mang điện tích hình thức ít
nhất thì bền hơn. Dựa trên quan điểm này, cấu trúc B kém bền hơn bởi
nó có hai nguyên tử mang điện tích hình thức, trong khi đó cấu trúc A
thì không. Cấu trúc A sẽ là cấu trúc cộng hưởng thành phần chính.

42 | Chuẩn bị kiến thức cho kì thi HSGQG Hóa học


3) Các cấu trúc với điện tích âm trên nguyên tử âm điện hơn thì bền
hơn. Khác biệt giữa hai cấu trúc dưới đây là vị trí đặt điện tích âm. Cấu
trúc B bền hơn và là cấu trúc cộng hưởng thành phần chính, bởi điện
tích âm ở trên nguyên tử oxygen âm điện hơn.

Tương tự, cấu trúc với điện tích dương trên trên nguyên tử kém âm điện
hơn thì bền hơn.

4) Các cấu trúc ít phân tách


các điện tích hình thức thì
bền hơn. Sự khác biệt giữa hai
cấu trúc dưới đây là vị trí
tương đối của các điện tích
dương và âm. Trong cấu trúc
A, các điện tích gần nhau hơn, làm cho nó bền hơn.

5) Các dạng cộng hưởng tương đương thì không khác nhau về độ bền.
Khi nhìn vào hai cấu trúc dưới đây thì không có sự khác biệt nào dựa
theo những tiêu chí đánh giá ở trên – có nghĩa rằng chúng tương đương
về độ bền và có sự đóng góp như nhau vào lai hóa cộng hưởng.

43 | Chuẩn bị kiến thức cho kì thi HSGQG Hóa học


Chủ đề 1: Sự phân cực của phân tử HCl
1) Hãy chọn ra phát biểu đúng:
A. Phân tử hydrogen chloride HCl phân cực bởi nó không có tâm đối
xứng.
B. Phân tử hydrogen chloride HCl phân cực bởi nó không có trục đối
xứng.
C. Phân tử hydrogen chloride HCl phân cực bởi độ âm điện của H và
Cl khác nhau.
D. Phân tử hydrogen chloride HCl phân cực bởi nó thiếu liên kết bội.
2) Khoảng cách tương tác trong phân tử HCl là dH−Cl = 127.4 pm. Hãy
tính moment lưỡng cực lí thuyết ptheo của phân tử HCl, giả sử rằng
đây là liên kết ion thuần túy.

A. ptheo = 2.04  10−29 C m B. ptheo = 1.02  10−29 C m

C. ptheo = 0.679 D D. ptheo = 3.06 D

3) Moment lưỡng cực thực nghiệm của phân tử HCl là pexp = 1.07 D. Hãy
tính tỉ lệ đặc trưng ion của liên kết H-Cl.
A. 63.5% B. 57.5% C. 35% D. 17.5%

44 | Chuẩn bị kiến thức cho kì thi HSGQG Hóa học


4) Trong bảng tuần hoàn nguyên tố, chlorine và bromine đều thuộc
chung nhóm (cùng cột) halogen, bromine nằm ở ngay dưới
chlorine.
A. Do chlorine và bromine thuộc cùng cột trong bảng tuần hoàn
nên tỉ lệ đặc trưng ion của các liên kết H-Cl và H-Br giống nhau.
B. Bán kính của nguyên tử chlorine nhỏ hơn của bromine, nên tỉ lệ
đặc trưng ion của liên kết H-Cl nhỏ hơn của liên kết H-Br.
C. Chlorine có độ âm điện lớn hơn bromine, tỉ lệ đặc trưng ion của
liên kết H-Cl cao hơn của liên kết H-Br.
D. Do chlorine có độ âm điện nhỏ hơn bromine nên tỉ lệ đặc trưng
ion của liên kết H-Cl cao hơn của liên kết H-Br.

45 | Chuẩn bị kiến thức cho kì thi HSGQG Hóa học


Hướng dẫn
1) Một phân tử bị phân cực khi trọng tâm G+ của các điện tích dương
không trùng với trọng tâm G- của các điện tích âm – điều này có nghĩa
là phân tử không có tâm đối xứng. Trong trường hợp phân tử lưỡng
nguyên tử, điều này chỉ xảy ra khi phân tử được tạo thành bởi hai
nguyên tử khác nhau, được đặc trưng bởi các độ âm điện khác nhau.
Chọn các phương án A và C.
2) Hãy xét sự phân bố điện tích toàn phần của các điện tích điểm, với
G+ và G- lần lượt là trọng tâm của các điện tích dương ở một phía và
trọng tâm của các điện tích âm ở phía còn lại. Đặt q là tổng điển tích
dương, bằng giá trị đối của tổng điện tích âm. Moment lưỡng cực điện
là: p = q G−G+ .

Đơn vị SI của moment lưỡng cực là Coulomb·metre (C·m), nhưng đơn vị


10−29
Debye (D) cũng thường được sử dụng: 1 D = C m
3
Moment lưỡng cực lí thuyết ptheo của phân tử HCl được tính với giả định
đây là liên kết thuần túy ion. Trong trường hợp này, G+ hợp nhất với hạt
nhân ion H+, G- hợp nhất với hạt nhân ion Cl-, và q = e.

ptheo = q G− G+ = e  dH−Cl


= 1.6  10−19  127.4  10−12 = 2.04  10−29 C m = 6.12 D
Chọn phương án A.
3) Tất cả các liên kết dị tố (tạo thành bởi hai nguyên tử khác loại) đều
phân cực. Nó có một phần đặc trưng cộng hóa trị, một phần đặc trưng
ion. Hãy tính tỉ lệ đặc trưng ion của liên kết H-Cl:
pexp 1.07
 100 =  100 = 17.5% (phương án D)
ptheo 6.12

4) Trong cùng một cột (nhóm) của bảng tuần hoàn thì độ âm điện tăng
theo chiều từ dưới lên trên. Do đó, chlorine có độ âm điện lớn hơn

46 | Chuẩn bị kiến thức cho kì thi HSGQG Hóa học


bromine. Điều này có nghĩa rằng điện tích riêng phần  + xuất hiện trên
nguyên tử hydrogen có vai trò quan trọng hơn (lớn hơn) trong trường
hợp HCl (so với trường hợp HBr), như minh họa trong giản đồ dưới đây:

Với các halogen acid HX thì tỉ lệ đặc trưng ion được viết là:
pexp +  dH−X +
 100 =  100 =  100
ptheo e  dH−X e

Vậy halogen acid có liên kết với đặc trưng ion càng lớn khi  + càng lớn.
Do đó, tỉ lệ đặc trưng ion của liên kết H-Cl lớn hơn của liên kết H-Br.
Chọn phương án C.

47 | Chuẩn bị kiến thức cho kì thi HSGQG Hóa học


Chủ đề 2: Các hợp chất thơm
1) Dichlorobenzene C6H4Cl2 tồn tại ở ba dạng đồng phân, khác nhau
về vị trí tương đối của hai nguyên tử chlorine trên vòng benzene:

A. Cả ba dạng đều phân cực.


B. Dạng ortho và meta phân cực, dạng para không phân cực.
C. Dạng meta phân cực, dạng ortho và para không phân cực.
D. Dạng para phân cực, dạng ortho và meta không phân cực.
2) Biểu diễn moment lưỡng cực của dạng meta của dichlorobenzene
ở dạng hàm của moment lưỡng cực của liên kết C-Cl.

3
A. pmeta = p B. pmeta = 3pC−Cl
2 C−Cl
C. pmeta = 0 D. pmeta = pC−Cl

3) Hình dưới đây là công thức cấu tạo của 1,4-dibromo-2-nitro-


benzene, có moment lưỡng cực là 4.3 D. Hãy tính moment lưỡng
cực của liên kết C-NO2 trong phân tử này.

A. pC−NO = 2.15 D B. pC−NO = 3.7 D


2 2

C. pC−NO = 4.3 D D. Không tính được.


2

48 | Chuẩn bị kiến thức cho kì thi HSGQG Hóa học


4) Dưới đây là công thức cấu tạo của 4-nitrotoluene và 4-
chlorotoluene.

A. Nhóm CH3 tạo nên cực dương của lưỡng cực C-CH3, còn các
nhóm NO2 và Cl tạo nên cực âm của các lưỡng cực C-NO2 và C-Cl.
B. Các nhóm NO2 và CH3 tạo nên cực dương của các lưỡng cực C-
NO2 và C-CH3, còn nhóm Cl tạo nên cực âm của lưỡng cực C-Cl.
C. Nhóm Cl tạo nên cực dương của lưỡng cực C-Cl, còn các nhóm
NO2 và CH3 tạo nên cực âm của các lưỡng cực C-NO2 và C-CH3.
D. Các nhóm NO2, CH3 và Cl tạo nên cực âm của các lưỡng cực C-
NO2, C-CH3 và C-Cl.
5) Moment lưỡng cực của 4-nitrotoluene bằng 4.7 D, của 4-
chlorotoluene bằng 2.4 D. Hãy suy ra moment lưỡng cực của các
liên kết C-CH3 và C-Cl trong các phân tử này.
A. pC−CH = 1 D và pC−Cl = 1.4 D. B. pC−CH = 1.4 D và pC−Cl = 1 D.
3 3

C. pC−CH = 0.4 D và pC−Cl = 2 D. D. pC−CH = 9 D và pC−Cl = 6.6 D.


3 3

49 | Chuẩn bị kiến thức cho kì thi HSGQG Hóa học


Hướng dẫn
1) Phân tử benzene (C6H6) được biểu diễn bởi lai hóa cộng hưởng tạo
thành bởi 6 liên kết carbon-carbon hoàn toàn không thể phân biệt: độ
dài của chúng giống nhau (là giá trị trung gian giữa độ dài liên kết đơn
C-C và độ dài liên kết đôi C=C) và các góc liên kết CCC = 120o. Do đó,
phân tử benzene có một tâm đối xứng và nó không phân cực. Ngược
lại, liên kết C-Cl thì bị phân cực. Do có độ âm điện lớn hơn carbon nên
chlorine là đầu âm của lưỡng cực này.
Dichlorobenzene là hai liên kết phân cực và moment lưỡng cực toàn
phần được xác định theo tổng vector: pC6H4Cl2 = pC−Cl(1) + pC−Cl(2)

Cấu trúc hình học chỉ rõ rằng:


- Dạng para không phân cực.
- Dạng ortho và meta phân cực, moment lưỡng cực của dạng
ortho lớn hơn của dạng meta.
Chọn phương án B.

50 | Chuẩn bị kiến thức cho kì thi HSGQG Hóa học


2) Trong dạng meta của dichlorobenzene (như biểu diễn trong hình
dưới đây) thì các vector pC−Cl(1) ,pC−Cl(2) và pC6H4Cl2 tạo thành tam giác cân

có góc  = . Do đó, đây còn là trường hợp đặc biệt (tam giác đều).
3

pmeta = pC−Cl

Chọn phương án D.
3)
- Phân tử benzene, với một tâm đối xứng, không phân cực.
- Liên kết C-Br phân cực. Bromine thuộc nhóm 17 trong bảng tuần
hoàn, còn carbon thuộc nhóm 14. Do đó, bromine có độ âm điện lớn
hơn carbon, và tạo thành đầu âm của lưỡng cực C-Br.
- Liên kết C-NO2 cũng phân cực. Phân tử nitrobenzene thực là lai hóa
cộng hưởng của ba dạng cộng hưởng thành phần như sau:

Ở dạng cộng hưởng đầu tiên, vòng thơm và nhóm NO2 trung hòa
điện. Mặt khác, ở hai dạng cộng hưởng còn lại, thì vòng thơm
mang điện tích dương, trong khi đó nhóm NO2 mang điện tích âm.
Do đó, nhóm NO2 tạo thành đầu âm của lưỡng cực C-NO2.
Vậy, phân tử 1,4-dibromo-2-nitro-benzene có ba liên kết phân cực và
moment lưỡng cực toàn phần của nó được xác định theo tổng vector:

pC6H3Br2NO2 = pC−Br(1) + pC−Br(2) + pC−NO2

51 | Chuẩn bị kiến thức cho kì thi HSGQG Hóa học


Các lưỡng cực C-Br(1) và C-Br(2) nằm ở các vị trí 1 và 4 trên vòng
benzene, gây ra hiệu ứng triệt tiêu lẫn nhau:

pC−Br(1) + pC−Br(2) = 0

Do đó: pC6H3Br2NO2 = pC−NO2  pC−NO = 4.3 D (phương án C).


2

4)
- Chlorine có độ âm điện lớn hơn carbon và tạo thành đầu âm của
lưỡng cực C-Cl.
- Như đã trình bày ở phần 3 thì NO2 tạo thành đầu âm của lưỡng cực
C-NO2.
- Mặt khác, liên kết C-CH3 được tạo thành bởi hai nguyên tử carbon
giống nhau (cùng độ âm điện). Hơn nữa, sự bền hóa cộng hưởng của
toluene cũng không thể hiện bất kì điện tích tích hình thức. Do đó,
sẽ rất hợp lí khi học sinh đặt ra câu hỏi là: Tại sao liên kết Cbenzene-
CH3 trong toluene lại phân cực, và định hướng của lưỡng cực
Cbenzene-CH3 là như thế nào? Đáp án cho câu hỏi này được tìm thấy
từ những liên kết liền kề. Đừng quên rằng 3 liên kết C-H của CH3 bị
phân cực, do mỗi liên kết đều tạo thành bởi hai nguyên tử khác
nhau. Do carbon có độ âm điện lớn hơn hydrogen nên các electron
từ mỗi cặp tạo thành liên kết C-H đều dịch chuyển (bị kéo lệch) về
carbon, gây ra lực đẩy tĩnh điện giữa các electron di động. Hệ quả
là cặp electron của liên kết Cbenzene-CH3 bị đẩy về phía vòng benzene
như minh họa trong hình dưới đây:

Cân bằng điện tích cho thấy có sự thiếu hụt điện tích trên nhóm
CH3, hệ quả là nó tạo thành đầu dương của lưỡng cực Cbenzene-CH3.

52 | Chuẩn bị kiến thức cho kì thi HSGQG Hóa học


Chọn phương án A.
5) Moment lưỡng cực của lưỡng cực C-NO2 đã được xác định ở ý 3 là:
pC−NO = 4.3 D
2

Phân tử 4-nitrotoluene có hai liên kết phân cực, và moment lưỡng cực
toàn phần của nó được xác định theo tổng vector:

pC6H4CH3NO2 = pC−CH3 + pC−NO2

Ta nhận được: pC H CH NO = pC−CH + pC−NO


6 4 3 2 3 2

 pC−CH = pC H CH NO − pC−NO = 4.7 − 4.3 = 0.4 D


3 6 4 3 2 2

Phân tử 4-chlorotoluene có hai liên kết phân cực, và moment lưỡng


cực toàn phần của nó được xác định theo tổng vector:

pC6H4CH3Cl = pC−CH3 + pC−Cl

53 | Chuẩn bị kiến thức cho kì thi HSGQG Hóa học


Ta nhận được: pC H CH Cl = pC−CH + pC−Cl
6 4 3 3

 pC−Cl = pC H CH Cl − pC−CH = 2.4 − 0.4 = 2 D


6 4 3 3

Chọn phương án C.

54 | Chuẩn bị kiến thức cho kì thi HSGQG Hóa học


Chủ đề 3: Hình học phân tử
Dạng hình học phân tử
Số lập
thể 0 cặp chưa 1 cặp chưa liên 2 cặp chưa liên 3 cặp chưa
liên kết kết kết liên kết
2
Thẳng

3
Tam giác
phẳng Gấp khúc

Tứ diện Tháp tam giác Gấp khúc

Lưỡng tháp
tam giác Bập bênh Dạng chữ T Thẳng

Bát diện Tháp đáy vuông Vuông phẳng

Lưỡng tháp Tháp đáy ngũ Ngũ phương


ngũ giác giác phẳng

Phương pháp VSEPR, có thể được áp dụng cho bất kì nguyên tử nào
trong một phân tử, để giúp dự đoán tương đối dạng hình học không gian
của phân tử trong vùng lân cận của nguyên tử này. Công thức VSEPR
cho một nguyên tử A trong phân tử được xác định bởi giản đồ Lewis
của phân tử và được viết ở dạng AXmEn. Trong công thức này:

55 | Chuẩn bị kiến thức cho kì thi HSGQG Hóa học


- m chỉ số nguyên tử X liên kết với A.
- n chỉ số cấu phần electron chưa liên kết (cặp hoặc electron
riêng lẻ) mà A có.
Mỗi công thức tương ứng với một dạng hình học nhất định. Các nhóm
công thức phổ biến nhất được cho trong bảng.
Sự biến đổi góc giữa các liên kết
Các giá trị góc liên kết cho trong bảng chỉ có ý nghĩa biểu thị tương đối.
Chúng chỉ đạt giá trị chính xác khi ba điều kiện dưới đây được đồng thời
thỏa mãn.
- n = 0 (tức là A không có cấu phần electron chưa liên kết);
- Toàn bộ các liên kết A-X đều có cùng độ bội (cùng là liên kết
đơn, hoặc đôi, …)
- Toàn bộ các nguyên tử X liên kết với nguyên tử A đều giống
nhau.
Trong tất cả những trường hợp khác, góc liên kết sẽ biến đổi. Sự phi
đối xứng có thể gây ra bởi: (1) sự hiện diện của một hoặc nhiều cặp
electron chưa liên kết trên nguyên tử trung tâm A; (2) (2) sự tồn tại của
các liên kết khác nhau về độ bội; (3) độ âm điện khác nhau giữa các
nguyên tử X khác nhau.
Dựa vào lí thuyết được cho ở trên để trả lời các câu hỏi sau đây:
1) Phân tử cyanamide có công thức cấu tạo N≡C−NH2. Xác định công
thức VSEPR của carbon và của nitrogen trong nhóm NH2.
A. Carbon: AX2E0; nitrogen: AX3E1
B. Carbon: AX2E0; nitrogen: AX3E0
C. Carbon: AX4E0; nitrogen: AX3E1
D. Carbon: AX4E0; nitrogen: AX3E0
2) Xác định dạng hình học của phân tử cyanamide:
A. Tam giác phẳng.
B. Tháp tam giác.
C. Tứ diện.

56 | Chuẩn bị kiến thức cho kì thi HSGQG Hóa học


D. Tất cả các nguyên tử, ngoại trừ một nguyên tử H, ở trên cùng
một mặt phẳng.
3) Xác định dạng hình học của phân tử methanal với công thức cấu
tạo thu gọn là H2C=O.
A. Phân tử thẳng. B. Tam giác phẳng.
C. Tháp tam giác. D. Tứ diện.
4) Xác định giá trị góc giữa hai liên kết C-H trong phân tử methanal.
A. 90o. B. 120o. C. 116o. D. 127o.
5) Trong Bảng tuần hoàn Nguyên tố, nitrogen (N) và phosphorus (P)
đều thuộc cột 15, với phosphorus ở ngay dưới nitrogen. Hãy xác
định góc giữa các liên kết trong phân tử ammonia NH3 và
phosphine PH3.
A. NH3: 109.47o; PH3: 109.47o B. NH3: 113; PH3: 119o
C. NH3: 113o; PH3: 111o D. NH3: 107o; PH3: 93o

57 | Chuẩn bị kiến thức cho kì thi HSGQG Hóa học


Hướng dẫn
1) Công thức Lewis của cyanamide:

- Nguyên tử carbon liên kết với hai nguyên tử khác, không có cấu
phần electron chưa liên kết. Công thức VSEPR của nó là AX2E0.
- Nguyên tử nitrogen của nhóm NH2 liên kết với ba nguyên tử khác,
và có một cấu phần electron chưa liên kết. Công thức VSEPR của
nó là AX3E1.
Chọn phương án A.
2) Phân tử cyanamide có hai nguyên tử trung tâm có thể áp dụng lí
thuyết VSEPR:
- Nguyên tử carbon, có công thức VSEPR là AX2E0 (xem ý 1), có nghĩa
hình học xung quanh nguyên tử này là dạng thẳng.
- Nguyên tử nitrogen của nhóm NH2, có công thức VSEPR là AX3E1
(xem ý 1), có nghĩa hình học xung quanh nguyên tử này là tháp tam
giác. Bởi hợp phần N≡C−N có dạng thẳng, nên dạng hình học toàn
phần của cấu trúc này chỉ đơn giản là hình chóp đáy tam giác. Có
nghĩa rằng toàn bộ các nguyên tử, ngoại trừ một nguyên tử
hydrogen, đều thuộc cùng một mặt phẳng.
Chọn các phương án B và D.
3) Công thức Lewis của formaldehyde:

Nguyên tử carbon liên kết với ba nguyên tử khác và không có cấu phần
electron chưa liên kết. Công thức VSEPR của nó là AX3E0.
Dạng hình học của phân tử methanal quanh nguyên tử carbon là tam
giác phẳng. Phân tử chỉ có một nguyên tử trung tâm nên bản thân toàn
cấu trúc này cũng là dạng tam giác.
Chọn phương án B.

58 | Chuẩn bị kiến thức cho kì thi HSGQG Hóa học


4) Như đã thấy ở ý trước, công thức VSEPR của nguyên tử carbon trong
phân tử methanal là AX3E0, cho thấy dạng hình học của toàn phân tử là
tam giác phẳng – với góc liên kết lí thuyết giữa các góc A-X là 120o.
Hãy dự đoán góc liên kết thực tế thì sao. Trong trường hợp methanal
thì n = 0 (carbon không có cấu phần electron chưa liên kết) nhưng ba
nguyên tử liên kết với carbon không giống nhau. Do mật độ electron
lớn, liên kết đôi C=O sẽ đẩy các liên kết đơn C-H sao cho góc  giữa các
liên kết C=O và C-H mở ra một chút (tức  > 120o), trong khi đó góc 
giữa hai liên kết C-H hẹp lại một chút (tức  < 120o), như minh họa trong
hình dưới đây:

Ngoài ra, oxygen có độ âm điện lớn hơn carbon, nên nó sẽ hút 4e tạo
liên kết đôi C=O về phía mình. Đồng thời, các nguyên tử hydrogen có
độ âm điện nhỏ hơn carbon sẽ đẩy các electron tạo liên kết đơn C-H về
phía carbon. Sự kết hợp cả hai hiệu ứng này khiến cho góc  mở rộng
một chút. Tuy nhiên, tác động này vẫn nhỏ hơn các yếu tố kể trên nên
phương án duy nhất chấp nhận được (với  <120o) là 116o (phương án C).
5) Công thức Lewis của ammonia và phosphine tương đồng với nhau:

Các nguyên tử nitrogen và phosphorus đều liên kết với ba nguyên tử,
và đều có một cấu phần electron chưa liên kết. Công thức VSEPR của
chúng đều là AX3E1. Dạng hình học của các phân tử ammonia và
phosphine đều là hình chóp (tháp) đáy tam giác.
Hãy tiến hành quy trình phân tích như đề bài đã cho.

59 | Chuẩn bị kiến thức cho kì thi HSGQG Hóa học


(1) n = 1, do đó góc lí thuyết tiêu chuẩn (109.5o) sẽ bị biến đổi. Do cặp
electron chưa liên kết sẽ gần với hạt nhân hơn cặp electron tạo
liên kết, nên nó sẽ đẩy ba liên kết đơn N-H (hoặc P-H) xích lại gần
nhau (kiểu như một chiếc ô đang đóng lại).
(2) Ba liên kết đơn N-H (hoặc P-H) giống nhau nên yếu tố này không
gây ảnh hưởng gì đến góc liên kết.
(3) Nitrogen và phosphorus đều âm điện hơn hydrogen nên sẽ hút các
electron của ba liên kết đơn N-H (hoặc P-H) về trung tâm, làm các
cặp electron liên kết dễ đẩy nhau hơn, gây ra sự tăng nhẹ góc giữa
các liên kết này. Hiệu ứng “mở góc” này ngược chiều với hiệu ứng
“đóng góc” gây ra bởi yếu tố (1), nhưng tác động của nó nhỏ hơn. Mặt
khác, do nitrogen âm điện hơn phosphorus nên các góc giữa các
liên kết N-H sẽ lớn hơn góc giữa các liên kết P-H.

Tổng kết lại, ta dự đoán được: PH  NH  109.5o


3 3

Phương án được chấp nhận là: PH = 93o và NH = 107o (D).
3 3

60 | Chuẩn bị kiến thức cho kì thi HSGQG Hóa học


Đọc thêm: Thuyết VSEPR
Lực đẩy giữa các cặp electron trong lớp vỏ hóa trị, hay VSEPR4, đóng
góp vào hình dạng không gian, dạng hình học của nguyên tử trung tâm.
Dạng hình học của nguyên tử trung tâm
- Sự sắp xếp của các cặp electron electron (liên kết và không liên
kết) theo nguyên tắc nhằm đảm bảo lực đẩy tương hỗ (lẫn nhau) tối
thiểu giữa chúng.
- Mỗi liên kết (đơn hoặc bội) và các cấu phần electron chưa liên kết
(cặp hoặc electron riêng lẻ) tạo ra một “vùng electron”. Số lượng
vùng electron (còn gọi là “số lập thể”) quanh mỗi nguyên tử tạo ra
dạng hình học lí tưởng (xem Bảng và Hình dưới).
Số lập thể và kiểu lai hóa/dạng hình học
Số lập thể Góc liên kết Kiểu lai hóa Dạng hình học
2 180o sp thẳng
3 120o sp2 tam giác
4 109.5o sp3 tứ diện
4 90o dsp2 vuông phẳng
5 90o, 120o dsp3 lưỡng tháp tam giác
6 90o d2sp3 bát diện
7 90o, 108o d3sp3 lưỡng tháp ngũ giác

4
valence shell electron pair repulsion (lực đẩy cặp electron vỏ hóa trị).

61 | Chuẩn bị kiến thức cho kì thi HSGQG Hóa học


Hình dạng không gian

lưỡng tháp
thẳng tam giác tứ diện bát diện
tam giác
90, 120 và
180 120 109.5 90 và 180
180

- Đôi khi, xảy ra sự biến dạng của dạng hình học lí tưởng, bởi không
phải tất cả các vùng electron đều chiếm không gian như nhau. Cụ
thể: cặp electron chưa liên kết > liên kết bậc 3 > liên kết bậc 2 > liên
kết đơn
- “Những đám mây electron dày đặc nhất trú ngụ ở những nơi rộng
rãi nhất” – có nghĩa là cặp electron chưa liên kết hoặc các liên kết
bội thường sẽ nằm ở vị trí xích đạo (vị trí biên). Các nhóm thế âm
điện (với các đám mây electron “mỏng”) có xu thế chiếm các vị trí
trục. Trong môi trường bát diện, hai cặp electron chưa liên kết
thường tạo thành định hướng dạng trans.
- Các phối tử có độ âm điện lớn làm thu hẹp các đám mây electron
của nguyên tử trung tâm, làm giảm lực đẩy tương hỗ giữa chúng và
do đó làm giảm độ lớn góc liên kết: góc F-N-F trong NF3 < góc H-N-
H trong NH3.
- Nguyên tử trung tâm càng lớn thì lực đẩy tương hỗ càng nhỏ: góc
H-P-H trong PH3 < góc H-N-H trong NH3.
- Khi đánh giá dạng hình học của một phân tử, chúng ta chỉ nhìn vào
dạng hình học của các cặp electron liên kết.

62 | Chuẩn bị kiến thức cho kì thi HSGQG Hóa học


Chủ đề 4: Số oxid hóa
1)
A. Số oxid hóa của nguyên tố trong phân tử bằng 0.
B. Số oxid hóa của nguyên tố chlorine trong phân tử Cl2 bằng 0
C. Số oxid hóa của một nguyên tố trong ion đơn nguyên tử bằng
giá trị tuyệt đối của điện tích.
D. Khi một nguyên tố bị oxid hóa thì số oxid hóa của nó giảm.
2) Ion thiosulfate S2O32- có công thức Lewis như sau:

Biết oxygen có độ âm điện lớn hơn lưu huỳnh.


A.
Hai nguyên tử oxygen tạo liên kết đơn có số oxid hóa là -1.
B.
Ba nguyên tử oxygen đều có số oxid hóa -2.
C.
Hai nguyên tử lưu huỳnh đều có số oxid hóa +2.
D.
Nguyên tử lưu huỳnh trung tâm có số oxid hóa +4, còn nguyên
tử ngoại biên có số oxid hóa bằng 0.
3) Hai tiểu phân Mn2+ và MnO4-
A. tạo cặp acid/base, trong đó Mn2+ là acid.
B. tạo cặp acid/base, trong đó Mn2+ là base.
C. tạo cặp oxid hóa-khử, trong đó Mn2+ là chất khử.
D. tạo cặp oxid hóa-khử, trong đó Mn2+ là chất oxid hóa.
4) Hai tiểu phân Cr2O72- và CrO42-
A. tạo cặp acid/base, trong đó Cr2O72- là acid.
B. tạo cặp acid/base, trong đó Cr2O72- là base.
C. tạo cặp oxid hóa-khử, trong đó Cr2O72- là chất khử.
D. tạo cặp oxid hóa-khử, trong đó Cr2O72- là chất oxid hóa.
5) Để cân bằng phản ứng oxid hóa-khử trong môi trường base sau
đây: MnO4− + I− MnO24− + IO3− thì phương trình
A. phải có H2O trong vai trò chất khử.
B. phải có H3O+ trong vai trò chất oxid hóa.
C. phải có H2O ở dạng sản phẩm.
D. phải có OH- trong vai trò chất khử.
6) Trong phương trình oxid hóa-khử đã cân bằng sau đây:

63 | Chuẩn bị kiến thức cho kì thi HSGQG Hóa học


IO3− + 5I− + 6H3O+ 3I2 + 9H2O

A. Chỉ iodine thay đổi số oxid hóa.


B. H2O đóng vai trò chất khử.
C. Có sự trao đổi 5 electron.
D. Có sự trao đổi 6 electron.

64 | Chuẩn bị kiến thức cho kì thi HSGQG Hóa học


Hướng dẫn
1) Tổng đại số của các số oxid hóa của toàn bộ các nguyên tố tạo thành
phân tử bằng 0, nhưng không có nghĩa là số oxid hóa của các nguyên
tố thành phần bằng 0. Phương án A sai.
Phân tử Cl2 là phân tử lưỡng nguyên tử đồng nhân – được tạo thành từ
các nguyên tử giống nhau – nghĩa là cùng độ âm điện và chia đôi cặp
electron tạo liên kết theo tỉ lệ 50%-50%. Mỗi nguyên tử chlorine trong
phân tử được “gán” với 7 electron – cũng giống như trong giản đồ Lewis
nguyên tử của nó, vậy điện tích hình thức của mỗi nguyên tử bằng 0.
Phương án B đúng.
Trong trường hợp ion đơn nguyên tử thì số oxid hóa của nguyên tố
thành phần bằng trị số điện tích của ion – nghĩa là có giá trị âm hoặc
dương và không phải giá trị tuyệt đối. Vậy phương án C sai.
Cuối cùng, khi một nguyên tố bị oxid hóa, nó sẽ nhường 1 hoặc nhiều
electron cho môi trường bên ngoài. Do đó, số oxid hóa của nó sẽ tăng
lên và phương án D sai.
2) Phương án B và D đúng.
3) Trong các tiểu phân Mn2+ và MnO4-, số oxid hóa của manganese khác
nhau, do đó Mn2+ và MnO4- tạo thành một cặp oxid hóa-khử, trong đó
tiểu phân có manganese với số oxid hóa cao hơn sẽ là chất oxid hóa.
Vậy MnO4- là chất oxid hóa, còn Mn2+ là chất khử.
Chọn phương án C.
4) Trong Cr2O72- và CrO42- không có sự khác biệt về số oxid hóa của
chromium (đều là +6). Vậy nên Cr2O72- và CrO42- không tạo thành cặp
oxid hóa-khử. Phương án C và D đều sai.
Phương trình cân bằng của phản ứng liên quan đến hai tiểu phân này
là: Cr2O27 − + 3H2O 2CrO24− + 2H3O+ - cân bằng này cho thấy trong môi
nước, ion Cr2O72- chuyển thành CrO42- và tạo ra H3O+; vậy Cr2O72- và CrO42-
tạo thành một cặp acid-base liên hợp, trong đó Cr2O72- đóng vai trò
acid. Chọn phương án A.
5) Chỉ ra sự thay đổi số oxid hóa của các nguyên tố

65 | Chuẩn bị kiến thức cho kì thi HSGQG Hóa học


+7 −1 +6 +5
MnO4− + I− Mn O24− + I O3−

Thiết lập hệ số cân bằng:


+7 +6
Mn+ 1e → Mn 6
−1 +5
1
I → I + 6e

6MnO4− + I− 6MnO24− + IO3−

Phương trình trên cho thấy sự thiếu hụt điện tích âm ở vế trái (7-) so với
vế phải (13-). Để cân bằng điện tích, sẽ cần bổ sung thêm 13 – 7 = 6 ion
hydroxide OH- vào vế trái:

6MnO4− + I− + 6OH− 6MnO24− + IO3−

Do đó, sẽ cần bổ sung thêm H2O ở vế phải (sản phẩm) để đảm bảo cho
sự bảo toàn nguyên tố hydrogen:

6MnO4− + I− + 6OH− 6MnO24− + IO3− + 3H2O

Phương trình phản ứng đã được cân bằng.


6) Phương án A và C.

66 | Chuẩn bị kiến thức cho kì thi HSGQG Hóa học


Chương 3: Tinh thể học
Chủ đề 1: Mạng tinh thể bạc
1) Dưới đây là mạng cấu trúc tinh
thể của kim loại bạc, có khối
lượng mol nguyên tử là M =
107.9 g·mol-1. Thông số mạng
a = 0.409 nm.
A. Bạc kết tinh theo hệ lập
phương đơn giản.
B. Bạc kết tinh theo hệ lục
phương.
C. Bạc kết tinh theo hệ lập
phương tâm khối.
D. Bạc kết tinh theo hệ lập phương tâm diện.
2) Một trục đối xứng của mạng tinh thể được xem là có bậc n nếu sự
2
quay quanh trục này một góc bằng thì tinh thể sẽ lập lại chính
n
nó. Theo đó:
A. Các đường chéo dài của khối lập phương là trục đối xứng bậc 3.
B. Các đường chéo dài của khối lập phương là trục đối xứng bậc 2.
C. Trục nối tâm các mặt đối diện có bậc đối xứng bằng 4.
D. Trục nối các trung điểm của các cạnh song song và đối nhau có
bậc đối xứng bằng 4.
3) Mật độ cư trú (Z) trong mỗi ô mạng là
A. Z = 4. B. Z = 5. C. Z = 7. D. Z = 14.
4) Số phối trí của nguyên tử bạc là
A. 4. B. 6. C. 8. D. 12.
5) Khối lượng riêng của kim loại bạc là

67 | Chuẩn bị kiến thức cho kì thi HSGQG Hóa học


A.  = 1830 kg m−3 B.  = 3670 kg m−3

C.  = 10500 kg  m−3 D.  = 13100 kg m−3

6) Xem các nguyên tử bạc là khối cầu bán kính R. Hãy xác định mối
liên hệ giữa thông số mạng tinh thể a với R:

A. a = ( )
3 −1 R B. a = 2 2R

3
C. a = 2R D. a = R
2
7) Độ đặc khí (c) của tinh thể bạc là
A. c = 0.29 . B. c = 0.68.
C. c = 0.74. D. c = 0.82.

68 | Chuẩn bị kiến thức cho kì thi HSGQG Hóa học


Hướng dẫn
1) Mạng lập phương có các nguyên tử bạc chiếm:
- 8 đỉnh.
- các tâm của 6 mặt.
Nên đây là hệ lập phương tâm diện. Chọn phương án D.
2) Xét đường chéo dài nối các điểm E và K của mạng tinh thể (xem hình
dưới). Sự quay 120o quanh trục EK không làm thay đổi E, K; và chuyển
- F thành H, H thành I, I thành F;
- G thành L, L thành J và J thành G.
Do đó, mạng tinh thể không thay đổi. Nguyên tắc này cũng được áp
dụng cho các sự quay góc 240o và 360o. Vậy n = 3.
Các đường chéo dài của khối lập phương là các trục đối xứng bậc 3, nên
câu A đúng, câu B sai.

Bây giờ, hãy xét trục nối tâm các mặt đối diện EFGH và IJKL. Sự quay
góc 90o quanh trục này chuyển:
- E thành F, F thành G, G thành H, H thành E.
- I thành J, J thành K, K thành L, L thành I.
Do đó, mạng tinh thể không thay đổi. Nguyên tắc này cũng được áp
dụng cho các sự quay góc 180o, 270o và 360o. Vậy n = 4.

69 | Chuẩn bị kiến thức cho kì thi HSGQG Hóa học


Các trục nối tâm các mặt đối diện là trục đối xứng bậc 4, nên câu C
đúng.

Cuối cùng, hãy xét trục nối trung điểm các cạnh song song và đối nhau
IJ và GH. Sự quay góc 180o quanh trục này chuyển:
- E thành K, F thành L.
- G thành H, H thành G.
- I thành J, J thành I.
- K thành E, L thành F.
Do đó, mạng tinh thể không thay đổi. Nguyên tắc này cũng được áp
dụng cho sự quay góc 360o. Vậy n = 2. Vậy các trục nối trung điểm các
cạnh song song và đối nhau có bậc đối xứng bằng 2. Vậy D sai.

70 | Chuẩn bị kiến thức cho kì thi HSGQG Hóa học


Chọn các phương án A và C.
3) Mật độ cư trú (Z) là số đơn vị thành phần (nguyên tử, ion hoặc phân
tử) có trong mỗi ô mạng đơn vị.
Mỗi nguyên tử bạc ở 8 đỉnh của mạng tinh thể được chia sẻ chung cho
8 mạng, và có thể xem rằng mỗi mạng có 1/8 nguyên tử. Do đó, tổng số
nguyên tử chiếm các vị trí đỉnh là:

1
8 = 1
8
Mỗi nguyên tử bạc đặt ở tâm của 6 mặt được chia sẻ chung cho 2 mạng,
và có thể xem rằng mỗi mạng có 1/2 nguyên tử. Do đó, tổng số nguyên
tử chiếm các vị trí tâm mặt là:
1
6 = 3
2
Vậy số nguyên tử trong mỗi ô mạng đơn vị là: Z = 1 + 3 = 4
4) Phối trí mẫu: Trong sự phối trí của một mẫu thì số phối trí (hay “chỉ
số phối trí”) là số lượng các mẫu lân cận ở khoảng cách ngắn nhất so
với mẫu được xét. Trong trường hợp này là “các nguyên tử bạc khác
xung quanh nguyên tử bạc được xét”.
Khoảng cách d ngắn nhất giữa hai nguyên tử bạc liền kề bằng ½ độ dài
của đường chéo mặt, tức là bằng:

71 | Chuẩn bị kiến thức cho kì thi HSGQG Hóa học


1 2 2 a
d= a +a =
2 2
Xét một nguyên tử bạc bất kì, ở đây ta chọn nguyên tử ở vị trí tâm mặt
(chia sẻ chung giữa hai ô mạng đơn vị) – kí hiệu bởi hình tròn màu đen.
Các nguyên tử lân cận (với khoảng cách gần nhất) được kí hiệu bởi các
hình tròn màu trắng.

Theo hình minh họa ở trên, nguyên tử bạc này (thuộc hai ô mạng đơn
vị) được xem là có 12 nguyên tử khác (màu trắng) xung quanh. Vậy số
phối trí của bạc là 12 (phương án D).
5) Khối lượng riêng của bạc được tính toán dựa trên lập luận theo một
ô mạng đơn vị:
Z M
=
NA  V

Trong đó:
- Z là số nguyên tử trong ô mạng đơn vị (đã tính được ở ý 3).
- M là khối lượng mol nguyên tử của bạc, biểu diễn theo kg·mol-1.
- NA là 6.02·1023 mol-1 là số Avogadro.
- V = a3 là thể tích ô mạng đơn vị.
Thay các giá trị số vào, tính được:

72 | Chuẩn bị kiến thức cho kì thi HSGQG Hóa học


4  0.1079
= = 10500 kg m−3 (phương án C).
( )
3
6.02  1023  0.409  10−9

La masse atomique molaire doit impérativement être convertie en


unité standard du système international d’unités, c’est-à-dire en
kg.mol−1
6) Các nguyên tử bạc
được xem là các khối cầu
bán kính R. Như đã xác
định ở ý 4, khoảng cách d
ngắn nhất giữa hai
nguyên tử bạc lân cận
bằng ½ độ dài đường chéo
a
mặt: d =
2
Các khối cầu đặc khít, đặt
trên các đường chéo mặt
của khối lập phương, sẽ
tiếp xúc với nhau (như
mình họa ở hình bên). Do
đó: d = 2R  a = 2 2R
Chú ý rằng phương án C (a = 2R) sai bởi nó có nghĩa rằng có sự tiếp xúc
giữa hai khối cầu trên cùng một cạnh – tương ứng với hệ lập phương
đơn giản!
7) Độ đặc khít (c) của một tinh thể là giá trị phi thứ nguyên, được đo bởi
tỉ lệ chiếm không gian thực của các khối cầu. Tức là:
thể tích bị chiếm Z × (thể tích một khối cầu)
c= =
thể tích sẵn có thể tích ô mạng đơn vị
Một điều kiện cần thỏa mãn là: 0 < c < 1 hay 0% < c < 100%.

73 | Chuẩn bị kiến thức cho kì thi HSGQG Hóa học


4 
Z   R3 
3 16R3
Ta thiết lập được biểu thức: c =  3  =
a 3a3

Theo ý 6 thì: a = 2 2R - thế vào biểu thức trên, ta có:

16R3 
c= = = 0.74 (phương án C).
( )
3
3  2 2R 3 2

74 | Chuẩn bị kiến thức cho kì thi HSGQG Hóa học


Chủ đề 2: Barium titanate
1) Dưới đây là ô mạng đơn vị lập phương của barium titanate, một chất
rắn ion có công thức BaxTiyOz.

Xác định các giá trị x, y, z.


A. x = 4, y = 1 và z = 3. B. x = 4, y = 1 và z = 3.
C. x = 4, y = 1 và z = 3. D. x = 4, y = 1 và z = 3.
2) Trong chất rắn ion này, nguyên tố barium tồn tại ở dạng ion Ba2+,
còn nguyên tố titanium tồn tại ở dạng
A. ion Ti+. B. ion Ti2+. C. ion Ti3+. D. ion Ti4+.
3) Mỗi ion barium được bao quanh bởi
A. 4 ion titanium và 8 ion oxygen.
B. 8 ion titanium và 12 ion oxygen.
C. 8 ion titanium và 16 ion oxygen.
D. 8 ion titanium và 24 ion oxygen.
4) Mỗi ion titanium được bao quanh bởi
A. 4 ion barium và 3 ion oxygen.
B. 4 ion barium và 6 ion oxygen.
C. 8 ion barium và 3 ion oxygen.
D. 8 ion barium và 6 ion oxygen.

75 | Chuẩn bị kiến thức cho kì thi HSGQG Hóa học


5) Mỗi ion oxygen được bao quanh bởi
A. 4 ion barium và 4 ion titanium.
B. 6 ion barium và 4 ion titanium.
C. 4 ion barium và 2 ion titanium.
D. 6 ion barium và 6 ion titanium.

76 | Chuẩn bị kiến thức cho kì thi HSGQG Hóa học


Hướng dẫn
1) Mật độ cư trú (Z) là số đơn vị thành phần (nguyên tử, ion hoặc phân
tử) trong mỗi ô mạng đơn vị. Khi xét mỗi nguyên tử hoặc ion như một
khối cầu thì sự đóng góp (chia sẻ) của chúng vào mỗi ô mạng là:
- 1 nếu nó nằm hoàn toàn trong ô mạng đơn vị (đặc biệt là ở vị
trí trung tâm);
1
- nếu nó nằm trên một mặt (đặc biệt là ở tâm mặt);
2
1
- nếu nó nằm trên một cạnh (đặc biệt là ở trung điểm);
4
1
- nếu nó nằm ở một đỉnh.
8
Hãy xác định mật độ cư trú (số phân tử barium titanate) của mỗi ô mạng
đơn vị:
▪ Có 8 ion barium được đặt ở 8 đỉnh của ô mạng, tương ứng với số
1
ion barium thuộc về mỗi ô mạng là: 8  = 1 ion.
8
▪ Có 1 ion titanium đặt ở tâm ô mạng, tương ứng với số ion
1
titanium thuộc về mỗi ô mạng là: 1 = 1 ion.
1
▪ Có 6 ion oxygen đặt ở 6 mặt của ô mạng, tương ứng với số ion
1
oxygen thuộc về mỗi ô mạng là: 6  = 3 ion.
2
Vậy mỗi ô mạng có 1 ion Ba, 1 ion Ti và 3 ion oxygen. Công thức hóa học
của barium titanate tương ứng với các giá trị nguyên tối giản của x, y,
z là BaTiO3 (x = 1, y = 1, z = 3). Chọn phương án B.
2) Nguyên tố barium và oxygen thường sẽ tồn tại ở các trạng thái quen
thuộc lần lượt là Ba2+ và O2-. Xét trạng thái oxid hóa của titanium trong
BaTiO3 là x, ta có: 1 ( +2 ) + 1 x + 3  ( −2 ) = 0  x = +4

Vậy titanium tồn tại ở dạng Ti4+ (phương án D).

77 | Chuẩn bị kiến thức cho kì thi HSGQG Hóa học


Theo thông tin trong bảng Tuần hoàn nguyên tố, titanium có 22 electron
– tương ứng với cấu hình electron 1s2 2s2 2p6 3s2 3p6 3d2 4s2 . Do đó, sẽ hợp
Ar

lí khi nguyên tử titanium mất đi 4 electron để tạo thành ion Ti4+ có cấu
hình electron bền của khí quý argon.
3) Trong hình dưới đây, với 8 ô mạng đơn vị liền kề nhau thì đỉnh chung
là ion Ba2+ (màu đen, khoanh tròn bên ngoài) có 8 ion Ti4+ (màu trắng)
liền kề.

78 | Chuẩn bị kiến thức cho kì thi HSGQG Hóa học


Trong hình dưới đây, ion Ba2+ được xét (màu đen, khoanh tròn bên
ngoài) có 12 ion O2- (màu xám, khoanh tròn bên ngoài) liền kề.

Vậy mỗi ion Ba2+ được bao xung quanh bởi 8 ion Ti4+ và 12 ion O2-
(phương án B).
4) Trong hình bên (biểu diễn một
ô mạng đơn vị) thì ion Ti4+ được xét
(màu trắng) có 8 ion Ba2+ (màu
đen) và 6 ion O2- (màu xám) liền kề.
Chọn phương án D.

79 | Chuẩn bị kiến thức cho kì thi HSGQG Hóa học


5) Trong hình dưới đây, với hai ô mạng đơn vị chung mặt, thì ion O2-
được xét (màu xám, khoanh tròn bên ngoài) có 4 ion Ba2+ (màu đen,
khoanh tròn bên ngoài) liền kề.

Tương tự, xung quanh ion O2- được xét có 2 ion Ti4+ liền kề (màu trắng).

Chọn phương án C.

80 | Chuẩn bị kiến thức cho kì thi HSGQG Hóa học


Chủ đề 3: Các hốc tinh thể
1) Nhôm, có khối lượng nguyên tử mol là M = 27 g·mol-1, kết tinh theo
hệ lập phương tâm diện. Khối lượng riêng của nhôm là
 = 2700 kg m−3 . Hãy tính độ dài cạnh a của ô mạng đơn vị.

A. a = 321 pm. B. a = 405 pm.


C. a = 436 pm. D. a = 615 pm.
2) Xác định giá trị bán kính nguyên tử R của nhôm:
A. R = 143 pm. B. R = 217 pm.
C. R = 218 pm. D. R = 556 pm.
3) Mỗi ô mạng đơn vị gồm:
A. 1 hốc bát diện và 4 hốc tứ diện.
B. 4 hốc bát diện và 4 hốc tứ diện.
C. 1 hốc bát diện và 8 hốc tứ diện.
D. 4 hốc bát diện và 8 hốc tứ diện.
4) Tỉ lệ bị chiếm của các hốc tứ diện là
A. 0%. B. 50%. C. 67%. D. 100%.
5) Biểu diễn bán kính rO của khối cầu lớn nhất có thể chứa vừa một hốc
bát diện mà không làm biến dạng cấu trúc tinh thể:

(
A. rO = 2 2 − 1 R ) B. rO =( 2 − 1)R
C. r = 2 (
O
2 − 1)R D. r = ( 2 − 2 )R
O

81 | Chuẩn bị kiến thức cho kì thi HSGQG Hóa học


6) Biểu diễn bán kính rT của khối cầu lớn nhất có thể chứa vừa một hốc
bát diện mà không làm biến dạng cấu trúc tinh thể:
 3   3
A. rT = 2  − 1 R B. rT =  1 − R
 2   2 
  

 3   3 
C. rT =  − 1 R D. rT =  + 1 R
 2   2 
   

82 | Chuẩn bị kiến thức cho kì thi HSGQG Hóa học


Hướng dẫn
1) Nhôm có cấu tạo mạng lập phương tâm diện (như bạc ở bài trước),
nên mật độ cư trú (tức số nguyên tử nhôm trong mỗi ô mạng đơn vị) là:
1 1
8  + 6  = 4 nguyên tử.
8 2
Khối lượng riêng của nhôm được tính toán dựa trên lập luận theo một
ô mạng đơn vị:
Z M
=
NA  V

Trong đó:
- Z là số nguyên tử trong ô mạng đơn vị.
- M là khối lượng mol nguyên tử, biểu diễn theo kg·mol-1.
- NA là 6.02·1023 mol-1 là số Avogadro.
- V = a3 là thể tích ô mạng đơn vị.

Z M Z M
= a= 3 . Thay các giá trị số vào, ta có:
NA  a3
NA p

4  27  10−3
a= 3 −23
= 4.05  10−10 m hay 405 pm (phương án B)
6.02  10  2700

2) Các nguyên tử nhôm được xem là những khối cầu có bán kính R.
Trong hệ lập phương tâm diện, khoảng cách d ngắn nhất giữa hai
nguyên tử nhôm bằng ½ độ dài đường chéo mặt:
1 2 2 a
d= a +a =
2 2
Những khối cầu liền kề, nằm trên đường chéo mặt của khối lập phương,
thì tiếp xúc với nhau như minh họa trong hình dưới đây:

83 | Chuẩn bị kiến thức cho kì thi HSGQG Hóa học


Do đó: d = 2R

a 405  10−12
Vậy: R = = = 1.43  10−10m hay 143 pm (phương án A)
2 2 2 2
3) Bất kì cấu trúc tinh thể được tạo thành bởi các quả cầu bán kính R
giống hệt nhau nào cũng cần phải có một phần không gian chưa bị
chiếm (độ đặc khít của nó nhỏ hơn 1): đó là các hốc tinh thể (đôi khi
được gọi là các “vị trí xen kẽ” hoặc “vị trí chèn”). Có hai kiểu hốc:
- Hốc bát diện: được bao xung quanh bởi 6 khối cầu đặt ở các đỉnh
của một bát diện (đa diện được tạo thành bởi 8 mặt tam giác).
Số phối trí của loại tâm này là 6;
- Hốc tứ diện: bị giới hạn bởi 4 khối cầu đặt ở các đỉnh của một tứ
diện (đa diện được tạo thành bởi 4 mặt tam giác). Số phối trí của
loại tâm này là 4.
Trong hình dưới đây, các hốc bát diện của tinh thể nhôm được biểu
diễn bởi hình màu đen:

84 | Chuẩn bị kiến thức cho kì thi HSGQG Hóa học


Chúng được đặt ở các vị trí:
- Có 1 hốc nằm ở tâm ô mạng đơn vị (thuộc hoàn toàn về ô mạng
này).
- Có 12 hốc ở trung điểm 12 cạnh, mỗi hốc chia sẻ chung cho 4 ô
1
mạng, tức là mỗi ô mạng có 12  = 3 hốc kiểu này.
4

Vậy mỗi ô mạng đơn vị có 1 + 3 = 4 hốc bát diện.


Tương tự, trong hình dưới đây, các hốc tức diện của tinh thể nhôm
được biểu diễn bởi hình màu đen:

a
Chúng nằm ở vị trí tâm của 8 khối lập phương nhỏ, với độ dài cạnh là
2
(các khối lập phương nhỏ này tạo thành ô mạng đơn vị). Các hốc này
thực sự là những tứ diện, bởi chỉ có 4 trong số 8 đỉnh của mỗi hình lập
phương nhỏ bị chiếm bởi các nguyên tử nhôm. Cả 8 hốc tứ diện này đều
thuộc ô mạng đơn vị. Chọn phương án D.

85 | Chuẩn bị kiến thức cho kì thi HSGQG Hóa học


4) Trong ô mạng tinh thể của nhôm, không có nguyên tử nào chiếm vị
trí hốc tứ diện. Do đó, tỉ lệ bị chiếm của loại hốc này bằng 0%. Chọn
phương án A.
5) Trong hình bên, khối
cầu lớn nhất có thể xếp
vừa vào hốc bát diện
nằm ở tâm của ô mạng
được biểu diễn bởi hình
màu đen. Sự tiếp xúc
giữa khối cầu bán kính
rO và 6 khối cầu bán kính
R, đại diện cho các
nguyên tử nhôm đặt ở
tâm của 6 mặt, biểu thị
cho điều kiện:
a
R + rO =
2

Ở ý 2, ta xác định được: a = 2 2R  rO = ( )


2 − 1 R (phương án B).

6) Trong hình dưới đây, ta biểu diễn hình màu đen là khối cầu lớn nhất
có thể xếp vừa vào hốc tứ diện ở vị trí tâm của một trong tám hình lập
a
phương nhỏ với cạnh bằng (tất cả đều thuộc ô mạng đơn vị).
2

86 | Chuẩn bị kiến thức cho kì thi HSGQG Hóa học


Sự tiếp xúc giữa khối cầu bán kính rT và 4 khối cầu bán kính R, đại diện
cho các nguyên tử nhôm chiếm 4 trong số 8 đỉnh của hình lập phương
nhỏ, biểu thị cho điều kiện:
1
R + rT =  (đường chéo khối của hình lập phương nhỏ)
2

1
R + rT =  (đường chéo khối của ô mạng)
4

1 3
R + rT =  a2 + a2 + a2 = a
4 4
Ở ý 2, ta xác định được:

3  3 
a = 2 2R  R + rT = R  rT =  − 1 R
2  2 
 
Chọn phương án C.

87 | Chuẩn bị kiến thức cho kì thi HSGQG Hóa học


Chủ đề 4: Ammonium chloride
1) Ammonium chloride NH4Cl là chất rắn ion, kết tinh theo cấu trúc
kiểu-cesium chloride (CsCl). Thông số mạng a = 387 pm. Hình nào
dưới đây biểu diễn cho ô mạng đơn vị của chất này:

A. Hình 1. B. Hình 2. C. Hình 3. D. Hình 4.


2) Mạng tinh thể này là kết quả của sự xếp chồng
a
A. hai mạng lập phương đơn giản được dịch chuyển dọc theo các
2
trục Ox, Oy và Oz.
a
B. hai mạng lập phương đơn giản được dịch chuyển dọc theo các
2
trục Ox và Oy.

88 | Chuẩn bị kiến thức cho kì thi HSGQG Hóa học


a
C. hai mạng lập phương tâm diện được dịch chuyển dọc theo các
2
trục Ox, Oy và Oz.
D. một mạng lập phương đơn giản và một mạng lập phương tâm
a
mặt, dịch chuyển theo các trục Ox và Oy.
2

3) Cho các giá trị khối lượng nguyên tử mol: M(H) = 1 g·mol-1; M(N) = 1.4
g·mol-1; M(Cl) = 35.5 g·mol-1. Hãy tính khối lượng riêng  của hợp chất
này.
−3 −3
A.  = 1230 kg m B.  = 1450 kg m
−3 −3
C.  = 1530 kg m D.  = 3060 kg m

4) Bán kính của ion chloride Cl- là RCl− = 187 pm. Hãy tính bán kính RNH +
4

của ion ammonium NH4+.


A. RNH = 87 pm
+
B. RNH = 148 pm
+
4 4

C. RNH = 173 pm
+
D. RNH = 296 pm
+
4 4

5) Xác định độ đặc khít (c) của tinh thể ammonium chloride.
A. c = 0.68. B. c = 0.71. C. c = 0.74. D. c = 0.85.

89 | Chuẩn bị kiến thức cho kì thi HSGQG Hóa học


Hướng dẫn
1) Cấu trúc tinh thể kiểu cesium chloride: CsCl là chất rắn ion, kết
tinh theo dạng mạng lập phương, trong đó
- các ion chloride Cl- chiếm 8 đỉnh mạng;
- các ion cesium Cs+ chiếm vị trí tâm mạng.
1 1
( ) ( )
Theo đó: Z Cl− = 8  = 1 và Z Cs+ = 1 = 1  Z (CsCl) = 1
8 1

Vậy hình 3 tương ứng với mạng tinh thể ammonium chloride (chọn
phương án C).
2) Hình minh họa dưới đây chỉ rõ rằng mạng tinh thể ammonium
chloride được tạo thành từ sự xếp chồng hai ô mạng lập phương đơn
a
giản được dịch chuyển dọc theo các trục Ox, Oy và Oz; ô mạng lập
2
phương đơn giản của các ion NH4+ được tịnh tiến theo vector
a a a
u =  ex +  ey +  ez so với ô mạng lập phương đơn giản của các ion
2 2 2
Cl_ (chọn phương án A).

3) Khối lượng riêng của nhôm được tính toán dựa trên lập luận theo một
ô mạng đơn vị:

90 | Chuẩn bị kiến thức cho kì thi HSGQG Hóa học


Z (NH4Cl) M(NH4Cl )
=
NA  V

Trong đó:
- Z(NH4Cl) là số phân tử NH4Cl trong ô mạng đơn vị. Ta đã xác định
được Z(NH4Cl) = 1.
- M(NH4Cl) là phân tử khối mol, biểu diễn theo kg·mol-1:
M(NH4Cl) = 14 + 4  1 + 35.5 = 53.5 g mol−1 = 5.35  10−2 kg mol−1

- NA là 6.02·1023 mol-1 là số Avogadro.


- V = a3 là thể tích ô mạng đơn vị.

Z (NH4Cl) M(NH4Cl )
Theo đó:  =
NA  a3

Thay các giá trị số vào, tính được:

1 5.35  10−2 kg mol−1


= = 1530kg m−3 (phương án C).
( )
3
23 −1 −12
6.02  10 mol  387  10 m

4) Các ion Cl- được xem là những khối cầu với bán kính RCl− = 187 pm và
các ion NH4+ là những khối cầu với bán kính RNH . Như minh họa trong
+
4

hình dưới đây, các khối cầu ion Cl- và NH4+ tiếp xúc dọc theo đường
chéo khối của ô mạng đơn vị, trong khi đó hai ion Cl- trên cùng cạnh
không tiếp xúc với nhau do 2RCl− = 2  187 = 374 pm  a = 387 pm.

91 | Chuẩn bị kiến thức cho kì thi HSGQG Hóa học


Sự tiếp xúc ở trên biểu thị cho điều kiện:
1
RCl− + RNH+ =  (đường chéo khối của ô mạng đơn vị)
4 2

1 2 2 2 3 3
RCl− + RNH+ = a +a +a = a  RNH+ = a − RCl−
4 2 2 4 2

3
Thay các giá trị số vào, tính được: RNH+ =  387 − 187 = 148 pm
4 2
Chọn phương án B.
5) Thể tích bị chiếm được xác định bởi công thức: V =  Zi  Vi - trong
i

đó Zi và Vi lần lượt là số lượng vi hạt và thể tích bị chiếm bởi khối cầu
đại diện cho vi hạt i. Theo đó, độ đặc khít của tinh thể ammonium
chloride được tính như sau:
4  4 3 
( ) 
( 
)
Z Cl−   RCl3 −  + Z NH4+   RNH
3 3
+ 
4

c= 3
a
4
3a 
( ) ( )
c = 3  Z Cl− RCl3 − + Z NH4+ RNH
3 
4 
+

Thay các giá trị số vào tính được:

92 | Chuẩn bị kiến thức cho kì thi HSGQG Hóa học


4  3.14
c= 
3  ( ) ( )
1 187  10−12 3 + 1 148  10−12 3  = 0.71

(
3  387  10−12 ) 

Chọn phương án B.

93 | Chuẩn bị kiến thức cho kì thi HSGQG Hóa học


Chủ đề 5: Germanium
1) Germanium Ge, với khối lượng nguyên tử mol M = 72.6 g·mol-1, kết
tinh ở dạng cấu trúc kiểu kim cương. Thông số mạng a = 566 pm.
Hãy xác định số nguyên tử Ge trong mỗi ô mạng đơn vị.
A. Z = 4. B. Z = 5. C. Z = 8. D. Z = 9.
2) Số phối trí của germanium trong ô mạng đơn vị là
A. 4. B. 6. C. 8. D. 12.
3) Tính bán kính cộng hóa trị của germanium:
A. R = 283 pm. B. R = 245 pm.
C. R = 200 pm. D. R = 123 pm.
4) Tính khối lượng riêng của germanium:
−3 −3
A.  = 5990 kg  m B.  = 2660 kg m
−3 −3
C.  = 3330 kg  m D.  = 53200 kg m

5) Xác định độ đặc khít (c) của tinh thể germanium:


A. c = 0.37. B. c = 0.68. C. c = 0.74. D. c = 0.34.

94 | Chuẩn bị kiến thức cho kì thi HSGQG Hóa học


Hướng dẫn
1) Trong cấu trúc tinh thể kiểu-kim cương, mạng tinh thể có dạng hình
lập phương và các nguyên tử chiếm:
- 8 đỉnh của ô mạng;
- các tâm của 6 mặt;
a
- tâm của 4 trong số 8 hình vuông nhỏ với độ dài cạnh thuộc ô
2
mạng đơn vị.

Mật độ cư trú, tức số nguyên tử Ge trong mỗi ô mạng đơn vị, là


1 1 1
Z = 8  + 6  + 4  = 1 + 3 + 4 = 8 (phương án C).
8 2 1

2) Trong hình minh họa bên,


nguyên tử germanium ở tâm
một hình lập phương nhỏ (màu
đen) có 4 nguyên tử lân cận
(màu trắng), đặt ở 4 trong số 8
đỉnh của hình lập phương nhỏ.
Vậy số phối trí của germanium
là 4 (phương án A).

95 | Chuẩn bị kiến thức cho kì thi HSGQG Hóa học


3) Các nguyên tử germanium được xem là những khối cầu bán kính R.
Khoảng cách ngắn nhất d giữa hai nguyên tử germanium kề cận bằng
một nửa độ dài đường chéo khối của hình lập phương nhỏ:
2 2 2
1 a a a 3
d=   +  +  = a
2 2 2 2 4

Các khối cầu kề cận, đặt trên đường chéo khối của hình lập phương
nhỏ, tiếp xúc với nhau như minh họa trong hình dưới đây:

3 3
d = 2R  R = a=  566 = 123 pm (phương án D).
8 8
4) Phương án D:

Z M Z M 8  72.6  10−3
= = = = 5320 kg  m−3
(
NA  V NA  a3 6.02  1023  566  10−12
)
3

4 
Z   R3 
3 32R3 32R3 3
5) Độ đặc khít: c =  3  = = = = 0.34
a 3a3
 8R 
3
16
3 
 3

96 | Chuẩn bị kiến thức cho kì thi HSGQG Hóa học


Phụ lục b: Phức chất và ứng dụng
của thuyết trường tinh thể

Chủ đề 1: Đồng phân vô cơ


Thường thì hiện tượng đồng phân hay được liên hệ với hóa học hữu cơ,
nhưng thực tế trong hóa học vô cơ cũng rất nhiều các ví dụ về đồng
phân. Ở cuối thế kỉ 19, Alfred Werner, người sau này (1913) giành được
giải Nobel Hóa học, đã đặt nền móng cho hóa học về các phức chất và
đã nghiên cứu về cấu trúc đặc trưng của các hợp chất này. Các kiểu
đồng phân sau đây rất đặc trưng với các phức chất:
- Đồng phân lập thể: các đồng phân có cùng thành phần phân tử và
cấu trúc hóa học, nhưng khác nhau về cách sắp xếp các nguyên tử
trong không gian. Đồng phân lập thể có thể là đồng phân quang học
(đối quang) hoặc đồng phân hình học (xuyên lập thể phân, hay đồng
phân dia).
- Đồng phân cấu tạo: có cùng công thức phân tử nhưng khác về trật
tự liên kết nguyên tử.

Hình 1. Cấu trúc của các phức chất bát diện, vuông phẳng và tứ diện.
1) Vẽ ra tất cả các đồng phân lập thể của các phức chất vuông phẳng
platinum(II):
a) Pt(py)(NH3)Br2 và chỉ ra đồng phân cis/trans (trong đó py =
pyridine, C5H5N).
b) Pt(NH2CH2CH2OH)BrCl
2) Vẽ ra tất cả các đồng phân lập thể của các phức chất bát diện
cobalt(III):

97 | Chuẩn bị kiến thức cho kì thi HSGQG Hóa học


a) [Co(ox)2(H2O)2]- và chỉ ra đồng phân cis/trans (trong đó ox = ion
oxalate).
b) [Co(NH2CH2CH2OH)3]3+
NH2CH2CH2OH (ethanolamine) và ox đều là các phối tử hai càng.
Tính chất từ, màu sắc và nhiệt hydrate hóa của các phức chất có thể
được giải thích theo mô hình thuyết trường tinh thể. Theo thuyết
trường tinh thể, trong một hợp chất phối trí, các kim loại và phối tử có
tương tác tĩnh điện với nhau, nghĩa là cation kim loại dương điện hút
cặp electron âm điện của phối tử. Trong một nguyên tử kim loại tự do,
các orbital d có cùng mức năng lượng. Khi phối tử tiếp cận với ion kim
loại, các electron trong các orbital d của kim loại và các electron của
phối tử sẽ đẩy nhau. Kết quả là năng lượng của các orbitla d của kim
loại sẽ thay đổi và tạo thành một số kiểu đa diện. Các orbital nguyên tử
hướng trực tiếp về phía các phối tử sẽ nhận lực đẩy lớn hơn và có năng
lượng cao hơn so với các orbital d ở xa các phối tử.

Hình 2. Sự sắp xếp không gian của các orbital d.


3) Hãy điền các orbital d của kim loại vào các giản đồ trường tinh thể
tương ứng với các đa diện (bát diện và tứ diện).

98 | Chuẩn bị kiến thức cho kì thi HSGQG Hóa học


4) Tương tự, hãy vẽ giản đồ trường tinh thể của phức vuông phẳng.
Trong trường hợp phức bát diện, các orbital d bị chia thành hai đa diện,
eg và t2g. Năng lượng phân li của trường tinh thể, Δo, là năng lượng tách
mức các đa diện eg và t2g. Trong sự tạo thành các phức bát diện, năng
3
lượng của các orbital d thuộc nhóm eg tăng lên  , còn năng lượng
5 o
2
của các orbital thuộc nhóm t2g giảm  .
5 o

99 | Chuẩn bị kiến thức cho kì thi HSGQG Hóa học


Hình 3. Sự phân bố các orbital d trong các đa diện của phức bát diện.
Năng lượng bền hóa trường tinh thể (crystal field stabilization energy,
CFSE) mô tả năng lượng của các electron d của kim loại biến đổi thế
nào trong sự tạo thành một phức chất. Độ lớn của CFSE được xác định
bởi hai yếu tố: sự biến đổi năng lượng electron do sự phân li trường tinh
thể, Δo, và tiêu hao năng lượng cho sự hình thành cặp electron, P - năng
lượng tạo thành cặp electron.
5) Xác định cấu hình electron của Fe2+ trong các phức spin cao và spin
thấp.
6) Tính năng lượng bền hóa trường tinh thể (CFSE, theo đơn vị kJ mol-
1
) của hai phức bát diện Fe2+ là [Fe(CN)6]4− và [Fe(H2O)6]2+ với các
trường hợp phức spin cao và spin thấp. Và xác định cấu hình
electron d đặc trưng của các phức chất này. Cho biết rằng:
Δo([Fe(CN)6]4−) = 32800 cm-1, Δo([Fe(H2O)6]2+) = 10400 cm-1, năng
lượng tạo thành cặp electron của hai phức giống nhau: P = 229.1 kJ
mol-1.
Trong đa số trường hợp, dạng hình học của các phức bát diện không
tương ứng với hình bát diện đối xứng lí tưởng, mà sẽ bị biến dạng. Hiện
tượng này được giải thích bởi hiệu ứng Jahn-Teller, lí thuyết này phát
biểu rằng phân tử với các orbital có năng lượng bằng nhau có xu hướng
bị biến dạng hình học và do đó năng lượng của phân tử sẽ giảm khi cấu

100 | Chuẩn bị kiến thức cho kì thi HSGQG Hóa học


hình electron thay đổi. Trong hình 4, δ1 và δ2 là năng lượng tách mức
biến dạng trong các orbital của các nhóm eg và t2g.

Hình 4. Sự rút ngắn hoặc kéo dài liên kết theo trục z trong
phức bát diện.
7) Trong giản đồ trường tinh thể của sự rút ngắn hoặc kéo dài liên kết
theo trục z trong phức bát diện, hãy gán các orbital d của kim loại
với các đa diện tương ứng.
8) Dựa vào các giản đồ trường tinh thể và sử dụng các năng lượng tách
mức biến dạng, hãy xác định các phức bát diện nào sau đây có xu
hướng biến dạng và cho biết sự rút ngắn hoặc kéo dài liên kết theo
trục z xảy ra với các phức này:
a) [CrCl6]4- (spin cao).
b) [Mn(CN)6]4− (spin thấp).
c) [Mn(H2O)6]2+ (spin cao).

101 | Chuẩn bị kiến thức cho kì thi HSGQG Hóa học


Hướng dẫn
1)
a) b)

đồng phân cis đồng phân


trans
2) a)

đồng phân trans

đồng phân cis


b)

102 | Chuẩn bị kiến thức cho kì thi HSGQG Hóa học


3)

4)

5) Cấu hình Fe2+ trong phức


- spin cao: (t2g)4(eg)2
- spin thấp: (t2g)6

103 | Chuẩn bị kiến thức cho kì thi HSGQG Hóa học


6) CFSE (spin cao) = 4∙(-2/5Δo) + 2∙(+3/5Δo) + 1∙P
CFSE (spin thấp) = 6∙(-2/5Δo) + 3∙P

- [Fe(CN)6]4−
Δo(kJ mol-1) = NAhcΔo(cm-1) = (6.02214·1023 mol–1)∙( 6.62608·10–34 J s)∙(
2.99793·1010 cm s–1)∙( 32800 cm-1) = 392380 J mol-1 = 392.38 kJ mol-1
CFSE (spin cao) = 4∙(-2/5 ∙ 392.38 kJ mol-1) + 2∙(+3/5 ∙ 392.38 kJ mol-1)
+ 1 ∙ 229.1 kJ mol-1 = + 72.15 kJ mol-1
CFSE (spin thấp) = 6∙(-2/5 ∙ 392.38 kJ mol-1) + 3 ∙ 229.1 kJ mol-1 = - 254.4
kJ mol-1
- [Fe(H2O)6]2+
Δo(kJ mol-1) = NAhcΔo(cm-1) = (6.02214·1023 mol–1)∙(6.62608·10–34 J s)∙(
2.99793·1010 cm s–1)∙( 10400 cm-1) = 124410 J mol-1 = 124.41 kJ mol-1
KLSE (spin cao) = 4∙(-2/5 ∙ 124.41 kJ mol-1) + 2∙(+3/5 ∙ 124.41 kJ mol-1) +
1 ∙ 229.1 kJ mol-1 = + 179.3 kJ mol-1
KLSE (spin thấp) = 6∙(-2/5 ∙ 124.41 kJ mol-1) + 3 ∙ 229.1 kJ mol-1 = + 388.7
kJ mol-1
7)

8) a) [CrCl6]4- cấu hình spin cao: (t2g)3

104 | Chuẩn bị kiến thức cho kì thi HSGQG Hóa học


Năng lượng bền hóa rút ngắn liên kết:
ΔE = 1∙ (-2δ1/3) + 2∙(+δ1/3) + 1∙ (-δ2/2) = -δ2/2
Năng lượng bền hóa kéo dài liên kết:
ΔE = 2∙ (-δ1/3) + 1∙(+2δ1/3) + 1∙(-δ2/2) = -δ2/2
Phức chất [CrCl6]4- có xu hướng bị biến dạng. Cả hai xu hướng kéo dài
và rút ngắn liên kết đều bằng nhau về mặt năng lượng.
b) [Mn(CN)6]4− cấu hình spin thấp: (t2g)5
Năng lượng bền hóa rút ngắn liên kết:
ΔE = 2∙(-2δ1/3) + 3∙(+δ1/3) = -δ1/3
Năng lượng bền hóa kéo dài liên kết:
ΔE = 4∙(-δ1/3) + 1∙(+2δ1/3) = -2δ2/3
Phức chất [Mn(CN)6]4− có xu hướng bị biến dạng. Xu hướng kéo dài liên
kết thuận lợi hơn về mặt năng lượng.
c) [Mn(H2O)6]2+ cấu hình spin cao: (t2g)3(eg)2
Năng lượng bền hóa rút ngắn liên kết:
ΔE = 1∙(-2δ1/3) + 2∙(+δ1/3) + 1∙ (-δ2/2) + 1∙ (+δ2/2) = 0
Năng lượng bền hóa kéo dài liên kết:
ΔE = 2∙(-δ1/3) + 1∙(+2δ1/3) + 1∙(-δ2/2) + 1∙(+δ2/2) = 0
Phức chất [Mn(H2O)6]2+ không có xu hướng biến dạng bởi năng lượng
của các electron d không giảm xuống.

105 | Chuẩn bị kiến thức cho kì thi HSGQG Hóa học


Chủ đề 1: Thuyết trường tinh thể
Một số tính chất của các hợp chất phức được giải thích rất tốt nhờ lí
thuyết trường tinh thể. Cả năm orbital 3d trên một nguyên tử không có
tương tác (hình 1) có năng lượng bằng nhau. Trong các hợp chất phức,
năng lượng của các orbital này bị thay đổi. Các phối tử tiếp cận với ion
kim loại đẩy các electron của ion kim loại (bởi các electron của chúng),
và tùy thuộc vào sự sắp xếp không gian mà sự thay đổi năng lượng của
các orbital d khác nhau. Các orbital nguyên tử hướng trực tiếp về các
phối tử chịu ảnh hưởng mạnh hơn và năng lượng của chúng trở nên lớn
hơn các orbital ở xa phối tử. Từ các orbital 3d, có bảy đa diện với năng
lượng khác nhau được tạo thành. Chênh lệch năng lượng giữa các đa
diện được gọi là năng lượng phân li của trường tinh thể và kí hiệu là Δ.

3dz2 3dx2– y2 3dxz 3dxy 3dyz


Hình 1. Hình dạng các orbital 3d.
1) Trong các giản đồ bên dưới, hãy chỉ ra orbital nguyên tử 3d nào đã
tạo thành các đa diện nếu các phối tử được sắp xếp để tạo thành:
a) bát diện
b) hình vuông
c) tứ diện

106 | Chuẩn bị kiến thức cho kì thi HSGQG Hóa học


Hình 2. Sự sắp xếp của các phối tử trong một ion phức bát diện, vuông
phẳng và tứ diện.
Khi phức chất được chiếu sáng, các electron trong các orbital năng
lượng thấp hơn có thể hấp thụ năng lượng quang tử (photon) và nhảy
lên các orbital năng lượng cao hơn. Năng lượng tách mức trường tinh
thể (Δo) của nhiều phức bát diện thuộc vùng ánh sáng khả kiến, nên các
hợp chất này có màu.
2) Tính bước sóng của ánh sáng được hấp thụ bởi [Co(NH3)5(H2O)]2+ và
chỉ ra màu nào bị hấp thụ và phức chất này có màu nào trong ánh
sáng ban ngày thông thường. Biết phức chất này có Δo = 4.04·10–19
J. Bước sóng của ánh sáng tím-xanh lam là 380-495 nm, xanh lá
495 - 570 nm, vàng 570 - 590 nm, da cam 590 - 620 nm, đỏ 620 -
750 nm.
3) Viết cấu hình electron của Pt2+ và Pt4+. Trong quá trình oxid hóa,
trước tiên platinum mất các electron ở orbital 4s.
4) Điền vào giản đồ năng lượng orbital của Pt2+ và Pt4+ khi các ion này
này tạo thành các phức bát diện, vuông phẳng và tứ diện. Xét các
phức spin thấp. Chỉ các electron hóa trị của kim loại được sử dụng
đến.
5) Cho biết dạng hình học thuận lợi nhất của phức chất được tạo thành
với: a) Pt(II); b) Pt(IV).
Xét sơ đồ chuyển hóa dưới đây:

107 | Chuẩn bị kiến thức cho kì thi HSGQG Hóa học


6) Platinum và vàng cực kì trơ, nhưng thậm chí là các kim loại này cũng
có thể được hòa tan trong nước cường toan, là hỗn hợp dung dịch
đặc của các chất A và B. Viết phương trình phản ứng cân bằng biểu
diễn phản ứng của platinum với hỗn hợp các chất A và B.
Số oxid hóa của platinum chỉ thay đổi trong các giai đoạn 1 và 2. Trong
hợp chất F, tỉ lệ mol của platinum và iodine là 1:1. B và E là các chất khí.
Trong sơ đồ, kí hiệu 2 eq (equivalent, hay đương lượng) có nghĩa là 2
mol NH3 phản ứng với 1 mol H.
7) Xác định công thức các chất D-I.
Các chất J’ và J’’ là đồng phân. J’ là một trong các hợp chất platinum
đầu tiên được sử dụng trong điều trị ung thư. Khi đi vào cơ thể, nó liên
kết với DNA và kích hoạt sự chết tế bào được lập trình (programmed
cell death – PCD). Còn J’’ thì không có hiệu ứng như vậy. Hiệu ứng động
học trans (kinetic trans effect) có vai trò rất quan trọng với sự tạo
thành các hợp chất J’ và J’’. Biết rằng các phản ứng thế trans-Kim loại-
Cl diễn ra nhanh hơn trans-Kim loại-NH3.
8) Vẽ cấu trúc J’ và J’’, có xét đến số oxid hóa của platinum và dạng
hình học thuận lợi nhất.

108 | Chuẩn bị kiến thức cho kì thi HSGQG Hóa học


Cấu trúc của D trông giống như một chuỗi polymer dài. F tạo thành
tetramer dạng hình lập phương chứa bốn nguyên tử platinum. G có thể
tạo thành chuỗi polymer hoặc một hexamer với sáu nguyên tử
platnium, cũng có hình lập phương. Chú ý đến số oxid hóa của platinum
trong các hợp chất này và dạng hình học thuận lợi nhất.
9) Vẽ bốn cấu trúc được kể đến ở trên.
Hướng dẫn
1)

2) E = hc / λ
Cụ thể, trong trường hợp này: Δo = hc / λ;
λ = hc / Δo
λ = 6.626∙10-34 J∙s ∙ 2.998∙108 m∙s-1 / 4.04·10-19 J
λ = 492nm
Phức hấp thụ ánh sáng màu xanh lá, xuất hiện màu đỏ tươi.
3) Pt2+ [Xe]4f145d8 ; Pt4+ [Xe]4f145d6
4)

109 | Chuẩn bị kiến thức cho kì thi HSGQG Hóa học


5) Pt (II) vuông phẳng; Pt (IV) bát diện. Các dạng hình học được chọn
có năng lượng thấp nhất theo các giản đồ orbital.
6) Pt + 4 HNO3 + 6HCl → H2PtCl6 + 4 NO2 + 4 H2O
A B C
7)
D E F
PtCl4 Cl2 Pt(CH3)3I
G H I
PtCl2 K2PtCl4 [Pt(NH3)4]Cl2
8)
J‘ J‘‘

110 | Chuẩn bị kiến thức cho kì thi HSGQG Hóa học


9)
D F

G lập phương G chuỗi

111 | Chuẩn bị kiến thức cho kì thi HSGQG Hóa học


Chủ đề 3: Tính chất từ
Theo thuyết trường tinh thể khi các phức chất được tạo thành thì các
orbital d bị tách mức năng lượng. Sự tạo thành các phức spin cao hay
spin thấp sẽ được quyết định bởi kiểu phối tử và kim loại trung tâm.
a) Trong trường phối tử bát diện, cấu hình electron nào có thể tạo
thành các phức spin cao và spin thấp? Giải thích ngắn gọn.
b) Thêm các electron d vào giản đồ orbital ở trang bên. Dự đoán tính
chất từ (thuận từ hay nghịch từ?).
Với các phức kim loại chuyển tiếp dãy đầu, thuận từ thì moment từ μ lí
thuyết (μtheo) - tính theo đơn vị Bohr magneton (BM) - được ước lượng
hiệu quả theo công thức spin sau:  theo = n  (n + 2) BM với n là số
electron chưa ghép đôi.
c) Tính giá trị moment từ spin với các nguyên tử kim loại có lần lượt 1,
2, 3, 4 và 5 electron chưa ghép đôi.

112 | Chuẩn bị kiến thức cho kì thi HSGQG Hóa học


Fe3+ Trường bát diện Trường tứ diện

Spin cao

Tính chất từ

Spin thấp

Tính chất từ
Co3+ Trường bát diện Trường tứ diện

Spin cao

Tính chất từ

Spin thấp

Tính chất từ
Mn3+ Trường bát diện Trường tứ diện

Spin cao

Tính chất từ

Spin thấp

Tính chất từ

113 | Chuẩn bị kiến thức cho kì thi HSGQG Hóa học


Bằng cách so sánh các giá trị moment từ lí thuyết (μtheo) với giá trị nhận
được từ dữ kiện thực nghiệm (μexp), bạn có thể xác định liệu một hợp
chất là phức spin cao hay thấp. μexp được tính theo một công thức phức
tạp mà trong trường hợp này có thể được đơn giản hóa như sau:

exp = 2.83  C(cm3 K mol−1) BM với C là hằng số Curie

C
= với T: nhiệt độ χ: độ cảm từ
T

Độ cảm từ của phức manganese(II) bát diện được đo ở các nhiệt độ


khác nhau như sau:

d) Xác định hằng số Curie (với hai chữ số thập phân) bằng cách vẽ đồ
thị 1/χ với T.
e) Tính μexp.
f) Phức manganese(II) trên là phức spin cao hay thấp? Giải thích.

114 | Chuẩn bị kiến thức cho kì thi HSGQG Hóa học


Hướng dẫn
a) Các phức spin cao và thấp chỉ tồn tại trong các phức có 4 đến 7
electron d (tức là d4 đến d7). Chỉ trong các trường hợp này thì mới có
các cách điền electron khác nhau vào các orbital phân tử.
b)
Fe3+ Trường bát diện Trường tứ diện

Spin cao

Tính chất từ thuận từ thuận từ

Spin thấp

Tính chất từ thuận từ thuận từ


Co3+ Trường bát diện Trường tứ diện

Spin cao

Tính chất từ thuận từ thuận từ

Spin thấp

Tính chất từ nghịch từ thuận từ


Mn3+ Trường bát diện Trường tứ diện

Spin cao

115 | Chuẩn bị kiến thức cho kì thi HSGQG Hóa học


Tính chất từ thuận từ thuận từ

Spin thấp

Tính chất từ thuận từ nghịch từ


c)
Số electron chưa ghép đôi 1 2 3 4 5
Moment từ μtheo (theo BM) 1.73 2.83 3.87 4.90 5.92
1 1
d) Vẽ đồ thị = T
thuan C

T, K 2 10 20 30 60 90 120 220 270 300

1
,

0.55 2.26 4.24 6.58 13.16 19.61 26.32 47.62 58.82 66.67
cm-3

mol

116 | Chuẩn bị kiến thức cho kì thi HSGQG Hóa học


1
Độ dốc: = 0.22 cm-3 mol  C = 4.55 cm3 K mol-1
C
e)

exp = 2.83  4.55= 6.04 BM

f) Nó phải là phức spin cao với 5 electron chưa ghép cặp với giá trị exp
và theo khá gần nhau.

117 | Chuẩn bị kiến thức cho kì thi HSGQG Hóa học


Chủ đề 4: Các ion kim loại trong phức chất
Khi điền electron vào các orbital thì orbital có mức năng lượng thấp
nhất sẵn có sẽ được điền đầu tiên, sau đó theo chiều tăng dần mức
năng lượng. Do đó, cần xét đến quy tắc Hund cũng như nguyên lí Pauli.
Có một số ngoại lệ với các quy tắc này.
a) Dẫn ra hai ví dụ về các nguyên tố với cấu hình electron (ở trạng thái
cơ bản) khác với cấu hình dự đoán thông thường.
Nguyên nhân của sự sai lệch so với dự đoán nay là bởi độ bền của một
số cấu hình electron cụ thể.
b) Những cấu hình nào đặc biệt thuận lợi về mặt năng lượng?
Các cấu hình thuận lợi đóng vai trò then chốt trong cấu hình của các
cation kim loại.
c) Viết cấu hình electron của các cation kim loại sau.
i) Fe3+ ii) Mn3+ iii) Pd4+ iv) Cr3+ v) Fe2+ vi) Pb2+ vii) Au3+ viii) Co2+ ix) Cu+ x) Ti2+
Các ion riêng lẻ được cho ở ý c chỉ tồn tại hình thức. Trong thực tế, ví
dụ trong các chất rắn hoặc các hợp chất phức, thì chúng luôn có một
cầu phối trí và được bao xung quanh bởi các vi hạt khác, tạo thành một
đa diện phối trí đều. Bạn thường thấy các số phối trí là bốn hoặc sáu.
d) Vẽ hình ảnh 3D của các đa diện với các số phối trí bốn và sáu.
e) Vẽ các đồng phân có thể có của các hợp chất phức MX2Y2, MX4Y2 và
MX3Y3. (M: vi hạt trung tâm; X và Y: các phối tử một càng).
Tất cả các hệ suy biến đều có xu hướng làm giảm sự suy biến orbital.
Các mức d suy biến của một cation kim loại cô lập bị tách mức ngay khi
chúng chịu tác động của một trường phối tử, tạo thành các mức năng
lượng cao hơn và thấp hơn.
f) Mô tả sự tách mức năng lượng của các mức electron d trong một
trường bát diện. Cho biết các orbital d khác nhau bị ảnh hưởng như
thế nào.
g) Xác định số electron chưa ghép cặp tồn tại trong trạng thái spin
cao và spin thấp của các cation ở ý c trong trường phối tử bát diện.

118 | Chuẩn bị kiến thức cho kì thi HSGQG Hóa học


Hướng dẫn
a)

b) Các phân lớp electron bão hòa (cấu hình khí hiếm, d10, f14, p6, s2) hay
bán bão hòa (d5, f7, p3) đặc biệt thuận lợi về mặt năng lượng.
c)

d)

e)

f)

119 | Chuẩn bị kiến thức cho kì thi HSGQG Hóa học


g)

120 | Chuẩn bị kiến thức cho kì thi HSGQG Hóa học


Chủ đề 5: Phức bis(salicylidene)ethylenediamine
Hợp chất bis(salicylidene)ethylenediamine (kí hiệu là H2(salene)) tạo
nhiều phức chất kim loại chuyển tiếp bền, được sử dụng làm xúc tác
oxid hóa khử trong tổng hợp hữu cơ.

a) Trong phức chất kim loại chuyển tiếp, H2(salene) thường tách hai
proton và đóng vai trò phối tử bốn càng. Vẽ cấu trúc (có biểu diễn điện
tích) của phức chất theo kiểu phối trí này giữa anion (salene)2– và ion
kim loại Mn+.
b) Phản ứng giữa H2(salene*) và manganese(II) acetate trong ethanol
có mặt oxygen không khí và LiCl, thu được phức chất T có công thức
[Mn(salene*)Clx]. Hòa tan 351,2 mg T trong nước rồi thêm vào 5,00 mL
dung dịch ascorbic acid 0,30 M, thu được [Mn(salene*)]. Lượng dư
ascorbic acid được chuẩn độ bằng dung dịch KI3 0,10 M thì hết 11,05
mL. Biết ascorbic acid (C6H8O6) bị oxid hóa thành dehydroascorbic
acid (C6H6O6).
Xác định nhóm thế R và chỉ số x trong T.
c) Biết T có moment từ spin là 4,90 BM. Biểu diễn dạng hình học và sử
dụng thuyết liên kết hóa trị (thuyết VB) để mô tả sự hình thành liên
kết phối trí trong T.
d) Dự đoán (có giải thích) moment từ spin của phức chất
[Mn(salene*)].
Cho biết: Khối lượng nguyên tử của H=1; C=12; N=14; O=16; F=19; S=32;
Cl=35,5; Mn=55; ZMn=25.

121 | Chuẩn bị kiến thức cho kì thi HSGQG Hóa học


Hướng dẫn
a) Cấu trúc phức chất:

b) Phương trình phản ứng, xác định nhóm thế R và chỉ số x:


+ Phương trình phản ứng (1/4):
x x
[Mn(salene* )Clx] + C6H8O6 ⎯⎯→ [Mn(salene* )] + C6H6O6 + xHCl
2 2
KI3 + C6H8O6 ⎯⎯
→ KI + 2HI + C6H6O6

+ Xác định nhóm thế R; chỉ số x:


nascorbic(tổng) = 0,30.5,00 = 1,50 (mmol) nI− = 0,10.11,05 = 1,105
3

(mmol)
x x
Vì nascorbic(tổng) = nI + .nX = 1,105 + .nX = 1,50

3 2 2
−3
0,79 351,2.10
nên nX = (mmol) và MX = = 444,55x.
x 0,79 −3
.10
x
Công thức phân tử của T có dạng: C16H12O2N2MnClxR2 nên MX = 319 +
35,5x + 2R.
Suy ra: 444,55x = 319 + 35,5x + 2R;
Do R là các gốc: -F; -COOH; -SO3H nên 19 ≤ R ≤ 81;
Suy ra: 0,783 ≤ x ≤ 1,176 nên x = 1.
Với x = 1 có R = 45 và R là -COOH.
c) Biểu diễn dạng hình học và thuyết VB:
Phức chất [Mn(salene*)Cl] (T) là phức chất với số phối trí 5 dạng chóp
đáy vuông của Mn(III). Vì ion Mn3+ có cấu hình electron hóa trị là 3d4, mà

122 | Chuẩn bị kiến thức cho kì thi HSGQG Hóa học


giá trị moment từ spin cho biết số electron độc thân là 4 nên
[Mn(salene*)Cl] (T) là phức chất spin cao trong đó ion Mn3+ ở trạng thái
lai hóa dsp3.

d) Dự đoán và giải thích moment từ spin của phức [Mn(salene*)]:


Tương tác giữa (salene*)2– và Mn2+ yếu hơn tương tác giữa (salene*)2– và
Mn3+. Như thấy ở ý c) phức [Mn(salene*)Cl] (T) của Mn3+ là phức chất spin
cao nên tương tác giữa (salene*)2– và Mn3+ không mạnh. Do vậy, tương
tác giữa (salene*)2– và Mn2+ yếu và [Mn(salene*)] cũng là phức spin cao.
Vì Mn2+ có cấu hình electron hóa trị là 3d5 nên phức spin cao
[Mn(salene*)] sẽ có 5 electron độc thân. Do đó, moment từ spin của
[Mn(salene*)] là: spin = 5.(5 + 2)  5,92 (BM).

123 | Chuẩn bị kiến thức cho kì thi HSGQG Hóa học


Chủ đề 6: Xác định phức chất
Hợp chất X có chứa kim loại M có số oxid hóa +4. Từ dung dịch nước, X
kết tinh ở dạng monohydrate. Phần trăm khối lượng M trong
monohydrate của X là khoảng 18%.
X không tan trong nước và bị phân hủy khi đun nóng. X phát ra ánh kim
loại màu tím đen. Trong chất rắn X, kim loại M có cầu phối trí bát diện,
bao xung quanh bởi các nguyên tử hydrogen.
Oxide đơn giản nhất của M kết tinh theo cấu trúc giống sodium
chloride.
Để tổng hợp X, cần các hợp chất A, B, C.
Hợp chất A tạo thành các tinh thể màu trắng, không hút ẩm. A có thể
được tổng hợp bằng cách oxid hóa sodium iodate hoặc sodium iodide
với chlorine hoặc bromine trong dung dịch kiềm. A tồn tại ở dạng ortho
(“giàu nước”). Phân tử khối của A lớn hơn 250 g/mol và hệ số tỉ lượng
của sodium và oxygen trong A là n(Na):n(O) = 1:2.
B là hợp chất ion. Anion của nó là chất oxid hóa mạnh và có thể được
tạo thành bằng cách oxid hóa các sulfate hoặc hydrogen sulfate bởi
các chất oxid hóa cực kì mạnh (như fluorine) hoặc bằng phương pháp
điện hóa. Cation tạo màu vàng trong thí nghiệm đổi màu ngọn lửa.
Dung dịch nước của hợp chất C có các phản ứng sau:
- Tạo thành kết tủa với một lượng dư dung dịch NaOH, chất này
không tan trong NaOH dư nhưng tan trong các acid.
- Tạo thành kết tủa trắng với barium chloride, kết tủa này không
tan trong các acid.
- Dung dịch trong ammonia tạo thành kết tủa đen với sulfur
hydrogen.
- Không tạo màu với dung dịch potassium cyanate.
a) Xác định công thức phân tử các chất A, B, C.
b) Xác định số oxid hóa của lưu huỳnh trong anion của B.
c) Xác định công thức của X và phản ứng tạo thành X⸱H2O. Chỉ rõ số
oxid hóa của các nguyên tử trong phương trình phản ứng.
d) Dự đoán tính chất từ của hợp chất X. Giải thích bởi giản đồ orbital.
Yếu tố nào ảnh hưởng đến sự chiếm các orbital.

124 | Chuẩn bị kiến thức cho kì thi HSGQG Hóa học


Kim loại M và titanium là thành phần chính của một trong các đại diện
tiêu biểu của nhóm hợp kim hiện đại.
e) Cho biết tên hợp chất này. Loại hợp kim này có tính chất đặc biệt
nào?
Hướng dẫn
a) A: Na3H2IO6 B: Na2S2O8 C: NiSO4
b)

c) X: NaNiIO6

hoặc

Hàm lượng nickel:

d) Nickel có số oxid hóa +IV và có hệ d6. Cấu hình electron spin thấp
hoặc spin cao thì phụ thuộc vào sự tách mức của trường phối tử. Sự
tách mức bị ảnh hưởng bởi các phối tử (trong trường hợp này là oxygen
– nằm ở khoảng giữa trong dãy phổ hóa học), bởi kim loại trung tâm (số
lượng tử càng lớn thì sự tách mức càng mạnh, trong trường hợp này là
nickel với hệ 3d – khá nhỏ) và bởi điện tích của nguyên tử trung tâm
(điện tích càng lớn, như trường hợp này, thì sự tách mức càng mạnh).
Trong trường hợp này, điện tích có thể đóng vai trò then chốt – hợp
chất có tính nghịch từ.

125 | Chuẩn bị kiến thức cho kì thi HSGQG Hóa học


Thực tế, có những tâm Ni(III) trong hợp chất này (đặc biệt sau quá
trình ủ), do đó bạn có thể quan sát được một lượng nhỏ có tính thuận
từ.
e) Hợp kim được nói đến là nitinol, là một hợp kim có khả năng ghi nhớ
hình dạng. Nếu bị biến dạng thì khi đun nóng, nó có thể khôi phục lại
hình dạng ban đầu.

126 | Chuẩn bị kiến thức cho kì thi HSGQG Hóa học


Chủ đề 7: Liên kết ngược
Hai phức chất chromium chứa các phối tử CO, PF3 và PCl3 ở dạng hình
học bát diện được cho dưới đây. Trong một phức bát diện, các orbital
phân tử tạo thành bởi sự phối trí có thể được xem là kết quả từ sự
nhường hai electron bởi mỗi trong số sáu phối tử σ-donor (hợp phần
nhường σ) vào các orbital d của kim loại, gọi là sự tạo thành liên kết σ.
Sự tạo thành liên kết π (Pi) trong các phức bát diện cũng có thể khi
phối tử có sẵn các orbital phân tử p, d hoặc π*. Các phối tử như CO, CN-
và các phosphine (công thức PR3) là các π acceptor (hợp phần nhận π),
với các orbital trống có thể tương tác với các orbital d theo kiểu liên
kết π. Trong đa số trường hợp, liên kết  ngược chiếm ưu thế, và mật
độ electron được chuyển từ kim loại đến phối tử. Sự tạo thành liên kết
π có thể ảnh hưởng đến năng lượng liên kết kim loại-phối tử và độ dài
liên kết các phức chất carbonyl và phosphine.
Xem xét tương tác π, trả lời các câu hỏi sau:
1) Trong phức chất nào, liên kết C-O ngắn hơn, Cr(CO)5(PF3) hay
Cr(CO)5(PCl3)?
2) Trong phổ hồng ngoại của phức chất nào sau đây, dải kéo căng C-
O có năng lượng cao hơn, Cr(CO)5(PF3) hay Cr(CO)5(PCl3)?

127 | Chuẩn bị kiến thức cho kì thi HSGQG Hóa học


Hướng dẫn
1) Cr(CO)5(PF3)
Do liên kết ngược (π-bonding) giữa kim loại và phối tử được tăng cường
nên liên kết kim loại-carbon được tăng cường và liên kết C-O bị suy
yếu. Bởi PF3 là hợp phần nhận π (π-acceptor) tốt hơn PCl3 (do độ âm
điện F cao hơn Cl). Do đó, PF3 làm giảm mật độ điện tích trên tâm kim
loại nhiều hơn PCl3 và sự nhường electron π từ kim loại đến các orbital
phản liên kết π* của CO bị giảm đi, dẫn đến liên kết trong phức thế PF3
mạnh hơn và ngắn hơn.
2) Cr(CO)5(PF3)

128 | Chuẩn bị kiến thức cho kì thi HSGQG Hóa học


Chủ đề 8: Hydrogen hóa xúc tác
Một trong những phương pháp để khử hóa alkene là dùng phản ứng
hydrogen hóa xúc tác. Có nhiều cách khác nhau để thực hiện phản ứng
này. Ví dụ, trong bình phản ứng phòng thí nghiệm, xúc tác kim loại
platinum ở dạng mạt giũa được cho vào dung dịch chứa alkene rồi sục
khí hydrogen (H2) vào. Sơ đồ dưới đây biểu diễn phản ứng hydrogen hóa
xúc tác alkene:

Ở cấp độ hiển vi, phản ứng diễn ra với sự hấp phụ các phân tử H2 trên
bề mặt xúc tác platinum. Liên kết H-H bị bẻ gãy, để lại các nguyên tử
hydrogen trên bề mặt và chúng sẽ liên kế với alkene tạo thành alkane,
như minh hoạ trong hình 1.

Hình 1: Sơ đồ hydrogen hóa xúc tác alkene. Các quả cầu lớn màu trắng
là các phân tử platinum, các quả cầu nhỏ màu xám là các nguyên tử
hydrogen.
1) Vẽ giản đồ orbital phân tử của H2 và xác định bậc liên kết H-H.
2) Viết cấu hình điện tử của platinum và vẽ 5 orbital d của nó.
3) Giải thích tại sao liên kết H-H trong phân tử H2 bị bẻ gãy khi xảy ra
sự hấp phụ trên bề mặt platinum.

129 | Chuẩn bị kiến thức cho kì thi HSGQG Hóa học


Hình 2: Cơ chế hydrogen hóa xúc tác alkene sử dụng xúc tác
Wilkinson.
Một quá trình thay thế để thực hiện phản ứng hydrogen hóa xúc tác là
sử dụng xúc tác đồng thể. Trong trường hợp này, một phương án khả
thi là sử dụng xúc tác Wilkinson (phức I). Trong trường hợp này, phản
ứng diễn ra theo cơ chế minh họa ở hình 2 bên dưới. (Chú ý: các số từ I
đến VI chỉ ra các phức chất tham gia vào mỗi giai đoạn của chu trình
xúc tác).
4) Vẽ lại cấu trúc phức chất I và biểu diễn tất cả các mặt phẳng đối
xứng của nó.
5) Tính số oxid hóa của các nguyên tử Rh trong các phức chất I và IV.
6) Biểu diễn giản đồ tách mức năng lượng của các orbital d Rh trong
các phức chất I, II và dự đoán tính chất từ của các phức chất này.
7) Các phối tử phosphine liên kết với ion Rh đóng vai trò quan trọng
để bền hóa các trung gian tạo thành trong phản ứng. Giả sử ba
nhóm PPh3 trong I bị thay thế PF3 thì hoạt tính xúc tác sẽ tăng, giảm
hay không bị ảnh hưởng? Giải thích.
Phổ hấp thụ hồng ngoại là một kĩ thuật quang phổ cho phép chúng ta
suy ra độ bền của các liên kết hóa học. Dao động kéo căng của phân tử
CO tự do xuất hiện ở 2143 cm-1. Xét phản ứng hóa học dưới đây của phức

130 | Chuẩn bị kiến thức cho kì thi HSGQG Hóa học


Ib, một chất tương tự với I, trong đó các nhóm phosphine đã bị thay thế
bởi các nhóm carbonyl.

8) Đối với phức Ib, dao động kéo căng phân tử CO sẽ xuất hiện ở số
sóng lớn hơn, nhỏ hơn hay bằng với CO tự do? Giải thích.
9) Đối với phức IIb, dao động kéo căng phân tử CO sẽ xuất hiện ở số
sóng lớn hơn, nhỏ hơn hay bằng với CO tự do? Giải thích.
10) Cho biết tên gọi dạng hình học của phức Ib. Vẽ tất cả các đồng phân
của IIb.

131 | Chuẩn bị kiến thức cho kì thi HSGQG Hóa học


Hướng dẫn
1) Bậc liên kết bằng 1:

2) [Xe]6s24f145d8

3) Khi phân tử H2 hấp phụ trên bề mặt platinum, các kim loại giàu
electron sẽ chuyển mật độ electron đến orbital σ* của H2, dẫn đến sự
suy yếu liên kết H-H. Do platinum có khả năng chuyển mật độ electron
cao, chỉ sau một thời gian bậc liên kết H-H sẽ trở về không và liên kết
sẽ bị bẻ gãy.

132 | Chuẩn bị kiến thức cho kì thi HSGQG Hóa học


4)

5) I: +1, IV: +3
6)

7) Sự thay thế PPh3 bởi PF3 sẽ làm giảm mật độ electron trong Rh và do
đó làm giảm hoạt tính xúc tác do trong nhiều giai đoạn, Rh có số oxid
hóa +3, không được bền hóa bởi các phối tử hút electron như fluorine.

133 | Chuẩn bị kiến thức cho kì thi HSGQG Hóa học


8) Dao động kéo căng phân tử CO sẽ xuất hiện ở số sóng thấp hơn, do
sự suy yếu của liên kết CO sẽ xảy đến khi liên kết M-C được tạo thành.
Liên kết M-C được tạo thành bởi sự dùng chung các electron của C với
kim loại. Do kim loại có các orbital bị chiếm có tính đối xứng π, chúng
sẽ nhường electron cho các orbital của phối tử có cùng tính đối xứng,
nhưng các orbital này là orbital phản liên kết của CO, dẫn đến sự suy
yếu liên kết CO.

9) Dao động kéo căng của phân tử CO sẽ xuất hiện ở số sóng lớn hơn.
Số oxid hóa cao hơn của Rh(III) trong phức chất IIb so với Ib (Rh(I)) sẽ làm
bền hóa thêm liên kết sigma, làm giảm mật độ electron của liên kết
cho nhận ngược (backbonding) thông qua liên kết pi. Sự suy yếu của
liên kết CO trong phức chất IIb kém hơn, do đó số sóng cho sự kéo căng
CO cao hơn.
10) Dạng hình học Ib: vuông phẳng.
Các đồng phân hình học của IIb:

134 | Chuẩn bị kiến thức cho kì thi HSGQG Hóa học


Phần B: Hóa Hữu cơ
Chương 4: Cấu trúc và hoạt tính
Chủ đề 1: Công thức phân tử - Cấu tạo khai triển
1) Dự đoán công thức phân tử khả dĩ cho các hợp chất sau:
a) CCla.
b) AlHb.
c) CHcCl2.
d) SiFd.
e) CH3NHe.
2) Biểu diễn công thức cấu tạo khai triển của các hợp chất sau, chỉ rõ
tất cả các electron chưa liên kết:
a) CHCl3, chloroform.
b) H2S, hydrogen sulfide.
c) CH3NH2, methylamine.
d) CH3Li, methyllithium.
3) Xác định công thức phân tử tương ứng với mỗi cấu tạo sau đây:

135 | Chuẩn bị kiến thức cho kì thi HSGQG Hóa học


4) Có bao nhiêu nguyên tử hydrogen liên kết với mỗi nguyên tử carbon
trong các hợp chất sau. Chỉ rõ công thức phân tử tương ứng của
mỗi chất.
a) Adrenaline b) Estrone

5) Hãy đề xuất khung sườn cấu tạo cho các hợp chất thỏa mãn công
thức phân tử sau đây. Mỗi trường hợp có thể có nhiều phương án
được đề xuất.
a) C5H12.
b) C2H7N.
c) C3H6O.
d) C4H9N.
6) Bổ sung các electron hóa trị chưa liên kết còn thiếu trong các cấu
tạo sau đây:

7) Quetiapine, có tên thương mại là Seroquel, là loại thuốc chống loạn


thần được kê đơn nhiều nhất, dùng trong điều trị chứng tâm thần
phân liệt và rối
loạn lưỡng cực.
Hãy chuyển đổi
cấu tạo dưới đây
thành cấu tạo khai
triển và cấu tạo
khung sườn –
đồng thời chỉ ra
công thức phân tử
của quetiapine.

136 | Chuẩn bị kiến thức cho kì thi HSGQG Hóa học


137 | Chuẩn bị kiến thức cho kì thi HSGQG Hóa học
Hướng dẫn
1) Carbon (thuộc nhóm 4A, hay 14) có 4 electron trong vỏ hóa trị và có
xu hướng tạo thành 4 liên kết cộng hóa trị để tạo thành cấu hình khí
quý tựa neon. Nên công thức khả dĩ là CCl4.
Tương tự AlH4, CH2Cl2 và SiF4. Với trường hợp e, nitrogen (thuộc nhóm
5A, hay 15) có 5 electron hóa trị và có xu hướng tạo thành 3 liên kết
cộng hóa trị - ngoài một liên kết với carbon của nhóm CH3 thì còn thêm
2 liên kết với hydrogen. Công thức phù hợp là CH3NH2.
2) Nếu bạn chưa quen, hãy bắt đầu với việc biểu diễn công thức Lewis
của phân tử.
Bước 1: Xác định số electron hóa trị của mỗi nguyên tử cấu thành phân
tử. Xét ví dụ với chloroform, chúng ta biết rằng carbon có 4 electron
hóa trị, hydrogen có 1, còn chlorine thì có 7.

Bước 2: Ta sử dụng hai electron hóa trị cho mỗi liên kết đơn.

Bước 3: Cuối cùng, sử dụng các electron còn lại để đạt tới cấu hình
electron khí quý cho nhiều nguyên tử nhất có thể. Để biểu diễn công
thức cấu tạo khai triển, chỉ cần thay mỗi cặp dấu chấm (electron) giữa
hai nguyên tử bởi một đường kẻ.

138 | Chuẩn bị kiến thức cho kì thi HSGQG Hóa học


3)
a) Aspirin C9H8O4
b) Vitamin C C6H8O6
c) Nicotine C10H14N2
d) Glucose C6H12O6
4)

139 | Chuẩn bị kiến thức cho kì thi HSGQG Hóa học


5) Một số phương án được đề xuất:

6)

7)

140 | Chuẩn bị kiến thức cho kì thi HSGQG Hóa học


Chủ đề 2: Sự lai hóa
a) Biểu diễn công thức cấu tạo của:
a) propene, CH3CH=CH2;
b) 1,3-butadiene, CH2=CH-CH=CH2.
c) propyne, CH3C  CH .

chỉ ra sự lai hóa các orbital của mỗi nguyên tử arbon và dự đoán giá
trị mỗi góc liên kết.
b) Dự đoán trạng thái lai hóa của mỗi nguyên tử carbon trong các
phân tử sau đây:

c) Dự đoán trạng thái lai hóa của mỗi nguyên tử carbon trong các
phân tử sau đây:

141 | Chuẩn bị kiến thức cho kì thi HSGQG Hóa học


d) Dự đoán trạng thái lai hóa của mỗi nguyên tử carbon trong các
phân tử sau đây:

(c)

e) Chỉ ra các cặp electron chưa liên kết trong các phân tử sau và dự
đoán dạng hình học của mỗi nguyên tử được chỉ rõ.
a) Oxygen trong dimethyl ether, CH3-O-CH3.
b) Nitrogen trong trimethylamine, (CH3)3N.
c) Phosphorus trong phosphine, PH3.
d) Lưu huỳnh trong methionine, CH3SCH2CH2CH(NH2)C(=O)OH.

142 | Chuẩn bị kiến thức cho kì thi HSGQG Hóa học


Hướng dẫn
1)
a)

Cả 4 nguyên tử gắn với liên kết đôi carbon-carbon nằm trong cùng một
mặt phẳng và toàn bộ các góc liên kết giữa các nguyên tử này đều là
120o. Góc liên kết giữa hydrogen và nguyên tử carbon lai hóa sp3 là 109o.
b)

Tất cả các nguyên tử đều nằm trong cùng một mặt phẳng và tất cả các
góc liên kết đều xấp xỉ 120o.
c)

Ba nguyên tử carbon của propyne nằm trên cùng một đường thẳng:
góc liên kết bằng 180o. Góc liên kết H − C1  C2 cũng bằng 180o. Góc liên
kết giữa hydrogen và carbon lai hóa sp3 là 109o.

143 | Chuẩn bị kiến thức cho kì thi HSGQG Hóa học


2)

3)

4)

(c)

144 | Chuẩn bị kiến thức cho kì thi HSGQG Hóa học


5) a) Nguyên tử oxygen lai hóa sp3 có dạng hình học tứ diện.

b) Nguyên tử nitrogen sp3 có dạng tháp tam giác.

c) Nguyên tử phosphorus sp3 có dạng tháp tam giác tương tự như


nitrogen trong NH3.

d) Nguyên tử lưu huỳnh sp3 có dạng gấp khúc.

145 | Chuẩn bị kiến thức cho kì thi HSGQG Hóa học


Chủ đề 3: Một số tiểu phân đặc biệt
1) Hãy lí giải tại sao chưa có ai thành công trong việc phân lập hợp
chất cyclopentyne?

2)
a) Allene, CH2=C=CH2, là hợp chất có chút bất thường vì có hai liên kết
đôi liền kề. Hãy vẽ giản đồ biểu diễn các orbital tham gia vào các
liên kết sigma và pi của allene. Nguyên tử carbon trung tâm có
trạng thái lai hóa sp2 hay sp? Dự đoán dạng hình học của allene.
b) Cấu tạo của allene cũng có nhiều điểm tương đồng với carbon
dioxide. Hãy biểu diễn giản đồ orbital của CO2 và chỉ ra trạng thái
lai hóa của nguyên tử carbon.
3) Đa số các tiểu phân hữu cơ bền đều có các nguyên tử carbon hóa
trị bốn, nhưng các tiểu phân chứa nguyên tử carbon hóa trị ba cũng
tồn tại. Carbocation là một nhóm các hợp chất như vậy. Dưới đây là
cấu tạo của carbocation đơn giản nhất:

a) Nguyên tử carbon mang điện tích dương có bao nhiêu electron


hóa trị?
b) Dự đoán trạng thái lai hóa của nguyên tử carbon này.
c) Dự đoán dạng hình học của carbocation.
4) Carbanion là tiểu phân chứa nguyên tử carbon hóa trị ba, mang
điện tích âm:

a) Cho biết mối liên hệ về mặt điện tử giữa carbanion và hợp chất
nitrogen hóa trị ba, ví dụ như NH3.

146 | Chuẩn bị kiến thức cho kì thi HSGQG Hóa học


b) Nguyên tử carbon mang điện tích âm có bao nhiêu electron hóa
trị?
c) Dự đoán trạng thái lai hóa của nguyên tử carbon này.
d) Dự đoán dạng hình học của carbanion.
5) Các tiểu phân chứa carbon hóa trị hai, carbene, có khả năng trong
thời gian rất ngắn. Ví dụ, carbene đơn giản nhất là methylene, :CH2.
Hai electron chưa chia sẻ trong methylene có thể ghép cặp trong
một orbital hoặc không ghép cặp và thuộc các orbital khác nhau.
Hãy dự đoán kiểu lai hóa phù hợp với singlet methylene (dạng ghép
cặp spin) và triplet methylene (dạng không ghép cặp spin).

147 | Chuẩn bị kiến thức cho kì thi HSGQG Hóa học


Hướng dẫn
1) Trong một hợp chất chứa liên kết ba carbon-carbon, các nguyên tử
gắn với các nguyên tử carbon lai hóa sp phải nằm trên một đường
thẳng. Không thể tạo thành vòng năm cạnh nếu như bốn nguyên tử
carbon phải thỏa mãn mối liên hệ thẳng hàng như vậy!
2)
a) Nguyên tử carbon trung tâm của allene tạo thành hai liên kết
sigma và hai liên kết pi; nguyên tử này lai hóa sp (còn hai carbon
đầu mạch thì lai hóa sp2). Góc liên kết tạo thành bởi ba nguyên
tử carbon này là 180o – chỉ ra dạng hình học thẳng hàng của các
nguyên tử carbon trong allene.

b)

148 | Chuẩn bị kiến thức cho kì thi HSGQG Hóa học


3)
a) Nguyên tử carbon mang điện tích dương được bao xung quanh
bởi 6 electron hóa trị (carbon có 3 electron hóa trị, và các
nguyên tử hydrogen đóng góp 3 electron còn lại).
b) Nguyên tử carbon này lai hóa sp2.
c) Dạng hình học quanh nguyên tử carbon mang điện tích dương
của carbocation là tam giác phẳng.
4)

a) Carbanion là tiểu phân đẳng điện tử (có cùng số electron) với


hợp chất nitrogen hóa trị ba.
b) Nguyên tử carbon mang điện tích âm của carbanion có 8
electron.
c) Nguyên tử này lai hóa sp3.
d) Dạng hình học quanh nguyên tử carbon mang điện tích âm của
carbanion là tứ diện.
5) Trong triplet methylene, carbon lai hóa sp, còn trong singlet
methylene thì carbon lai hóa sp2.

149 | Chuẩn bị kiến thức cho kì thi HSGQG Hóa học


Chủ đề 4: Sự phân cực liên kết
1) Tra cứu bảng độ âm điện (từ các tài liệu tham khảo hoặc Internet)
và chỉ ra nguyên tử có độ âm điện lớn nhất trong mỗi phân tử sau:

2) Sử dụng quy ước  + / − để chỉ ra chiều phân cực dự đoán cho mỗi
liên kết dưới đây:

3) Hãy dự đoán chiều phân cực trong các nhóm chức phổ biến sau:

4)
a) Dự đoán chiều phân cực của liên kết C-Cl trong chloromethane:

b) Ethylene glycol, HOCH2CH2OH, có moment lưỡng cực bằng


không mặc dù các liên kết carbon-oxygen đều phân cực mạnh
và oxygen có hai cặp electron chưa liên kết. Hãy giải thích tại
sao.
5) Biểu diễn cấu trúc ba chiều của mỗi phân tử sau và dự đoán liệu
mỗi phân tử có moment lưỡng cực không. Nếu có, hãy biểu diễn
chiều của nó.

6) Maleic acid có moment lưỡng cực, nhưng đồng phân của nó là


fumaric acid (chất tham gia vào chu trình citric acid chuyển hóa
thực phẩm) thì không. Hãy giải thích nguyên do.

150 | Chuẩn bị kiến thức cho kì thi HSGQG Hóa học


Hướng dẫn
1)

2)

3)

4)
a)

151 | Chuẩn bị kiến thức cho kì thi HSGQG Hóa học


b) Moment lưỡng cực của ethylene glycol bằng không bởi sự phân
cực liên kết của hai liên kết carbon-oxygen triệt tiêu lẫn nhau.

5)

6) Trong maleic acid, các moment lưỡng cực riêng lẻ tổ hợp tạo thành
moment lưỡng cực toàn phần; trong khi đó các moment lưỡng cực
riêng lẻ trong fumaric acid lại triệt tiêu lẫn nhau.

152 | Chuẩn bị kiến thức cho kì thi HSGQG Hóa học


Chủ đề 5: Điện tích hình thức
1) Xác định điện tích hình thức cho các nguyên tử khác hydrogen
trong các phân tử sau:

2) Các nhóm chức phosphate tồn tại phổ biến trong các phân tử sinh
học. Hãy xác định điện tích hình thức trên các nguyên tử oxygen
trong dianion methyl phosphate:

3) Xác định điện tích hình thức của các nguyên tử được chỉ ra bởi các
mũi tên dưới đây:

153 | Chuẩn bị kiến thức cho kì thi HSGQG Hóa học


4) Thiamin diphosphate (TPP), một dẫn xuất của vitamin B1 – cần thiết
cho sự chuyển hóa glucose – là acid yếu, có thể bị deproton hóa bởi
các base. Hãy gán điện tích hình thức cho các nguyên tử phù hợp
trong TPP và sản phẩm deproton hóa của nó:

154 | Chuẩn bị kiến thức cho kì thi HSGQG Hóa học


Hướng dẫn
1) Nhắc lại công thức tính điện tích hình thức (FC hay “formal charge”):
1
FC = [Số electron hóa trị] - [Số electron liên kết] – [Số electron chưa liên kết]
2

a)

8
carbon: FC = 4 − − 0 = 0
2
8
nitrogen ( 1) : FC = 5 − − 0 = +1
2
4
nitrogen ( 2 ) : FC = 5 − − 4 = −1
2
b)

8
carbon ( 1) : FC = 4 − − 0 = 0
2
8
carbon ( 2 ) : FC = 4 − − 0 = 0
2
8
nitrogen: FC = 5 − − 0 = +1
2
2
oxygen: FC = 6 − − 6 = −1
2
c)

155 | Chuẩn bị kiến thức cho kì thi HSGQG Hóa học


8
carbon ( 1) : FC = 4 − − 0 = 0
2
6
carbon ( 2 ) : FC = 4 − − 0 = −1
2
8
nitrogen: FC = 5 − − 0 = +1
2
2)

4
oxygen ( 1) : FC = 6 − − 4 = 0
2
4
oxygen ( 2 ) : FC = 6 − − 4 = 0
2
2
oxygen ( 3 ) : FC = 6 − − 6 = −1
2
2
oxygen ( 4 ) : FC = 6 − − 6 = −1
2
3)

156 | Chuẩn bị kiến thức cho kì thi HSGQG Hóa học


4)

157 | Chuẩn bị kiến thức cho kì thi HSGQG Hóa học


Chủ đề 6: Sự cộng hưởng
1) Mỗi cặp cấu tạo sau đây có phải là các dạng cộng hưởng không?

2) 1,3-cyclobutadiene là phân tử hình chữ nhật, với hai liên kết đôi
ngắn hơn và hai liên kết đơn dài hơn. Tại sao các cấu tạo dưới đây
lại không phải các dạng cộng hưởng?

3) Hãy biểu diễn các công thức cộng hưởng, theo số lượng cấu trúc
được cho trong dấu ngoặc đơn, cho mỗi tiểu phân sau:

158 | Chuẩn bị kiến thức cho kì thi HSGQG Hóa học


4) Hãy biểu diễn (nhiều nhất có thể) các dạng cộng hưởng cho mỗi
tiểu phân sau đây:

5) Carbocation, tiểu phân chứa nguyên tử carbon hóa trị ba, có thể
phản ứng với nước tạo thành alcohol.

Bạn hãy giải thích tại sao carbocation sau đây tạo thành hỗn hợp
hai alcohol khi phản ứng với nước:

6) Các hợp chất azide, như azidobenzene, chứa hợp phần gồm ba
nguyên tử nitrogen cạnh nhau. Dưới đây là một trong dạng cộng
hưởng của azidobenzene. Hãy vẽ thêm ba dạng cộng hưởng khác
của chất này và gán điện tích hình thức phù hợp cho các nguyên tử
nitrogen trong cả bốn dạng.

159 | Chuẩn bị kiến thức cho kì thi HSGQG Hóa học


7) Biểu diễn các cấu trúc cộng hưởng của gốc benzyl, C6H5CH2• - được
tạo thành trong phản ứng bromo hóa toluene bởi NBS.
8) Biểu diễn các cấu trúc cộng hưởng của các tiểu phân dưới đây:

160 | Chuẩn bị kiến thức cho kì thi HSGQG Hóa học


Hướng dẫn
1) Các dạng cộng hưởng thì không khác nhau về vị trí của các nguyên
tử. Hai cấu tạo ở ý a không phải là các dạng cộng hưởng bởi vị trí của
các nguyên tử carbon và hydrogen ở ngoài vòng sáu cạnh của hai
dạng này khác nhau.

Các cặp b – d thì đều biểu diễn các dạng cộng hưởng.
2) Bởi vị trí của các nguyên tử carbon trong hai dạng này khác nhau.
3)
a)

b)

c)

d)

161 | Chuẩn bị kiến thức cho kì thi HSGQG Hóa học


4)

5) Cation này có thể được biểu diễn bởi hai dạng cộng hưởng. Phản ứng
với nước có thể diễn ra ở hai vị trí carbon mang điện tích dương, dẫn
đến sự tạo thành hai sản phẩm:

6)

162 | Chuẩn bị kiến thức cho kì thi HSGQG Hóa học


7)

8)

163 | Chuẩn bị kiến thức cho kì thi HSGQG Hóa học


Chủ đề 7: Tổng quan về tính acid-base
1) Amino acid phenylalanine có pKa = 1.83, còn tryptophan có pKa =
2.83.

2) Cho biết ammonia, NH3, có pKa xấp xỉ 36, còn acetone thì xấp xỉ 19.
Hãy cho biết phản ứng sau có thể được xem là có xảy ra không?

3) Hãy giải thích theo quan điểm về


sự cộng hưởng tại sao hydrogen
O-H trong acetic acid lại có tính
acid mạnh hơn nhiều so với các
hydrogen C-H.
4) Dù đều chứa liên kết O-H nhưng phenol, C6H5OH, lại có lực acid
mạnh hơn methanol, CH3OH, nhiều. Hãy biểu diễn cấu tạo các
anion tạo thành từ quá trình tách H+ khỏi phenol và methanol, và
dựa vào quan điểm sự cộng hưởng để giải thích cho sự khác biệt về
tính acid:

164 | Chuẩn bị kiến thức cho kì thi HSGQG Hóa học


Hướng dẫn
1) Acid càng mạnh thì pKa càng nhỏ, còn acid càng yếu thì pKa càng lớn.
Theo đó, phenylalanine (pKa = 1.83) là acid mạnh hơn tryptophan (pKa =
2.83).
2) Phản ứng có thể được xem là diễn ra gần như hoàn toàn bởi chênh
lệch giữa các giá trị pKa lớn.

3) Bởi oxygen âm điện hơn carbon nhiều nên liên kết -O-H bị phân cực
mạnh hơn các liên kết -C-H, dẫn đến việc H+ dễ phân li hơn. Ngoài ra,
anion acetate tạo thành được bền hóa bởi sự cộng hưởng.

4) Khi mất proton, anion tạo thành từ phenol được bền hóa cộng
hưởng, còn anion từ methanol thì không:

165 | Chuẩn bị kiến thức cho kì thi HSGQG Hóa học


Chủ đề 8: Acid-base theo Lewis
1) Biểu diễn công thức Lewis của các phân tử sau, chỉ ra các cặp
electron chưa liên kết và cho biết hợp chất nào có thể là acid
Lewis, hợp chất nào có thể là base Lewis.

2) Sử dụng các mũi tên cong, hãy chỉ ra các tiểu phân ở nhóm (a) có
thể đóng vai trò base Lewis như thế nào trong phản ứng với HCl; và
các tiểu phân ở nhóm (b) có thể đóng vai trò acid Lewis như thế nào
trong phản ứng với OH-:

3) Imidazole tạo thành một phần cấu trúc của amino acid histidine và
có thể đóng vai trò của cả acid lẫn base.

Hình dưới đây cho biết nguyên tử hydrogen có tính acid mạnh nhất
và nguyên tử nitrogen có tính base mạnh nhất trong imidazole.

Hãy vẽ các cấu trúc cộng hưởng của sản phẩm tạo thành khi
imidazole bị proton hóa bởi acid và deproton hóa bởi base.

166 | Chuẩn bị kiến thức cho kì thi HSGQG Hóa học


4) Xác định các acid và base trong mỗi phản ứng sau:

167 | Chuẩn bị kiến thức cho kì thi HSGQG Hóa học


Hướng dẫn
1)

Các acid Lewis có thể nhận một cặp electron bởi chúng có orbital
trống, hoặc có thể nhường H+. Còn base Lewis thì có các cặp electron
chưa liên kết.

2) Chỉ ra các cặp electron của base Lewis và vẽ mũi tên cong đi từ cặp
electron đến Lewis acid.

168 | Chuẩn bị kiến thức cho kì thi HSGQG Hóa học


3)

4)

169 | Chuẩn bị kiến thức cho kì thi HSGQG Hóa học


Chủ đề 9: Nhóm chức
1) Xác định các nhóm chức trong mỗi hợp chất sau đây:
a) Amino acid methionine:

b) Thuốc giảm đau Ibuprofen:

c) Capsaicin – chất tạo vị cay trong ớt:

2) Xác định các nhóm chức trong các hợp chất sau:

170 | Chuẩn bị kiến thức cho kì thi HSGQG Hóa học


3) Xác định nhóm chức trong các phân tử sau, và chỉ ra sự phân cực
của chúng:

171 | Chuẩn bị kiến thức cho kì thi HSGQG Hóa học


Hướng dẫn
1)

2)

172 | Chuẩn bị kiến thức cho kì thi HSGQG Hóa học


3)

173 | Chuẩn bị kiến thức cho kì thi HSGQG Hóa học


Chủ đề 10: Độ bất bão hòa
1) Hãy tính độ bất bão hòa của mỗi công thức phân tử sau và biểu
diễn một số công thức cấu tạo tương ứng:
a) C4H8
b) C4H6
c) C3H4
2) Hãy tính độ bất bão hòa của mỗi công thức phân tử sau
a) C6H5N
b) C6H5NO2
c) C8H9Cl3
d) C9H16Br2
e) C10H12N2O3
f) C20H32ClN

174 | Chuẩn bị kiến thức cho kì thi HSGQG Hóa học


Hướng dẫn
1)

2)
a) C6H5N: 5
b) C6H5NO2: 5
c) C8H9Cl3: 3
d) C9H16Br2: 1
e) C10H12N2O3: 6
f) C20H32ClN: 5

175 | Chuẩn bị kiến thức cho kì thi HSGQG Hóa học


Chủ đề 11: Đồng phân cấu tạo
1) Có bao nhiêu đồng phân tương ứng với các mô tả sau:
a) Alcohol với công thức C3H8O.
b) Bromoalkane với công thức C4H9Br.
c) Thioester với công thức C4H8OS.
2) Biểu diễn cấu tạo của
a) Các ester có công thức C5H10O2.
b) Các nitrile có công thức C4H7N.
c) Các disulfide có công thức C4H10S2.
3) Biểu diễn cấu tạo của
a) Các alcohol có công thức C4H10O.
b) Các amine có công thức C5H13N.
c) Các ketone có công thức C5H10O.
d) Các aldehyde có công thức C5H10O.
e) Các ester có công thức C4H8O2.
f) Các ether có công thức C4H10O.

176 | Chuẩn bị kiến thức cho kì thi HSGQG Hóa học


Hướng dẫn
1)

2)
(a)

(b)

(c)

177 | Chuẩn bị kiến thức cho kì thi HSGQG Hóa học


3)
a)

b)

c)

d)

178 | Chuẩn bị kiến thức cho kì thi HSGQG Hóa học


e)

f)

179 | Chuẩn bị kiến thức cho kì thi HSGQG Hóa học


Chủ đề 12: Tính electrophile và nucleophile
Trước khi đi vào từng câu hỏi cụ thể, hãy nhớ rằng
▪ Electrophile là tiểu phân thiếu hụt electron, bởi nó mang điện tích
dương, hoặc bởi nó có nhóm chức phân cực hóa dương điện, hoặc
bởi nó có orbital trống.
▪ Nucleophile là tiểu phân giàu electron. bởi nó mang điện tích âm,
hoặc bởi nó có nhóm chức chứa cặp electron chưa liên kết, hoặc
bởi nó có nhóm chức phân cực hóa âm điện.
▪ Một số tiểu phân có thể đóng vai trò nucleophile hoặc electrophile
– tùy thuộc vào điều kiện phản ứng.

1) Hãy dự đoán các tiểu phân dưới đây sẽ ưu tiên thể hiện tính
electrophile hay nucleophile?

2) Hãy dự đoán các tiểu phân dưới đây sẽ ưu tiên thể hiện tính
electrophile hay nucleophile?

180 | Chuẩn bị kiến thức cho kì thi HSGQG Hóa học


3) Xác định các tâm nucleophile (n) và electrophile (e) khả dĩ trong
các phân tử sau:

181 | Chuẩn bị kiến thức cho kì thi HSGQG Hóa học


Hướng dẫn
1) a) Ion nitronium NO2+ là electrophile bởi nó mang điện tích dương.

b) Ion cyanide :C  N là nucleophile bởi nó mang điện tích âm.

c) Methylamine CH3NH2 có thể là nucleophile hoặc electrophile – tùy


thuộc vào điều kiện cụ thể. Cặp electron chưa liên kết trên nguyên tử
nitrogen làm cho tiểu phân này có thể là nucleophile; trong khi đó các
liên kết N− − H+ phân cực mạnh khiến cho nó có thể thể hiện tính acid
(electrophile).
d) Ion trimethylsulfonium (CH3)3S+ là electrophile bởi nó mang điện tích
dương.
2)

3)

182 | Chuẩn bị kiến thức cho kì thi HSGQG Hóa học


Chủ đề 13: Sử dụng mũi tên cơ chế phản ứng
Dưới đây là một vài quy tắc cơ bản:
Quy tắc 1: Electron dịch chuyển từ nguồn có tính nucleophile (Nu:
hoặc Nu:-) đến vùng nhận có tính electrophile (E hoặc E+). Nguồn có
tính nucleophile phải có một cặp electron, thường là cặp electron chưa
liên kết hoặc liên kết bội.

Trong khi đó, vùng nhận có tính electrophile phải có khả năng nhận
cặp electron, thường là bởi nó có nguyên tử mang điện tích dương hoặc
nhóm chức chứa nguyên tử bị phân cực hóa dương điện.

Quy tắc 2: Nucleophile có thể mang điện tích âm hoặc trung hòa
điện. Nếu nucleophile mang điện tích âm thì nguyên tử nhường cặp
electron sẽ trở thành trung hòa điện.

Còn nếu nucleophile trung hòa điện thì nguyên tử nhường cặp electron
sẽ mang điện tích dương.

183 | Chuẩn bị kiến thức cho kì thi HSGQG Hóa học


Quy tắc 3: Electrophile có thể mang điện tích dương hoặc trung hòa
điện. Nếu electrophile mang điện tích dương thì nguyên tử mang điện
tích sẽ trở nên trung hòa điện sau khi nhận cặp electron.

Nếu electrophile trung hòa điện thì nguyên tử nhận cặp electron sẽ
mang điện tích âm. Tuy nhiên, để điều này xảy ra, điện tích âm phải
được bền hóa bởi được mang trên nguyên tử âm điện như oxygen,
nitrogen hoặc halogen. Carbon và hydrogen thường không bền hóa
điện tích âm.

Quy tắc 4: Quy tắc bát tử thường được thỏa mãn.

1) Hãy sử dụng mũi tên cơ chế để biểu diễn dòng electron trong phản
ứng phân cực sau:

184 | Chuẩn bị kiến thức cho kì thi HSGQG Hóa học


2) Bổ sung các mũi tên cong phù hợp vào mỗi phản ứng phân cực sau
để mô tả dòng electron:

3) Bổ sung các mũi tên cong phù hợp vào mỗi phản ứng phân cực sau
để mô tả dòng electron:

4) Bổ sung mũi tên cong để chỉ rõ sự dịch chuyển electron trong cơ


chế phản ứng sau:

185 | Chuẩn bị kiến thức cho kì thi HSGQG Hóa học


5) Ammonia phản ứng với acetyl chloride (CH3COCl) tạo thành
acetamide (CH3CONH2). Hãy vẽ các mũi tên cong để biểu diễn dòng
electron trong mỗi giai đoạn của phản ứng.

186 | Chuẩn bị kiến thức cho kì thi HSGQG Hóa học


Hướng dẫn
1) Hãy nhìn vào phản ứng và xác định những sự thay đổi về liên kết sẽ
diễn ra. Trong trường hợp này, liên kết C-Br bị phá vỡ và liên kết C-C
được tạo thành. Sự tạo thành liên kết C-C liên quan đến sự nhường cặp
electron từ nguyên tử carbon có tính nucleophile (của chất phản ứng
bên trái) cho nguyên tử carbon có tính electrophile (của CH3Br), vậy
nên ta sẽ vẽ mũi tên cong đi từ cặp electron chưa liên kết trên nguyên
tử carbon mang điện tích âm và hướng về nguyên tử carbon của CH3Br.
Đồng thời với sự tạo thành liên kết C-C, liên kết C-Br phải bị phá vỡ để
không vi phạm quy tắc octet. Do đó, ta sẽ vẽ mũi tên cong thứ hai, đi
từ liên kết C-Br đến Br. Bây giờ, bromine trở thành ion Br- bền.

2) a)

b) Một liên kết được tạo thành giữa oxygen và carbon của
bromomethane, trong khi đó liên kết giữa carbon và bromine bị phá
vỡ. CH3O- là nucleophile, còn bromoethane là electrophile.

c) Một liên kết đôi được tạo thành giữa oxygen và carbon, và liên kết
carbon-chlorine bị phá vỡ. Các electron dịch chuyển từ oxygen để tạo
thành liên kết đôi và từ carbon đến chlorine.

187 | Chuẩn bị kiến thức cho kì thi HSGQG Hóa học


3)

4)

5)

188 | Chuẩn bị kiến thức cho kì thi HSGQG Hóa học


Chủ đề 14: Dự đoán sản phẩm phản ứng
1) Phản ứng dưới đây là một giai đoạn của chu trình citric (chuyển hóa
thực phẩm). Hãy dự đoán sản phẩm tạo thành dựa vào các dòng
electron (được chỉ ra bởi các mũi tên cong):

2) Dự đoán sản phẩm tạo thành trong các phản ứng sinh học sau:

189 | Chuẩn bị kiến thức cho kì thi HSGQG Hóa học


Hướng dẫn
1) Không cần quan tâm đến hóa lập thể của phản ứng:

2)

190 | Chuẩn bị kiến thức cho kì thi HSGQG Hóa học


Chủ đề 15: Giản đồ năng lượng phản ứng
1) Một phản ứng có năng lượng hoạt hóa là G‡ = +45 kJ/mol và phản
ứng thứ hai có G‡ = +70 kJ/mol. Hãy cho biết phản ứng nào diễn
ra nhanh hơn?
2) Hãy đề xuất giản đồ năng lượng cho phản ứng một-giai đoạn diễn
ra nhanh và tỏa nhiệt mạnh.
3) Hãy đề xuất giản đồ năng lượng cho phản ứng hai-giai đoạn tỏa
nhiệt, trong đó giai đoạn thứ hai có trạng thái chuyển tiếp với năng
lượng cao hơn của giai đoạn thứ nhất.
4) Hãy đề xuất giản đồ năng lượng cho phản ứng hai-giai đoạn trong
đó cả hai giai đoạn đều tỏa nhiệt và giai đoạn thứ hai có trạng thái
chuyển tiếp với năng lượng cao hơn của giai đoạn thứ nhất. Kí hiệu
các phần trên giản đồ tương ứng với chất phản ứng, sản phẩm, chất
trung gian, G‡ và Go toàn phản ứng.
5) Cho giản đồ dưới đây:

a) Năng lượng tự do Gibbs của toàn phản ứng âm hay dương? Hãy
chỉ rõ trên giản đồ.
b) Phản ứng có bao nhiêu trạng thái chuyển tiếp? Hãy chỉ rõ trên
giản đồ.

191 | Chuẩn bị kiến thức cho kì thi HSGQG Hóa học


6) Khi xử lí isopropylidenecyclohexane với acid mạnh ở nhiệt độ
thường thì xảy ra phản ứng đồng phân hóa theo cơ chế trình bày
dưới đây, tạo thành 1-isopropylcyclohexene:

Khi cân bằng, trong hỗn hợp phản ứng có khoảng 30%
isopropylidenecyclohexane và khoảng 70% 1-isopropylcyclo
hexene.
a) Hãy tính giá trị gần đúng của Keq.
b) Hãy phác thảo giản đồ năng lượng của phản ứng.
c) Bổ sung các mũi tên cơ chế để mô tả sự dịch chuyển electron
trong mỗi giai đoạn phản ứng.

192 | Chuẩn bị kiến thức cho kì thi HSGQG Hóa học


Hướng dẫn
1) Năng lượng hoạt hóa càng nhỏ thì phản ứng càng diễn ra nhanh.
2)

Phản ứng diễn ra nhanh thì có năng lượng hoạt hóa G‡ càng nhỏ, còn
“tỏa nhiệt mạnh” tương ứng với giá trị năng lượng tự do Gibbs Go rất
âm.
3)

193 | Chuẩn bị kiến thức cho kì thi HSGQG Hóa học


Phản ứng hai-giai đoạn thì sẽ có hai trạng thái chuyển tiếp và một
trung gian phản ứng ở giữa. Năng lượng hoạt hóa G‡ của toàn phản
ứng là biến thiên năng lượng giữa các chất phản ứng và trạng thái
chuyển tiếp có năng lượng cao nhất – trong trường hợp này là trạng
thái chuyển tiếp của giai đoạn thứ hai. Phản ứng xỏa nhiệt thì sẽ có Go
toàn phản ứng âm.
4)

5)

194 | Chuẩn bị kiến thức cho kì thi HSGQG Hóa học


6)
san pham 0.70
a) Keq = = = 2.3
chat phan ung 0.30

b)

c)

195 | Chuẩn bị kiến thức cho kì thi HSGQG Hóa học


Chương 5: Hydrocarbon và dẫn
xuất halide
Chủ đề 1: Alkane
1) Hãy biểu diễn các đồng phân của nhánh alkyl 5C.
2) Xác định bậc của mỗi nguyên tử carbon trong các alkane dưới đây.
Sử dụng kí hiệu p = bậc một; s = bậc hai; t = bậc ba; q = bậc bốn.

3) Biểu diễn cấu tạo của các alkane


a) có hai carbon bậc ba.
b) có một nhóm isopropyl.
c) có một carbon bậc bốn và một carbon bậc hai.
4) Xác định danh pháp IUPAC của

5) Biểu diễn cấu tạo và gọi tên các đồng phân C7H16.
6) Biểu diễn cấu tạo của các hợp chất sau:
a) 3,4-Dimethylnonane
b) 3-Ethyl-4,4-dimethylheptane
c) 2,2-Dimethyl-4-propyloctane
d) 2,2,4-Trimethylpentane

196 | Chuẩn bị kiến thức cho kì thi HSGQG Hóa học


7) Giải thích tại sao các tên gọi dưới đây không chính xác:
a) 2,2-Dimethyl-6-ethylheptane
b) 4-Ethyl-5,5-dimethylpentane
c) 3-Ethyl-4,4-dimethylhexane
d) 5,5,6-Trimethyloctane
e) 2-Isopropyl-4-methylheptane

197 | Chuẩn bị kiến thức cho kì thi HSGQG Hóa học


Hướng dẫn
1)

2)

3)

4)
(a)

198 | Chuẩn bị kiến thức cho kì thi HSGQG Hóa học


(b)

(c-d)

5)

199 | Chuẩn bị kiến thức cho kì thi HSGQG Hóa học


6)

7) Hãy phân tích từng tên gọi:


a) 2,2-Dimethyl-6-ethylheptane

Mạch dài nhất là octane và chỉ có các nhánh methyl, tên chính xác phải
là 2,2,6-trimethyloctane.
b) 4-Ethyl-5,5-dimethylpentane

Mạch dài nhất là hexane. Việc đánh số nên bắt đầu từ phía đối diện
của mạch carbon, gần với nhánh đầu tiên hơn. Tên chính xác là 4-
ethyl-3,3-dimethylhexane.
c) 3-Ethyl-4,4-dimethylhexane

200 | Chuẩn bị kiến thức cho kì thi HSGQG Hóa học


Việc đánh số nên bắt đầu từ phía đối diện của mạch carbon. Tên chính
xác là 4-ethyl-3,3-dimethylhexane.
d) 5,5,6-Trimethyloctane

Việc đánh số nên bắt đầu từ phía đối diện của mạch carbon. Tên chính
xác là 3,4,4-trimethyloctane.
e) 2-Isopropyl-4-methylheptane

Mạch dài nhất là octane. Tên chính xác là 2,3,5-trimethyloctane.

201 | Chuẩn bị kiến thức cho kì thi HSGQG Hóa học


Chủ đề 2: Cấu dạng alkane
1) Hãy phác họa một giản đồ năng lượng của các cấu dạng của
propane ở dạng hàm của góc quay.
2) Dọc theo trục liên kết C2-C1 của 2-methylpropane (isobutane) hãy
vẽ công thức chiếu Newman cho cấu dạng
1) bền nhất.
2) kém bền nhất.
3) Dọc theo trục liên kết C2-C1 của 2,3-dimethylbutane hãy vẽ công
thức chiếu Newman cho cấu dạng bền nhất.
4) Đề xuất một giản đồ [định tính] của thế năng cho sự quay quanh
liên kết C-C của 1,2-dibromoethane. Dự đoán cấu dạng nào bền
nhất và kém bền nhất. Chỉ rõ các cấu dạng đối lệch và bán lệch của
1,2-dibromoethane.
5) Biểu diễn cấu dạng bền nhất của:
a) pentane.
b) 1,4-dichlorobutane.
Sử dụng các kí hiệu đường nét đậm-nét rời để biểu diễn các liên kết
đi ra-đi vào mặt phẳng giấy.

202 | Chuẩn bị kiến thức cho kì thi HSGQG Hóa học


Hướng dẫn
1)

2) Cấu dạng bền nhất là xen kẽ, còn kém bền nhất là che khuất.

3) Cấu dạng dưới đây bền nhất bởi nó là dạng xen kẽ và có tương tác
bán lệch CH3  CH3 cực tiểu.

203 | Chuẩn bị kiến thức cho kì thi HSGQG Hóa học


4)

5)
a)

b)

204 | Chuẩn bị kiến thức cho kì thi HSGQG Hóa học


Chủ đề 3: Cycloalkane
1) Biểu diễn 5 đồng phân cycloalkane tương ứng với công thức C5H10.
2) Gọi tên các cycloalkane sau theo danh pháp IUPAC:

3) Xác định cấu tạo các hợp chất sau:


a) 1,1-Dimethylcyclooctane
b) 3-Cyclobutylhexane
c) 1,2-Dichlorocyclopentane
d) 1,3-Dibromo-5-methylcyclohexane
4) Gọi tên các hợp chất sau, gồm cả tiền tố cis hoặc trans phù hợp:

5) Biểu diễn cấu tạo của:


a) trans-1-Bromo-3-methylcyclohexane
b) cis-1,2-Dimethylcyclobutane
c) trans-1-tert-Butyl-2-ethylcyclohexane

205 | Chuẩn bị kiến thức cho kì thi HSGQG Hóa học


6) Prostaglandin F2α là hormone gây co bóp tử cung trong quá trình
sinh nở, có cấu trúc được dẫn ra dưới đây. Hai nhóm hydroxy (-OH)
của vòng cyclopentane có vị trí tương đối với nhau là dạng cis hay
trans? Vị trí tương đối của hai mạch carbon gắn vào vòng thì sao?

206 | Chuẩn bị kiến thức cho kì thi HSGQG Hóa học


Hướng dẫn
1) Hai cấu tạo cuối là các đồng phân trans và cis:

2)

3)

207 | Chuẩn bị kiến thức cho kì thi HSGQG Hóa học


4)

5)

6) Hai nhóm OH có vị trí cis; hai mạch carbon có vị trí trans.

208 | Chuẩn bị kiến thức cho kì thi HSGQG Hóa học


Chủ đề 4: Sức căng vòng
1) Mỗi tương tác che khuất H  H trong ethane tương ứng với năng
lượng khoảng 4.0 kJ/mol. Trong cyclopropane có bao nhiêu tương
tác như vậy? Tính tỉ lệ sức căng xoắn trong tổng năng lượng sức
căng toàn phần 115 kJ/mol (27.5 kJ/mol) của cyclopropane.
2) cis-1,2-dimethylcyclopropane có sức căng lớn hơn trans-1,2-
dimethylcyclopropane. Hãy giải thích cho sự khác biệt này. Đồng
phân nào bền hơn?
3) Tồn tại bao nhiêu tương tác che khuất H  H (4.0 kJ/mol) nếu
cyclopentane có cấu trúc phẳng? Ước lượng sức căng xoắn của
cyclopentane phẳng. Biết rằng sức căng toàn phần đo được của
cyclopentane là 26 kJ/mol, vậy sự gấp khúc đã làm giảm bớt bao
nhiêu phần sức căng xoắn? Cần lưu ý là trong trường hợp
cyclopentane thì sức căng góc (gây ra bởi sự ép hoặc mở rộng góc
liên kết) gần như không còn.
4) Dưới đây là hai cấu dạng của cis-1,3-dimethylcyclobutane. Hãy dự
đoán cấu dạng nào bền hơn?

209 | Chuẩn bị kiến thức cho kì thi HSGQG Hóa học


Hướng dẫn
1) Tất cả các nguyên tử hydrogen ở
cùng phía của vòng cyclopropane
đều bị che khuất bởi các hydrogen
bên cạnh. Theo đó, sẽ có 6 tương
tác che khuất H  H (xem hình, mỗi
phía có ba tương tác). Giả sử mỗi
tương tác này cũng tương tự như
trong ethane, đều tương ứng với 4.0
24
kJ/mol, thì tổng cộng 6 tương tác là 24.0 kJ/mol. Vậy  100%  21%
115
năng lượng sức căng toàn phần của cyclopropane là gây ra bởi sức
căng xoắn.
2) Nguyên nhân chính là bởi đồng phân cis có tương tác che khuất
CH3  CH3 - dẫn đến đồng phân này có sức căng lớn hơn và kém bền
hơn đồng phân trans.

3) Nếu cyclopentane có cấu trúc


phẳng thì sẽ có 10 tương tác che
khuất hydrogen-hydrogen, với tổng
năng lượng là 10  4.0 = 40 kJ/mol.
Sức căng vòng đo được là 26 kJ/mol
– cho thấy có 40 – 26 = 14 kJ/mol
14
(tương ứng với  100% = 35% ) sức
40
căng che khuất của cyclopentane đã được giảm thiểu nhờ sự gấp khúc.
4) Cấu dạng 1 bền hơn bởi hai nhóm methyl ở xa nhau hơn.

210 | Chuẩn bị kiến thức cho kì thi HSGQG Hóa học


Chủ đề 5: Độ bền cấu dạng
1) Vẽ hai cấu dạng ghế khác nhau của cyclohexanol
(hydroxycyclohexane) và biểu diễn tất cả các nguyên tử hydrogen.
Chỉ rõ vị trí của chúng là trục (a) hay biên (e).
2) Vẽ hai cấu dạng ghế khác nhau của trans-1,4-dimethylcyclohexane
và chỉ rõ các vị trí trục (a) và biên (e).
3) Tại sao sự xuất hiện một nhóm thế cyano (-CN) ở vị trí trục lại gần
như không gây ra sức căng không gian 1,3-diaxial (chỉ khoảng 0.4
kJ/mol)?
4)
a) Galactose là một loại đường tương tự glucose, có chứa vòng 6
cạnh với tất cả các nhóm thế, ngoại trừ nhóm OH được chỉ ra
bên dưới, đều ở vị trí biên. Hãy biểu diễn cấu dạng ghế bền nhất
của galactose.

b) Biểu diễn hai cấu dạng ghế của menthol và chỉ ra cấu dạng bền
hơn. Giải thích tại sao.

5) Tại sao đồng phân cyclohexane hai nhóm thế 1,3-cis lại bền hơn
đồng phân 1,3-trans?
6) Đồng phân cyclohexane hai nhóm thế 1,3-cis hay 1,3-trans bền
hơn?
7) cis-1,2-Dimethylcyclobutane thì kém bền hơn đồng phân trans của
nó, nhưng cis-1,3-dimethylcyclobutane thì lại bền hơn đồng phân
trans tương ứng. Hãy vẽ các cấu dạng bền nhất và giải thích cho
hiện tượng này.

211 | Chuẩn bị kiến thức cho kì thi HSGQG Hóa học


8)
a) Đồng phân nào, cis-decalin hay trans-decalin, bền hơn?

b) Giải thích tại sao cis-bicyclo[4.1.0]heptane lại bền hơn đồng


phân trans tương ứng.

c) Amantadine là chất kháng virus, có hoạt tính chống bệnh cúm A.


Hãy biểu diễn cấu trúc ba chiều của amantadine, trong đó tất cả
các vòng cyclohexane đều ở cấu dạng ghế.

9) Khi xử lí với CrO3, các alcohol bị oxid hóa tạo thành các hợp chất
carbonyl. Ví dụ, 2-tert-butylcyclohexanol tạo thành 2-tert-
butylcyclohexanone. Biết các nhóm -OH ở vị trí trục thường có hoạt
tính mạnh hơn so với đồng phân ở vị trí biên, hãy dự đoán xem đồng
phân nào (cis hay trans) của 2-tert-butylcyclohexanol sẽ bị oxid
hóa nhanh hơn? Giải thích tại sao.

212 | Chuẩn bị kiến thức cho kì thi HSGQG Hóa học


Hướng dẫn
1)

2)

3) Có sự khác biệt rất nhỏ về mặt năng lượng giữa nhóm cyano ở vị trí
trục và vị trí biên, bởi nhóm này có cấu trúc thẳng, chiếm vị trí không
gian rất nhỏ và gần như không tạo ra tương tác không gian 1,3-diaxial.

213 | Chuẩn bị kiến thức cho kì thi HSGQG Hóa học


4) a)

b) Từ hình vẽ vòng phẳng, bạn có thể thấy rằng nhóm methyl và nhóm
OH có vị trí tương đối là cis, còn nhóm isopropyl có vị trí tương đối là
trans với cả hai nhóm này. Hãy biểu diễn vòng cyclohexane dạng ghế
rồi gắn các nhóm thế vào ứng với mối liên hệ vị trí phù hợp:

Trong cấu dạng này, tất cả các nhóm thế đều ở vị trí biên. Bây giờ, hãy
thực hiện thao tác lật vòng:

Cấu dạng thứ hai snày rõ ràng kém bền hơn bởi tất cả các nhóm thế
đều ở vị trí trục.
5) Đồng phân cis-1,3 gần như tồn tại ở cấu dạng lưỡng biên
(diequatorial), không có tương tác 1,3-diaxial. Đồng phân trans thì phải
có một nhóm ở vị trí trục, dẫn đến sự tồn tại các tương tác 1,3-diaxial.
Vậy nên, đồng phân trans kém bền hơn đồng phân cis. Khi một phân tử

214 | Chuẩn bị kiến thức cho kì thi HSGQG Hóa học


có hai cấu dạng khả dĩ thì nó sẽ tồn tại chủ yếu ở cấu dạng có năng
lượng thấp hơn.

6) Đồng phân trans-1,4 bền hơn bởi cả hai nhóm thế đều ở vị trí biên.

7) Trong cis-1,2-dimethylcyclobutane tồn tại hai kiểu liên kết: một


kiểu xuất hiện giữa hai nhóm methyl – gần như bị che khuất. Kiểu còn
lại là tương tác giữa nhóm methyl ở vị trí 1 trong vòng và hydrogen ở vị
trí 3 (xem hình dưới). Trong đồng phân trans thì không có những kiểu
tương tác này nên nó bền hơn đồng phân cis.

Trong trans-1,3-dimethylcyclobutane, tồn tại tương tác giữa nhóm


methyl ở vị trí 3 của vòng và hydrogen ở vị trí 1 (xem hình dưới). Còn

215 | Chuẩn bị kiến thức cho kì thi HSGQG Hóa học


trong đồng phân cis thì không có tương tác như vậy, do đó nó bền hơn
đồng phân trans.

8) a) Trans-decalin bền hơn cis-


decalin bởi có ba tương tác 1,3-
diaxial hiện diện trong đồng phân
cis. Bạn có thể nhận ra các tương
tác này nếu bạn hình dung các
phần được khoanh tròn của cis-
decalin tương tự như các nhóm
methyl ở vị trí trục. Các tương tác
bán lệch xuất hiện với các nhóm
methyl axial cũng tồn tại trong
đồng phân cis.
b) Trong cả hai trường hợp, hệ ngưng tụ giữa vòng sáu cạnh với vòng
ba cạnh làm biến dạng các góc liên kết của cả hai vòng, gây ra sức căng
góc. Trong trường hợp đồng phân trans, sức căng vòng lớn hơn so với
đồng phân cis – dẫn đến đồng phân cis bền hơn.

216 | Chuẩn bị kiến thức cho kì thi HSGQG Hóa học


c)

10) Dưới đây là các cấu dạng của hai đồng phân. Cấu dạng thứ hai của
mỗi cặp đều có sức căng không gian lớn – do mỗi đồng phân chủ
yếu tồn tại theo cấu dạng thứ nhất. Do chỉ đồng phân cis có nhóm
hydroxyl ở vị trí trục, nên nó sẽ bị oxid hóa nhanh hơn đồng phân
trans.

217 | Chuẩn bị kiến thức cho kì thi HSGQG Hóa học


Chủ đề 6: Độ hơn cấp và tâm thủ tính
1) Sắp xếp các nhóm thế trong mỗi trường hợp sau theo trình tự giảm
dần độ hơn cấp:
a) -H, -OH, -CH2CH3, -CH2CH2OH
b) -COOH, -COOCH3, -CH2OH, -OH
c) -CN, -CH2NH2, -CH2NHCH3, -NH2
d) -SH, -CH2SCH3, -CH3, -SSCH3
2) Sắp xếp các nhóm thế trong mỗi trường hợp sau theo trình tự giảm
dần độ hơn cấp:

3) Hãy cho biết trong mỗi trường hợp dưới đây, nguyên tử carbon
trung tâm có cấu hình R hay S? Cho biết các giá trị (1) – (4) để chỉ độ
hơn cấp của các nhóm thế (1 – lớn nhất; 4 – nhỏ nhất).

4) Xác định cấu hình tuyệt đối (R/S) của mỗi tâm thủ tính trong các
hợp chất sau:

218 | Chuẩn bị kiến thức cho kì thi HSGQG Hóa học


5) Biểu diễn cấu trúc tứ diện của:
a) (S)-2-pentanol.
b) (S)-2-Chlorobutane
c) (R)-3-Chloro-1-pentene [H2CPCHCH(Cl)CH2CH3]
6) Xác định cấu hình tuyệt đối (R/S) của mỗi tâm thủ tính trong các
hợp chất sau:

7) Xác định cấu hình tuyệt đối (R/S) của mỗi tâm thủ tính trong các
hợp chất sau:
a) b)

c)

d)

219 | Chuẩn bị kiến thức cho kì thi HSGQG Hóa học


8) Xác định cấu hình tuyệt đối (R/S) của mỗi tâm thủ tính trong các
hợp chất có công thức chiếu Newman sau:

9) Xylose là chất đường phổ biến, được tìm thấy trong nhiều loại gỗ,
bao gồm cả gỗ thích và gỗ anh đào. Bởi hoạt tính ít gây sâu răng hơn
so với sucrose nên xylose được sử dụng trong nhiều loại kẹo ngọt
và kẹo cao su. Hãy xác định cấu hình tuyệt đối (R/S) cho các tâm
thủ tính trong xylose.

10) Một trong các phân tử dưới đây, (a – d), là D-erythrose 4-phosphate
– trung gian trong chu trình quang hợp Calvin mà thực vật biến
lượng CO2 hấp thu được thành các phân tử carbohydrate. Biết rằng
cả hai tâm thủ tính của D-erythrose 4-phosphate đều có cấu hình
R, hãy cho biết:
a) Cấu trúc của D-erythrose 4-phosphate.
b) Trong ba cấu trúc còn lại, đâu là đối quang của D-erythrose 4-
phosphate và đâu là các xuyên lập thể phân (đồng phân dia)?

220 | Chuẩn bị kiến thức cho kì thi HSGQG Hóa học


Hướng dẫn
1) a) -OH > -CH2CH2OH > -CH2CH3 > -H
b) -OH > -COOCH3 > -COOH > -CH2OH
c) -NH2 > -CN > -CH2NHCH3 > -CH2NH2
d) -SSCH3 > -SH > -CH2SCH3 > -CH3
2)

3)

221 | Chuẩn bị kiến thức cho kì thi HSGQG Hóa học


4)

5) a) Hãy sắp xếp các nhóm thế quanh nguyên tử carbon thủ tính theo
trình tự giảm dần độ hơn cấp:

-OH > -CH2CH2CH3 > -CH3 > -H


Biểu diễn cấu trúc tứ diện của (S)-2-pentanol:

b)

222 | Chuẩn bị kiến thức cho kì thi HSGQG Hóa học


c)

6)

7)

(c)

(d)

223 | Chuẩn bị kiến thức cho kì thi HSGQG Hóa học


8)

9)

10) Kí hiệu nhóm phosphate là P. Ta sẽ xác định cấu hình của mỗi tâm
thủ tính trong từng cấu trúc a – d. Dưới đây là ví dụ về cấu trúc a:

Vậy a là D-erythrose 4-phosphat và d là đối quang của nó; còn các cấu
trúc b và c là các xuyên lập thể phân.

224 | Chuẩn bị kiến thức cho kì thi HSGQG Hóa học


Chủ đề 7: Đồng phân meso
1) Morphine là một loại thuốc giảm đau thuộc họ thuốc phiện được
tìm thấy tự nhiên trong một số thực vật và động vật, bao gồm cả
con người. Morphine tác động trực tiếp lên hệ thần kinh trung ương
để giảm cảm giác đau.

a) Morphine có bao nhiêu tâm thủ tính?


b) Về mặt lí thuyết, morphine có thể có bao nhiêu đồng phân lập
thể?
2) Những cấu trúc nào sau đây là hợp chất meso (nội tiêu triền)?

3) Vẽ các dạng meso của các phân tử sau và chỉ rõ mặt phẳng đối xứng
trong mỗi trường hợp:

4) Hợp chất nào dưới đây có đồng phân meso?


a) 2,3-Butanediol
b) 2,3-Pentanediol
c) 2,4-Pentanediol

225 | Chuẩn bị kiến thức cho kì thi HSGQG Hóa học


5) Ribose là thành phần quan trọng của ribonucleic acid (RNA). Khi
phản ứng với khí hydrogen có xúc tác platinum, ribose chuyển hóa
thành ribitol.
a) Vẽ cấu trúc đối quang của ribose.
b) Hãy cho biết ribitol có tính quang hoạt hay không? Giải thích.

6) Vẽ tất cả các đồng phân lập thể có thể có của:


a) 1,2-cyclobutanedicarboxylic acid
b) 1,3-cyclobutanedicarboxylic acid
Trong mỗi trường hợp, hãy chỉ ra mối liên hệ giữa các đồng phân và
cho biết đồng phân nào có tính quang hoạt.

226 | Chuẩn bị kiến thức cho kì thi HSGQG Hóa học


Hướng dẫn
1) Morphine có năm tâm thủ tính, nên về mặt lí thuyết, có thể có 25 = 32
đồng phân lập thể. Tuy nhiên, đa số các đồng phân này không tồn tại
bởi sức căng quá lớn.

2) Để xác định một cấu trúc có thể hiện tính chất nội tiêu triền (tức có
phải đồng phân meso) hay không, hãy thử xác định xem liệu có tồn tại
một mặt phẳng đối xứng, chia phân tử thành hai nửa là ảnh gương.

Tương tự với cấu trúc d:

Còn các cấu trúc b, c không phải đồng phân meso.

227 | Chuẩn bị kiến thức cho kì thi HSGQG Hóa học


3)

4) a) Butanediol có thể tồn tại ở dạng một cặp đối quang hoặc một
đồng phân meso, tùy thuộc vào cấu hình của các nguyên tử carbon 2
và 3.

b) 2,3-Pentanediol không có mặt phẳng đối xứng nên không có đồng


phân meso.
c) 2,4-Pentanediol có thể tồn tại dạng meso:

228 | Chuẩn bị kiến thức cho kì thi HSGQG Hóa học


5)
a)

b) Phản ứng hydrogen hóa xúc tác đã chuyển nhóm chức aldehyde
thành nhóm chức hydroxyl, làm cho cấu trúc của ribitol xuất hiện một
mặt phẳng đối xứng, chia phân tử thành hai nửa là ảnh gương của nhau.
Ribitol là một hợp chất meso (hợp chất nội tiêu triền) và không có tính
quang hoạt.

6)
a) B và C là các đối quang và có tính quang hoạt. A là xuyên lập thể
phân của chúng và là hợp chất meso, không có tính quang hoạt.

b) Hai đồng phân này đều không có tính thủ tính và không quang hoạt.

229 | Chuẩn bị kiến thức cho kì thi HSGQG Hóa học


Chủ đề 8: Hóa lập thể của sản phẩm phản ứng
1) Biết rằng acetic acid (CH3COOH) phản ứng được với (S)-2-butanol
tạo thành một ester. Hãy dự đoán hóa lập thể của sản phẩm tạo
thành.

2) Hãy dự đoán hóa lập thể của các sản phẩm tạo thành từ phản ứng
của (±)-lactic acid với (S)-1-phenylethylamine.

3) Amino acid tồn tại trong tự nhiên cysteine có một tâm thủ tính với
cấu hình S. Khi xử lí với tác nhân oxid hóa êm dịu, hai phân tử
cysteine có thể nối với nhau, tạo thành cystine – một disulfide. Giả
sử rằng tâm thủ tính không bị ảnh hưởng bởi phản ứng, hãy cho biết
cysteine có tính quang hoạt hay không? Giải thích.

230 | Chuẩn bị kiến thức cho kì thi HSGQG Hóa học


Hướng dẫn
1) Sản phẩm tạo thành là S-ester tinh khiết. Không có tâm thủ tính nào
mới được tạo thành trong quá trình phản ứng và cấu hình của tâm thủ
tính trong (S)-2-butanol không thay đổi.

2) Hai muối tạo thành là các xuyên lập thể phân.

3) Cystine có cấu hình (S, S) và có tính quang hoạt:

231 | Chuẩn bị kiến thức cho kì thi HSGQG Hóa học


Chủ đề 9: Tính chất tiền-thủ tính
1) Xác định các nguyên tử hydrogen được chỉ ra trong mỗi phân tử
dưới đây là dạng pro-S hay pro-R?

2) Xác định các nguyên tử hydrogen được chỉ ra trong mỗi phân tử
dưới đây là dạng pro-S hay pro-R?

3) Hãy cho biết các mặt được chỉ ra dưới đây của các nguyên tử
carbon là mặt Re hay Si?

4) Lactic acid được tạo ra trong cơ bắp và tích tụ trong quá trình tập
luyện thể dục thể thao có nguồn gốc từ pyruvate. Biết phản ứng
diễn ra với sự cộng hợp hydrogen vào mặt Re của pyruvate. Hãy xác
định hóa lập thể của sản phẩm.

232 | Chuẩn bị kiến thức cho kì thi HSGQG Hóa học


5) Phản ứng cộng hợp nước vào cis-aconitate được xúc tác bởi
aconitase trong chu trình citric acid diễn ra với hóa lập thể như sau.
Hãy cho biết sự cộng hợp nhóm OH diễn ra trên mặt Re hay Si của
chất nền? Hydrogen và nhóm OH được cộng hợp vào cùng phía của
liên kết đôi hay khác phía?

6) Một trong các giai đoạn của chuyển hóa chất béo là phản ứng hợp
nước (hydrate hóa) crotonate, tạo thành 3-hydroxyburyrate. Phản
ứng diễn ra với sự cộng nhóm -OH vào mặt Si ở C3, sau đó proton
hóa ở C2, cũng ở mặt Si. Hãy vẽ cấu trúc sản phẩm phản ứng và
trình bày hóa lập thể của mỗi bước.

7) Ketone phản ứng được với sodium acetylide (muối sodium của
acetylene, Na+ −C  CH ), tạo thành các alcohol. Ví dụ như phản ứng
của sodium acetylide với 2-butanone tạo thành 3-methyl-1-
pentyn-3-ol:

233 | Chuẩn bị kiến thức cho kì thi HSGQG Hóa học


a) Giả sử rằng phản ứng diễn ra với khả năng (xác suất) bằng nhau
ở cả hai mặt, Re và Si, của nhóm carbonyl. Hãy dự đoán sản
phẩm có quang hoạt hay không? Giải thích.
b) Nếu phản ứng tương tự được tiến hành giữa sodium acetylide
và (R)-2-phenylpropanal thì tạo thành 4-phenyl-1-pentyn-3-ol.
Giả sử rằng phản ứng ưu tiên diễn ra ở mặt Re của nhóm
carbonyl, hãy xác định sản phẩm chính và sản phẩm phụ của
phản ứng. Hỗn hợp tạo thành có tính quang hoạt hay không?

234 | Chuẩn bị kiến thức cho kì thi HSGQG Hóa học


Hướng dẫn
1) Với mỗi phân tử, hãy thay thế hydrogen bên trái bởi deuterium (2H).
Sau đó, xác định độ hơn cấp của các nhóm thế rồi gán cấu hình R, S
phù hợp cho các tâm thủ tính. Nếu cấu hình là R, thì hydrogen bị thay
thế là pro-R; tương tự với cấu hình S.

2)

3) Hãy vẽ mặt phẳng chứa carbon sp2 và các nhóm thế của nó, rồi sắp
xếp các nhóm thế theo độ hơn cấp. Với mặt phía trên, vẽ mũi tên đi từ
nhóm có độ hơn cấp cao nhất đến các nhóm thấp hơn ( 1 → 2 → 3 ).
Nếu chiều quay theo chiều kim đồng hồ, thì đó là mặt Re; ngược lại, thì
đó là mặt Si.

235 | Chuẩn bị kiến thức cho kì thi HSGQG Hóa học


4)

5) Sự cộng -OH diễn ra ở mặt Re của C2 trong aconitate. Sự cộng -H


diễn ra ở mặt Re của C3 tạo thành (2R,3S)-isocitrate. Sự cộng H và OH
diễn ra ở hai mặt ngược nhau của liên kết đôi.

236 | Chuẩn bị kiến thức cho kì thi HSGQG Hóa học


6)

7) a) Phản ứng diễn ra với sự cộng anion acetylide vào nhóm carbonyl
với xác suất diễn ra ở cả hai phía của carbon ketone phẳng bằng nhau:

Sản phẩm tạo thành là hỗn hợp đối quang với tỉ lệ 1:1 (hỗn hợp racemic)
và không quang hoạt.
b) Sản phẩm tạo thành là hỗn hợp các xuyên lập thể phân (3R,4R) và
(3S,4R) của 4-phenyl-1-pentyn-3-ol. Không thể dự đoán tỉ lệ sản phẩm
nhưng chắc chắn là khác 50:50. Hỗn hợp sản phẩm có tính quang hoạt.

237 | Chuẩn bị kiến thức cho kì thi HSGQG Hóa học


Chủ đề 10: Các hợp chất không có tâm thủ tính
1) Các allene là những hợp chất với các liên kết đôi carbon-carbon
liền kề. Nhiều allene có tính thủ tính, mặc dù chúng không có bất kì
tâm thủ tính nào. Ví dụ như mycomycin, một kháng sinh tự nhiên,
được phân lập từ vi khuẩn Nocardia acidophilus có tính thủ tính với
   = −130. Hãy giải thích cho tính thủ tính của mycomycin.
D

2) Thí nghiệm phân giải 4-methylcyclohexylideneacetic acid thành


hai đối quang đã được tiến hành từ trước khi các allene thủ tính
được biết đến. Hãy cho biết tại sao hợp chất này có tính thủ tính và
dạng hình học của nó có điểm gì tương đồng với các allene?

3) (S)-1-Chloro-2-methylbutane có phản ứng với Cl2 khi chiếu sáng,


tạo thành hỗn hợp các sản phẩm, trong đó có chứa 1,4-dichloro-2-
methylbutane và 1,2-dichloro-2-methylbutane.
a) Viết phương trình phản ứng, chỉ rõ hóa lập thể của chất phản
ứng.
b) Một trong hai sản phẩm kể trên có tính quang hoạt, nhưng sản
phẩm còn lại thì không. Hãy cho biết đâu là sản phẩm có tính
quang hoạt.

238 | Chuẩn bị kiến thức cho kì thi HSGQG Hóa học


Hướng dẫn
1) Mycomycin không chứa carbon thủ tính nhưng vẫn có tính quang
hoạt. Để hiểu tại sao, trước tiên hãy biểu diễn cấu trúc không gian của
hợp chất này. Để đơn giản hóa, hợp phần −CH = CHCH = CHCH2COOH
và −C  CC  CH lần lượt được kí hiệu là A và B. Các nguyên tử carbon
của allene có mối liên hệ thẳng hàng và các liên kết pi được tạo thành
vuông góc với nhau. Hãy gắn các nhóm thế vào các carbon sp2.

Chú ý rằng các nhóm thế A, Ha và tất cả các nguyên tử carbon nằm trên
một mặt phẳng vuông góc với mặt phẳng chứa B, Hb và tất cả các
nguyên tử carbon. Bây giờ, hãy thử biểu diễn một cấu trúc giống như
cấu trúc đầu tiên, ngoại trừ việc thay đổi vị trí A và Ha. Allene mới này
không thể chồng khít vào allene ban đầu – do đó, hai allene (cũ và mới)
này là các đối quang và có tính thủ tính bởi chúng không có mặt phẳng
đối xứng.

2) Nguyên nhân gây ra tính thủ tính của 4-methylcyclo


hexylideneacetic acid cũng tương tự như mycomycin: Nó không có
mặt phẳng đối xứng và không thể chồng khít với ảnh gương của mình.
Cũng tương tự như trường hợp allene, hai nhóm ở một đầu của phân tử
nằm trên mặt phẳng vuông góc với mặt phẳng chứa hai nhóm của đầu
kia.

239 | Chuẩn bị kiến thức cho kì thi HSGQG Hóa học


3) a)

b) Phản ứng chlor hóa ở carbon C4 tạo thành sản phẩm quang hoạt bởi
tâm thủ tính ở C2 không bị ảnh hưởng. Còn phản ứng chlor hóa ở carbon
C2 thì tạo thành sản phẩm racemic không quang hoạt.

240 | Chuẩn bị kiến thức cho kì thi HSGQG Hóa học


Chủ đề 11: Danh pháp alkene và cycloalkene
1) Xác định danh pháp IUPAC của các alkene sau:

2) Biểu diễn cấu tạo của các alkene sau:


a) 2-Methyl-1,5-hexadiene
b) 3-Ethyl-2,2-dimethyl-3-heptene
c) 2,3,3-Trimethyl-1,4,6-octatriene
d) 3,4-Diisopropyl-2,5-dimethyl-3-hexene
3) Xác định danh pháp IUPAC của các cycloalkene sau:

4) Hợp chất nào sau đây tồn tại đồng phân cis-trans; nếu có, hãy biểu
diễn cấu trúc mỗi cặp và chỉ ra cấu hình tương ứng:

5) Biểu diễn cấu trúc của cis-4,5-dimethyl-2-hexene và trans-6-


methyl-3-heptene.

241 | Chuẩn bị kiến thức cho kì thi HSGQG Hóa học


6) Sử dụng quy tắc Cahn–Ingold–Prelog, sắp xếp các nhóm thế sau
theo trình tự giảm dần độ hơn cấp

7) Gán cấu hình E-Z cho các alkene sau:

8) Biểu diễn cấu trúc của:


a) (4E)-2,4-Dimethyl-1,4-hexadiene
b) cis-3,3-Dimethyl-4-propyl-1,5-octadiene
c) 4-Methyl-1,2-pentadiene
d) (3E,5Z)-2,6-Dimethyl-1,3,5,7-octatetraene
e) 3-Butyl-2-heptene
f) trans-2,2,5,5-Tetramethyl-3-hexene
9) Fucoserratene và ectocarpene
là các pheromone giới tính do
tảo nâu biển tạo ra. Hãy xác
định tên gọi hệ thống của
chúng.

242 | Chuẩn bị kiến thức cho kì thi HSGQG Hóa học


Hướng dẫn
1)

2)

3)

243 | Chuẩn bị kiến thức cho kì thi HSGQG Hóa học


4) Các hợp chất (c), (d), (f) có thể tồn tại ở dạng đồng phân cis-trans:

5)

6)

7) a) Z; b) E; c) Z; d) E.

244 | Chuẩn bị kiến thức cho kì thi HSGQG Hóa học


8)

9)

245 | Chuẩn bị kiến thức cho kì thi HSGQG Hóa học


Chủ đề 12: Độ bền của alkene
1) Cho biết hợp chất nào bền hơn trong mỗi cặp sau đây:

2) Giải thích tại sao:


a) trans-2-butene bền hơn cis-2-butene chỉ khoảng 4 kJ/mol
nhưng trans-2,2,5,5-tetramethyl-3-hexene thì bền hơn đồng
phân cis của nó khoảng 39 kJ/mol.
b) Thường thì trans alkene bền hơn đồng phân cis, nhưng trans-
cyclooctene lại kém bền hơn cis-cyclooctene 38.5 kJ/mol.
c) Cyclodecene có thể tồn tại ở dạng cis và trans, nhưng
cyclohexene thì không.

246 | Chuẩn bị kiến thức cho kì thi HSGQG Hóa học


Hướng dẫn
1)

2) a) Trong cả hai trường hợp, đồng phân trans đều bền hơn đồng phân
cis. Chênh lệch về độ bền đồng phân cis-trans trở nên rõ ràng hơn
nhiều trong hợp chất tetramethyl. Hãy thử biểu diễn cấu tạo của cis-
2,2,5,5-tetramethyl-3-hexene, bạn sẽ nhận thấy sức căng không gian
lớn gây ra bởi các nhóm methyl.

247 | Chuẩn bị kiến thức cho kì thi HSGQG Hóa học


b) Trong trường hợp trans-cyclooctene thì sức căng xoắn làm cho
phân tử trở nên kém bền hơn.

c) Trong trường hợp cyclohexene, vòng sáu cạnh quá nhỏ để có thể
chứa một liên kết đôi với cấu hình trans mà không gây ra sức căng vòng
lớn.

Các hệ vòng lớn, ví dụ như vòng mười cạnh của cyclodecene, thì đủ linh
hoạt để đáp ứng cấu hình cis hoặc trans của liên kết đôi; dù rằng đồng
phân cis thường bền hơn đồng phân trans.

248 | Chuẩn bị kiến thức cho kì thi HSGQG Hóa học


Chủ đề 13: Phản ứng cộng electrophile vào alkene
1) Dự đoán cấu trúc và gọi tên sản phẩm chính tạo thành từ các phản
ứng:

2) Đề xuất alkene có thể sử dụng để tổng hợp các dẫn xuất sau:

3) Đề xuất cấu trúc của carbocation trung gian bền hơn được tạo
thành trong mỗi phản ứng sau:

4) Khi xử lí với HBr, vinylcyclohexane xảy ra phản ứng cộng hợp và


chuyển vị, tạo thành 1-bromo-1-ethylcyclohexane. Sử dụng các
mũi tên cơ chết, hãy đề xuất cơ chế để giải thích kết quả này.

249 | Chuẩn bị kiến thức cho kì thi HSGQG Hóa học


5) Dự đoán sản phẩm chính của phản ứng cộng HBr vào các alkene
sau:

6) Mỗi carbocation sau có thể chuyển vị tạo thành ion bền hơn. Hãy
đề xuất sản phẩm của mỗi quá trình chuyển vị:

7) Phản ứng cộng HCl và 1-isopropenyl-1-methylcyclopentane tạo


thành 1-chloro-1,2,2-trimethylcyclohexane. Hãy đề xuất cơ chế
phản ứng.

8) Vinylcyclopropane phản ứng với HBr tạo thành một alkyl bromide.
Dựa vào các dòng electron được biểu diễn bởi các mũi tên cong sau
đây, hãy biểu diễn cấu trúc của carbocation trung gian (trong dấu
ngoặc vuông) và sản phẩm cuối.

250 | Chuẩn bị kiến thức cho kì thi HSGQG Hóa học


9) Limonene, là một hydrocarbon có mùi thơm, được tìm thấy trong
chanh và cam, được sinh tổng hợp từ geranyl diphosphate theo
hướng như dưới đây. Hãy bổ sung các mũi tên cong để biểu diễn cơ
chế của mỗi giai đoạn. Giai đoạn nào là phản ứng cộng electrophile
vào alkene. Cho biết ion OP2O64- là diphosphate ion; còn “Base” là kí
hiệu tác nhân có tính base chưa xác định trong enzyme, đóng vai
trò xúc tác cho phản ứng.

10) epi-Aristolochene, một hydrocarbon được tìm thấy trong cả hạt


tiêu và thuốc lá, được sinh tổng hợp theo hướng như dưới đây. Hãy
bổ sung các mũi tên cong để biểu diễn cơ chế của mỗi giai đoạn

251 | Chuẩn bị kiến thức cho kì thi HSGQG Hóa học


11) Iodine azide, IN3, cộng hợp vào alkene theo cơ chế electrophile
tương tự như bromine. Nếu phản ứng với alkene một nhóm thế, ví
dụ như but-1-ene, thì chỉ tạo thành một sản phẩm:

a) Biểu diễn các dạng cộng hưởng của iodine azide.


b) Chỉ ra sự phân cực của liên kết I-N3.
c) Đề xuất cơ chế phản ứng.
12) Đề xuất cơ chế phản ứng:

252 | Chuẩn bị kiến thức cho kì thi HSGQG Hóa học


Hướng dẫn
1)

2)

253 | Chuẩn bị kiến thức cho kì thi HSGQG Hóa học


3)

4) Phản ứng diễn ra qua các giai đoạn sau:


▪ Giai đoạn 1: Cộng electrophile H+ vào liên kết đôi.
▪ Giai đoạn 2: Chuyển dịch hydride, tạo thành carbocation bậc
ba bền hơn.
▪ Giai đoạn 3: Phản ứng của carbocation với Br-.

254 | Chuẩn bị kiến thức cho kì thi HSGQG Hóa học


5)

Ở ý (c), sản phẩm cuối cùng là kết quả của quá trình chuyển vị tạo thành
carbocation bậc ba bền hơn.
6) a) Carbocation bậc một chuyển vị thành carbocation bậc hai bền
hơn:

b) Carbocation bậc hai chuyển vị thành carbocation bậc ba bền hơn:

c) Vòng bốn cạnh chuyển dịch alkyl, mở rộng thành vòng bền hơn.

255 | Chuẩn bị kiến thức cho kì thi HSGQG Hóa học


7)

8) Các electron pi của liên kết đôi tấn công vào H+ tạo thành
carbocation dưới đây:

Sau đó, diễn ra sự chuyển dịch liên kết (chuyển dịch alkyl), tạo thành
trung gian như trong dấu ngoặc vuông, tiểu phân này sau đó phản ứng
với Br- tạo thành 1-bromo-2-methylcyclobutane.

9)

Giai đoạn thứ hai, trong đó các electron của liên kết đôi tấn công vào
liên kết đôi còn lại, là phản ứng cộng electrophile vào alkene.

256 | Chuẩn bị kiến thức cho kì thi HSGQG Hóa học


10)

11)
a)

b)

c)

257 | Chuẩn bị kiến thức cho kì thi HSGQG Hóa học


12)

258 | Chuẩn bị kiến thức cho kì thi HSGQG Hóa học


Chủ đề 14: Phản ứng cộng electrophile
1) Dự đoán sản phẩm tạo thành từ phản ứng cộng chlorine vào 1,2-
dimethylcyclohexene? Biểu diễn hóa lập thể của sản phẩm.
2) Biểu diễn cấu trúc (chỉ rõ hóa lập thể) của hỗn hợp sản phẩm tạo
thành từ phản ứng cộng HCl vào 1,2-dimethylcyclohexene.
3) Dự đoán sản phẩm phản ứng oxymercury hóa–demercury hóa các
alkene sau:

4) Dự đoán sản phẩm được tạo thành từ phản ứng hdyrate hóa xúc tác
acid của 4-methylcyclopentene?
5) 10-Bromo-a-chamigrene là hợp chất được phân lập từ tảo biển.
Các nhà nghiên cứu cho rằng chất này được sinh tổng hợp từ γ-
bisabolene theo chuyển hóa như dưới đây. Hãy vẽ cấu trúc của
trung gian bromoium và carbocation vòng. Đề xuất cơ chế cho cả
ba giai đoạn.

6) Phản ứng của HBr với 3-methylcyclohexene tạo thành hỗn hợp bốn
sản phẩm: cis- và trans-1-bromo-3-methylcyclohexane cùng với
cis- và trans-1-bromo-2-methylcyclohexane. Phản ứng tương tự
của HBr với 3-bromocyclohexene tạo thành trans-1,2-
dibromocyclohexane là sản phẩm duy nhất. Hãy biểu diễn cấu trúc
của các trung gian khả dĩ, và giải thích tại sao trong phản ứng của
3-bromocyclohexene chỉ tạo thành một sản phẩm duy nhất?

259 | Chuẩn bị kiến thức cho kì thi HSGQG Hóa học


7)
a) Phản ứng của cyclohexene với mercury(II) acetate trong CH3OH
thay vì H2O, sau đó xử lí với NaBH4, sẽ tạo thành cyclohexyl
methyl ether, thay vì cyclohexanol. Hãy đề xuất cơ chế phản
ứng.

b) Đề xuất cơ chế cho phản ứng dưới đây:

260 | Chuẩn bị kiến thức cho kì thi HSGQG Hóa học


8) Khi xử lí 4-penten-1-ol với dung dịch Br2 thì tạo thành một bromo
ether vòng, thay vì bromohydrin. Hãy đề xuất cơ chế phản ứng:

9) Phản ứng cộng hợp HBr vào liên kết đôi có nhóm thế ether (-OR)
diễn ra với sự chọn lọc vị trí tạo thành sản phẩm trong đó -Br và -
OR liên kết với cùng carbon. Hãy biểu diễn cấu trúc của hai trung
gian carbocation khả dĩ trong phản ứng cộng electrophile này và
giải thích, bởi cấu trúc cộng hưởng, tại sao quan sát thấy sản phẩm
như vậy được tạo thành.

261 | Chuẩn bị kiến thức cho kì thi HSGQG Hóa học


Hướng dẫn
1)

2) Phản ứng cộng hydrogen halide liên quan đến sự tạo thành một
carbocation vòng mở (không phải trung gian halonium như phản ứng
cộng halogen). Carbocation này, với lai hóa sp2 và có cấu trúc phẳng,
có thể bị tấn công bởi chloride từ hai phía (trên và dưới), tạo thành hỗn
hợp sản phẩm, trong đó các nhóm methyl có thể ở vị trí cis hoặc trans
với nhau.

3)

262 | Chuẩn bị kiến thức cho kì thi HSGQG Hóa học


4) Cơ chế phản ứng:

Có hai carbocation được tạo thành. Mỗi carbocation có thể phản ứng
với nước theo hướng từ phía trên hoặc dưới, tạo thành tổng cộng bốn
đồng phân lập thể.

263 | Chuẩn bị kiến thức cho kì thi HSGQG Hóa học


5)

6) Trong phản ứng của 3-methylcyclohexene với HBr, hai carbocation


trung gian với độ bền xấp xỉ bằng nhau được tạo thành. Cả hai đều phản
ứng với ion bromide từ các mặt trên và dưới để tạo thành bốn sản phẩm
khác nhau:

264 | Chuẩn bị kiến thức cho kì thi HSGQG Hóa học


Trung gian cation bền nhất từ phản ứng proton hóa 3-
bromocyclohexene là ion bromonium, sẽ bị Br- tác kích từ hướng
ngược lại, tạo thành sản phẩm trans.

7) a)

b) Phản ứng diễn ra qua bốn giai đoạn dưới đây:


▪ Giai đoạn 1: Cộng hợp Hg(OAc)2 vào một trong các liên kết đôi, tạo
thành ion mercurinium vòng.
▪ Giai đoạn 2: Phản ứng của liên kết đôi thứ hai với ion mercurinium,
tạo thành vòng sáu cạnh.
▪ Giai đoạn 3: Vòng hóa lần hai, tạo thành vòng thứ hai.
▪ Giai đoạn 4: Tách loại H, tạo thành liên kết đôi.

265 | Chuẩn bị kiến thức cho kì thi HSGQG Hóa học


8)

9)

266 | Chuẩn bị kiến thức cho kì thi HSGQG Hóa học


Chủ đề 15*: Phản ứng của iodine chloride
Năm 2005, trong một bài báo được đăng trên tạp chí Journal of Organic
Chemistry, các tác giả đã mô tả phản ứng giữa iodine(I) chloride và các
2-ethynyl biphenyl có nhóm thế A. Các chất B tạo thành được phản ứng
với bromine và các tác nhân electrophile khác, tạo thành các sản
phẩm thế và không tạo thành các sản phẩm cộng.

1) Xác định sản phẩm B1 của phản ứng giữa iodine(I) chloride và hợp
chất A1 có X = Y = H và R = CH2Si(CH3)3, biết rằng hàm lượng carbon
trong B1 là 55.38 % và hydrogen là 4.87 %. Phổ 1H NMR của hợp chất
B1 chứa các tín hiệu ở 0.1 và 3.2 ppm, và một bộ tí hiệu trong khoảng
7.5-8.7 ppm theo tỉ lệ 9:2:8.
2) Sắp xếp trật tự giảm dần hoạt tính với ICl của các hợp chất A2-A6,
với X = Y = H và R = 4-ZC6H4 khi Z = H, CH3, CO2CH3, NO2, OCH3. Trong
phản ứng của chất A5 (Z = NO2) và A6 (Z = OCH3), ngoài các hợp chất
B còn có các sản phẩm C5 và C6 được tạo thành. Viết cấu trúc các
sản phẩm này, biết theo kết quả phân tích nguyên tố, hàm lượng
carbon trong C5 và C6 lần lượt là 52.0 và 47.0 %.
3) Sắp xếp theo trật tự tăng dần hoạt tính của các hợp chất A7 (X = Y
= H), A8 (X = NO2, Y = H), A9 (X = H, Y = NO2), trong các chất này R =
C6H5. Trong phản ứng của một trong số chúng, có sản phẩm phụ D
được tạo thành. Xác định cấu trúc của D.
4) Xác định chất nào trong số các đồng phân A10 và A11 tạo thành sản
phẩm B nhanh hơn trong phản ứng với ICl.

267 | Chuẩn bị kiến thức cho kì thi HSGQG Hóa học


5) Khi đun nóng với ICl, các hợp chất A12-A14 có thể tạo thành các
sản phẩm đồng phân B. Xác định sản phẩm chính tạo thành từ mỗi
hợp chất này.

Hướng dẫn
1) Các tín hiệu ở 0.1 và 3.2 pm trong sản phẩm B1 là do sự hiện diện của
nhóm CH2Si(CH3)3 (tỉ lệ 9:2). Ngoài ra, phổ chứa các tín hiệu khác của 8
nguyên tử hydrogen trong vùng thơm. Dựa vào dữ liệu phổ NMR ta thấy
hợp chất B1 có 19 nguyên tử hydrogen. Từ đây, có thể kết luận khối
lượng phân tử của hợp chất này là MB1 = 19:0.0487 = 390. Số nguyên tử
carbon trong B1 là nC = 390∙0.5538:12 = 18. Số nguyên tử carbon trong
A1 cũng là 18, vậy trong phản ứng với iodine thì số carbon không thay
đổi. Sản phẩm phản ứng với ICl chứa ít hơn 1 nguyên tử hydrogen so với
trong A1. Do đó, có thể giả sử rằng hydrogen trong A1 bị thay thế bởi
iodine. Thực tế MB1 tương đương với phân tử C18H19ISi. Khả năng đầu
tiên: nguyên tử hydrogen bị thay thế bởi iodine thuộc vòng benzene.
Tuy nhiên, các hợp chất trong phản ứng với bromine và tác nhân
electrophile khác không tạo thành sản phẩm cộng mà tạo thành sản
phẩm thế. Do đó, chúng phải không chứa liên kết ba carbon-carbon
ban đầu và bất kì liên kết bội nào khác, trừ các liên kết đôi của hệ thơm.
Vậy, ICl đã tấn công vào liên kết ba. Sự tấn công này nhanh hơn tương
tác với nhóm phenyl, do tính thơm của vòng benzene đã bị suy giảm.
ICl phản ứng với liên kết đôi theo cơ chế cộng electrophile thông
thường, qua giai đoạn tạo thành các phức π và σ. Tùy thuộc vào độ bền
của phức σ mà nó có thể phản ứng với ion bù trừ điện tích, tạo thành
sản phẩm cộng hoặc tấn công nội phân tử vào nhóm phenyl tạo thành
sản phẩm thế nhân thơm.

268 | Chuẩn bị kiến thức cho kì thi HSGQG Hóa học


Có hai hướng vòng hóa khả thi để tạo thành dẫn xuất phenanthrene
hoặc 9-methylenfluorene thế. Hợp chất sau chứa 1 liên kết đôi ngoại
vòng, có thể phản ứng với bromine, tạo thành sản phẩm cộng, do đó
không phù hợp với điều kiện đề bài. Vậy nên, hợp chất B là dẫn xuất
phenanthrene. Công thức B1 là:

2) Tất cả các chất A2-A6 đều có những yếu tố không gian giống nhau,
chỉ khác nhau nhóm thế ở xa. Việc đưa nhóm thế nhường electron làm
tăng tốc độ phản ứng với electrophile, còn nhóm thế hút electron làm
giảm tốc độ phản ứng. Do đó, trật tự hoạt tính của các chất biến đổi
như sau: 4-MeOC6H4 > 4-MeC6H4 > C6H5 > 4-MeO2CC6H4 > 4-O2NC6H4.
Chất A5 chứa 20 nguyên tử carbon. Sản phẩm C5 có thể cũng có cùng
số carbon. Do đó khối lượng phân tử của nó là MC5 = 20∙12:0.52 = 461.5
gam/mol. Sẽ hợp lí khi giả định C5 có 1 nguyên tử chlorine. Thực tế, sự
giảm bền hóa iodonium ion bởi nhóm para-nitro dẫn đến sự vòng hóa
dẫn đến sự tạo thành sản phẩm B5. Công thức phân tử của sản phẩm
này là C20H13ClINO2, khối lượng phân tử là 461.5.

269 | Chuẩn bị kiến thức cho kì thi HSGQG Hóa học


Chất A6 chứa 21 nguyên tử carbon. Nếu sản phẩm C6 chứa cùng số
nguyên tử carbon như vậy thì khối lượng phân tử là MC6 = 21∙12:0.47 =
536 gam/mol. Khối lượng phân tử A6 là 284. Sự chênh lệch 252 tương
ứng với chênh lệch khối lượng 2 nguyên tử iodine và 2 nguyên tử
hydrogen. Do đó, có thể kết luận rằng C6 là C21H12I2O. Ta đã biết dẫn
xuất thế phenol dễ phản ứng với các tác nhân electrophile theo cơ chế
thế electrophile. Do đó, nguyên tử iodine thứ hai đi vào phân tử C6 bởi
sự iodine hóa vòng benzene chứa nhóm methoxy nhường electron ở vị
trí ortho.

3) Rõ ràng, chất không có nhóm thế hoạt động hơn chất chứa các
nhóm nitro hút electron. Nhóm nitro trong vòng benzene của A8 làm
cho phản ứng vòng hóa diễn ra chậm hơn nhiều so với A9 (nhóm nitro
ở vòng thứ 2). Do đó, thứ tự hoạt tính A8 < A9 < A7.

4) Trong phản ứng của A11, ion trung gian tấn công vào nhóm phenyl
còn với A10 là C2 trong benzofuran. Sự mất năng lượng thơm hóa trong
trường hợp đầu tiên lớn hơn nhiều so với trường hợp thứ hai. Do đó hợp
chất A10 cho sản phẩm vòng B nhanh hơn A11.
5) Cả 3 chất đều có thể tạo thành 2 sản phẩm vòng hóa đồng phân: với
A12, sự vòng hóa có thể diễn ra ở nguyên tử C2 và C4 thiophene, với
A13 là ở vị trí ortho và para đối với nhóm methoxy, với A14 là ở nguyên

270 | Chuẩn bị kiến thức cho kì thi HSGQG Hóa học


tử C1 và C3 napthalene. Trong trường hợp các dẫn xuất thiophene và
naphthalene, phản ứng diễn ra qua ion bền hơn, tạo thành trong sự tấn
công vào vị trí α. Với dẫn xuất anisole, ưu tiên tấn công vào vị trí para
do cation bền hơn (vì những yếu tố không gian).

271 | Chuẩn bị kiến thức cho kì thi HSGQG Hóa học


Chủ đề 16: Phản ứng hydroboron hóa
1) Dự đoán sản phẩm tạo thành từ phản ứng hydroboron hóa–oxid hóa
các alkene sau:

2) 1,4-dimethylcyclopent-1-ene tạo thành hỗn hợp hai alcohol khi


tham gia phản ứng hydroboron hóa, sau đó oxid hóa. Hãy biểu diễn
cấu trúc các sản phẩm tạo thành và giải thích cho kết quả này.
3) Tại sao hợp chất dưới đây khi tham gia phản ứng hydroboron hóa,
sau đó oxid hóa thì chỉ tạo thành một sản phẩm chứ không phải hỗn
hợp sản phẩm.

4) Phản ứng hydroboron hóa 2-methyl-2-pentene ở 25 °C sau đó


oxide hóa với H2O2 trong kiềm, tạo thành 2-methyl-3-pentanol,
nhưng hydroboron hóa ở 160 °C sau đó oxid hóa thì lại thu được 4-
methyl-1-pentanol. Hãy đề xuất cơ chế phản ứng để giải thích.

272 | Chuẩn bị kiến thức cho kì thi HSGQG Hóa học


Hướng dẫn
1)

2) Phản ứng cộng BH3 vào liên kết đôi của cycloalkene có thể diễn ra
từ phía trên hoặc phía dưới, tạo thành các trạng thái chuyển tiếp như
sau:

Phản ứng của hai sản phẩm cộng alkylborane trung hòa với hydrogen
peroxide sẽ tạo thành hai đồng phân alcohol. Trong một đồng phân,
hai nhóm methyl có mối quan hệ cis, còn trong đồng phân kia là quan
hệ trans.

273 | Chuẩn bị kiến thức cho kì thi HSGQG Hóa học


3) Dưới đây là hai alcohol có thể được tạo thành từ phản ứng
hydroboron hóa/oxid hóa alkene trên. Một sản phẩm theo hướng tấn
công từ phía trên (thực tế không được tạo thành), và sản phẩm còn lại
là phản ứng cộng từ phía dưới (được tạo thành). Phản ứng cộng từ phía
trên không xảy ra bởi nhóm methyl đầu cầu của hệ hai vòng (bicyclic)
cản trở sự tiếp cận của borane.
4) Hydroboron hóa 2-methyl-2-pentene ở 160 oC là phản ứng thuận
nghịch. Trung gian cơ-borane ban đầu có thể tách loại BH3 theo hai
hướng, tạo thành 2-methyl-2-pentene hoặc 4-methyl-2-pentene, sau
đó chúng có thể trải qua phản ứng hydroboron hóa thuận nghịch, tạo
thành 4-methyl-2-pentene hoặc 4-methyl-1-pentene. Tác động của
các phản ứng thuận nghịch này làm dịch chuyển liên kết đôi dọc mạch
carbon. Phản ứng hydroboron hóa cuối cùng tạo thành cơ-borane bền
nhất (bậc một) rồi sau đó oxid hóa tạo thành 4-methyl-1-pentanol.

274 | Chuẩn bị kiến thức cho kì thi HSGQG Hóa học


Chủ đề 17: Phản ứng thế allyl
1) Tại sao sản phẩm chính của phản ứng giữa methylenecyclohexane
với N-bromosuccinimide là 1-(bromomethyl)cyclohexene?

2) Dự đoán sản phẩm của phản ứng giữa các alkene sau với NBS:

3)
a) Dự đoán các sản phẩm tạo thành từ phản ứng của 1,4-
hexadiene với NBS. Chỉ ra cấu trúc của trung gian gốc bền nhất.
b) Dự đoán các sản phẩm tạo thành từ phản ứng của 1-phenyl-2-
butene với NBS.

275 | Chuẩn bị kiến thức cho kì thi HSGQG Hóa học


Hướng dẫn
1)

Sản phẩm B được ưu tiên tạo thành hơn bởi phản ứng ở đầu bậc một
của gốc allylic tạo thành sản phẩm với liên kết đôi có ba nhóm thế.
2)

276 | Chuẩn bị kiến thức cho kì thi HSGQG Hóa học


3)

277 | Chuẩn bị kiến thức cho kì thi HSGQG Hóa học


Trung gian bên phải bền hơn bởi electron chưa ghép cặp được giải tỏa
trên nhiều nguyên tử hơn, dẫn đến các sản phẩm tạo thành từ trung
gian này sẽ chiếm ưu thế.
4) Có hai gốc allylic có thể được tạo thành:

Gốc thứ hai (bên dưới) bền hơn, bởi nó vừa là allylic, vừa là benzylic. Do
đó, sản phẩm dưới đây sẽ ưu tiên được tạo thành:

278 | Chuẩn bị kiến thức cho kì thi HSGQG Hóa học


Chủ đề 18: Phản ứng oxid hóa
1) Dự đoán sản phẩm tạo thành từ phản ứng của cis-2-butene với
meta-chloroperoxybenzoic acid? Chỉ rõ hóa lập thể.
2) Đề xuất phương án điều chế các hợp chất sau đây từ các alkene:

3) Dự đoán sản phẩm tạo thành từ phản ứng của 1-methylcyclohexene


với các tác nhân sau:
a) Dung dịch KMnO4 được acid hóa.
b) O3, sau đó là Zn/CH3COOH.
4) Hãy dự đoán sản phẩm tạo thành khi các hợp chất dưới đây phản
ứng lần lượt với (1) meta-chloroperoxybenzoic acid, (2) KMnO4 trong
acid và (3) O3, sau đó xử lí với Zn trong acetic acid.
a) 2,5-Dimethyl-2-heptene
b) 3,3-Dimethylcyclopentene
5) Hợp chất A có công thức C10H16. Khi hydrogen hóa xúc tác trên
palladium thì nó chỉ phản ứng với 1 đương lượng H2. Khi tiến hành
ozone phân A, sau đó xử lí với kẽm, thì tạo thành diketone đối xứng
B (C10H16O2). Xác định cấu trúc của A, B và viết các phương trình
phản ứng xảy ra.
6) Hydrocarbon A có công thức phân tử C6H12, phản ứng được với 1
đương lượng mol H2 trên xúc tác palladium. Hydrocarbon A cũng
phản ứng được với OsO4 tạo thành diol B. Khi oxid hóa với KMnO4
trong dung dịch acid thì A bị phân cắt thành hai mảnh. Một mảnh
phản ứng với propanoic acid, CH3CH2CO2H, và mảnh còn lại là
ketone C. Xác định cấu trúc A, B, C. Viết phương trình các phản ứng
diễn ra.

279 | Chuẩn bị kiến thức cho kì thi HSGQG Hóa học


Hướng dẫn
1)

2)

280 | Chuẩn bị kiến thức cho kì thi HSGQG Hóa học


3)

4)

281 | Chuẩn bị kiến thức cho kì thi HSGQG Hóa học


5)

6)

282 | Chuẩn bị kiến thức cho kì thi HSGQG Hóa học


Chủ đề 19: Phản ứng cộng carbene vào alkene
1) Đề xuất sản phẩm tạo thành từ các phản ứng sau:

2) Các đồng phân (cis/trans) của 2-butene tạo thành các sản phẩm
cyclopropane khác nhau trong phản ứng Simmons–Smith. Hãy
biểu diễn cấu trúc của chúng và giải thích cho sự khác nhau này.

3) Dự đoán các sản phẩm tạo thành từ mỗi chuyển hóa dưới đây. Biết
rằng trong tất cả các trường hợp, vòng thơm đều không tham gia
phản ứng:

283 | Chuẩn bị kiến thức cho kì thi HSGQG Hóa học


4) Hãy dự đoán sản phẩm được tạo thành từ các phản ứng sau:

5) Phản ứng Simmons–Smith của cyclohexene với diiodomethane


tạo thành một sản phẩm cyclopropane duy nhất, nhưng phản ứng
tương tự của cyclohexene với 1,1-diiodoethane thì tạo thành (theo
hiệu suất thấp) hỗn hợp hai sản phẩm methylcyclopropane đồng
phân. Giải thích sự khác biệt này.

284 | Chuẩn bị kiến thức cho kì thi HSGQG Hóa học


Hướng dẫn
1)

2) Phản ứng Simmons–Smith diễn ra theo hóa lập thể syn. Chỉ có cis-
1,2-dimethylcyclopropane được tạo thành từ cis-2-butene, và chỉ có
trans-1,2-dimethylcyclopropane được tạo thành từ trans-2-butene.

3)

285 | Chuẩn bị kiến thức cho kì thi HSGQG Hóa học


286 | Chuẩn bị kiến thức cho kì thi HSGQG Hóa học
4)

5) Hãy tập trung vào hóa lập thể của vòng ba cạnh. Phản ứng
Simmons–Smith của 1,1-diiodoethane với liên kết đôi diễn ra theo hóa
lập thể syn và có thể tạo thành hai đồng phân. Trong một đồng phân
này (A), nhóm methyl ở cùng phía của vòng ba cạnh như các nguyên tử
carbon của vòng cyclohexane; còn trong B thì nhóm methyl và các
carbon của vòng cyclohexane ở khác phía của vòng ba cạnh.

287 | Chuẩn bị kiến thức cho kì thi HSGQG Hóa học


Chủ đề 20: Tổng hợp từ alkene
1) Hãy đề xuất tác nhân để thực hiện các phản ứng sau:

2) Trong ý tưởng tổng hợp một hợp chất thì việc biết được hướng nào
không nên thực hiện cũng quan trọng như hướng nên thực hiện.
Hãy giải thích tại sao không nên thực hiện các chuyển hóa sau:

288 | Chuẩn bị kiến thức cho kì thi HSGQG Hóa học


Hướng dẫn
1)

2) a) Phản ứng cộng HI sẽ diễn ra theo sự chọn lọc Markovnikov – tức


là iodine sẽ cộng vào carbon nhiều nhóm thế hơn.
b) Phản ứng hydroxyl hóa liên kết đôi sẽ tạo thành cis, chứ không phải
trans, diol.
c) Ozone phản ứng với cả hai liên kết đôi của 1,4-cyclohexadiene.
d) Phản ứng hydroboron hóa là cộng hợp syn.

289 | Chuẩn bị kiến thức cho kì thi HSGQG Hóa học


Chủ đề 21: Danh pháp alkyne
a) Gọi tên các hợp chất sau theo danh pháp IUPAC:

b) Biểu diễn cấu tạo các hợp chất sau:


a) 3,3-Dimethyl-4-octyne
b) 3-Ethyl-5-methyl-1,6,8-decatriyne
c) 2,2,5,5-Tetramethyl-3-hexyne
d) 3,4-Dimethylcyclodecyne
e) 3,5-Heptadien-1-yne
f) 3-Chloro-4,4-dimethyl-1-nonen-6-yne
g) 3-sec-Butyl-1-heptyne
h) 5-tert-Butyl-2-methyl-3-octyne

290 | Chuẩn bị kiến thức cho kì thi HSGQG Hóa học


Hướng dẫn
1)

2)

291 | Chuẩn bị kiến thức cho kì thi HSGQG Hóa học


292 | Chuẩn bị kiến thức cho kì thi HSGQG Hóa học
Chủ đề 22: Phản ứng hóa học
1) Dự đoán sản phẩm tạo thành từ phản ứng hydrate hóa các alkyne
sau:

2) Đề xuất alkyne ban đầu để điều chế các ketone sau:

3) Đề xuất alkyne ban đầu để điều chế các hợp chất sau bởi phản ứng
hydroboron hóa-oxid hóa:

4) Đề xuất alkyne ban đầu để điều chế các hợp chất sau bởi các phản
ứng khử hóa phù hợp:
a) trans-2-Octene
b) cis-3-Heptene
c) 3-Methyl-1-pentene
5) Dự đoán sản phẩm của các phản ứng sau:

293 | Chuẩn bị kiến thức cho kì thi HSGQG Hóa học


6) Đề xuất cấu trúc tiền chất hydrocarbon ban đầu sẽ tạo thành các
sản phẩm sau trong phản ứng oxid hóa phân cắt bởi KMnO4 hoặc
O3:

7) Các alkyne đầu mạch phản ứng với Br2 và nước, tạo thành bromo
ketone. Hãy đề xuất cơ chế của phản ứng dưới đây:

294 | Chuẩn bị kiến thức cho kì thi HSGQG Hóa học


Hướng dẫn
1) a) Alkyne đối xứng chỉ tạo thành một sản phẩm:

b)

2)

Ở trường hợp b thì ketone mong muốn chỉ có thể được điều chế ở dạng
thành phần một hỗn hợp.
3) Hãy nhớ rằng phản ứng hydroboron hóa sẽ tạo thành aldehyde từ
alkyne đầu mạch và ketone từ alkyne trong mạch.

295 | Chuẩn bị kiến thức cho kì thi HSGQG Hóa học


4)

5)

296 | Chuẩn bị kiến thức cho kì thi HSGQG Hóa học


6)

7) Cơ chế phản ứng này cũng tương tự sự tạo thành halohydrin:

297 | Chuẩn bị kiến thức cho kì thi HSGQG Hóa học


Chủ đề 23: Tổng hợp từ alkyne
1) Từ 4-octyne là nguồn alkyne duy nhất, cùng với bất kì các tác nhân
vô cơ cần thiết nào khác, hãy đề xuất quy trình tổng hợp các hợp
chất sau:
a) cis-4-Octene
b) Butanal
c) 4-Bromooctane
d) 4-Octanol
e) 4,5-Dichlorooctane
f) Butanoic acid
2) Từ acetylene cùng bất kì alkyl halide và các tác nhân vô cơ cần
thiết, hãy đề xuất quy trình tổng hợp các hợp chất sau:
a) Decane
b) 2,2-Dimethylhexane
c) Hexanal
d) 2-Heptanone
3) Đề xuất quy trình tổng hợp các hợp chất sau từ các tiền chất không
quá 4C, cùng các tác nhân cần thiết khác:

4) Xác định các tác nhân a – c để thực hiện các chuyển hóa sau:

298 | Chuẩn bị kiến thức cho kì thi HSGQG Hóa học


5) Đề xuất cách thực hiện các chuyển hóa sau:

6) Đề xuất giải pháp thực hiện chuyển hóa sau:

7) Đề xuất giải pháp thực hiện chuyển hóa sau:

299 | Chuẩn bị kiến thức cho kì thi HSGQG Hóa học


Hướng dẫn
1) a) Sử dụng tác nhân H2 với xúc tác Lindlar để khử hóa liên kết ba
thành liên kết đôi với chọn lọc lập thể cis:

b) Khử hóa 4-octyne thành 4-octene rồi phân cắt liên kết đôi bởi O3,
thu được aldehyde mong muốn:

c) Cộng hợp HBr vào cis-4-octene tạo thành 4-bromooctane.

d)

e)

300 | Chuẩn bị kiến thức cho kì thi HSGQG Hóa học


f)

2) a)

b)

c)

d)

301 | Chuẩn bị kiến thức cho kì thi HSGQG Hóa học


3) a)

b)

4)

302 | Chuẩn bị kiến thức cho kì thi HSGQG Hóa học


5)

6)

7)

303 | Chuẩn bị kiến thức cho kì thi HSGQG Hóa học


304 | Chuẩn bị kiến thức cho kì thi HSGQG Hóa học
Chủ đề 24: Tính nucleophile
i) Lấy ví dụ về các nucleophile chứa С-, N-, O-, S-.
ii) Hãy dẫn ra định nghĩa về tính base và tính nucleophile. Trong một
chu kì và một nhóm của bảng tuần hoàn, tính base và nucleophile
của các hợp chất tương ứng biến đổi như thế nào?
iii) Độ mạnh-yếu của nucleophile phụ thuộc vào một vài yếu tố. Hãy
cho biết phát biểu nào sau đây không đúng:
(A) Anion là nucleophile mạnh hơn nguyên tử trung hòa hoặc phân
tử.
(B) Các nguyên tử càng phân cực mạnh thì tính nucleophile càng
mạnh.
(C) Mức độ solvate hóa ảnh hưởng đến tính nucleophile.
(D) Trong một chu kì của Bảng tuần hoàn, tính nucleophile và tính
base của các hợp chất tương ứng biến đổi cùng chiều.
(E) Trong một nhóm của Bảng tuần hoàn, tính nucleophile và tính
base của các hợp chất tương ứng biến đổi cùng chiều.
iv) Sắp xếp các anion và hợp chất trung hòa sau theo trình tự giảm dần
tính nucleophile trong dung dịch nước.

v) Sắp xếp các anion và hợp chất trung hòa sau theo trình tự giảm dần
tính nucleophile.
CF3CH2O− CH3CH2O− CH3COO− CH3CH2S− (CH CH ) O
3 2 2
CH3CH2Li

vi) Phân loại các anion và phân tử trung hòa sau thành các nhóm theo
độ mạnh tương đối của tính nucleophile so với methanol.
RCOOH,Br − ,HS− ,H2O,RO− ,RCOO− ,OH− ,I− ,NH3 ,Cl− ,ROH,F − ,NC− ,RS− ,N3−

Rất tốt
Tốt
Chấp nhận được
Yếu
Rất yếu

305 | Chuẩn bị kiến thức cho kì thi HSGQG Hóa học


vii) Tính nucleophile của các tiểu phân trong dãy I-, Br-, Cl-, F- biến đổi
như thế nào từ các dung môi aprotic sang dung môi protic?
Hướng dẫn
i)

ii) Tính base là khả năng phản ứng của một phân tử hoặc ion với proton
hoặc các Brønsted acid, còn tính nucleophile là khả năng phản ứng với
carbon có tính electrophile, nghĩa là với carbocation (sp2) hoặc với
nguyên tử carbon lai hóa sp3 mang điện tích dương riêng phần. Trong
một nhóm của bảng tuần hoàn, theo chiều từ trên xuống, tính
nucleophile của các hợp chất tương ứng sẽ tăng, trong khi đó tính base
sẽ giảm (với các phản ứng trong dung môi protic). Còn trong một chu
kì, tính nucleophile và tính base đều tăng theo chiều từ phải sang trái.
iii) Phát biểu e sai. Trong một nhóm của Bảng tuần hoàn, tính
nucleophile và tính base của các hợp chất tương ứng biến đổi ngược
chiều.
iv) Nucleophile
yếu nhất là
methanol trung
hòa. Các
nucleophile chứa oxygen được sắp xếp theo độ mạnh của các acid liên
hợp (pKa: HOH (15.7), phenol (10), acetic acid (4.76)). Do lưu huỳnh có
kích thước lớn hơn oxygen nên CH3S- > HO-.
v)
CH3CH2Li  CH3CH2S−  CH3CH2O−  CF3CH2O−  CH3COO−  (CH3CH2 ) O
2

vi)
Rất tốt I− ,HS− ,RS−
Tốt Br − ,HO− ,RO− ,NC− ,N3−

306 | Chuẩn bị kiến thức cho kì thi HSGQG Hóa học


Chấp nhận được NH3 ,Cl− ,F − ,RCOO−

Yếu H2O,ROH

Rất yếu RCOOH


vii) Trong các dung môi aprotic (ví dụ như dimethylformamide,
acetonitrile, acetone, tetrahydrofuran, dimethyl sulfoxide,
hexamethyl phosphotriamide) thì tính nucleophile biến đổi như sau: I-
< Br- < Cl- < F-. Trong các dung môi này, các nucleophile không bị
solvate hóa mạnh như trong các dung môi protic. Sự tiêu hao năng
lượng để khử-solvate hóa biến đổi trong khoảng hẹp. Hoạt tính của các
nucleophile chủ yếu được quyết định năng lượng thu được khi hình
thành liên kết giữa nucleophile và nguyên tử carbon của chất phản
ứng, nghĩa là ái lực với tâm electrophile này – cũng giống như ái lực đối
với proton, đạt giá trị lớn nhất trong trường hợp fluorine.
Trong các dung môi protic (ví dụ như nước, formic acid, methanol,
ethanol, acetic acid): I- > Br- > Cl- > F-. Trong các dung môi protic, tác
nhân nucleophile được solvate hóa mạnh hơn nhiều so với trong các
dung môi aprotic, do các liên kết hydrogen xuất hiện giữa nucleophile
và dung môi. Sự tiêu hao năng lượng để khử-solvate hóa của
nucleophile có thể so sánh với năng lượng thu được từ việc tạo thành
liên kết halogen-carbon mới và phụ thuộc vào khả năng phân cực (bán
kính) của ion halide.
Do đó, trong các dung môi aprotic, tính nucleophile của ion halide tỉ lệ
thuận với tính base của chúng, còn trong dung môi protic thì chủ yếu
được xác định bởi khả năng phân cực của chúng.

307 | Chuẩn bị kiến thức cho kì thi HSGQG Hóa học


Chủ đề 25: Phản ứng thế
1) Đề xuất cách điều chế các dẫn xuất halide sau từ alcohol tương
ứng:

2) Dự đoán sản phẩm tạo thành từ phản ứng thế theo cơ chế SN2 của
OH- với (R)-2-bromobutane.
3) Dự đoán sản phẩm phản ứng thế nucleophile của các tác nhân sau
với 1-bromobutane.

4) Sắp xếp các hợp chất dưới đây theo chiều tăng dần hoạt tính với
phản ứng thế SN2:
CH3Br, CH3OTos, (CH3)3CCl, (CH3)2CHCl
5) Dự đoán sản phẩm tạo thành từ phản ứng của (S)-3-chloro-3-
methyloctane với acetic acid.
6) Sắp xếp các các hợp chất dưới đây theo chiều giảm dần hoạt tính
với phản ứng thế SN2:

7) 3-Bromo-1-butene và 1-bromo-2-butene trải qua phản ứng thế SN1


với tốc độ gần như bằng nhau, mặc dù chúng là các halide khác bậc.
Hãy giải thích tại sao.
8) Hãy dự đoán mỗi phản ứng dưới đây ưu tiên xảy ra theo cơ chế SN1
hay SN2?

308 | Chuẩn bị kiến thức cho kì thi HSGQG Hóa học


Hướng dẫn
1) Các halogen acid được sử dụng để chuyển hóa alcohol bậc ba thành
alkyl halide. PBr3 và SOCl2 được sử dụng để chuyển alcohol bậc một và
bậc hai thành alkyl halide. Còn (CH3CH2)2NSF3 và HF trong pyridine có
thể dùng để tạo thành các alkyl fluoride.

2)

309 | Chuẩn bị kiến thức cho kì thi HSGQG Hóa học


3)

4)

5) Trong phản ứng SN1 này, acetate có thể tác kích từ hai phía của trung
gian carbocation phi thủ tính, dẫn đến sự tạo thành hỗn hợp các
acetate đối quang R và S. Tỉ lệ của các đối quang này xấp xỉ 50:50.

310 | Chuẩn bị kiến thức cho kì thi HSGQG Hóa học


6)

7) Do trong giai đoạn tốc định, cả hai bromobutene này đều tạo thành
cùng carbocation:

8) Cả hai chất nền đều có nhóm allylic và có thể phản ứng theo cả
hướng SN1 và SN2. Cơ chế phản ứng được quyết định bởi nhóm rời đi,
dung môi hoặc nucleophile.
a) Phản ứng này có khả năng xảy ra theo cơ chế SN1 hơn. HCl chuyển
hóa nhóm rời đi kém -OH thành nhóm rời đi rất tốt -OH2+ và dung môi
phân cực làm bền hóa trung gian carbocation tạo thành.
b) Phản ứng xảy ra với một nucleophile mang điện tích âm trong dung
môi phi-proton, phân cực. Nhiều khả năng xảy ra theo cơ chế SN2.

311 | Chuẩn bị kiến thức cho kì thi HSGQG Hóa học


Chủ đề 26: Ảnh hưởng của dung môi
i) Phát biểu nào sau đây về ảnh hưởng của dung môi trong các phản
ứng thế nucleophile là không chính xác:
(A) Các dung môi phân cực với hằng số điện môi lớn tạo thuận lợi
cho phản ứng SN1.
(B) Các dung môi aprotic phân cực tạo thuận lợi cho phản ứng SN2.
(C) Các alkyl halide vẫn có thể tham gia phản ứng thế SN2 trong các
dung môi phân cực với hằng số điện môi lớn.
(D) Các alkyl halide bậc ba vẫn có thể tham gia phản ứng SN1 trong
các dung môi không phân cực.
(E) Sự chọn lọc lập thể của các phản ứng với sự tham gia của alkyl
halide bậc hai không thể bị tác động bởi dung môi.
ii) Giải thích xu hướng biến đổi giá trị tốc độ tương đối của phản ứng
giữa n-butyl bromide với sodium azide trong các dung môi khác
nhau.
CH3CH2CH2CH2Br + NaN3 → CH3CH2CH2CH2N3 + NaBr
Dung môi CH3OH H2O DMSO DMFA CH3CN [(CH3)2N]3P=O
Tốc độ tương đối 1 7 1300 2800 5000 200000

Cho biết:

iii) Giải thích ảnh hưởng của dung môi đến các phản ứng bậc hai sau:
a) Tốc độ phản ứng tương đối trong H2O / trong CH3OH / trong
C2H5OH = 1/16/44.
131 −
I + CH3I → CH3131I + I− 131 −
I + CH3I → CH3131I + I−

b) Tốc độ phản ứng tương đối trong hexane / trong chloroform =


1/13000.
(CH CH CH ) N + CH I → (CH CH CH ) N (CH )I
3 2 2 3 3 3 2 2 3
+
3

312 | Chuẩn bị kiến thức cho kì thi HSGQG Hóa học


Hướng dẫn
i) Đánh giá các phát biểu:
a) (A) đúng. Các phản ứng SN1 cần các dung môi phân li proton với
hằng số điện môi ε lớn. Vai trò của các dung môi như vậy (H2O,
HCOOH, CH3OH, CH3COOH) là để làm bền hóa các cation và anion.
b) (B) đúng. Với các phản ứng SN2, không thực sự cần dung môi protic,
do chỉ có cation (ion bù trừ của nucleophile) cần được solvate hóa.
Các dung môi tốt có thể kể đến là: acetone, các ether (diethyl
ether, dioxane, tetrahydrofuran), acetonitrile, dimethylform-
amide, dimethyl sulfoxide, hexamethylphosphotriamide.
c) (C) và (D) đúng, phản ứng vẫn có thể xảy ra nhưng với tốc độ rất
chậm.
d) (E) sai vì sự chọn lọc lập thể của các phản ứng thế của alkyl halide
bậc hai có thể bị ảnh hưởng bởi dung môi.
ii) Do n-butyl bromide là alkyl halide bậc một nên phản ứng thế ưu tiên
diễn ra theo kiểu SN2. Các dung môi protic (CH3OH, H2O) làm chậm tốc
độ phản ứng, trong khi đó các dung môi aprotic phân cực (DMSO, DMF,
CH3CN, [(CH3)2N]3P=O) thì phù hợp với phản ứng SN2. Các dung môi
protic làm chậm tốc độ phản ứng SN2 bởi chúng solvate hóa
nucleophile, làm giảm năng lượng và hoạt tính của nó. Các dung môi
aprotic phân cực thì hòa tan được nhiều muối nhưng chúng solvate các
cation thay vì anion có tính nucleophile. Kết quả là các anion không bị
solvate hóa có thêm hoạt tính nucleophile và làm tăng tốc độ phản ứng
SN2.
iii) a) Dung môi với độ phân cực kém hơn (ethanol kém phân cực hơn
nước) sẽ thuận lợi hơn.

b) Trong trường hợp này, trạng thái chuyển tiếp phân cực hơn, do đó
dung môi phân cực hơn (chloroform) sẽ thuận lợi hơn.

313 | Chuẩn bị kiến thức cho kì thi HSGQG Hóa học


Chủ đề 27: Hóa lập thể phản ứng thế
1) Chu trình Walden được tiến hành như sau. Hãy giải thích các kết quả
và chỉ ra sự nghịch đảo Walden diễn ra khi nào:

2) Phản ứng của (R)-2-hydroxysuccinic acid với phosphorus


trichloride tạo thành sản phẩm B.

a) Xác định cấu trúc (R)-2-hydroxysuccinic acid.


b) Xác định cấu trúc trung gian A và sản phẩm B, chỉ rõ hóa lập
thể.
c) Đề xuất cấu trúc trạng thái chuyển tiếp trong các chuyển hóa
của A, B.
3) Phản ứng của (1S, 2S)-trans-p-bromobenzenesulfonate của
iodocyclohexyl (W) với acetic acid tạo thành trans-acetate của 2-
iodocyclohexyl (T).
a) Viết cấu dạng ghế của W(e,e) và W(a,a). Với e là equatorial
(biên), còn a là axial (trục).

314 | Chuẩn bị kiến thức cho kì thi HSGQG Hóa học


Phản ứng diễn ra theo sơ đồ dưới đây:

b) Xác định các trung gian X+, Y+ và T trong giản đồ trên và viết cấu
dạng của chúng.
c) Xác định cấu hình R/S của các nguyên tử carbon thủ tính trong
T.
4) Các phản ứng SN2 diễn ra với sự nghịch đảo cấu hình, còn các phản
ứng SN1 diễn ra với sự racemic (ngoại tiêu triền) hóa. Tuy nhiên,
phản ứng thế sau đây, diễn ra với sự bảo toàn cấu hình hoàn toàn.
Hãy đề xuất cơ chế để giải thích.

315 | Chuẩn bị kiến thức cho kì thi HSGQG Hóa học


Hướng dẫn
1) Trong chuỗi phản ứng này, một alcohol được chuyển hóa thành một
ether theo hai hướng khác nhau. Các ether tạo thành giống nhau về
cấu tạo nhưng khác nhau về dấu góc quay cực riêng. Do đó, ở một vài
bước trong chuỗi phản ứng này, phải xảy ra sự nghịch đảo cấu hình ở
carbon thủ tính. Hãy nghiên cứu về mỗi bước để xác định khi nào xảy
ra sự nghịch đảo:

Ở bước 1, alcohol phản ứng với potassium tạo thành potassium


alkoxide. Do liên kết giữa carbon và oxygen không bị phá vỡ nên không
xảy ra sự nghịch đảo cấu hình.
Potassium alkoxide đóng vai trò như nucleophile trong phản ứng thế
SN2 trên CH3CH2Br trong bước 2. Tuy nhiên, liên kết C-Br của
bromoethane, chứ không phải liên kết C-O của alkoxide, bị phá vỡ.
Trong bước này cũng không xảy ra sự nghịch đảo cấu hình tâm thủ tính.
Alcohol ban đầu phản ứng với tosyl chloride ở bước 3. Lại một lần nữa,
liên kết O-H, chứ không phải C-O, của alcohol bị phá vỡ, nên vẫn không
xảy ra sự nghịch đảo cấu hình.
Do đó, sự nghịch đảo cấu hình phải diễn ra ở bước 4, khi nhóm -O-Tos
bị thay thế bởi CH3CH2OH. Liên kết C-O của tosylate (-OTos) bị phá vỡ,
và liên kết C-O mới được tạo thành.

316 | Chuẩn bị kiến thức cho kì thi HSGQG Hóa học


Chú ý đến các góc quay cực riêng của hai sản phẩm đối quang. Sản
phẩm ở bước 1 và 2 phải là đối quang tinh khiết bởi không có phản ứng
nào tác động đến liên kết C-O. Phản ứng 4 diễn ra với một phần
racemic hóa ở tâm thủ tính, dẫn đến giá trị    có trị tuyệt đối nhỏ
D

hơn.
2) a)

b)

c)

3) a)

317 | Chuẩn bị kiến thức cho kì thi HSGQG Hóa học


b-c)

4)

318 | Chuẩn bị kiến thức cho kì thi HSGQG Hóa học


Chủ đề 28: Phản ứng tách
1) Dự đoán sản phẩm tạo thành từ phản ứng tách hydrogen halide của
các alkyl halide sau. Bỏ qua hóa lập thể của liên kết đôi trong sản
phẩm.

2) Các alkene sau có thể được tạo thành từ alkyl halide nào:

3)
a) Dự đoán hóa lập thể của sản phẩm phản ứng tách E2 của
(1R,2R)- 1,2-dibromo-1,2-diphenylethane.
b) Dự đoán cấu trúc của alkene ba nhóm thế được tạo thành từ
phản ứng tách E2 của hợp chất dưới đây khi xử lí với KOH.

c) Dự đoán đồng phân nào, trans hay cis-1-bromo-4-tert-


butylcyclohexane, xảy ra phản ứng tách E2 nhanh hơn. Biểu
diễn mỗi phân tử ở cấu dạng ghế bền nhất và đưa ra giải thích
hợp lí.

319 | Chuẩn bị kiến thức cho kì thi HSGQG Hóa học


4) Dự đoán mỗi phản ứng sau đây nhiều khả năng diễn ra theo cơ chế
SN1, SN2, E1, E1cB, hay E2?

5)
a) 1-Chloro-1,2-diphenylethane có thể trải qua phản ứng tách E2
tạo thàn cis- hoặc trans-1,2-diphenylethylene (stilbene). Hãy
biểu diễn công thức chiếu Newman của các cấu dạng hoạt tính,
dẫn đến sự tạo thành các sản phẩm khả dĩ và đề xuất lí do vì sao
trans alkene là sản phẩm chính.

b) Tosylate của (2R,3S)-3-phenyl-2-butanol trải qua phản ứng


tách E2 khi xử lí với sodium ethoxide tạo thành (Z)-2-phenyl-2-
butene. Sử dụng công thức chiếu Newman để giải thích lí do.

320 | Chuẩn bị kiến thức cho kì thi HSGQG Hóa học


c) Dự đoán sản phẩm tạo thành từ phản ứng tách E2 của tosylate
của (2R,3R)-3-phenyl-2-butanol. Alkene nào có thể được tạo
thành từ phản ứng tách E2 của (2S,3R) và (2S,3S) tosylate?
d) Giải thích tại sao trans-1-bromo-2-methylcyclohexane tạo
thành sản phẩm không tuân theo quy tắc Zaitsev, 3-
methylcyclohexene, khi xử lí với base.

321 | Chuẩn bị kiến thức cho kì thi HSGQG Hóa học


Hướng dẫn
1)

2) Để có hiệu suất tốt nhất, alkyl halide ban đầu không nên tạo thành
một hỗn hợp sản phẩm phản ứng tách.

322 | Chuẩn bị kiến thức cho kì thi HSGQG Hóa học


3) a) Trước tiên, cần biểu diễn đúng hóa lập thể của chất phản ứng:

Chuyển cấu trúc này thành công thức chiếu Newman, biểu diễn theo
cấu dạng với dạng hình học đối song (xen kẽ) đối với -H và -Br:

Alkene tạo thành từ phản ứng tách E2 là (Z)-1-bromo-1,2-


diphenylethylene:

b) Biểu diễn theo công thức chiếu Newman sao cho -H và -Br ở vị trí
đối song:

Sản phẩm chính sẽ là (Z)-3-methyl-2-pentene. Ngoài ra cũng có một


lượng nhỏ 3-methyl-1-pentene được tạo thành.

323 | Chuẩn bị kiến thức cho kì thi HSGQG Hóa học


c) Dưới đây là cấu dạng ghế bền nhất của mỗi đồng phân. Nhóm tert-
butyl lớn hơn luôn ở vị trí biên trong cấu dạng bền hơn. Đồng phân cis
sẽ trải qua phản ứng E2 nhanh hơn bởi -Br và -H có sự sắp xếp đối song,
thuận lợi cho phản ứng tách E2.

4) a) Phản ứng diễn ra theo cơ chế SN2 bởi chất nền là dẫn xuất halide
bậc một, nucleophile không có tính base và sản phẩm đã cho là sản
phẩm thế.

b) Đây là phản ứng tách E2 giữa alkyl halide bậc hai với base mạnh:

c) Chất nền bậc ba gần như chỉ tạo thành sản phẩm thế SN1:

324 | Chuẩn bị kiến thức cho kì thi HSGQG Hóa học


d) Đây là phản ứng tác E1cb bởi nhóm rời đi cách nhóm carbonyl hai
nguyên tử carbon:

6) a)

Cả hai công thức chiếu Newman đặt –H và –Cl vào dạng hình học đối
song phù hợp với phản ứng tách E2.

Trạng thái chuyển tiếp A‡ hoặc B‡ có thể tạo thành khi 1-chloro-1,2-
diphenylethane trải qua phản ứng tách E2. Tương tác không gian giữa
hai nhóm phenyl cồng kềnh trong A‡ khiến cho trạng thái chuyển tiếp
này (và sản phẩm tạo thành) có năng lượng hoạt hóa cao hơn trạng thái
chuyển tiếp B‡ . Do đó, sự tạo thành sản phẩm từ B‡ thuận lợi hơn, và
trans-1,2-diphenylethylene.

325 | Chuẩn bị kiến thức cho kì thi HSGQG Hóa học


b)

Vẽ công thức chiếu Newman của tosylate của (2R,3S)-3-phenyl-2-


butanol và quay công thức chiếu cho đến khi -OTos và -H trên các
nguyên tử carbon liền kề ở vị trí đối song. Mặt dù cấu dạng này có một
vài tương tác bán lệch nhưng là cấu dạng duy nhất mà -OTos và -H cách
xa 180o.

Phản ứng tách tạo thành đồng phân Z của 2-phenyl-2-butene.

326 | Chuẩn bị kiến thức cho kì thi HSGQG Hóa học


c)

Đồng phân (2S,3S) cũng tạo thành E alkene, còn đồng phân (2S,3R) thì
tạo thành Z alkene.
d) Sản phẩm tách E2 đòi hỏi hai nguyên tử bị tách phải có mối liên hệ
vị trí đối song. Do không thể có cấu dạng này với bromine ở C1 và
hydrogen ở C2 nên phải xảy ra sự lật vòng và phản ứng tách diễn ra
theo hướng không tuân theo quy tắc Zaitsev, tạo thành 3-
methylcyclohexene.

327 | Chuẩn bị kiến thức cho kì thi HSGQG Hóa học


Chủ đề 29: Phản ứng ghép cặp
1) Đề xuất cách thực hiện chuyển hóa dưới đây trên cơ sở chính là
phản ứng ghép cặp của hợp chất cơ-đồng:

2) Dự đoán sản phẩm của các phản ứng sau:

3) Xác định các tác nhân a – c để thực hiện các chuyển hóa sau:

328 | Chuẩn bị kiến thức cho kì thi HSGQG Hóa học


Hướng dẫn
1) a)

b)

c)

329 | Chuẩn bị kiến thức cho kì thi HSGQG Hóa học


2)

Sản phẩm chính chứa liên kết đôi bốn nhóm thế, còn sản phẩm phụ
chứa liên kết đôi ba nhóm thế.

330 | Chuẩn bị kiến thức cho kì thi HSGQG Hóa học


3)

331 | Chuẩn bị kiến thức cho kì thi HSGQG Hóa học


Chủ đề 30: Rượu lá
Các hợp chất A và B được dùng làm thành phần điều chế nhiều loại
nước hoa và tinh chất thực phẩm. Khi pha loãng, chất chất đầu có mùi
của lá và cỏ tươi, còn chất sau có mùi của lá tử linh lan (violet) - chính
đây là nguồn gốc tên gọi thông thường của chúng: “rượu lá” và “rượu lá
tử linh lan”. Hợp chất B có công thức C9H16O, nhận được từ A qua sơ đồ
dưới đây. Còn để điều chế A thì có 3 phương pháp được sử dụng:

1) Xác định cấu tạo các hợp chất A-J.


Những biến đổi tương đối nhỏ trong cấu trúc “rượu lá” cũng dẫn đến
thay đổi lớn về mùi của hợp chất. Các đồng đẳng khác nhau 2 và 3
nguyên tử carbon có mùi xạ hương, còn hợp chất K, đồng phân của A,
có mùi một loại trái cây màu xanh lá. Chất K, được dùng làm thành phần
tinh chất thực phẩm, có thể nhận được theo sơ đồ sau:

Điều thú vị là M, khi pha loãng, có mùi táo xanh. Chất này được sử dụng
làm hương liệu “táo xanh”.
2) Xác định cấu trúc các hợp chất K, L, M.
Mùi cỏ và rau quả tươi cũng xuất hiện với hợp chất N, đồng phân của
các chất H và J, nhận được từ phản ứng giữa dimethyl vinyl carbinol và
isobutyric aldehyde (i-PrCHO) khi có mặt p-toluenesulfonic acid. Phổ
cộng hưởng từ hạt nhân của N có 6 tín hiệu ở 1.0, 1.6, 1.65, 2.1, 5.3 và 9.4
ppm với tỉ lệ cường độ 6:3:3:2:1:1.
3) Xác định cấu trúc chất N.

332 | Chuẩn bị kiến thức cho kì thi HSGQG Hóa học


Hướng dẫn
1) Cấu trúc hợp chất A có thể được xác định dựa vào quy trình tổng hợp
từ acetylene và but-1-yne. Phân tích sơ đồ ta thấy A là cis-alkene tạo
thành bởi phản ứng hydrogen hóa alkyne trên xúc tác Lindlar. Hợp chất
D không phải là alkyne đầu mạch, 1 nhóm thế là C2H5 và nhóm kia là
CH2CH2OH (but-1-yen bị deproton hóa bởi base mạnh, anion bị alkyl
hóa bởi alkyl halide bậc 1, sau đó nhóm bảo vệ tetrahydropyranyl bị loại
bỏ bởi phản ứng thủy phân acetal trong môi trường acid.) Hợp chất E là
sản phẩm là sản phẩm cộng syn của triethylaluminum vào acetylene.
Sản phẩm này, là hợp chất cơ kim, phản ứng với ethylene oxide bởi
phản ứng cộng cộng mở vòng, tạo thành “rượu lá” A.
Hợp chất A trong phản ứng với thionyl chloride bị chuyển thành
chloride G tương ứng. Tác nhân Grignard tạo thành từ chloride phản
ứng với acrolein tạo thành allyl alcohol bậc 3 H. Dưới tác động của acid,
alcohol bị đồng phân hóa tạo thành “rượu lá tử linh lan” B. Cấu trúc của
B có thể được xác định bởi phần dưới của sơ đồ. Hợp chất J cũng là allyl
alcohol đồng phân của H và được tạo thành bởi phản ứng giữa acrolein
và hợp chất cơ kim nhận được từ G. Rõ ràng cấu trúc của hợp chất này
khác biệt chỉ từ kim loại của tác nhân Grignard tương ứng. Hướng khả
thi duy nhất là hợp chất cơ đồng tạo thành sản phẩm công thức C9H16O,
khác với H, là sản phẩm cộng 1,4 - nghĩa là sự tấn công của hợp chất cơ
kim vào nguyên tử carbon β của hệ aldehyde không no α, β, tạo thành
một aldehyde bão hòa J. Aldehyde này được tạo thành từ B bởi phản
ứng oxid hóa (MnO2 là tác nhân oxid hóa các alcohol thành aldehyde
không no α, β) và sau đó là khử hóa aldehyde bởi sodium trong
ammonia.
Vấn đề khó nhất trong sơ đồ này là việc xác định cấu trúc hợp chất F.
Chất này được thấy là sản phẩm tạo thành bởi phản ứng cộng, xúc tác
acid, của formaldehyde vào 1,3-pentadiene. Phản ứng bắt đầu với sự
proton hóa một nguyên tử có mật độ điện tích lớnn hất. Rõ ràng nguyên
tử như vậy là oxygen trong formaldehyde. Hydroxylmethyl cation
HOCH2+ tạo thành tấn công vào nguyên tử C1 của penta-1,3-diene, tạo
thành allylic cation được bền hóa bởi tương tác nội phân tử với nhóm
hydroxy tạo thành vòng 6 cạnh. Phản ứng vòng hóa tạo thành vòng 4
cạnh thì không phù hợp bởi những hạn chế về mặt không gian. Do đó,
sơ đồ diễn ra như sau:

333 | Chuẩn bị kiến thức cho kì thi HSGQG Hóa học


2) Hợp chất K là đồng phân của A, nghĩa là có cùng công thức phân tử
C6H12O. Nó được tạo thành từ M bởi sự khử với lithium aluminum
hydride. Sẽ hợp lí khi giả sử rằng K là một alcohol không no, và M là hợp
chất carbonyl tương ứng.
Dựa vào cấu trúc của hợp chất đầu có thể kết luận rằng trong K phải có
một nhóm propyl. Xét quy trình tổng hợp L. Hợp chất này chứa 12
nguyên tử carbon, nghĩa là được tạo thành từ 2 phân tử (hợp chất đầu
và một hợp chất khác nữa). Phản ứng phải bắt đầu bởi sự tấn công của
Lewis acid vào nguyên tử có mật độ điện tích cao. Trong trường hợp
này là nguyên tử oxygen của nhóm carbonyl. Hydroxyalkyl cation tạo
thành phản ứng với nguyên tử carbon đầu mạch của ethyl vinyl ether

334 | Chuẩn bị kiến thức cho kì thi HSGQG Hóa học


tạo thành một hydroxyalkyl cation mới. Có hai hướng có thể xảy ra:
phân tử phản ứng thứ ba có thể là ethyl vinyl ether hoặc aldehyde ban
đầu:

Trong số hai hợp chất có thể tạo thành, L và L’, aldehyde không no chỉ
có thể tạo thành khi một chất trong số đó thủy phân và L’ không thỏa
mãn điều kiện này. Do đó, chúng ta xác định được cấu trúc L, M, K là:

3) Hợp chất N có công thức C9H16O, có nghĩa đây là sản phẩm ngưng tụ
của dimethyl vinyl carbinol và isobutyric aldehyde (i-PrCHO). Cấu trúc
của N có thể được dẫn ra từ dữ kiện phổ NMR. Tín hiệu ở 9.4 ppm với
cường độ mạnh cho thấy sự xuất hiện của 1 nhóm aldehyde. Tín hiệu ở
5.3 ppm tương ứng với nguyên tử hydrogen nhóm vinyl. Do đó, N là một
aldehyde không no, liên kết đôi C=C có 3 nhóm thế. 2 tín hiệu có cường
độ 3 ở 1.6 và 1.65 tương ứng với 2 nhóm methyl, một trong số đó được

335 | Chuẩn bị kiến thức cho kì thi HSGQG Hóa học


đặt ở vị trí cis so với nhóm thế thứ 3 và nhóm còn lại ở vị trí trans. Có 8
nguyên tử hydrogen, 6 trong số chúng rõ ràng thuộc về 2 nhóm methyl
giống hệt nhau, và 2 còn lại thuộc về nhóm CH2, nối với liên kết đôi. Hợp
chất N thu được như là kết quả của: a) sự tạo thành allyl cation bậc ba
bề từ dimethyl vinyl carbinol; b) sự enol hóa isobutyric aldehyde; c) tấn
công của nguyên tử carbon ít nhóm thế trong ally cation vào enol tạo
thành aldehdye.

336 | Chuẩn bị kiến thức cho kì thi HSGQG Hóa học


Chủ đề 31: Hợp chất cơ lithium
Các hợp chất cơ lithium có giá trị to lớn trong tổng hợp hữu cơ hiện đại.
Có 3 phương pháp cơ bản để điều chế chúng. Đầu tiên là đưa lithium
vào liên kết C-X (trong đó X thường là halogen). Ví dụ, n-butyllithium
được tạo thành khi xử lí lithium với dung dịch n-butyl iodide trong
hexane. Sản phẩm tạo thành là cụm phức chất có công thức (RLi)6.
1) Độ kết hợp của các alkyl lithium biến đổi như thế nào khi thay dung
môi hydrocarbon bằng các ether (diethyl ether, THF)? Giải thích.
Trong điều chế n-butyllithium với phản ứng trong ether, hiệu suất bị
hạ thấp do sự tạo thành các sản phẩm phụ là hợp chất X, Y, Z với hàm
lượng carbon lần lượt là 85.7 %, 84.2 %, 82.8 %.
2) Xác định công thức cấu tạo các hợp chất X, Y, Z.
Phương pháp thứ hai để điều chế các hợp chất cơ kim là trao đổi
halogen với lithium bởi phản ứng của RHal với lượng dư R’Li (hiệu quả
đạt được cao nhất với tert-butyllithium). Phản ứng với RI diễn ra ngay
tức thì thậm chí ở -78 oC, với các bromide khá nhanh nhưng lại gần như
không hiệu quả với các chloride.
Thêm 19 mL dung dịch tert-butyllithium 1 M trong diethyl ether được
thêm vào 2.32 gam 2-phenylethyl iodide. Sau vài phút, thêm lượng dư
benzaldehyde vào hỗn hợp phản ứng, sau đó là nước. Phân tích hỗn hợp
sản phẩm cho thấy chỉ có duy nhất một alcohol W có mặt trong đó.
3)
a) Xác định cấu trúc alcohol W và gọi tên nó theo danh pháp
IUPAC.
b) Cho biết những sản phẩm nào khác có trong hỗn hợp sản phẩm.
Viết tất cả các phản ứng diễn ra trong thí nghiệm này.
c) Tính lượng chất W (theo gam), giả sử rằng khi benzaldehyde
được thêm vào thì RLi chuyển hóa hoàn toàn thành alcohol.
Phương pháp thứ ba để tổng hợp các hợp chất cơ lithium là phản ứng
giữa các dẫn xuất alkyllithium với hợp chất chứa 1 nguyên tử hydrogen
có tính acid cao. Ví dụ như các tri- và diarylmethane, alkyne đầu mạch
dễ tham gia vào phản ứng. Tuy nhiên, toluene không phản ứng với RLi
bất chấp độ bền cao của các anion kiểu benzyl. Đồng thời, butyllithium

337 | Chuẩn bị kiến thức cho kì thi HSGQG Hóa học


có thể deproton hóa các alkene, hợp chất thơm và thậm chí cả các gốc
hydrocarbon no, nếu chúng có chứa oxygen, nitrogen, sulfur, …
4) Từ anisol và acetyl chloride, tùy thuộc vào điều kiện tiến hành, có
thể nhận được para hoặc ortho-methoxyacetophenon, với hiệu
suất tạo thành mỗi đồng phân là 80-90 %. Đề xuất các phương
pháp để điều chế các methoxyacetophenon, chỉ ra tác nhân và
điều kiện phù hợp cho phản ứng.

338 | Chuẩn bị kiến thức cho kì thi HSGQG Hóa học


Hướng dẫn
1) Không như hexane và các alkane khác, dung môi ether có thể tạo
phối trí với lithium cation do sự tạo thành các tương tác cho-nhận giữa
lithium và các cặp electron trên nguyên tử oxygen. Do đó, độ kết hợp
của các alkyl lithium trong dung môi ether sẽ thấp hơn so với trong
dung môi hydrocarbon.
2) Theo điều kiện điều chế, X, Y, Z chỉ có thể chứa carbon, hydrogen,
iodine và lithium. Hơn nữa, trong phản ứng của n-butyl iodide với
lithium, chỉ có các hợp chất chứa 4n nguyên tử carbon có thể được tạo
thành. Xét hàm lượng carbon trong phân tử, đối với X ta có: 85.7 %
carbon - 4n nguyên tử. Với n = 1, MX = 48/0.857 = 56. Giá trị này tương
ứng với công thức C4H8 (butene). Chất này có thể được tạo thành bởi sự
tách HI từ butyl iodide dưới tác động của butyl lithium (base). Do đó A
là but-1-ene. Với n = 2, MX = 96/0.857 = 112. Tuy nhiên, lời giải thích hợp
lí cho sự tạo thành một phân tử (C8H16 và C8H9Li) dưới điều kiện như vậy
là không thể đưa ra. Biện luận tiếp ta xác định được Z là C4H10 (n-
butane) và Y là C8H18 (n-octane).

3) a) Theo điều kiện đã cho, các iodide tham gia vào phản ứng trao đổi
gần như tức thì. Kết quả là 2-phenylethyl lithium được tạo thành từ 2-
phenylethyl iodide, chất này phản ứng với benzaldehyde tạo thành
alcoholate ion, được trung hòa bởi nước tạo thành alcohol.

Danh pháp của W theo IUPAC: 1,3-diphenylpropan-1-ol.


b) 2.32 gam 2-phenylethyl iodide tương ứng với 0.01 mol; 19 mL dung
dịch tert-buthyllithium - 0.019 mol. Nếu chỉ các phản ứng được đề cập

339 | Chuẩn bị kiến thức cho kì thi HSGQG Hóa học


ở trên diễn ra thì chỉ còn lại 0.009 mol t-butyllithium còn lại và cũng sẽ
phản ứng với benzaldehyde tạo thành alcohol thứ hai, điều này mâu
thuẫn với điều kiện. Mặt khác, khi phản ứng với các alkyl halide, các tác
nhân alkyllithium có thể đóng vai trò như base, dẫn đến sự tạo thành
sản phẩm tách, hoặc (mặc dù rất hiếm khi - như tác nhân nucleophile,
tạo thành sản phẩm thế.) Tert-butyl iodide tham gia vào phản ứng thế
nucleophile không đáng kể, đặc biệt là khi tương tác với base mạnh.
Phản ứng trao đổi halogen-kim loại diễn ra trong phản ứng của tert-
butyllithium với tert-butyl iodide không thể diễn ra. Do đó, phản ứng
tách là quá trình duy nhất xảy ra giữa chúng. Phản ứng tách cũng là một
quá trình thứ cấp trong tương tác giữa tert-butyllithium với chất đầu
2-phenylethyl iodide.

c) 1.908 gam.
4) Điều chế p-methoxyacetophenone là phản ứng acyl hóa Friedel-
Crafts cổ điển. Dưới những điều kiện của phản ứng này, đồng phân
ortho được tạo thành với lượng rất nhỏ. o-methoxyacetophenone
được điều chế bằng cách deproton hóa các hợp chất thơm chứa 1
oxygen. Do sự phối trí của lithium ion trên nguyên tử oxygen của
anisole, quá trình deproton hóa diễn ra chủ yếu ở vị trí ortho. Phản ứng
của anion tạo thành với một tác nhân acyl hóa (acetyl chloride,
anhydride, ethyl acetate, ….) tạo thành o-methoxyacetophenone
mong muốn.

340 | Chuẩn bị kiến thức cho kì thi HSGQG Hóa học


Chủ đề 32: Phản ứng cộng electrophile vào hệ liên
hợp mở
1)
a) Đề xuất sản phẩm cộng 1,2 và 1,4 của 1 đương lượng HCl vào 1,3-
pentadiene.

b) Biểu diễn các trung gian carbocation tạo thành trong phản ứng
cộng ở trên. Dự đoán các sản phẩm cộng 1,2 và cộng 1,4 chiếm
ưu thế.
2) Xác định cấu trúc các sản phẩm cộng 1,2 và 1,4 của 1 đương lượng
HCl vào 2,3-dimethylcyclohexa-1,3-diene.
3) Dự đoán các sản phẩm cộng khả dĩ của diene sau với 1 đương lượng
HCl:

4) Các sản phẩm cộng 1,2 và 1,4 được tạo thành từ phản ứng cộng HBr
với 1,3-butadiene (theo tỉ lệ 1:1) tồn tại trong cân bằng ở 40 oC. Hãy
đề xuất cơ chế giải thích sự chuyển hóa qua lại giữa các sản phẩm
này.

341 | Chuẩn bị kiến thức cho kì thi HSGQG Hóa học


Hướng dẫn
1) a)

b)

Carbocation A và D, được bền hóa cộng hưởng, dễ tạo thành hơn B và


C. Điện tích dương trên carbocation allylic A được giải tỏa trên hai
nguyên tử carbon bậc hai, trong khi đó trong D thì điện tích dương được
giải tỏa trên một carbon bậc hai và một carbon bậc một. Do đó, ta dự
đoán rằng carbocation A là sản phẩm trung gian chính và 4-chloro-2-
pentene chiếm ưu thế. Chú ý rằng sản phẩm này được tạo thành cả từ
phản ứng cộng 1,2 lẫn 1,4.

342 | Chuẩn bị kiến thức cho kì thi HSGQG Hóa học


2)

3) Sản phẩm cộng 1,2 và 1,4 lần lượt là 3-bromo-1-butene và 1-bromo-


2-butene:

Các dẫn xuất allylic halide có thể trải qua sự phân li chậm, tạo thành
carbocation được bền hóa (như trong phản ứng thế SN1). Cả 3-bromo-
1-butene và 1-bromo-2-butene đều tạo thành cùng một allylic
carbocation như trên. Phản ứng cộng của ion bromide vào allylic
carbocation diễn ra sau đó tạo thành một hỗn hợp của các
bromobutene. Phản ứng diễn ra trong điều kiện cân bằng và 1-bromo-
2-butene bền nhiệt động hơn chiếm ưu thế.

343 | Chuẩn bị kiến thức cho kì thi HSGQG Hóa học


4)

344 | Chuẩn bị kiến thức cho kì thi HSGQG Hóa học


Chủ đề 33: Phản ứng Diels-Alder
1) Biểu diễn trạng thái chuyển tiếp và dự đoán sản phẩm của phản
ứng cộng Diels–Alder sau:

2) Dự đoán hợp chất nào dưới đây sẽ là dineophile tốt cho phản ứng
Diels-Alder:

3) Diene nào sau đây có cấu hình s-cis và diene nào có cấu dạng s-
trans? Trong số các diene s-trans, chất nào có thể dễ dàng bị quay
cấu dạng thành s-cis?

4) Biểu diễn sản phẩm phản ứng Diels-Alder của diene sau với 3-
buten-2-one, CH2=CHCOCH3. Chỉ rõ hóa lập thể của sản phẩm.

345 | Chuẩn bị kiến thức cho kì thi HSGQG Hóa học


5) Biểu diễn cấu trúc, chỉ rõ hóa lập thể, của sản phẩm phản ứng
Diels-Alder sau:

6) Đề xuất chất phản ứng ban đầu để điều chế các hợp chất sau bởi
phản ứng Diels-Alder:

7) Dự đoán sản phẩm tạo thành từ phản ứng của 1,3-cyclohexadiene


với mỗi tác nhân sau:
a) 1 đương lượng Br2 trong CH2Cl2.
b) O3 sau đó xử lí với kẽm.
c) 1 đương lượng HCl trong ether.
d) 1 đương lượng DCl trong ether.
e) 3-buten-2-one (CH2=CHCOCH3).
f) OsO4 dư, sau đó với NaHSO3.

346 | Chuẩn bị kiến thức cho kì thi HSGQG Hóa học


8)
a) Aldrin – một loại thuốc trừ sâu chứa chloro, ngày nay đã bị cấm
sử dụng ở Mỹ - có thể được tạo thành bởi phản ứng Diels-Alder
giữa hexachloro-1,3-cyclopentadiene với norbornadiene. Biểu
diễn cấu trúc của aldrin.

b) Có thể điều chế norbornadiene từ phản ứng của chloroethylene


với 1,3-cyclopentadiene, sau đó xử lí sản phẩm tạo thành với
sodium ethoxide. Viết sơ đồ phản ứng và cho biết hai phản ứng
trên thuộc loại nào?
9) Đề xuất cơ chế cho phản ứng sau:

10) Benzenediazonium carboxylate phân hủy khi đun nóng tạo thành N2,
CO2 và một hợp chất rất hoạt động (không thể phân lập được). Khi đun
nóng benzenediazonium carboxylate trong furan thì quan sát được
phản ứng sau:

347 | Chuẩn bị kiến thức cho kì thi HSGQG Hóa học


Hướng dẫn
1)

2) Các dienophile tốt thường có nhóm hút electron liên hợp với liên kết
đôi, tức là các chất:

3) a) Diene này có cấu dạng s-cis.


b) Diene này có cấu dạng s-trans. Bởi các liên kết đôi thuộc hệ vòng
dung hợp nên chusng không thể quay để tạo thành cấu dạng s-cis.
c) Sự quay cấu dạng có thể diễn ra quanh liên kết đơn của diene s-
trans này. Tuy nhiên, cấu dạng s-cis tạo thành có tương tác không gian
kém thuận lợi của nhóm methyl với hydrogen ở carbon 1.

348 | Chuẩn bị kiến thức cho kì thi HSGQG Hóa học


4)

5) Diene quay thành cấu dạng s-cis. Mối liên hệ trans của hai nhóm
ester trong dienophile được bảo toàn trong sản phẩm.

6)

349 | Chuẩn bị kiến thức cho kì thi HSGQG Hóa học


7)

350 | Chuẩn bị kiến thức cho kì thi HSGQG Hóa học


8) a)

b)

351 | Chuẩn bị kiến thức cho kì thi HSGQG Hóa học


9) Phản ứng Diels-Alder giữa α-pyrone (diene) và alkyne (dienophile)
tạo thành sản phẩm sau:

Các liên kết đôi trong sản phẩm này không liên hợp và sản phẩm bền
hơn có thể được tạo thành bởi sự tách CO2:

10)

352 | Chuẩn bị kiến thức cho kì thi HSGQG Hóa học


Chủ đề 34: Hợp chất thơm
1) Xác định danh pháp IUPAC của các hợp chất sau:

2) Biểu diễn cấu tạo của các hợp chất sau:


a) p-Bromochlorobenzene
b) p-Bromotoluene
c) m-Chloroaniline
d) 1-Chloro-3,5-dimethylbenzene
3) Theo quy tắc Hückel, hydrocarbon thơm phải có 4n + 2 electron pi
và có hệ liên hợp vòng. Tuy nhiên, 1,3,5,7,9-cyclodecapentaene
thỏa mãn một trong các tiêu chí này nhưng không thỏa mãn tiêu
chí còn lại, và ngăn trở mọi nỗ lực tổng hợp hợp chất này. Giải thích.

4)
a) Biểu diễn 5 công thức cộng hưởng của anion cyclopentadienyl.
Trong anion này, các liên kết carbon-carbon có tương đương
nhau không?
b) Khi xử lí 3-chlorocyclopropene với AgBF3 thì tạo thành kết tủa
AgCl và dung dịch của một sản phẩm bền. Dự đoán cấu trúc của
sản phẩm và giải thích tính bền của nó.

353 | Chuẩn bị kiến thức cho kì thi HSGQG Hóa học


c) Cyclopropanone có hoạt tính cao (dễ phản ứng) bởi có sức căng
góc lớn. Nhưng methylcyclopropenone, dù có sức căng góc
thậm chí còn lớn hơn cyclopropanone, nhưng lại tương đối bền
và thậm chí có thể được chưng cất. Dựa vào sự phân cực của
nhóm carbonyl, hãy giải thích sự khác biệt này.

d) Cycloheptatrienone thì bền, nhưng cyclopentadienone lại rất


dễ tham gia phản ứng, đến độ không thể phân lập được. Hãy giải
thích dựa vào sự phân cực của nhóm carbonyl.

5)
a) Cyclooctatetraene dễ phản ứng với kim loại potassium tạo
thành dianion cyclooctatetraene, C8H82-. Tại sao phản ứng này
dễ xảy ra? Dự đoán dạng hình học của dianion này.

b) Hãy dự đoán tiểu phân (carbocation, carbanion, gốc) cyclo-


nonatetraenyl nào bền nhất? Tại sao?
6)
a) Azulene, một hydrocarbon có màu xanh dương rất đẹp, là đồng
phân của naphthalene. Hợp chất này có tính thơm không? Biểu
diễn dạng cộng hưởng còn lại của azulene.

354 | Chuẩn bị kiến thức cho kì thi HSGQG Hóa học


b) Biểu diễn các dạng cộng hưởng của anthracene.

c) Calicene là một hydrocarbon với moment lưỡng cực lớn bất


thường. Hãy giải thích lí do.

d) Pentalene là một trong những phân tử rất kém bền, chỉ được
phân lập ở nhiệt độ cực kì thấp (bởi nitrogen lỏng). Tuy nhiên,
dianion pentalene thì lại tương đối bền. Hãy giải thích cho sự
khác biệt về độ bền này.

355 | Chuẩn bị kiến thức cho kì thi HSGQG Hóa học


Hướng dẫn
1)

2)

3) Cyclodecapentaene có 4n + 2 electron π (n = 2) nhưng không phẳng.


Nếu hợp chất này phẳng, các nguyên tử hydrogen đánh dấu hoa thị sẽ
có tương tác lẫn nhau như minh họa. Để tránh tương tác này, hệ vòng
phải biến dạng khỏi dạng phẳng.

4) a) Cấu trúc thực của anion cyclopentadienyl là dạng lai hóa của tất
cả các cấu trúc dưới đây và chỉ chứa duy nhất một loại nguyên tử

356 | Chuẩn bị kiến thức cho kì thi HSGQG Hóa học


carbon và một loại nguyên tử hydrogen. Tất cả các liên kết carbon-
carbon đều tương đương nhau.

b) Sản phẩm tạo thành có chứa cation cyclopropenyl C3H3+ - nó là một


hệ thơm 2 electron pi với các cấu trúc cộng hưởng như dưới đây:

c) Trong cấu trúc cộng hưởng A, methylcyclopropenone là hợp chất


liên hợp vòng với ba electron π trong vòng. Tuy nhiên, do oxygen âm
điện hút các electron π của liên kết carbon-oxygen nên có thể biểu
diễn cấu trúc cộng hưởng B, trong đó các electron π của carbonyl được
định xứ trên oxygen, nên trong vòng chỉ còn lại 2 electron π. Khi đó,
vòng methylcyclopropenone có tính thơm và được dự đoán là bền dù
chỉ là vòng ba cạnh.

d)

357 | Chuẩn bị kiến thức cho kì thi HSGQG Hóa học


Tương tự như ý c, ta có thể biểu diễn các dạng cộng hưởng trong đó cả
hai electron π của carbonyl đều định xứ trên oxygen. Vòng
cycloheptatrienone trong B có 6 electron π và có tính thơm theo quy
tắc Hückel; trong khi đó vòng cyclopentadienone trong D có 4
electron π và có tính phản thơm.
5) a) Khi cyclooctatetraene nhận 2 electron, nó trở thành các ion thơm
với (4n + 2) electron π. Dianion tạo thành có cấu trúc phẳng với góc liên
kết carbon-carbon là 135o (của bát giác đều).
b) Cấu trúc của các tiểu phân carbocation – gốc – carbanion được lần
lượt biểu diễn như sau:

Theo đó, hệ 10 electron π (anion) là bền nhất bởi nó là hệ thơm.


6) a) Azulene có tính thơm bởi nó có vòng liên hợp 10 electron π.

b)

c) Sự tái phân bố các electron π trong calicene tạo thành một dạng
cộng hưởng trong đó cả hai vòng đều có tính thơm và có moment lưỡng
cực lớn.

358 | Chuẩn bị kiến thức cho kì thi HSGQG Hóa học


d) Pentalene có 8 electron π và là hệ phản thơm, nhưng dianion
pentalene thì có 10 electron π và là hệ thơm.

359 | Chuẩn bị kiến thức cho kì thi HSGQG Hóa học


Chủ đề 35: Dị vòng thơm
1) Biểu diễn giản đồ orbital của pyridine và furan để giải thích tính
thơm của chúng:

2) Mỗi nguyên tử nitrogen trong purine đóng góp bao nhiêu electron
vào hệ π thơm?

3) Trong phản ứng với acid, 4-pyrone bị proton hóa trên oxygen của
nhóm carbonyl tạo thành sản phẩm bền có tính cation. Dựa vào các
cấu trúc cộng hưởng và quy tắc Hückel, hãy giải thích tại sao sản
phẩm proton hóa lại bền.

4) Hợp chất N-phenylsydnone – được đặt tên nhằm lưu danh Đại học
Sydney, Australia là nơi đầu tiên tổng hợp thành công – có những
tính chất tương tự một hệ thơm điển hình. Giải thích tại sao.

360 | Chuẩn bị kiến thức cho kì thi HSGQG Hóa học


Hướng dẫn
1) Hai hệ vòng liên hợp phẳng này đều được tạo thành bởi 6 electron π:

2) Purine là phân tử thơm 10 electron π. Nguyên tử nitrogen N-H trong


vòng 5 cạnh đóng góp cả hai electron của cặp electron chưa liên kết
cho hệ thơm, còn mỗi nguyên tử nitrogen khác đóng góp một electron.

3) Sự proton hóa 4-pyrone tạo thành cấu trúc A, với các dạng cộng
hưởng B – F. Trong E và F, cặp electron chưa liên kết của oxygen trong
vòng được giải tỏa vào vòng, tạo thành hệ 6 thơm electron π.

4)

361 | Chuẩn bị kiến thức cho kì thi HSGQG Hóa học


Dạng cộng hưởng thứ hai của N-phenylsydnone thể hiện rõ tính thơm
của vòng năm cạnh. Ở dạng này, oxygen trong vòng đóng góp 2
electron vào hệ π, mỗi nguyên tử nitrogen đóng góp 1 electron và mỗi
nguyên tử carbon đóng góp 1 electron (oxygen của nhóm carbonyl
mang điện tích âm hình thức). Đây là hệ vòng liên hợp phẳng với 6
electron π, thỏa mãn quy tắc Hückel.

362 | Chuẩn bị kiến thức cho kì thi HSGQG Hóa học


Chủ đề 36: Phản ứng thế electrophile nhân thơm
1) Sắp xếp các nhóm sau theo trình tự giảm dần khả năng tham gia
phản ứng thế electrophile nhân thơm:
a) Nitrobenzene, phenol, toluene, benzene
b) Phenol, benzene, chlorobenzene, benzoic acid
c) Benzene, bromobenzene, benzaldehyde, aniline
2) Dự đoán sản phẩm chính của các phản ứng sau:
a) Nitro hóa bromobenzene.
b) Bromo hóa nitrobenzene.
c) Chloro hóa phenol.
d) Bromo hóa aniline.
3) Giải thích tại sao acetanilide có hoạt tính kém hơn aniline trong
phản ứng thế electrophile nhân thơm.

4) Biểu diễn các cấu trúc cộng hưởng của các trung gian phản ứng thế
electrophile vào các vị trí ortho, meta, para của nitrobenzene.
Trung gian nào bền nhất?
5) Dự đoán vị trí diễn ra phản ứng thế electrophile trong mỗi hợp chất
sau đây:

6)
a) Có thể thực hiện phản ứng iodo hóa nhân thơm với một số tác
nhân, bao gồm iodine monochloride, ICl. Xác định chiều phân
cực của ICl và đề xuất cơ chế phản ứng iodo hóa của một vòng
thơm bởi ICl.
b) Phản ứng sulfo hóa nhân thơm với SO3 và H2SO4 là phản ứng
thuận nghịch. Nghĩa là, khi đun nóng benzenesulfonic acid với

363 | Chuẩn bị kiến thức cho kì thi HSGQG Hóa học


H2SO4 tạo thành benzene. Hãy trình bày cơ chế phản ứng của
phản ứng desulfo hóa.
7) Nhóm N,N,N trimethylammonium -N+(CH3)3, là một trong số ít các
nhóm phản hoạt hóa, định hướng meta mà không gây hiệu ứng cộng
hưởng hút electron. Giải thích đặc tính này.

364 | Chuẩn bị kiến thức cho kì thi HSGQG Hóa học


Hướng dẫn
1) a) Phenol > toluene > benzene > nitrobenzene
b) Phenol > benzene > chlorobenzene > benzoic acid
c) Aniline > benzene > bromobenzene > benzaldehyde
2)
a)

3) Với acetanilide, sự giải tỏa cộng hưởng của cặp electron chưa liên
kết của nitrogen vào vòng thơm kém thuận lợi hơn vì điện tích dương
trên nitrogen cạnh nhóm carbonyl phân cực dương điện. Sự giải tỏa

365 | Chuẩn bị kiến thức cho kì thi HSGQG Hóa học


cộng hưởng của oxygen carbonyl thuận lợi hơn bởi điện tích âm của
oxygen. Bởi vậy, mật độ electron nhường vào vòng thơm trong trường
hợp acetanilide kém hơn trong aniline, dẫn đến acetanilide có hoạt
tính kém hơn aniline (trong phản ứng thế electrophile nhân thơm).

4) Trung gian tạo thành từ hướng tác kích meta là thuận lợi nhất:

366 | Chuẩn bị kiến thức cho kì thi HSGQG Hóa học


5)

367 | Chuẩn bị kiến thức cho kì thi HSGQG Hóa học


6) a)

b)

7) Khi electrophile tác kích vào vòng thơm mang nhóm -C(CH3)3+:

Nhóm N,N,N-trimethylammonium không có hiệu ứng cộng hưởng


electron bởi nó không có orbital p trống để xen phủ với hệ orbital π của

368 | Chuẩn bị kiến thức cho kì thi HSGQG Hóa học


vòng thơm. Tuy nhiên, nhóm -N+(CH3)3 gây phản ứng hoạt hóa cảm ứng,
bởi nó mang điện tích dương. Nó là nhóm định hướng meta bởi trung
gian cationic tạo thành từ sự tác kích meta bền hơn đôi chút so với sản
phẩm tạo thành từ sự tác kích ortho và para.

369 | Chuẩn bị kiến thức cho kì thi HSGQG Hóa học


Chủ đề 37: Phản ứng Friedel-Crafts
1) Xác định sản phẩm một lần thế chính của phản ứng alkyl hóa
Friedel–Crafts giữa benzene với 1-chloro-2-methylpropane khi có
mặt AlCl3.
2) Đề xuất các carboxylic acid chloride ban đầu để điều chế mỗi
acylbenzene sau đây bởi phản ứng acyl hóa Friedel–Crafts:

3) Dự đoán sản phẩm của các phản ứng sau:

4) Carbocation electrophile trong phản ứng Friedel–Crafts có thể


được tạo thành theo các cách khác ngoài phản ứng alkyl chloride
với AlCl3. Ví dụ, phản ứng của benzene với 2-methylpropene khi có
mặt H3PO4 tạo thành tert butylbenzene. Hãy đề xuất cơ chế cho
phản ứng này.
5) Triphenylmethane có thể được điều chế bởi phản ứng của benzene
và chloroform khi có mặt AlCl3. Hãy đề xuất cơ chế phản ứng.

370 | Chuẩn bị kiến thức cho kì thi HSGQG Hóa học


6)
a) Đề xuất cơ chế của phản ứng benzene với 2,2,5,5
tetramethyltetrahydrofuran:

b) Đề xuất cơ chế của phản ứng sau:

c) Trong phản ứng Gatterman–Koch, nhóm formyl (-CHO) được


đưa trực tiếp vào vòng benzene. Ví dụ, phản ứng của toluene với
CO và HCl khi có mặt CuCl/AlCl3, tạo thành p-methyl
benzaldehyde. Hãy đề xuất cơ chế phản ứng:

371 | Chuẩn bị kiến thức cho kì thi HSGQG Hóa học


Hướng dẫn
1) Carobcation isobutyl tạo thành ban đầu khi 1-chloro-2-
methylpropane phản ứng với AlCl3 sẽ chuyển vị qua sự chuyển dịch
hydride, tạo thành carbocation tert-butyl bền hơn, tiểu phân này sau
đó sẽ alkyl hóa benzene, tạo thành tert-butylbenzene.

2)

3)

372 | Chuẩn bị kiến thức cho kì thi HSGQG Hóa học


4)

5)

Dichloromethyl)benzene có thể phản ứng với hai đương lượng benzene


nữa bởi cơ chế tương tự để tạo thành triphenylmethane.

373 | Chuẩn bị kiến thức cho kì thi HSGQG Hóa học


6) a)

374 | Chuẩn bị kiến thức cho kì thi HSGQG Hóa học


b)

c) Các bước phản ứng:

375 | Chuẩn bị kiến thức cho kì thi HSGQG Hóa học


Chủ đề 38: Phản ứng thế nucleophile
Xử lí p-bromotoluene với NaOH ở 300 oC tạo thành hỗn hợp hai sản
phẩm, nhưng khi xử lí m-bromotoluene với NaOH trong điều kiện tương
tự thì tạo thành hỗn hợp ba sản phẩm. Giải thích tại sao.
Hướng dẫn
1) Xử lí m-bromotoluene với NaOH dẫn đến sự tạo thành hai trung gian
benzyne khả dĩ, tiểu phân này phản ứng với nước tạo thành ba sản
phẩm methylphenol.

376 | Chuẩn bị kiến thức cho kì thi HSGQG Hóa học


Chủ đề 39: Phản ứng oxid hóa
1) Dự đoán sản phẩm thơm có thể tạo thành từ phản ứng oxid hóa bởi
KMnO4 các hợp chất sau:

2) Dự đoán sản phẩm tạo thành từ các phản ứng sau:

377 | Chuẩn bị kiến thức cho kì thi HSGQG Hóa học


Hướng dẫn
1) Phản ứng diễn ra ở vị trí benzylic. KMnO4 oxid hóa nhóm methyl
nhưng sẽ để lại nhóm tert-butyl.

2)

378 | Chuẩn bị kiến thức cho kì thi HSGQG Hóa học


379 | Chuẩn bị kiến thức cho kì thi HSGQG Hóa học
Chủ đề 40: Tổng hợp hữu cơ
1) Đề xuất quy trình tổng hợp các hợp chất sau đây từ benzene:
a) m-Chloronitrobenzene
b) m-Chloroethylbenzene
c) 4-Chloro-1-nitro-2-propylbenzene
d) 3-Bromo-2-methylbenzenesulfonic acid
2) Đề xuất quy trình tổng hợp các hợp chất sau đây từ benzene:

3) Xác định các tác nhân a – e trong sơ đồ sau:

380 | Chuẩn bị kiến thức cho kì thi HSGQG Hóa học


Hướng dẫn
1) a)

b)

c)

d)

381 | Chuẩn bị kiến thức cho kì thi HSGQG Hóa học


2)

3) a) CH3CH2COCl, AlCl3; b) H2, Pd/C; c) Br2, FeBr3; d) NBS, (PhCO2)2; e)


KOH, ethanol.

382 | Chuẩn bị kiến thức cho kì thi HSGQG Hóa học


Chủ đề 41: Cơ chế phản ứng
1) Đề xuất cơ chế phản ứng fluor hóa benzene bởi F-TEDA-BF4.

2) Herbicide oxyfluorfen có thể được điều chế bởi phản ứng giữa
phenol và một aryl fluoride. Hãy đề xuất cơ chế phản ứng.

383 | Chuẩn bị kiến thức cho kì thi HSGQG Hóa học


Hướng dẫn
1) Các electron pi của benzene tác kích fluorine của F-TEDA-BF4 và
trung gian không thơm tách hydrogen, tạo thành sản phẩm fluoro hóa.

2) Bước 1 là phản ứng cộng hợp nucleophile, tiếp theo là tách loại ion
fluoride. Nhóm nitro làm cho vòng suy giảm mật độ electron và dễ bị
tác kích bởi nhóm nucleophile RO-. Nó cũng giúp bền hóa phức
Meisenheimer mang điện tích âm.

384 | Chuẩn bị kiến thức cho kì thi HSGQG Hóa học


Chủ đề 42*: Chuyển hóa của alkene
Alkane là những hợp chất kém hoạt động, tuy nhiên, vẫn tham gia vào
một số phản ứng thế theo cơ chế gốc tự do với một số chất, bao gồm
chất lỏng không màu A. Một số chuyển hóa của A và sản phẩm phản
ứng của nó với alkane X được biểu diễn trong sơ đồ sau:

- K1 và K2 là các đồng phân tạo thành trong phản ứng giữa A và


X.
- Sự xuất hiện của màu sắc đặc trưng khi tạo thành M1 và M2 cho
phép chúng ta đánh giá bản chất sự thay thế các nhóm chức
của K1, K2 và các đồng đẳng của chúng.
- Chất lỏng A, sản phẩm của phản ứng giữa nitric acid khan, dư
với acetic anhydride, được sử dụng trong hóa sinh để biến tính
amino acid tyrosine (Tyr). Sản phẩm phản ứng của A với Tyr có
màu vàng tương tự như màu xuất hiện trên da khi tiếp xúc với
nitric acid mà không có găng tay bảo vệ.

385 | Chuẩn bị kiến thức cho kì thi HSGQG Hóa học


- Đôi khi, A cũng được dùng để tách các đối quang của các
hydrocarbon thơm do nó dễ tạo thành các phức chất kém bền
với chúng.
1) Xác định cấu trúc của tất cả các chất chưa biết trong sơ đồ.
2) Xác định cấu trúc các dạng meso của sản phẩm tạo thành từ phản
ứng của Tyr với A. Cho biết dạng nào trong số đó chịu trách nhiệm
cho sự tạo màu của sản phẩm.
3) Đưa ra cấu trúc của bất kì hydrocarbon quang hoạt nào không
chứa các nguyên tử carbon no.

386 | Chuẩn bị kiến thức cho kì thi HSGQG Hóa học


Hướng dẫn
1) Từ chuỗi chuyển hóa A → B → C, có thể giả sử rằng B là sản phẩm
cộng (có thể là một phức chất) của anthracene với A và khi phản ứng
với một tác nhân dienophile hoạt động (tetracyanoethylene) thì sẽ
tách loại A, tạo thành sản phẩm bền C (Công thức phân tử của C tương
ứng với sản phẩm cộng của tetracyanoethylene và anthracene). Do đó
có thể suy ra công thức phân tử của A = B (C15H10N4O8) + C6N4 - C
(C20H10N4) = CN4O8. Đồng phân bền duy nhất ứng với công thức này chỉ
có thể là tetranitromethane - C(NO2)4.
Tương tự, công thức phân tử của chất D cũng được dễ dàng xác định:
D = B (C15H10N4O8) - H (C14H9NO2) = CHN3O6 - D chỉ có thể là
trinitromethane. D và H được tạo thành khi đun nóng sản phẩm cộng
của anthracene với tetranitromethane (A), vậy H là sản phẩm nitro hóa
của anthracene.
Khi đun nóng A với dung dịch kiềm - alcohol thì tạo thành D và E. Có
thể xác định được rằng: Е = А + KOH - D = CN4O8 + KOH - CHN3O6 = KNO3.
Khi đun nóng, KNO3 bị phân hủy thành KNO2 và O2, nghĩa là G = KNO2.

Đun nóng A với alkane X tạo thành một sản phẩm thế mono theo cơ
chế gốc tự do (K). Sự tạo thành 2 đồng phân cho thấy sự tồn tại của 2
loại hydrogen trong alkane ban đầu. Bản chất của chúng có thể được
đánh giá từ thực tế rằng phản ứng phân hủy trong môi trường acid các
đồng phân dẫn xuất nitro của K tạo thành các hợp chất carbonyl L có
tốc độ khác nhau. (Các hợp chất đồng phân C9H18O không thể là allyl
alcohol, epoxide hoặc cycloalkanol do tính kém bền của chúng trong
môi trường acid và việc chúng không thể tạo thành từ dẫn xuất nitro).

387 | Chuẩn bị kiến thức cho kì thi HSGQG Hóa học


Đồng phân alkane C9 duy nhất thỏa mãn các điều kiện này là 2,2,4,4-
tetramethylpentane (X).

Màu sắc của M1 là do sự tồn tại của một hệ liên hợp; hợp chất M2 có
màu xanh lục đặc trưng của các hợp chất nitroso.
2) Màu sắc là do đóng góp của cấu trúc quinoid (ii), sự tạo thành cấu
trúc này được hỗ trợ bởi liên kết hydrogen nội phân tử trong (i).

388 | Chuẩn bị kiến thức cho kì thi HSGQG Hóa học


3) Ví dụ về các hợp chất như vậy:

389 | Chuẩn bị kiến thức cho kì thi HSGQG Hóa học


Chủ đề 43*: Chuyển hóa của cycloalkene
Thông thường, các đồng đẳng chỉ khác nhau 1 hoặc 2 nhóm CH2 (không
liên kết trực tiếp với nhóm chức) thì sẽ có phản ứng giống nhau với các
tác nhân. Tuy nhiên, đôi khi những thay đổi về mặt cấu trúc tưởng
chừng không đáng kể lại dẫn tới những khác biệt đáng chú ý trong tính
chất hóa học của các hợp chất. Dưới đây là sơ đồ phản ứng có sự tham
gia của 3 cycloalkene đơn giản: cyclopentene, cyclohexene và
cycloheptene.

*ПФК là PFC (polyphosphoric acid).


Khi xử lí F5 - F7 với N-bromosuccinimide thì trong mỗi trường hợp sẽ
tạo thành 3 dẫn xuất monobromo. Đồng thời, một trong các sản phẩm
F, khi đun nóng với platinum trong không khí, tạo thành hydrocarbon
G có màu xanh dương; một chất khác trong cùng điều kiện sẽ chuyển
thành hợp chất H không màu; còn chất thứ ba hoàn toàn không phản
ứng.
1) Xác định cấu trúc của các hợp chất A - H.
2) Tính chênh lệch hàm lượng (%) bromine trong các dibromide tạo
thành bằng cách cộng hợp bromine vào các hợp chất G và H.
3) Có thể tạo thành các sản phẩm E không nếu các hợp chất C được
xử lí trước với polyphosphoric acid, sau đó là với hydrazine?

390 | Chuẩn bị kiến thức cho kì thi HSGQG Hóa học


Hướng dẫn
1) Trong số các hydrocarbon, màu xanh dương là đặc trưng của chỉ các
dẫn xuất azulene chứa các hệ vòng ngưng tụ 5 hoặc 7 cạnh. Rõ ràng,
dẫn xuất azulene được tạo thành trong quá trình dehydrogen hóa một
trong các hợp chất F trên platinum. Sẽ hợp lí khi giả sử rằng hợp chất
này không thể là F6, bởi khi dehydrogen hóa vòng cyclohexane tạo
thành dẫn xuất tương ứng của benzene.
Hãy xét sơ đồ tổng hợp. Giai đoạn đầu tiên là phản ứng acyl hóa của các
alkene theo Friedel-Crafts (hay phản ứng Kondakov). Sau đó, anion
của malonic ether được cộng hợp vào các ketone không no theo
Michael:

(Tùy thuộc vào các điều kiện, phản ứng acyl hóa của các alkene theo
Kondakov sẽ tạo thành các ketone không no (đa số, đặc biệt là khi đun
nóng) hoặc các ketochloride - là sản phẩm của phản ứng cộng acyl
chloride vào alkene. Khi phản ứng với malonic ester thì ketochloride,
cũng chuyển thành hợp chất B. Cả hai cách trả lời đều có thể được
chấp nhận là đúng.)
Các giai đoạn tiếp theo rất rõ ràng: chúng dẫn tới sự tạo thành các
hợp chất ba vòng.

Do vòng trung tâm chứa 7 nguyên tử carbon nên rõ ràng là dẫn xuất
azulene được tạo thành từ F5. Hợp chất F6 được chuyển hóa thành
dibenzo cycloheptatriene; trong cả hai trường hợp này, động lực cho

391 | Chuẩn bị kiến thức cho kì thi HSGQG Hóa học


sự dehydrogen hóa là quá trình thơm hóa (với F5 là toàn phân tử, với F6
là vòng 6 cạnh). Do phản ứng dehydrogen hóa F7 dẫn tới sự tạo thành
một hợp chất phản thơm, thay vì hợp chất thơm, nên quá trình này
không thuận lợi về mặt nhiệt động học và thực tế là không xảy ra.

2) Hợp chất H phản ứng với bromine tạo thành một dibromine (đây là
phản ứng cộng electrophile thông thường). Ngược lại, hợp chất G phản
ứng với bromine lại theo cơ chế phản ứng thế electrophile nhân thơm.
Phản ứng diễn ra tương đối dễ dàng bởi sự đóng góp cấu trúc lưỡng cực
của azulene G’. Sự đóng góp của cấu trúc này cũng xác định hướng tấn
công của tác nhân electrophile:

Công thức phân tử của các dibromine: C14H8Br2 và C15H12Br2. Hàm lượng
bromine trong các phân tử lần lượt là 47.62 % và 45.45 %, do đó chênh
lệch hàm lượng bromine là 2.17 %.

392 | Chuẩn bị kiến thức cho kì thi HSGQG Hóa học


3) Không thể. Trước tiên là bởi hợp chất C, không giống như D, thể hiện
hoạt tính yếu trong các phản ứng thế electrophile và chỉ xảy ra phản
ứng trong những điều kiện khắc nghiệt. Ngược lại, các alkyl ketone
phản ứng tương đối dễ dàng với các acid mạnh, tạo thành hỗn hợp sản
phẩm:

Thứ hai, ngay cả khi một số phân tử tham gia phản ứng theo Friedel-
Crafts thì diketone tạo thành trong khi tham gia phản ứng Wolff -
Kishner cũng sẽ không chuyển thành E, mà tạo thành dẫn xuất no
tương ứng:

393 | Chuẩn bị kiến thức cho kì thi HSGQG Hóa học


Chủ đề 44*: Hydrocarbon khung
Trong nửa sau thế kỉ 20, các hydrocarbon khung (ví dụ như cubane - có
hình dạng của một khối lập phương, hoặc adamantane) thu hút được
nhiều sự chú ý của các nhà nghiên cứu. Một mặt, cấu trúc 3 chiều phức
tạp của chúng làm xuất hiện những tính chất lí hóa bất thường; mặt
khác, chúng gây ấn tượng mạnh mẽ về mặt nghệ thuật với các nhà hóa
tổng hợp. Quy trình tổng hợp của một trong những hydrocarbon khung
có tên là tristerine (T) đã được tiến hành từ năm 1965 theo sơ đồ sau:

Nếu giai đoạn đầu tiên được tiến hành mà không có đồng thì ngoài B và
C còn có một đồng phân E được tạo thành. Trong các điều kiện dùng
để tổng hợp tristerane, là chuyển thành sản phẩm G.

Khi đun nóng với lượng A dư gấp 5 lần trong một ống kín, có platiunum
ở 250 oC, T chuyển thành hỗn hợp các chất H, I và J với hiệu suất thấp.
Khi đun nóng trong cùng điều kiện, G bị chuyển hóa không hoàn toàn
thành hỗn hợp J, K, L và M. Biết rằng: H là tiền chất của các đồng phân
I và J, còn K là tiền chất của các đồng phân của J, L và M.
1) Xác định cấu tạo của các hợp chất A - M và T, biết rằng cả A và T
đều có 2 tín hiệu trong phổ 1H NMR với tỉ lệ cường độ 1:1.
2) Ước lượng tỉ lệ (lớn hơn/bé hơn) của các sản phẩm B và C trong giai
đoạn đầu tiên của tổng hợp triasterane.
3) Giải thích tại sao các chuyển hóa của T và G khi đun nóng với A là
không hoàn toàn.

394 | Chuẩn bị kiến thức cho kì thi HSGQG Hóa học


Hướng dẫn
Hợp chất A có độ bất bão hòa bằng 3, có thể là hexatriene,
cyclohexadiene, bicyclohexene hoặc tricyclohexan. Từ dữ kiện phổ
NMR, có thể thấy A có 4 nguyên tử hydrogen cùng loại và 4 nguyên tử
khác thuộc một loại khác. Điều kiện này chỉ có thể được thỏa mãn với
1,4-cyclohexadiene.

Việc xác định cấu tạo của T là vấn đề phức tạp hơn. Từ sơ đồ tổng hợp,
có thể thấy rằng vòng cyclohexane vẫn được bảo toàn trong phân tử,
do các tác nhân có thể phá vỡ liên kết C-C không được sử dụng trong
bất kì giai đoạn nào. 3 nguyên tử carbon được thêm vào rõ ràng là đóng
góp từ diazomethane, CH2N2 (1 nguyên tử C) và diazoacetic ether (2
nguyên tử C). Do B tham gia vào phản ứng thủy phân kiềm, nên 2
nguyên tử này không phải carbon của nhóm ethyl (N2CHCO2Et). Giai
đoạn cuối của tổng hợp T là sự khử nhóm carbonyl theo Wolff -
Kishner. Do vậy, sản phẩm T phải chứa ít nhất 1 nhóm CH2. Theo dữ liệu
phổ NMR, trong phân tử T phải có 6 nguyên tử hydrogen cùng loại và 6
nguyên tử còn lại thuộc một loại khác. Vậy phân tử không chỉ chứa 1,
mà là 3 nhóm CH2. Có thể giả sử rằng 2 nhóm CH2 của 1,4-
cyclohexadiene vẫn không thay đổi, và 6 nguyên tử hydrogen là của 6
nhóm CH. Do đó, phân tử phải có cấu trúc đối xứng với các nhóm CH và
CH2 giống nhau. Điều này cho phép chúng ta xác định cấu trúc của
tristerine:

Sự chính xác của cấu trúc này được xác định bởi quy trình tổng hợp.
Các hợp chất diazo kém bền nhiệt, khi đun nóng sẽ tách các phân tử
nitrogen tạo thành carbene:
o
t
N2CHCO2Et ⎯⎯ → :CHCO2Et
Có 2 dạng carbene: singlet và triplet. Trong dạng đầu tiên, 2 electron
chiếm 1 orbital p, còn 1 orbital khác vẫn trống; còn ở dạng thứ hai, mỗi

395 | Chuẩn bị kiến thức cho kì thi HSGQG Hóa học


orbital có 1 electron (dạng gốc đôi). Khi các phản ứng tạo thành
carbene được xúc tác bởi đồng thì chỉ có singlet carbene được tạo
thành, tiểu phân này phản ứng với 1,4-cyclohexadiene tạo thành các
dẫn xuất vòng cyclopropane B và C (khác nhau về định hướng của
nhóm ethoxycarbonyl với hợp phần cyclohexene). Xem rằng dạng
endo là B và exo là C.

Đồng phân exo C bền về mặt nhiệt động học hơn đồng phân endo B, do
không có tương tác đẩy giữa nhóm ethoxycarbonyl với các nguyên tử
hydrogen của hợp phần cyclohexene. Do đó, C được tạo thành với
lượng nhiều hơn B. Phản ứng thủy phân kiềm của ester tạo thành
carboxylic acid, chất này được chuyển hóa hoàn toàn thành acyl
chloride và diazoketone D:

Khi đun nóng diazoketone có xúc tác đồng sẽ tạo thành một singlet
carbene, tiểu phân này sẽ tấn công nội phân tử vào liên kết đôi thứ hai:

Khi đun nóng ketone tạo thành với hydrazine và một base sẽ tạo thành
hydrocarbon tương ứng (triasterane):

396 | Chuẩn bị kiến thức cho kì thi HSGQG Hóa học


Trong trường hợp nhiệt phân không xúc tác diazoacetic ether thì cả
hai dạng singlet và triplet carbene đều được tạo thành. Do đó, ngoài B
và C còn có sản phẩm của triplet carbene (gốc đôi) được tạo thành qua
liên kết C-H allyl.

Chuyển hóa tiếp theo của E được thực hiện theo sơ đồ sau:

Biết rằng khi đun nóng 1,4-cyclohexadiene với platinum thì xảy ra sự
dehydrogen hóa (tách các phân tử hydrogen ra). Trong bình kín,
hydrogen giải phóng phản ứng với các hợp chất không no. Liên kết đôi
C=C bị hydrogen hóa dễ dàng hơn vòng cyclopropane, nhưng trong
điều kiện để dehydrogen hóa cyclohexadiene thì cả hai nhóm chức đều
phản ứng dễ dàng. Do đó, khi đun nóng tristerine và 1,4-
cyclohexadiene thì xảy ra các phản ứng sau:

397 | Chuẩn bị kiến thức cho kì thi HSGQG Hóa học


Phản ứng dị phân cyclohexadiene có sự tạo thành benzene và
cyclohexane (qua sự tạo thành trung gian cyclohexene) là quá trình
chính trong những điều kiện này, do đó triastere chỉ phản ứng một
phần. Tương tự, khi đun nóng A và G, sự khử G chỉ xảy ra một phần bởi
phản ứng hydrogen hóa cạnh tranh của cyclohexadiene. Trong trường
hợp này, J, K, L và M được tạo thành:

398 | Chuẩn bị kiến thức cho kì thi HSGQG Hóa học


Chủ đề 45*: Chuyển hóa của hydrocarbon vòng
Phản ứng của dung dịch hydrochloric acid của hai hợp chất X, Y đồng
phân quang hoạt, có công thức C10H19N với dung dịch sodium nitrite đã
được nghiên cứu kĩ. Từ hỗn hợp sản phẩm phức tạp, đã cô lập được các
hợp chất A không quang hoạt, có công thức C10H18O, trong phổ hồng
ngoại có các dải hấp thụ rộng trong vùng 3100-3600 cm-1; và các
hydrocarbon B có công thức C10H16.
Các nhà nghiên cứu thấy rằng trong những điều kiện này, chất X ưu tiên
tạo thành 2 hợp chất A1, A2 và 3 hydrocarbon B1, B2, B3. Phản ứng
hydrogen hóa xúc tác B1-B3 đều tạo ra cùng hydrocarbon C, trong phổ
NMR của chất này chỉ có duy nhất 1 tín hiệu. Trong phản ứng oxid hóa
cắt mạch B1-B3 chỉ tạo thành duy nhất một acid đơn giản D, và cần
12.95 mL dung dịch potassium hydroxide 0.126 M để chuẩn độ 142.9
mg acid D.
3 hợp chất A (A3, A4, A5) và 5 hydrocarbon B (B1, B4-B7) đã được cô
lập khi nghiên cứu về chuyển hóa của chất Y. Khi hydrogen hóa, 2 trong
số các hydrocarbon tạo thành hợp chất C, và 3 hydrocarbon còn lại
chuyển thành một chất lỏng bay hơi ở khoảng 187-196 oC và chứa hai
cấu tử E và F, có những tính chất hóa-lí rất giống nhau, gồm cả phổ IR
và NMR. Hỗn hợp E, F chứa 87 % carbon về khối lượng. Nếu đun nóng
hỗn hợp này với bột sulfur thì tạo thành hydrocarbon G chứa 93.75 %
carbon về khối lượng.
1) Xác định công thức cấu tạo X, Y. Gọi tên các chất này theo danh
pháp hệ thống IUPAC.
2) Xác định cấu trúc các hợp chất A-G.
3) Đề xuất sơ đồ giải thích sự tạo thành các chất A, B từ các chất X,
Y. Bỏ qua hiện tượng đồng phân hình học trong các chất A, B.

399 | Chuẩn bị kiến thức cho kì thi HSGQG Hóa học


Hướng dẫn
X, Y là các amine bậc 1, phản ứng với sodium nitrite trong môi trường
acid tạo thành alcohol (dữ kiện phổ IR) và các hợp chất không no theo
sơ đồ sau. Phản ứng diễn ra theo cơ chế SN1 được chứng minh bởi sự
racemic hóa và tạo thành đồng thời cả sản phẩm tách và thế.

Sự hydrogen hóa các hydrocarbon C10H16 về cơ bản sẽ tạo thành các


dẫn xuất C10H18, C10H20 và C10H22. Tuy nhiên, có thể lập luận rằng
hydrocarbon C tạo thành trong phản ứng hydrogen hóa của các
hydrocarbon B1-B3 là cyclodecane bởi đây là dẫn xuất duy nhất mà tất
cả các proton (trong 10 nhóm CH2) tương đương nhau. Do đó, các
hydrocarbon B1-B3 là những cyclodecadiene, các hợp chất A1 và A2 là
những cyclodecenol và nguyên liệu đầu là aminocyclodecene.
Theo dữ kiện chuẩn độ, khối lượng đương lượng của acid nhận được
như là sản phẩm oxid hóa duy nhất của B1 là 0.142/(0.01295∙0.126) = 87,
tương ứng với dicarboxylic acid có khối lượng 174 - octanedicarboxylic
acid, chất này được tạo thành từ 1,3-cyclodecadiene. Do vậy một trong
các hydrocarbon là 1,3-cyclodecadiene.
Trong phản ứng diazoni hóa thông thường, 1,3-cyclodecane cần phải
được tạo thành từ 3- aminocyclopentane hoặc 4-aminocyclopentane.
Amine đầu tiền có thể tạo thành (bỏ qua hiện tượng đồng phân cis-
trans) 1 diene và 1 alcohol (3-hydroxycyclodecene). Còn 4-
aminocyclodecene có thể tạo thành 1 alcohol (3-hydroxycyclodecene)
và 2 hydrocarbon: 1,3-cyclodeacadiene (chủ yếu) và 1,4-
cyclodecadiene. Tương tự, 5- và 6-aminocyclodecene có thể tạo
thành 1 alcohol và 2 diene. Tuy nhiên, số alcohol và diene nhận được là
lớn hơn. Hiện tượng đồng phân hóa đơn giản (chuyển hydride ion từ 1
nguyên tử carbon trong cation sang cation khác, hoặc loại 1 proton rồi
sau đó gắn nó vào 1 nguyên tử carbon khác của liên kết đôi mới tạo
thành) cũng không thể giải thích kết quả nhận được, do trong trường
hợp này, một hỗn hợp thậm chí còn phức tạp hơn có thể được tạo
thành. Để giải quyết vấn đề này, cần nhớ rằng các vòng chứa 5 hoặc
nhiều carbon hơn là không phẳng, và có các dạng không gian (cấu
dạng) khác nhau. Đối với vòng 6 cạnh, cấu dạng chiếm ưu thế nhất là
dạng “ghế bành” (armchair), nhưng cả dạng “thuyền” và dạng “xoắn”

400 | Chuẩn bị kiến thức cho kì thi HSGQG Hóa học


cũng có thể được tạo thành. Có thể giả sử rằng cyclodecene và các
dẫn xuất của nó có thể tạo các cấu dạng tương tự như cyclohexane: 2
nguyên tử carbon là cầu nối chung của 2 cấu dạng cyclohexane. Do đó,
các cấu dạng sau có thể phù hợp với cyclodecene:

Một trong số chúng có C5 và C10 ở gần nhau, nếu tâm cation được tạo
thành trên nguyên tử C5 thì có thể chuyển hydride ion từ C10 để tạo
thành allylic cation tương đối bền. Sau đó, sự tấn công của nucleophile
quanh tâm này sẽ dẫn đến sự tạo thành alcohol khác, nghĩa là, cùng với
5-cyclodecenol thì còn có 3-cyclodecenol được tạo thành. Sự tách
hydrogen cũng sẽ tạo thành 1,3-cyclodecadiene liên hợp cùng với các
diene được tạo thành từ cation ban đầu (1,4- và 1,5-). Do đó, X là (Z)-5-
amino-1-cyclodecene (trong cấu trúc của (Z)-6-aminocyclodecene,
tâm cation bị loại từ liên kết đôi, do đó không xảy ra phản ứng phụ.)

Quá trình này trong trường hợp của Y thậm chí còn phức tạp hơn. Thay
vì chỉ có 2 hydrocarbon và 1 alcohol như dự đoán thì có đến 5
hydrocarbon và alcohol được tạo thành. Y cũng là dẫn xuất
cyclodecene, do 2 trong số 5 hydrocarbon bị hydrogen hóa thành
cyclodecane (C). Các sản phẩm hydrogen hóa của 3 hydrocarbon khác
có tỉ lệ:

Do đó công thức của chúng là C10H18. Do không bị hydrogen hóa nên


chúng không có liên kết bội và là các hợp chất hai vòng. Tỉ lệ C:H xác
định công thức của G là C10H8, tương ứng với hydrocarbon thơm hai
vòng naphthalene. Sự tạo thành cấu trúc hai vòng có thể được giải
thích nếu ta xem xét cẩn thận cấu dạng trans-cyclodecene. Trong cấu

401 | Chuẩn bị kiến thức cho kì thi HSGQG Hóa học


trúc này, liên kết đôi C=C gần với nguyên tử C6, do đó nếu điện tích
dương xuất hiện trên nguyên tử này thì có thể xảy ra sự tấn công
electrophile liên kết đôi ở vị trí C1. Trong trường hợp này, một điện tích
dương được tạo thành trên nguyên tử C2. Cation như vậy có thể tạo
thành nhiều hơn 1 alcohol và 2 hydrocarbon no. Nói cách khác, một
cation bậc 2 có thể bị đồng phân hóa thành cation bậc 3. Do đó chúng
ta sẽ có các alcohol và hydrocarbon no khác.

Các hydrocarbon không no B5-B7 có thể dehydrogen hóa tạo thành


naphthalene, và khi hydrogen hóa chúng sẽ chuyển hóa thành hỗn hợp
cis- và trans-decalin.

Vậy Y là (E)-6-aminocyclodecane.

402 | Chuẩn bị kiến thức cho kì thi HSGQG Hóa học


Chủ đề 46*: Chuyển hóa của terpene
Terpene và terpenoid có tầm quan trọng to lớn với hóa học tổng hợp
hữu cơ, thường được sử dụng làm nước hoa và cũng thể hiện nhiều hoạt
tính sinh lí. Các loại thực vật khác nhau tạo thành các terpene với cấu
trúc khác nhau, bao gồm:

Trong tự nhiên, tiền chất của tất cả các terpene này là diphosphate
của geraniol (trans-3,7-dimethyl-2,6-octadiene-1-ol).
1) Vẽ giản đồ chuyển hóa của geranyldiphosphate thành các terpene
này, chỉ rõ các hợp chất trung gian quan trọng (cả phân tử trung
hòa lẫn mang điện.)
Nhiệt độ sôi của myrcene, limonene và 3-isopropyl-6-
methylenecyclohexene rất gần nhau, do đó không thể tách riêng các
hợp chất này bằng phương pháp chưng cất.
2) Đề xuất phương pháp hóa học để phân lập myrcene từ hỗn hợp này.
Viết các phương trình phản ứng tương ứng.
Quá trình sinh tổng hợp của (+)-mentofuran từ (-)-limonene có sự tạo
thành (+)-pulegone.

Trong công nghiệp, chuyển hóa này được tiến hành bởi phản ứng của
pulegone với sulfuric acid bốc khói trong acetic anhydride để tạo
thành hợp chất trung gian chứa 56.1 % carbon và 6.5 % hydrogen, rồi
nhiệt phân hợp chất trung gian này.

403 | Chuẩn bị kiến thức cho kì thi HSGQG Hóa học


3) Viết công thức phân tử và câu tạo của tiểu hợp chất trung gian tạo
thành.
Dưới đây là một hướng sinh chuyển hóa khác của (+)-pulegone thành (-
)-menthol.

Trong tinh dầu bạc hà, ngoài (-)-menthol còn có một ít (+)-neomenthol.
Khi đun nóng với HI, chúng tạo thành cùng hydrocarbon A. Khi dẫn qua
copper oxide ở 230-240 oC thì (-)-menthol bị chuyển thành hợp chất B,
hydrogen hóa hợp chất này trên Raney nickel tạo thành menthol,
neomenthol và hai đồng phân dia C, D không được tìm thấy trong tự
nhiên.
4) Xác định cấu trúc của (+)-neomenthol, A-D. Tại sao phản ứng khử
sinh hóa (+)-pulegone và các sản phẩm hydrogen hóa của nó tạo
thành (-)-menthol là chủ yếu, thay vì các đồng phân dia?

404 | Chuẩn bị kiến thức cho kì thi HSGQG Hóa học


Hướng dẫn
1)

2) Để cô lập myrcene từ hỗn hợp này, có thể sử dụng tính chất thuận
nghịch của phản ứng Diels-Alder, do trong số các hợp chất này chỉ nó
là có thể tham gia phản ứng Diels-Alder. Ví dụ sản phẩm cộng maleic
acid có thể tan trong các dung môi phân cực, giúp cho việc tách riêng
nó khỏi limonene và 3-isopropyl-6-methylenecyclohexene kị nước.
Sau đó, nhiệt phân sản phẩm cộng vòng tạo thành myrcene ban đầu:

405 | Chuẩn bị kiến thức cho kì thi HSGQG Hóa học


3) Tỉ lệ C:H trong hợp chất trung gian bằng 5:7. Có thể giả sử rằng hợp
chất trung gian chứ 10 nguyên tử carbon (tương ứng với 14 hydrogen
[số chẵn]). Khối lượng phân tử hợp chất trung gian là 214, có nghĩa
oxygen và các nguyên tử còn lại nặng 80 đơn vị, tương ứng với 5 oxygen
hoặc 3 oxygen+1 sulfur. Do rất khó để hình dung làm thế nào mà 4
nguyên tử oxygen mới có thể xuất hiện trong hợp chất trung gian, để
sau đó phân cắt thành mentofuran nên biến thể thứ hai dường như hợp
lí hơn. Có nghĩa là, so với pulegone thì hợp chất trung gian chứa ít hơn
2 nguyên tử hydrogen, nhưng nhiều hơn 2 oxygen và 1 sulfur. Trong
trường hợp này, hợp chất trung gian: a) được tạo thành bởi phản ứng
của sulfuric acid với pulegone; b) phân hủy SO2. Chỉ có duy nhất 1 cấu
trúc thỏa mãn các điều kiện này:

Khi dẫn qua copper oxide, (-)-menthol bị oxid hóa thành ketone tương
ứng (menthone). Khử menthone tạo thành (-)-menthol, neomenthol và
2 đồng phân dia. Khi đun với HI, neomenthol và (-)-menthol bị khử
thành cùng hydrocarbon. Điều này dẫn đến kết luận neomenthol là
đồng phân quang học của (-)-menthol, chính là (1S,2S,5R)-2-isopropyl-
5-methyl cyclohexanol.

406 | Chuẩn bị kiến thức cho kì thi HSGQG Hóa học


Sự tạo thành hai đồng phân dia C và D trong quá trình khử ketone B là
do sự epimer hóa tâm bất đối ở vị trí α của nhóm carbonyl do tác động
của vết base có trong Raney nickel. Do đó, khử hóa B tạo thành hỗn
hợp sản phẩm sau:

Trong phân tử menthol, tất cả các nhóm thế ở vị trí biên (equatorial):

Do đó menthol là dạng bền nhất trong 8 đồng phân dia có thể có. Đây
là nguyên nhân lí giải tại sao nó được tạo thành chủ yếu trong cả phản
ứng khử hóa học lẫn sinh hóa.

407 | Chuẩn bị kiến thức cho kì thi HSGQG Hóa học


Chủ đề 47*: Phản ứng hoán vị
Năm 1967, N. Calderon, H. Yu Chen và K.W. Scott đã lần đầu tiên mô tả
về phản ứng hoán vị (metathesis) của các olefin dưới điều kiện xúc tác
bởi tungsten hexachloride và ethylaluminum dichloride. Hoán vị (có
nguồn gốc từ tiếng Hi Lạp μετάθεσις nghĩa là “thay đổi vị trí”) là phản
ứng trao đổi các nguyên tử (nhóm nguyên tử) giữa các phân tử. Hoán vị
của các olefin từ quan điểm thông thường là phản ứng trao đổi các liên
kết đôi tạo thành các mảnh alkene:

Qua nhiều năm nghiên cứu để tối ưu các điều kiện phản ứng, Yves
Chauvin (Pháp), Robert H. Grubbs (Mỹ) và Richard R. Schrock (Mỹ) đã
phát triển các hệ xúc tác độc nhất cho phản ứng hoán vị các olefin,
ngày nay đang được sử dụng rộng rãi trong tổng hợp hữu cơ hiện đại,
và cũng được đưa vào các quy trình sản xuất công nghiệp các polymer
và dược phẩm. Nhờ thành tựu này, năm 2005, họ đã cùng nhau dành
được giải Nobel Hóa học.
Nhóm phản ứng hoán vị alkene quan trọng nhất là vòng hóa (CM [ЦМ])
và polymer hóa mở vòng (PMRC [ПМРЦ]). Các xúc tác thường là phức
carbene loại LxM=CHR. Các quá trình này có thể biểu diễn ở dạng sơ đồ
như sau:

1) Đề xuất cấu trúc sản phẩm trong các phản ứng của các diene sau
khi xúc tác bởi phức chất LxM=CHR:

408 | Chuẩn bị kiến thức cho kì thi HSGQG Hóa học


2) Giải thích tại sao phản ứng hoán vị alkene đã được thực hiện thành
công giữa dec-9-enoic acid và nitrile của nó nhưng không hữu hiệu
khi tiến hành giữa 10-bromodec-5-en-1-ol và (hept-4- en-1-yl)
cyclobutane.
Hướng dẫn
1) Mặc dù các hợp chất với hai liên kết C=C có thể được sử dụng để tạo
thành các dẫn xuất vòng bởi phản ứng hoán vị, nhưng không phải mọi
diene đều phản ứng theo cách thông thường. Diene (a) không thể tham
gia vào phản ứng vòng hóa nội phân tử, do tạo thành cyclopropene có
sức căng vòng cao. Do đó, phản ứng dimer hóa (nhị hợp) của 1,4-
pentadiene ban đầu tạo thành hỗn hợp cis- và trans-octa-1,4,7-triene.
Đồng phân cis có thể tiếp tục tạo thành một sản phẩm vòng hóa: 1,4-
cyclohexadiene; đồng phân trans không thể tham gia vào phản ứng
như vậy nên chuyển hóa tiếp của nó sẽ tạo thành một hợp chất polymer
(-CH=CH-CH2-)n.

Tình huống tương tự cũng xảy ra với trường hợp 2-methyl-1,5-


heptadiene (b): phản ứng vòng hóa nội phân tử tạo thành cyclobutene
không diễn ra do quá trình thu nhiệt lớn, và sự dimer hóa diene tạo
thành hỗn hợp 2,6- và 2,9-dimethyldeca- 5,9-triene. Các đồng phân Z
tiếp tục chuyển thành 1,2- và 1,5-dimethyl-1,5-cyclooctadiene, còn
các đồng phân E bị polymer hóa (oligomer hóa.)

409 | Chuẩn bị kiến thức cho kì thi HSGQG Hóa học


Với 2,7-octadiene (c), sự vòng hóa nội phân tử diễn ra, tạo thành
cyclopentene.

1-allyl-3-methylenecyclopentane (d) không thể tạo thành sản phẩm


vòng hóa do sản phẩm như vậy sẽ có chứa một liên kết đôi C=C ở
carbon vị trí đầu cầu. Theo quy tắc Bredt, trường hợp này chỉ xảy ra với
các vòng rất lớn. Do đó chất (d) sẽ tạo thành một polymer (oligomer):

Cuối cùng, trong trường hợp (e), phân tử được biểu diễn ở cấu dạng này
(trông như một chiến binh Viking) thì rõ ràng không thể tạo thành sản
phẩm vòng hóa. Tuy nhiên, một phép quay đơn giản quanh liên kết C-
C sẽ tạo thành một cấu dạng khác có thể tham gia vào phản ứng vòng
hóa, tạo thành dibenzanthracene:

2) Trong phản ứng giữa dec-9-enoic acid và nitrile của nó, một sản
phẩm là hợp chất có khối lượng phân tử đủ lớn và (quan trọng nhất)

410 | Chuẩn bị kiến thức cho kì thi HSGQG Hóa học


nhiệt độ sôi cao. Một sản phẩm khác là ethylene, một sản phẩm khí.
Việc tách loại ethylene từ hỗn hợp phản ứng chuyển cân bằng về hướng
tạo thành sản phẩm mong muốn:

Còn trong phản ứng giữa 10-bromodec-5-en-1-ol và (hept-4-en-1-yl)


cyclobutane, các sản phẩm khác không có những yếu tố để chuyển cân
bằng về hướng chỉ tạo thành bất kì một sản phẩm nào. Việc tạo thành
một hỗn hợp sản phẩm phản ứng không được xem là một cách hữu hiệu
để tổng hợp chúng.

411 | Chuẩn bị kiến thức cho kì thi HSGQG Hóa học


Chủ đề 48*: Phản ứng hoán vị
1) Cuối thập niên 1980, một phương pháp đã được đề xuất để điều chế
một số polymer khó hoặc không thể tổng hợp bằng các phương
pháp cổ điển. Dưới đây là sơ đồ của một trong các chuỗi phản ứng
đó. Xác định cấu trúc sản phẩm L:

2) Một số hệ xúc tác hoạt tính cao đã được phát triển và một vài hệ
trong đó không những có khả năng xúc tác cho phản ứng hoán vị
alkene mà còn các alkyne trong các phản ứng “tiếp đôi” [tandem].
(Tiếp đôi là các phản ứng trong đó sản phẩm đầu (là một hợp chất
bền và dưới những điều kiện cụ thể có thể được cô lập ở dạng tinh
khiết) sẽ tham gia vào các chuỗi chuyển hóa liên tiếp). Xác định các
sản phẩm của các phản ứng tiếp đôi sau:

3) Hoàn thành sơ đồ của một trong những phương pháp hữu hiệu nhất
để tổng hợp các hệ nhóm chức đa vòng sử dụng phản ứng hoán vị.
Các hệ này có những tính chất thú vị đối với việc phát triển các tác
nhân kháng khuẩn mới.

412 | Chuẩn bị kiến thức cho kì thi HSGQG Hóa học


Hướng dẫn
1) Sản phẩm của quá trình tổng hợp 2-giai đoạn là polyacetylene (hợp
chất L). Ở giai đoạn đầu tiên, sự tạo thành polymer K diễn ra theo sơ đồ
PMRC (với liên kết C=C hoạt động nhất), chất này tiếp đó xảy ra phản
ứng retro-Diels-Alder (Diels-Alder nghịch), tách loại 1,2-bis
(trifluoromethyl) benzene.

2) a) Do 2 liên kết đôi đầu mạch được đặt đủ xa nhau nên xúc tác trước
tiên sẽ tương tác với một trong các liên kết đôi đó, sau đó phức
carbene phản ứng với liên kết đôi của cyclopentene, rồi sau đó là với
liên kết đôi còn lại.

b) Chất đầu chứa 2 liên kết đôi và 1 liên kết ba. Trước tiên, xúc tác tương
tác với liên kết đôi hoạt động nhất. Phức carbene tạo thành tấn công
vào liên kết ba dễ phản ứng hơn. Tiểu phân trung gian tạo thành tiếp
tục phản ứng với liên kết đôi:

413 | Chuẩn bị kiến thức cho kì thi HSGQG Hóa học


3)

414 | Chuẩn bị kiến thức cho kì thi HSGQG Hóa học


Chương 6: Các nhóm chức chứa
oxygen
Chủ đề 1: Alcohol - Phenol
1) Xác định tên gọi theo danh pháp IUPAC của các hợp chất sau:

2) Xác định cấu tạo của các alcohol tương ứng với tên gọi sau:
a) (Z)-2-Ethyl-2-buten-1-ol
b) 3-Cyclohexen-1-ol
c) trans-3-Chlorocycloheptanol
d) 1,4-Pentanediol
e) 2,6-Dimethylphenol
f) o-(2-Hydroxyethyl)phenol
3) Gọi tên các hợp chất sau theo danh pháp IUPAC:

415 | Chuẩn bị kiến thức cho kì thi HSGQG Hóa học


4) Sắp xếp các hợp chất trong mỗi nhóm sau theo trình tự tăng dần
lực acid:

416 | Chuẩn bị kiến thức cho kì thi HSGQG Hóa học


Hướng dẫn
1)

2)

417 | Chuẩn bị kiến thức cho kì thi HSGQG Hóa học


3)

4)

418 | Chuẩn bị kiến thức cho kì thi HSGQG Hóa học


Chủ đề 2: Tổng hợp alcohol
1) Dự đoán sản phẩm của các phản ứng sau:

2) Đề xuất tác nhân phù hợp để thực hiện các chuyển hóa sau:

3) Xác định sản phẩm tạo thành từ phản ứng cộng giữa methyl-
magnesium bromide với các hợp chất sau rồi xử lí với acid:
a) Cyclopentanone
b) Benzophenone (diphenyl ketone)
c) 3-Hexanone
4) Sử dụng tác nhân Grignard để điều chế các alcohol sau:

419 | Chuẩn bị kiến thức cho kì thi HSGQG Hóa học


a) 2-Methyl-2-propanol
b) 1-Methylcyclohexano
c) 3-Methyl-3-pentanol
d) 2-Phenyl-2-butanol
e) Benzyl alcohol
f) 4-Methyl-1-pentanol
5) Từ nguồn hữu cơ duy nhất là benzene và các alcohol không nhiều
hơn 6C, cùng với các tác nhân vô cơ cần thiết khác, hãy đề xuất sơ
đồ tổng hợp các hợp chất dưới đây:

420 | Chuẩn bị kiến thức cho kì thi HSGQG Hóa học


Hướng dẫn
1)

2) Một số lưu ý khi lựa chọn tác nhân khử hóa:


- NaBH4 khử hóa chọn lọc nhóm carbonyl (aldehyde/ketone)
nhưng không ảnh hưởng đến các nhóm thế khác.
- LiAlH4 là chất khử mạnh hơn NaBH4, có thể khử hóa cả nhóm
carbonyl lẫn ester; tuy nhiên không ảnh hưởng đến liên kết bội.

421 | Chuẩn bị kiến thức cho kì thi HSGQG Hóa học


3)

4) a) 2-Methyl-2-propanol là alcohol bậc ba. Để tổng hợp alcohol bậc


ba, nên bắt đầu với một ketone.

Nếu hai hoặc nhiều nhóm alkyl liên kết với nguyên tử C-OH đều giống
nhau thì alcohol đó có thể được tổng hợp từ ester và tác nhân Grignard:

422 | Chuẩn bị kiến thức cho kì thi HSGQG Hóa học


b) 1-methylcyclohexanol là alcohol bậc ba, nên hãy bắt đầu từ ketone:

c)

d)

423 | Chuẩn bị kiến thức cho kì thi HSGQG Hóa học


e)

f)

5)

424 | Chuẩn bị kiến thức cho kì thi HSGQG Hóa học


425 | Chuẩn bị kiến thức cho kì thi HSGQG Hóa học
Chủ đề 3: Chuyển hóa của alcohol
1) Đề xuất cách tiến hành chuyển hóa sau – là một giai đoạn trong
tổng hợp thương mại (S)-ibuprofen.

2) Dự đoán các sản phẩm tách nước các alcohol sau với POCl3 trong
pyridine. Chỉ ra sản phẩm chính trong mỗi trường hợp.

3)
a) Bằng chứng về các carbocation trung gian trong phản ứng tách
nước xúc tác acid của các alcohol đến từ quan sát rằng các
chuyển vị đôi khi vẫn diễn ra. Hãy đề xuất cơ chế giải thích cho
sự tạo thành 2,3-dimethyl-2-butene từ 3,3-dimethyl-2-
butanol.

b) Phản ứng tách nước xúc tác acid của 2,2 dimethylcyclohexanol
tạo thành hỗn hợp của 1,2 dimethylcyclohexene và
isopropylidenecyclopentane. Hãy đề xuất cơ chế phản ứng để
giải thích cho sự tạo thành hai sản phẩm này.

426 | Chuẩn bị kiến thức cho kì thi HSGQG Hóa học


c) Các epoxide cũng phản ứng được với tác nhân Grignard tạo
thành alcohol. Hãy đề xuất cơ chế phản ứng:

d) Khi xử lí epoxide sau với dung dịch acid tạo thành trung gian
carbocation có thể phản ứng với nước tạo thành sản phẩm diol.
Hãy biểu diễn cấu trúc của carbocation và đề xuất cơ chế cho
giai đoạn thứ hai:

e) Testosterone là một trong những steroid giới tính quan trọng


nhất. Khi testosterone bị tách nước bởi xử lí với acid thì xảy ra
sự chuyển vị, tạo thành sản phẩm như bên dưới. Hãy đề xuất cơ
chế để giải thích cho phản ứng này.

427 | Chuẩn bị kiến thức cho kì thi HSGQG Hóa học


4) Đề xuất các tác nhân phù hợp để thực hiện các chuyển hóa sau:

428 | Chuẩn bị kiến thức cho kì thi HSGQG Hóa học


Hướng dẫn
1) -OH là nhớm rời đi rất kém trong điều kiện phản ứng SN2, nhưng
toluenesulfonate lại là nhóm rời đi rất tốt và phản ứng của
toluenesulfonate của alcohol với CN- sẽ diễn ra rất dễ dàng dưới điều
kiện phản ứng SN2 để tạo thành sản phẩm mong muốn kèm theo sự
nghịch đảo cấu hình ở tâm thủ tính:

2) a) Sản phẩm chính có liên kết đôi nhiều nhóm thế nhất:

b) Trong phản ứng tách E2, sự tách nước diễn ra dễ dàng khi hai nhóm
bị tách có vị trí đối song. Trong hợp chất này, hydrogen duy nhất với
mối liên hệ lập thể phù hợp với nhóm -OH ở C6. Vậy nên, sản phẩm phi-
Zaitsev sẽ được tạo thành.

429 | Chuẩn bị kiến thức cho kì thi HSGQG Hóa học


c)

d) Sản phẩm chính có liên kết đôi nhiều nhóm thế nhất:

e)

430 | Chuẩn bị kiến thức cho kì thi HSGQG Hóa học


3) a)

b)

431 | Chuẩn bị kiến thức cho kì thi HSGQG Hóa học


c)

d)

e)

4) (a) NaBH4, sau đó với H3O+ (b) PBr3 (c) Mg, ether, sau HCHO (d) Dess–
Martin periodinane, CH2Cl2 (e) C6H5CH2MgBr, sau đó với H3O+ (f) POCl3,
pyridine.

432 | Chuẩn bị kiến thức cho kì thi HSGQG Hóa học


Chủ đề 4*: Lasiol
Năm 1990, Lloyd và đồng nghiệp đã phát hiện ra một chất hấp dẫn tình
dục của kiến Lasius meridionalis, được gọi là lasiol (công thức dưới
đây):

Sau này, cấu trúc của laziol đã được chứng minh rõ ràng bằng tổng hợp
đối chứng, quy trình này cũng cho phép xác định cấu hình tuyệt đối của
các tâm bất đối (những nguyên tử carbon có 4 nhóm thế khác nhau).
Trong giai đoạn đầu tiên, cis-4,5-dimethylcyclohexene phản ứng với
meta-chloroperbenzoic acid (mCPBA). Hai sản phẩm A và B được tạo
thành. Sản phẩm chính A được xử lí với base bất đối và thu được một
hợp chất C quang hoạt, chất này là đồng phân của A và B.

Dưới đây là những chuyển hóa sau đó của C:

1) Xác định tên hệ thống của lasiol theo danh pháp IUPAC. Xác định
cấu hình tuyệt đối của các tâm bất đối trong lasiol, sử dụng danh
pháp R/S theo hệ thống Cahn-Ingold-Prelog.
2) Xác định cấu trúc các hợp chất trung gian A, B, D.
3) Hợp chất E có thể tồn tại ở 3 dạng hỗ biến (tautomer): 2 dạng không
vòng và 1 dạng vòng (như trong trường hợp các phân tử đường). Xác
định cấu trúc 3 dạng hỗ biến này.
4) Nếu sản phẩm ozone phân của C không phải được xử lí với sodium
borohydride mà với kẽm trong acetic acid hoặc với các hợp chất
phosphorus hóa trị 3 thì tạo thành sản phẩm F, cũng đặc trưng bởi

433 | Chuẩn bị kiến thức cho kì thi HSGQG Hóa học


hiện tượng hỗ biến mạch vòng. Xác định các dạng hỗ biến có vòng
và không vòng của F.
Hướng dẫn
1) (2S, 3S) -2,3,6-trimethylhept-5-en-1-ol.
2) Giai đoạn đầu là phản ứng epoxide hóa alken với peracid. Trong
trường hợp này, có hai peroxide được tạo thành: một với các nhóm
methy ở vị trí cis, một là trans. Vì các nguyên nhân về tương tác không
gian, có thể dự đoán rằng sản phẩm trans bền hơn. Vị trí trans- của
oxygen cũng được xác thực bởi cấu trúc sản phẩm C, trong đó nguyên
tử oxygen ở vị trí trans so với các nhóm methyl trong vòng
cyclohexane. Base bất đối deproton hóa nguyên tử carbon ở vị trí cạnh
vòng oxirane, dẫn đến sự đồng phân hóa epoxide thành alcohol không
no.

Phản ứng ozone hóa allyl alcohol sau đó khử ozonide bởi sodium
borohydride tạo thành một triol chứa 2 nhóm hydroxyl ở các nguyên tử
carbon cạnh nhau và 1 ở vị trí xa. Vicial diol bị phân cắt khi xử lí với
sodium periodate. Đồng thời, tạo thành aldehyde alcohol, chất này
phản ứng với tác nhân Wittig tạo thành lasiol.

3) Các dạng hỗ biến của E:

434 | Chuẩn bị kiến thức cho kì thi HSGQG Hóa học


4)

435 | Chuẩn bị kiến thức cho kì thi HSGQG Hóa học


Chủ đề 5: Phenol - Ether
1) p-Nitrophenol và 2,6-dimethyl-4-nitrophenol đều có pKa xấp xỉ 7.15
nhưng 3,5-dimethyl-4-nitrophenol thì có pKa = 8.25. Hãy giải thích
tại sao 3,5-dimethyl-4-nitrophenol lại có tính acid kém hơn hai
chất còn lại?

2) Đề xuất cơ chế phản ứng của p-


methylphenol với 2-methylpropene và
xúc tác H3PO4 để tạo thành phụ gia thực
phẩm BHT (có tên gọi theo danh pháp
IUPAC là 2,6-ditert-butyl-4-
methylphenol).
3)
a) Đề xuất quy trình tổng hợp anethole từ phenol và các tác nhân
cần thiết khác:

b) Đề xuất quy trình tổng hợp benzyl phenyl từ benzene, phenol


và các tác nhân cần thiết khác.
c) Safrole, một hợp chất được phân lập từ dầu cây xá xị, được sử
dụng làm chất tạo mùi nước hoa. Hãy đề xuất quy trình tổng
hợp safrole từ catechol (1,2-benzenediol).

436 | Chuẩn bị kiến thức cho kì thi HSGQG Hóa học


4) Đề xuất sơ đồ thực hiện các chuyển hóa sau:

5)
a) Trình bày cơ chế phản ứng thủy phân cis-5,6-epoxydecane bởi
dung dịch acid. Chỉ rõ hóa lập thể của sản phẩm, giả sử rằng
phản ứng diễn ra theo hướng tác kích SN2 từ phía sau như thông
thường.

b) Xác định hóa lập thể của phản ứng thủy phân xúc tác acid của
trans-5,6-epoxydecane? Sản phẩm này khác gì sản phẩm tạo
thành ở ý a?

437 | Chuẩn bị kiến thức cho kì thi HSGQG Hóa học


6)
a) Khử hóa các epoxide bởi lithium aluminum hydride thì tạo
thành các alcohol. Hãy đề xuất cơ chế cho phản ứng dưới đây:

b) Biểu diễn cấu trúc và hóa lập thể của alcohol có thể được tạo
thành khi khử hóa 1,2-epoxycyclohexane bởi lithium aluminum
deuteride, LiAlD4.
7) Xử lí 1,1-diphenyl-1,2-epoxyethane với dung dịch acid tạo thành sản
phẩm chính là diphenylacetaldehyde. Hãy đề xuất cơ chế phản ứng.

8) Đề xuất các tác nhân phù hợp cho các chuyển hóa sau:

438 | Chuẩn bị kiến thức cho kì thi HSGQG Hóa học


Hướng dẫn
1) Anion phenoxide được bền hóa bởi hiệu ứng cộng hưởng hút electron
của nhóm p-nitro. Các nhóm methyl ở vị trí ortho với phenol không có
tác động gì đến tính acid nhưng các nhóm methyl làm cho nhóm nitro
của đồng phân 3,5 phải lệch phẳng với vòng thơm và giảm sự xen phủ
orbital với các orbital π của vòng, dẫn đến sự bền hóa cộng hưởng của
nhóm nitro giảm, và pKa của phenol trở nên cao hơn (cho thấy tính acid
giảm đi).
2) Phosphoric acid proton hóa 2-methylpropene, tạo thành tert-butyl
carbocation:

Carbocation tert-butyl đóng vai trò electrophile và sẽ alkyl hóa p-


cresol. Phản ứng alkyl hóa diễn ra ở vị trí ortho so với nhóm -OH bởi các
lí do điện tử và lập thể.

Sau đó carbocation tert-butyl thứ hai sẽ alkyl hóa tiếp tạo thành BHT.

439 | Chuẩn bị kiến thức cho kì thi HSGQG Hóa học


3) a)

b)

c)

440 | Chuẩn bị kiến thức cho kì thi HSGQG Hóa học


4)

441 | Chuẩn bị kiến thức cho kì thi HSGQG Hóa học


5) a) Sản phẩm thủy phân của cis-5,6-epoxydecane là hỗn hợp
racemic của các diol R,R và S,S.

442 | Chuẩn bị kiến thức cho kì thi HSGQG Hóa học


b)

6) a)

b)

443 | Chuẩn bị kiến thức cho kì thi HSGQG Hóa học


7)

8) a) CH3MgBr, ether; b) H2SO4, H2O; c) NaH, sau đó với CH3I; d) m-


ClC6H4COOOH; e) OH-, H2O.

444 | Chuẩn bị kiến thức cho kì thi HSGQG Hóa học


Chủ đề 6: Aldehyde-Ketone
1) Gọi tên các hợp chất sau theo danh pháp IUPAC:

2) Biểu diễn cấu trúc của các hợp chất sau:


a) 3-Methylbutanal
b) 4-Chloro-2-pentanone
c) Phenylacetaldehyde
d) cis-3-tert-Butylcyclohexanecarbaldehyde
e) 3-Methyl-3-butenal
f) 2-(1-Chloroethyl)-5-methylheptanal
3) Đề xuất cách thực hiện các chuyển hóa sau:

445 | Chuẩn bị kiến thức cho kì thi HSGQG Hóa học


Hướng dẫn
1)

2)

3)

446 | Chuẩn bị kiến thức cho kì thi HSGQG Hóa học


447 | Chuẩn bị kiến thức cho kì thi HSGQG Hóa học
Chủ đề 7: Phản ứng cộng nucleophile
1) Xử lí một aldehyde hoặc ketone với ion cyanide (CN-), sau đó proton
hóa trung gian ion alkoxide tứ diện thì tạo thành cyanohydrin. Hãy
biểu diễn cấu trúc của cyanohydrin tạo thành từ cyclohexane.
2) Khi hòa tan vào nước, trichloroacetaldehyde tồn tại chủ yếu ở dạng
hydrate hóa, gọi là chloral hydrate. Hãy biểu diễn cấu trúc của dạng
này.
3) Oxygen trong nước chủ yếu là 16O (99.8%), nhưng cũng có loại nước
được làm giàu đồng vị oxygen nặng 18O. Khi hòa tan aldehyde hoặc
ketone trong nước làm giàu 18O, thì đồng vị đánh dấu được đưa vào
nhóm carbonyl. Hãy giải thích nguyên nhân.

4) Tại sao cyclohexanone tạo thành cyanohydrin với hiệu suất tốt
nhưng 2,2,6-trimethylcyclohexanone thì không?
5)
a) Biểu diễn các sản phẩm có thể tạo thành phản ứng xúc tác acid
của cyclohexanone với ethylamine, CH3CH2NH2, và với
diethylamine, (CH3CH2)2NH.
b) Sự tạo thành imine có tính thuận nghịch. Hãy trình bày tất cả
các bước liên quan trọng phản ứng xúc tác bởi acid của một
imine với nước (thủy phân), tạo thành một aldehyde hoặc
ketone và amine bậc một.
6) Đề xuất sơ đồ thực hiện tổng hợp các hợp chất dưới đây từ 4-
methyl-3-penten-2-one và các tác nhân cần thiết khác:

7) Trình bày tất cả các giai đoạn của quá trình xúc tác acid cho sự tạo
thành acetal vòng từ ethylene glycol và một aldehyde hoặc ketone.
8)
a) Khi xử lí 4-hydroxybutanal với methanol có mặt xúc tác acid thì
tạo thành 2-methoxytetrahydrofuran. Hãy đề xuất cơ chế phản
ứng.

448 | Chuẩn bị kiến thức cho kì thi HSGQG Hóa học


b) Phản ứng SN2 của (dibromomethyl)benzene, C6H5CHBr2, với
NaOH tạo thành benzaldehyde thay vì (dihydroxymethyl)
benzene, C6H5CH(OH)2. Giải thích nguyên nhân.
c) Phản ứng 2-butanone với HCN tạo thành sản phẩm thủ tính. Xác
định hóa lập thể của sản phẩm.
9) Phản ứng Meerwein–Ponndorf–Verley liên quan đến sự khử hóa
ketone bằng cách xử lí với lượng dư aluminum triisopropoxide,
(CH ) CHO Al. Cơ chế của quá trình này gần với phản ứng
 3 2 3
Cannizzaro, trong đó ion hydride đóng vai trò nhóm rời đi. Hãy đề
xuất cơ chế phản ứng.

449 | Chuẩn bị kiến thức cho kì thi HSGQG Hóa học


Hướng dẫn
1)

2)

3)

4) Sự tạo thành cyanohydrin là quá trình cân bằng. Bởi phản ứng cộng
CN- vào 2,2,6-trimethylcyclohexanone bị cản trở không gian (bởi ba
nhóm methyl) nên cân bằng ưu tiên về hướng ketone chưa phản ứng.

450 | Chuẩn bị kiến thức cho kì thi HSGQG Hóa học


5)
a)

b)

6)

7) Trước tiên là sự tạo thành hemiacetal (a), sau đó là acetal (b):

451 | Chuẩn bị kiến thức cho kì thi HSGQG Hóa học


8) a) 4-Hydroxybutanal tạo thành hemiacetal vòng khi oxygen của
hydroxyl oxygen cộng hợp vào nhóm aldehyde:

Sau đó, methanol phản ứng với hemiacetal vòng, tạo thành 2-
methoxytetrahydrofuran.

452 | Chuẩn bị kiến thức cho kì thi HSGQG Hóa học


b) Phản ứng thế SN2 của ion hydroxide vào C6H5CHBr2 tạo thành
bromoalcohol kém bền, trung gian này sẽ mất Br- tạo thành
benzaldehyde.

c) Sự tác kích có thể diễn ra với xác suất bằng nhau ở cả hai phía của
nhóm carbonyl phẳng, tạo thành hỗn hợp sản phẩm racemic (ngoại
tiêu triền) không quang hoạt.

9) Bước 1: Nhôm, trong vai trò acid Lewis, tạo phức với oxygen của
nhóm carbonyl.
Bước 2: Sự tạo phức với nhôm làm cho nhóm carbonyl tăng tính
electrophile và tạo thuận lợi cho sự chuyển hydride từ isopropoxide.
Bước 3: Xử lí hỗn hợp phản ứng với acid để phân cắt liên kết nhôm-
oxygen và tạo thành cylohexanol.

453 | Chuẩn bị kiến thức cho kì thi HSGQG Hóa học


Cả phản ứng Meerwein–Ponndorf–Verley và Cannizzaro đều là sự
chuyển hydride trong đó nhóm carbonyl bị khử bởi nhóm alkoxide. Chú
ý rằng mỗi phân tử aluminum triisopropoxide có khả năng khử ba phân
tử ketone.

454 | Chuẩn bị kiến thức cho kì thi HSGQG Hóa học


Chủ đề 8: Phản ứng Wittig
1) Đề xuất các hợp chất carbonyl và phosphorus ylide có thể sử dụng
để điều chế mỗi hợp chất sau đây:

2) β-Carotene, chất tạo màu thực phẩm màu vàng và là nguồn vitamin
A, có thể được điều chế bởi phản ứng Wittig kép giữa 2 đương lượng
β-ionylideneacetaldehyde và một diylide. Biểu diễn cấu trúc của
sản phẩm β-carotene.
3) Các aldehyde có thể được điều chế bởi phản ứng Wittig, sử dụng
(methoxymethylene)triphenylphosphorane làm tác nhân Wittig rồi
sau đó thủy phân sản phẩm với acid, ví dụ:

a) Đề xuất cách điều chế phosphorane cần thiết.


b) Đề xuất cơ chế cho giai đoạn thủy phân.
4)
a) Các aldehyde và ketone phản ứng với thioacetal như khi phản
ứng với alcohol, tạo thành các acetal. Hãy dự đoán sản phẩm
của phản ứng sau và đề xuất cơ chế phù hợp.

455 | Chuẩn bị kiến thức cho kì thi HSGQG Hóa học


b) Ketone phản ứng với dimethylsulfonium methylide tạo thành
epoxide, ví dụ như phản ứng dưới đây. Hãy đề xuất cơ chế phản
ứng.

5) Các trans alkene bị chuyển thành đồng phân cis tương ứng và
ngược lại qua phản ứng epoxid hóa, sau đó xử lí epoxide với
triphenylphosphine. Hãy đề xuất cơ chế cho phản ứng epoxide →
alkene:

456 | Chuẩn bị kiến thức cho kì thi HSGQG Hóa học


Hướng dẫn
1)

457 | Chuẩn bị kiến thức cho kì thi HSGQG Hóa học


2)

3)

458 | Chuẩn bị kiến thức cho kì thi HSGQG Hóa học


4) a)

b)

5) Phản ứng diễn ra theo các bước như sau:


- Bước 1: Cộng hợp nucleophile phosphine.
- Bước 2: Xoay liên kết C-C.
- Bước 3: Tách triphenylphosphine oxide.

459 | Chuẩn bị kiến thức cho kì thi HSGQG Hóa học


Bước cuối tương tự như trong phản ứng Wittig.

460 | Chuẩn bị kiến thức cho kì thi HSGQG Hóa học


Chủ đề 9: Hợp chất carbonyl không bão hòa
1) Xử lí 2-cyclohexenone với HCN/KCN tạo thành keto nitrile bão hòa
thay vì cyanohydrin không bão hòa. Hay biểu diễn cấu trúc sản
phẩm và đề xuất cơ chế cho phản ứng.
2) Có thể sử dụng phản ứng cộng liên hợp của tác nhân lithium
diorganocopper như thế nào để tổng hợp các hợp chất dưới đây:

3) Mỗi sơ đồ chuyển hóa dưới đây đều có một hoặc vài điểm không hợp
lí. Hãy giải thích và sửa lại cho đúng:

4) Xử lí ketone không bão hòa , với hydrogen peroxide trong dung
dịch kiềm thì tạo thành epoxy ketone. Phản ứng này đặc trưng với
các ketone không bão hòa; chỉ riêng các liên kết đôi alkene thì
không có phản ứng. Hãy đề xuất cơ chế phản ứng.

461 | Chuẩn bị kiến thức cho kì thi HSGQG Hóa học


Hướng dẫn
1) Sản phẩm tạo thành phản ứng cộng 1,4 của CN-, sau đó proton hóa:

2) Để chọn được tác nhân phù hợp nhằm tạo thành sản phẩm cộng liên
hợp mong muốn, hãy thực hiện như sau:
- Đánh số carbon của nhóm aldehyde hoặc ketone là “1”, và đếm
thêm hai carbon từ đây. Liên kết đôi trong chất đầu không bão
hòa α, β sẽ nối các nguyên tử carbon “2” và “3”.
- Nhóm nối với carbon “3” (được khoanh tròn dưới đây) đến từ tác
nhân alkyllithium.

462 | Chuẩn bị kiến thức cho kì thi HSGQG Hóa học


3) a) Phản ứng cộng Grignard vào ketone liên hợp tạo thành sản phẩm
1,2 chứ không phải sản phẩm 1,4. LiAlH4 sẽ khử ketone thành alcohol.
Sơ đồ đúng phải là:

b) Phản ứng oxid hóa alcohol bởi CrO3 trong môi trường acid chuyển
alcohol bậc một thành carboxylic acid, chứ không phải aldehyde. Sơ
đồ đúng phải là:

c) Xử lí cyanohydrin với H3O+ sẽ tạo thành carboxylic acid chứ không


phải amine. Sơ đồ đúng phải là:

4)

463 | Chuẩn bị kiến thức cho kì thi HSGQG Hóa học


Chủ đề 10: Cơ chế phản ứng
1) Đề xuất cơ chế phản ứng tạo thành 5-dimethylpyrazole từ
hydrazine và 2,4-pentanedione.

2) Đề xuất cơ chế cho sự tạo thành 3,5-dimethylisoxazole từ


hydroxylamine và 2,4-pentanedione.

3) Các amine bậc một phản ứng với ester tạo thành amide. Hãy đề
xuất cơ chế chi phản ứng sau của một ester không bão hòa α, β.

4) Phản ứng Wharton chuyển hóa một epoxy ketone thành một allylic
alcohol bởi phản ứng với hydrazine. Hãy đề xuất cơ chế phản ứng.

464 | Chuẩn bị kiến thức cho kì thi HSGQG Hóa học


Hướng dẫn
1) Trước tiên là phản ứng cộng nucleophile của một nitrogen trong
hydrazine vào một trong các nhóm carbonyl, sau đó tách loại nước, tạo
thành hydrazone:

Tương tự như trên, nitrogen còn lại của hydrazine có thể cộng hợp vào
nhóm carbonyl khác của 2,4-pentanedione, tạo thành pyrazole.

Động lực của phản ứng này là sự tạo thành vòng thơm.

465 | Chuẩn bị kiến thức cho kì thi HSGQG Hóa học


2) Tương tự như ý 1.

466 | Chuẩn bị kiến thức cho kì thi HSGQG Hóa học


3)

4)

467 | Chuẩn bị kiến thức cho kì thi HSGQG Hóa học


Chủ đề 11: Tổng hợp hữu cơ
1) Đề xuất hướng tổng hợp các hợp chất dưới đây từ 2-cyclohexenone
cùng các tác nhân cần thiết khác:

2) Từ benzaldehyde và các tác nhân cần thiết khác, hãy đề xuất cách
tổng hợp các hợp chất sau đây:

3) Đề xuất quy trình tổng hợp các hợp chất dưới đây từ cyclohexanone
và các tác nhân cần thiết khác:
a) 1-Methylcyclohexene
b) 2-Phenylcyclohexanone
c) cis-1,2-Cyclohexanediol
d) 1-Cyclohexylcyclohexanol
4) 6-Methyl-5-hepten-2-one là thành phần của tinh dầu sả. Hãy đề
xuất quy trình tổng hợp chất này từ methyl 4-oxopentanoate.

5) Tamoxifen là loại thuốc được sử dụng trong điều trị ung thư vú. Hãy
đề xuất phương án tổng hợp tamoxifen từ benzene, ketone dưới
đây và các tác nhân cần thiết khác:

468 | Chuẩn bị kiến thức cho kì thi HSGQG Hóa học


469 | Chuẩn bị kiến thức cho kì thi HSGQG Hóa học
Hướng dẫn
1)

470 | Chuẩn bị kiến thức cho kì thi HSGQG Hóa học


2)

Cách thứ hai:

471 | Chuẩn bị kiến thức cho kì thi HSGQG Hóa học


3)

472 | Chuẩn bị kiến thức cho kì thi HSGQG Hóa học


4)

5)

473 | Chuẩn bị kiến thức cho kì thi HSGQG Hóa học


474 | Chuẩn bị kiến thức cho kì thi HSGQG Hóa học
Chủ đề 12: Carboxylic acid - Nitrile
1) Xác định danh pháp IUPAC của các hợp chất sau:

2) Biểu diễn cấu trúc của


a) 2,3-Dimethylhexanoic acid
b) 4-Methylpentanoic acid
c) trans-1,2-Cyclobutanedicarboxylic acid
d) o-Hydroxybenzoic acid
e) (9Z,12Z)-9,12-Octadecadienoic acid
f) 2-Pentenenitrile
3) Đề xuất quy trình tổng hợp các carboxylic acid sau đây:

4) Đề xuất hướng thực hiện các chuyển hóa sau:

5) Đề xuất quy trình tổng hợp 1-phenyl-2-butanone từ benzyl


bromide.

475 | Chuẩn bị kiến thức cho kì thi HSGQG Hóa học


6) Đề xuất quy trình thực hiện các chuyển hóa sau, mỗi chuyển hóa
có thể gồm nhiều bước:

7) Đề xuất hướng thực hiện các chuyển hóa sau:

476 | Chuẩn bị kiến thức cho kì thi HSGQG Hóa học


8) Đề xuất hướng thực hiện các chuyển hóa sau:
a)

b)

c)

d)

9) Đề xuất tác nhân để thực hiện các chuyển hóa sau:

477 | Chuẩn bị kiến thức cho kì thi HSGQG Hóa học


10)
a) Các carboxylic acid có nhóm carbonyl thứ hai cách xa hai
nguyên tử sẽ bị mất CO2 (decarboxyl hóa) qua một trung gian
ion enolate khi xử lí với base. Hãy trình bày cơ chế phản ứng
decarboxyl hóa dưới đây:

b) Trong phản ứng Ritter, alkene phản ứng với nitrile khi có mặt
dung dịch acid mạnh, tạo thành amide. Hãy đề xuất cơ chế phản
ứng:

478 | Chuẩn bị kiến thức cho kì thi HSGQG Hóa học


Hướng dẫn
1)

2)

3)

479 | Chuẩn bị kiến thức cho kì thi HSGQG Hóa học


4)

5)

6) (1)

(2) Từ chuyển hóa (1):

(3) Từ chuyển hóa (2):

480 | Chuẩn bị kiến thức cho kì thi HSGQG Hóa học


(4) Từ chuyển hóa (2):

(5) Từ chuyển hóa (2):

(6)

(7)

(8)

(9)

(10)

481 | Chuẩn bị kiến thức cho kì thi HSGQG Hóa học


7)

8) a)

b)

482 | Chuẩn bị kiến thức cho kì thi HSGQG Hóa học


c)

d)

9) (a) BH3, THF, sau đó xử lí với H2O2, OH-.


(b) PBr3.
(c) Mg, sau đó xử lí với CO2, rồi H3O+ (hoặc CN-, sau đó với H3O+).
(d) LiAlH4, sau đó xử lí với H3O+.
(e) Dess–Martin periodinane, CH2Cl2.
(f) H2NNH2, KOH.

483 | Chuẩn bị kiến thức cho kì thi HSGQG Hóa học


10) a)

b)

484 | Chuẩn bị kiến thức cho kì thi HSGQG Hóa học


Chủ đề 13: Dẫn xuất carboxylic acid
1) Xác định tên gọi các hợp chất sau đây:

2)
a) Phenyl benzoate
b) N-Ethyl-N-methylbutanamide
c) 2,4-Dimethylpentanoyl chloride
d) Methyl 1-methylcyclohexanecarboxylate
e) Ethyl 3-oxopentanoate
f) Methyl p-bromobenzenethioate
g) Formic propanoic anhydride
h) cis-2-Methylcyclopentanecarbonyl bromide
3) Đề xuất cơ chế phản ứng thế acyl nucleophile sau:

485 | Chuẩn bị kiến thức cho kì thi HSGQG Hóa học


4) Dự đoán sản phẩm của các phản ứng thế sau:

5) Dự đoán sản phẩm của các phản ứng sau:

486 | Chuẩn bị kiến thức cho kì thi HSGQG Hóa học


6) Đề xuất hướng thực hiện các chuyển hóa sau, mỗi chuyển hóa có
thể diễn ra qua nhiều giai đoạn:

7)
a) Đề xuất cơ chế phản ứng của 3,4,5-trimethoxybenzoyl chloride
và morpholine tạo thành trimetozine.

b) Đề xuất cơ chế cho phản ứng sau:

487 | Chuẩn bị kiến thức cho kì thi HSGQG Hóa học


c) Khi xử lí 5-aminopentanoic acid với DCC (dicyclohexyl-
carbodiimide) thì tạo thành một lactam. Biểu diễn cấu trúc của
sản phẩm và trình bày cơ chế phản ứng.

488 | Chuẩn bị kiến thức cho kì thi HSGQG Hóa học


Hướng dẫn
1)

2)

489 | Chuẩn bị kiến thức cho kì thi HSGQG Hóa học


3)

4)

490 | Chuẩn bị kiến thức cho kì thi HSGQG Hóa học


5)

491 | Chuẩn bị kiến thức cho kì thi HSGQG Hóa học


6)

492 | Chuẩn bị kiến thức cho kì thi HSGQG Hóa học


7) a)

b)

493 | Chuẩn bị kiến thức cho kì thi HSGQG Hóa học


c)

494 | Chuẩn bị kiến thức cho kì thi HSGQG Hóa học


Chủ đề 14*: Phổ cộng hưởng từ
Để xác định cấu trúc các hợp chất hữu cơ, phương pháp cung cấp
nhiều thông tin nhất là phổ cộng hưởng từ hạt nhân, chủ yếu là 1H NMR.
Phương pháp này dựa trên cơ sở là mỗi nguyên tử hydrogen không
tương đương sẽ có 1 tín hiệu. Cường độ tương đối của tín hiệu được xác
định bởi số nguyên tử hydrogen của mỗi loại. Tín hiệu này này được
biểu diễn ở dang 1 hoặc nhiều vạch, số lượng của chúng tùy thuộc vào
số nguyên tử hydrogen xung quanh mỗi loại. Ví dụ, phổ của methyl
tert-butyl ether (H3O-O-C(CH3)3 chứa 2 mũi đơn (tín hiệu có 1 vạch,
singlet) với tỉ lệ tương đối là 1:3, do nhóm CH3O có 3 nguyên tử oxygen
cùng loại còn nhóm C(CH3)3 có 9 nguyên tử hydrogen của loại khác. Phổ
của 1,1,2-trichloroethane Cl2CH-CH2Cl chứa 2 tín hiệu với cường độ
tương đối 1:2. Tín hiệu đầu tiên xuất hiện ở dạng mũi ba (3 vạch, triplet),
do nguyên tử hydrogen của nhóm Cl2CH tương tác với 2 nguyên tử
hydroge của nhóm CH2Cl. Tín hiệu thứ hai xuất hiện ở dạng mũi đôi (2
vạch, doublet), do 2 nguyên tử tương đương tương tác với nguyên tử
hydrogen của loại khác (của nhóm Cl2CH). Trong phổ của 1,1-
dichloroethane Cl2CH-CH3, có 2 tín hiệu: 1 mũi đôi và 1 mũi bốn (4 vạch,
quadruplet) với cường độ 3:1. Trong các trường hợp phức tạp hơn, các
tín hiệu xuất hiện ở dạng mũi đa (multiplet). Ví dụ như trong phổ 2-
chlorobutane CH3CHClCH2CH3 có 4 tín hiệu với cường độ tương đối là
3:1:2:3 - mũi đôi, 2 mũi đa và mũi ba.
1) Xác định số tín hiệu, cường độ tương đối và độ bội trong phổ NMR
của phân tử isooctane (đặc trưng tham khảo của nhiên liệu xăng).
Carnitine là vitamin tăng trường (vitamin BT). Nó có hiệu ứng đồng hóa,
gây thèm ăn và tăng cân. Dưới đây là sơ đồ tổng hợp vitamin:

2) Thiết lập cấu trúc của hợp chất A-L biết rằng có 3 tín hiệu (mũi đơn,
mũi ba và mũi bốn) trong phổ 1H NMR của hợp chất A và trong phổ
1
H NMR của hợp chất B có 4 tín hiệu: 2 mũi đơn, 1 mũi ba và 1 mũi
bốn.

495 | Chuẩn bị kiến thức cho kì thi HSGQG Hóa học


Hướng dẫn
1) Isooctane, hay 2,2,4-trimethylpentane:

Phân tử này có 9 nguyên tử hydrogen cùng loại (nhóm C(CH3)3). Do


không có nguyên tử hydrogen nào ở nguyên tử carbon trung tâm của
nhóm này nên tín hiệu này là vạch đơn. Phân tử còn có 6 nguyên tử
hydrogen của một loại khác (2 nhóm methyl trong mảnh CH(CH3)2). Do
có 1 nguyên tử hydrogen ở nguyên tử carbon kế cận, tín hiệu từ 2 nhóm
methyl này sẽ xuất hiện ở dạng mũi đôi. Nguyên tử hydrogen trong
mảnh này tạo ra tín hiệu thứ ba, xuất hiện ở dạng mũi đa (tương tác với
nhóm CH2 và 2 nhóm methyl). Cuối cùng, tín hiệu từ nhóm CH2 xuất hiện
ở dạng mũi đôi (tương tác với 1 nguyên tử hydrogen của nhóm CH). Vậy
phổ có chứa 4 tín hiệu: mũi đơn, mũi đôi, mũi đôi, mũi đa với cường độ
tương đối 9:6:2:1.
2) Trong phổ của hợp chất có công thức C4H8O2 có 3 tín hiệu: mũi đơn,
mũi ba và mũi bốn. Sự có xuất hiện của mũi bốn có nghĩa là sự xuất hiện
của nhóm CH3. Sự có mặt của mũi ba cho thấy sự hiện diện của nhóm
CH2. Do tín hiệu thứ ba là mũi đơn, có thể kết luận rằng các nhóm CH2
và CH3 liên kết với nhau trong nhóm ethyl CH2CH3. Phần còn lại của
phân tử có công thức C2H3O2 và có 1 nhóm CH3, vậy công thức của A có
thể là CH3COOCH2CH3 hoặc CH3CH2COOCH3. Khi xử lí A với sodium
ethylate thì nó chuyển thành B, có công thức C6H10O3. Trong phổ NMR
có 2 mũi đơn, mũi ba và mũi bốn. Như trong trường hợp của A, chúng
ta có thể kết luận rằng có một nhóm ethyl CH2CH3 trong phân tử này.
Chú ý đến công thức phân tử, 2 mũi đơn tương ứng với các nhóm CH3
và CH2, không liên kết với nhau hoặc với nhóm CH2CH3. Do đó trong
phản ứng, mảnh CH2O đã được đưa ra vào phân tử. Phân tử B là
CH3COCH2COOCH2CH3. Nó được tạo thành bởi sự ngưng tụ hai phân tử
ethyl acetate (ngưng tụ Claisen), xúc tác bởi sodium ethoxide. Chuyển
hóa của B thành C cũng được tiến hành dưới tác động của sodium
ethoxide, chất này deproton hóa nguyên tử carbon có tính acid mạnh
nhất. Enolate ion tạo thành phản ứng với BrCH2CN tạo thành
CH3COCH(CH2CN)COOCH2CH3. Khi xử lí C với sodium ethoxide gây ra
phân cắt dẫn xuất acetoacetic ester, tạo thành ethyl acetate (phân tử

496 | Chuẩn bị kiến thức cho kì thi HSGQG Hóa học


A) và ethyl ester của cyanopropionic acid G, NCCH2CH2COOC2H5. Xử lí
hợp chất này với diborane tạo thành sản phẩm D (C6H13NO2). Công thức
phân tử khác G chỉ 4 nguyên tử hydrogen. Có nghĩa là dưới tác động của
borane, nhóm CN bị khử thành CH2NH2, trong khi đó nhóm ester vẫn
giữ nguyên. Các giai đoạn sau đó rất rõ ràng: đấy là phản ứng acyl hóa
nhóm amino, thủy phân ester và phản ứng Gel-Folgard-Zelinsky. Do
đó, hợp chất E có công thức СН3СОNHCH2CH2CH2COOC2H5, F là acid
tương ứng và Z là dẫn xuất bromo của nó. Xử lí Z với dung dịch kiềm
rượu, đun nóng tạo thành hợp chất H có công thức C4H7NO2. Vậy ở giai
đoạn này diễn ra: a) tách HBr; b) thủy phân amide. Hợp chất H có cấu
tạo là H2NCH2CH=CHCOOH. Methyl hóa với methyl iodide dư khi có
base tạo thành hợp chất K: (CH3)3N+CH2CH=CHCOO-. Giai đoạn cuối
cùng là oxi-mercury hóa liên kết đôi, sau đó khử loại thủy ngân. Phản
ứng diễn ra theo cơ chế tấn công electrophile vào nguyên tử carbon
của liên kết đôi C=C, proton hóa tạo thành cation bền hơn. Do đó chuỗi
chuyển hóa đi qua các giai đoạn sau:

497 | Chuẩn bị kiến thức cho kì thi HSGQG Hóa học


Chủ đề 15*: Bất đối xứng hóa các diol đối xứng
Các nhà hóa học hữu cơ thường phải đối diện với vấn đề phải bất đối
xứng hóa các diol đối xứng để nhận được các dẫn xuất quang hoạt. Vấn
đề này đã được giải quyết bằng cách sử dụng các enzyme hoặc vi sinh
vật nhất định đóng vai trò như xúc tác. Dưới đây là sơ đồ tổng hợp chất
tương đồng vòng carbon của các nucleoside (1R,3S)-1-[(adenine-9-
yl)methyl]-3-(hydroxy methyl)indane (X), thể hiện hoạt tính chống ung
thư:

Chất đầu A được điều chế theo sơ đồ sau:

1) Xác định cấu trúc của các chất B-G, biết rằng E là hợp chất trung
gian kém bền.
Nếu giai đoạn đầu tiên của quy trình tổng hợp này được thực hiện mà
không có cyclopentadiene hoặc bất kì hợp chất hoạt động nào, thì hợp
chất trung gian E bị chuyển thành sản phẩm H.
2) Viết công thức của H.
3) Xác định nguyên nhân chính tại sao vinyl acetate, chứ không phải
ethyl acetate, được sử dụng trong giai đoạn chuyển hóa A thành B.
4) Những đồng phân acetate có công thức C5H8O2 nào là hiệu quả
trong phản ứng này?

498 | Chuẩn bị kiến thức cho kì thi HSGQG Hóa học


Hướng dẫn
1) Khi A phản ứng với vinyl acetate, có mặt Novozyme 435 (enzyme) thì
công thức phân tử bị biến dổi C2H2O (từ C11H14O2 thành C13H16O3). Do đó,
quá trình này đã chuyển một nhóm acetyl (ester chéo hóa) từ vinyl
acetate đến một trong các nhóm alcohol của hợp chất A. Hợp chất B
tạo thành phản ứng với methanesulfonyl chloride, khi đó alcohol
chuyển thành sulfonate C tương ứng. Các alkylsulfonate là những tác
nhân alkyl hóa hiệu quả hơn các alkyl acetate. Khi phản ứng với 6-
chloropurine, đã xảy ra sự tạo thành liên kết C-N có trong sản phẩm
cuối với sự tham gia của mảnh alkylsulfonate. Điều này cho phép
chúng ta xác định vị trí trong hợp chất C của nhóm acetyl và nhóm
sulfonate, và qua đó biết được hóa lập thể của sản phẩm ester chéo
hóa B. Khi D phản ứng với ammonia, hai phản ứng diễn ra đồng thời, sự
thay thế chlorine trong mảnh purine bởi nitrogen tạo thành dẫn xuất
adenine; và sự thủy phân alkyl acetate tạo thành alcohol.

Phân tích chuỗi chuyển hóa thứ hai thì đơn giản nhất là bắt đầu với
chuyển hóa F thành G và G thành A. Quá trình đầu tiên là sự ozone
phân-khử liên kết đôi, tạo thành dialdehyde; thứ hai là sự khử các
nhóm aldehyde tạo thành các alcohol. Do đó, có thể xác định cấu trúc
F và G là:

Hợp chất F được tạo thành bởi phản ứng của hợp chất trung gian kém
bền E với cyclopentadiene. Chú ý đến công thức của sản phẩm tạo

499 | Chuẩn bị kiến thức cho kì thi HSGQG Hóa học


thành (C11H10), có thể kết luận rằng đã xảy ra phản ứng Diels-Alder giữa
cyclopentadiene (C5H6) và dehydrobenzene (C6H4). Thực tế, dưới những
điều kiện này magnesium tách 2 nguyên tử halogen của
bromofluorobenzene tạo thành dehydrobenzene E.

2) Nếu dehydrobenzene đươc tạo thành mà không có các hợp chất


hoạt động thì xảy ra phản ứng dimer hóa:

3) Enzyme xúc tác cho quá trình ester chéo hóa cả theo chiều thuận
và nghịch. Do đó, khi sử dụng ethyl acetate (và bất kì alkyl acetate nào
tạo thành alcohol bền trong phản ứng deacetyl hóa) thì phản ứng sẽ
không diễn ra hoàn toàn. Cách duy nhất để chuyển cân bằng theo
chiều sang phải là sử dụng một lượng ethyl acetate cưc kì dư để vô hoạt
enzyme. Phản ứng ester chéo hóa của các alcohol bởi vinyl acetate tạo
thành vinyl acetate, chất này kém bền và nhanh chóng bị tautomer hóa
(hỗ biến) tạo thành dạng bền hơn, acetic aldehyde. Do đó, phản ứng
ester chéo hóa theo chiều nghịch không thể xảy ra.
4) Có 4 acetate tương ứng với công thức C5H8O2:

Allyl acetate khi bị ester chéo hóa tạo thành allyl alcohol, chất này có
thể tham gia phản ứng nghịch. Ngược lại các đồng phân E- và Z- của
(prop-1-yl) acetate và isopropenyl acetate khi tách nhóm acetyl tạo
thành các alcohol dạng vinyl kém bền, nhanh chóng bị tautomer hóa
thành propionic aldehyde và acetone rồi bị loại khỏi phản ứng. Do đó,
chỉ có allyl acetate là không hữu hiệu với phản ứng, còn các đồng phân
khác có thể tạo thành sản phẩm ester chéo hóa với hiệu quả cao.

500 | Chuẩn bị kiến thức cho kì thi HSGQG Hóa học


Chủ đề 16: Keto-enol
1) Biểu diễn cấu trúc các dạng hỗ biến enol của các hợp chất sau:
a) Cyclopentanone
b) Methyl thioacetate
c) Ethyl acetate
d) Propanal
e) Acetic acid
f) Phenylacetone
Chỉ ra những nguyên tử hydrogen có tính acid mạnh nhất (dễ bị tách
loại bởi base mạnh) trong mỗi phân tử trên.
2) Biểu diễn cấu trúc cộng hưởng của anion nitrile để giải thích cho
tính acid [yếu] của các nitrile:

3) Đề xuất cơ chế của phản ứng deutero hóa acetone khi xử lí với
D3O+ như sau:

4) Đề xuất quy trình điều chế các hợp chất sau đây, sử dụng phương
pháp tổng hợp malonic ester:

5) Đề xuất các alkyl halide để điều chế các ketone sau qua phương
pháp tổng hợp acetoacetic ester:

501 | Chuẩn bị kiến thức cho kì thi HSGQG Hóa học


6) Đề xuất hướng tổng hợp các hợp chất dưới đây, với giai đoạn then
chốt là phản ứng alkyl hóa:

7) Đề xuất hướng điều chế các hợp chất dưới đây, sử dụng phương
pháp tổng hợp acetoacetic ester hoặc malonic ester:

8) Dự đoán sản phẩm của các phản ứng sau:

502 | Chuẩn bị kiến thức cho kì thi HSGQG Hóa học


9) Aprobarbital là một barbiturate từng sử dụng để điều trị chứng mất
ngủ, được tổng hợp qua ba bước từ diethyl malonate. Trình bày
cách tổng hượp trung gian dialkyl hóa cần thiết và đề xuất cơ chế
cho phản ứng của trung gian này với urea để tạo thành
aprobarbital:

503 | Chuẩn bị kiến thức cho kì thi HSGQG Hóa học


Hướng dẫn
1) Những nguyên tử hydrogen có tính acid, dễ tách loại được in đậm:

Trong các trường hợp (d) và (f), có thể tồn tại cis và trans enol.

504 | Chuẩn bị kiến thức cho kì thi HSGQG Hóa học


2) Các nitrile là acid yếu bởi anion tạo thành có thể được bền hóa cộng
hưởng với sự tham gia của các liên kết pi trong nhóm nitrile:

3)

4)

505 | Chuẩn bị kiến thức cho kì thi HSGQG Hóa học


5)

506 | Chuẩn bị kiến thức cho kì thi HSGQG Hóa học


6)

507 | Chuẩn bị kiến thức cho kì thi HSGQG Hóa học


7)

508 | Chuẩn bị kiến thức cho kì thi HSGQG Hóa học


8)

9) Dialkyl hóa diethylmalonate:

509 | Chuẩn bị kiến thức cho kì thi HSGQG Hóa học


Cơ chế phản ứng:

Chuỗi chuyển hóa được lặp lại để tạo thành vòng sáu cạnh.

510 | Chuẩn bị kiến thức cho kì thi HSGQG Hóa học


Chủ đề 17: Cơ chế phản ứng
1) Phản ứng Favorskii liên quan đến việc xử lí một α-bromo ketone với
base, tạo thành sản phẩm rút gọn vòng. Ví dụ, phản ứng của 2-
bromocyclohexanone với dung dịch NaOH tạo thành
cyclopentanecarboxylic acid. Hãy đề xuất cơ chế phản ứng:

2) Xử lí một ketone vòng với diazomethane là phương pháp để thực


hiện một phản ứng mở rộng vòng. Ví dụ, khi xử lí cyclohexanone
với diazomethane sẽ tạo thành cycloheptanone. Hãy đề xuất cơ
chế phản ứng:

3) Các ketone phản ứng chậm với benzeneselenenyl chloride khi có


mặt HCl, tạo thành các α-phenylseleno ketone. Hãy đề xuất cơ chế
cho phản ứng thế α xúc tác acid dưới đây:

4) Giai đoạn cuối của một thử nghiệm tổng hợp laurene (hydrocarbon
được phân lập từ tảo biển Laurencia glandulifera), liên quan đến
phản ứng Wittig như dưới đây. Tuy nhiên, sản phẩm tạo thành
không phải là laurene mà là một đồng phân. Hãy đề xuất cơ chế
phản ứng để giải thích cho kết quả ngoài dự kiến này.

511 | Chuẩn bị kiến thức cho kì thi HSGQG Hóa học


5) Giai đoạn then chốt của một tổng hợp trong phòng thí nghiệm của
sativene (hydrocarbon phân lập từ nấm mốc Helminthosporium
sativum) bao gồm quá trình xử lí một keto tosylate với base như
sau. Phản ứng diễn ra như thế nào? Đề xuất hướng hoàn thành tổng
hợp.

6) Khi đun nóng carvone với dung dịch sulfuric acid thì nó chuyển hóa
thành carvacrol. Hãy đề xuất cơ chế phản ứng cho quá trình đồng
phân hóa này:

512 | Chuẩn bị kiến thức cho kì thi HSGQG Hóa học


Hướng dẫn
1) Phản ứng diễn ra qua các bước:
(1) Mất proton ở vị trí alpha.
(2) Thế bromide.
(3) Cộng nucleophile (OH-).
(4) Mở vòng.
(5) Chuyển proton.
(6) Proton hóa.

2)

3) Các bước phản ứng:


(1) Enol hóa xúc tác acid.
(2) Các electron pi của enol tác kích vào phenylselenyl chloride,
tách chloride.
(3) Tách proton.

513 | Chuẩn bị kiến thức cho kì thi HSGQG Hóa học


4) Sự khác biệt giữa laurene với sản phẩm quan sát được là cấu hình
hóa lập thể ở carbon α với nhóm methylene. Do đó là vị trí α của nhóm
carbonyl trong tiền chất nên trong quá trình phản ứng, phải xảy ra sự
enol hóa và đồng phân hóa.

514 | Chuẩn bị kiến thức cho kì thi HSGQG Hóa học


5)

6)

515 | Chuẩn bị kiến thức cho kì thi HSGQG Hóa học


Chủ đề 18: Phản ứng ngưng tụ
1) Dự đoán sản phẩm enone có thể nhận được từ phản ứng ngưng tụ
aldol mỗi hợp chất sau:

2) Những hợp chất nào sau đây có thể trải qua phản ứng sự ngưng tụ
aldol? Dự đoán cấu trúc sản phẩm trong mỗi trường hợp.
(a) Trimethylacetaldehyde (b) Cyclobutanone
(c) Benzophenone (diphenyl ketone) (d) 3-Pentanone
(e) Decanal (f) 3-Phenyl-2-propenal
3) Dự đoán sản phẩm tạo thành khi xử lí 1,6-cyclodecanedione với
base:

4) Dự đoán sản phẩm tạo thành bởi phản ứng ngưng tụ Claisen các
ester sau:
a) ethyl 3-methylbutanoate
b) ethyl phenylacetate
c) ethyl cyclohexylacetate
5) Dự đoán sản phẩm của phản ứng sau:

516 | Chuẩn bị kiến thức cho kì thi HSGQG Hóa học


6) Phản ứng vòng hóa Dieckmann của diethyl 3-methylheptanedioate
tạo thành hỗn hợp hai sản phẩm β-keto ester. Hãy chỉ ra cấu tạo
sản phẩm này và giải thích tại sao có sự tạo thành hỗn hợp.

7)
a) Dự đoán sản phẩm tạo thành từ phản ứng vòng hóa aldol của
hexanedial.

b) Phản ứng vòng hóa aldol nội phân tử của 2,5-heptanedione với
dung dịch NaOH tạo thành hỗn hợp hai sản phẩm enone với tỉ lệ
xấp xỉ là 9:1. Giải thích sự tạo thành hỗn hợp này.
8) Dự đoán các sản phẩm ngưng tụ có thể tạo thành khi xử lí các hợp
chất sau đây với sodium ethoxide trong ethanol:
a) Ethyl butanoate
b) Cycloheptanone
c) 3,7-Nonanedione
d) 3-Phenylpropanal
9) Tại sao mỗi phản ứng dưới đây không thể đạt hiệu suất cao?

517 | Chuẩn bị kiến thức cho kì thi HSGQG Hóa học


10) Đề xuất tác nhân để thực hiện các chuyển hóa sau:

Hướng dẫn
1)

2) Các hợp chất (a), (c), (f) không có hydrogen alpha nên không trải
qua phản ứng tự ngưng tụ aldol.

518 | Chuẩn bị kiến thức cho kì thi HSGQG Hóa học


3) Xảy ra phản ứng ngưng tụ aldol nội phân tử:

519 | Chuẩn bị kiến thức cho kì thi HSGQG Hóa học


4)

5)

6) Diethyl 3-methylheptanedioate có cấu trúc không đối xứng nên có


thể có hai enolate khác nhau được tạo thành và mỗi tiểu phân sẽ vòng
hóa tạo thành một sản phẩm.

520 | Chuẩn bị kiến thức cho kì thi HSGQG Hóa học


7) a)

b)

521 | Chuẩn bị kiến thức cho kì thi HSGQG Hóa học


8)

9) a) Ngoài sản phẩm đã chỉ ra còn có một số sản phẩm khác. Phản ứng
tự ngưng tụ của acetaldehyde và acetone có thể diễn ra, và tạo thành
sản phẩm hỗn tạp khác.
b) Có hai vấn đề với phản ứng này: (1) Phản ứng Michael của mono-
ketone diễn ra với hiệu suất thấp. Sự tạo thành enamine, sau đó là
phản ứng Michael, cho hiệu suất cao hơn; (2) Phản ứng cộng có thể
diễn ra ở cả hai phía của ketone để tạo thành hỗn hợp các sản phẩm.

522 | Chuẩn bị kiến thức cho kì thi HSGQG Hóa học


c) Phản ứng ngưng tụ aldol hóa nội phân tử của 2,6-heptanedione có
thể tạo thành vòng bốn cạnh hoặc sáu cạnh. Vòng sáu cạnh dễ tạo
thành hơn bởi có sức căng vòng ít hơn.

10)
(a) LiAlH4, sau đó xử lí với H3O+. (b) POCl3, pyridine.
(c) KMnO4, H3O+. (d) CH3CH2OH, H+.
(e) NaOEt. (f) H3O+.
(g) NaOEt, sau đó xử lí với CH3Br. (h) H3O+, đun nóng.

523 | Chuẩn bị kiến thức cho kì thi HSGQG Hóa học


Chủ đề 19: Các phản ứng ngưng tụ quan trọng
1) Dự đoán sản phẩm có thể nhận được từ phản ứng Michael xúc tác
base của 2,4-pentanedione với các hợp phần nhận α,β không bão
hòa sau đây:
a) 2-Cyclohexenone
b) Propenenitrile
c) Ethyl 2-butenoate
2) Dự đoán sản phẩm có thể nhận được từ phản ứng Michael xúc tác
base của 3-buten-2-one với các hợp phần nhường nucleophile sau
đây:

3) Dự đoán sản phẩm tạo thành sau khi thủy phân enamine nhận được
từ phản ứng của cyclopentanone và pyrrolidine với các hợp phần
nhận α,β không bão hòa sau đây:

4) Đề xuất cách sử dụng phản ứng enamine để điều chế các hợp chất
sau:

5) Dự đoán sản phẩm của phản ứng ngưng tụ Robinson giữa 2-methyl-
1,3-cyclopentanedione với 3-buten-2-one.

524 | Chuẩn bị kiến thức cho kì thi HSGQG Hóa học


6) Đề xuất hướng điều chế hợp chất sau bởi phản ứng ngưng tụ
Robinson giữa một β-diketone và một ketone không bão hòa α, β.

7) Ketone Wieland–Miescher là nguyên liệu đầu thường dùng để tổng


hợp các steroid. Đề xuất hướng điều chế hợp chất này từ 1,3-
cyclohexanedione.

525 | Chuẩn bị kiến thức cho kì thi HSGQG Hóa học


Hướng dẫn
1)

2)

526 | Chuẩn bị kiến thức cho kì thi HSGQG Hóa học


3)

4)

527 | Chuẩn bị kiến thức cho kì thi HSGQG Hóa học


5)

6)

7)

528 | Chuẩn bị kiến thức cho kì thi HSGQG Hóa học


Chủ đề 20: Ngưng tụ các hợp chất carbonyl
Khi xử lí với các base, các aldehyde và ketone tham gia vào phản ứng
ngưng tụ aldol. Cơ chế phản ứng ngưng tụ của acetic aldehyde được
biểu diễn như sau:

Khi sử dụng hai aldehyde hoặc ketone khác nhau thì một hỗn hợp sản
phẩm sẽ được tạo thành.
1) Viết tất cả các sản phẩm ngưng tụ, chứa không quá 8 nguyên tử
carbon, có thể tạo thành giữa acetone (CH3COCH3) và
cyclobutanone.
Tuy nhiên có những trường hợp trong đó quá trình ngưng tụ diễn ra
chọn lọc tạo thành một sản phẩm. Ngoài ra, chỉ có các ketoalcohol
(aldoalcohol) được tạo thành bởi sự ngưng tụ.
2) Trong phản ứng ngưng tụ giữa benzaldehyde và acetone, sản phẩm
X chứa 82.2 % carbon được tạo thành với hiệu suất cao. Xác định
cấu trúc của X.

529 | Chuẩn bị kiến thức cho kì thi HSGQG Hóa học


Hướng dẫn
1) Phản ứng ngưng tụ giữa acetone và cyclobutanone diễn ra theo cơ
chế thông thường, cả acetone và cyclobutanone đều có thể xảy ra
phản ứng trên nhóm carbonyl (đóng vai trò như một cấu tử carbonyl)
và phản ứng với nhóm CH có tính acid (đóng vai trò cấu tử methylene).
Do đó, sự tạo thành 4 sản phảm là có thể (các sản phẩm có thể được
tạo thành như là chuỗi ngưng tụ thì có ít nhất 9 carbon, không thỏa mãn
với điều kiện đề bài đã nêu.)

Về cơ bản, trong phản ứng ngưng tụ của benzaldehyde với acetone sẽ


tạo thành hỗn hợp có thành phần 1:1; một sản phẩm được tạo thành từ
2 phân tử benzaldehyde và 1 phân tử acetone (2:1) hoặc một sản phẩm
tạo thành từ 1 phân tử benzaldehyde và 2 phân tử acetone (1:2). Sản
phẩm cộng đầu tiên (1:1) chứa 10 carbon, thứ hai-13, thứ ba-17. Với bất
kì sản phẩm nào, khối lượng của số carbon này phải chiếm 82.2 % khối
lượng phân tử. Từ đây, ta có thể xác định khối lượng mol của các sản
phẩm khả dĩ. Với sản phẩm 1:2, đó là 12∙13/0.822 = 189.8 (không tương
ứng với bất kì công thức phân tử của hợp chất nào được tạo thành từ 2
chất đầu [chứa C, H, O].). Với sản phẩm cộng 2:1, M = 248.2, chất này
cũng không có đáp án nào thỏa mãn. Với sản phẩm cộng 1:1, khối lượng
phân tử là 12∙10/0.822 = 146.0 - tương ứng với C10H10O, chất này khác
công thức sản phẩm của phản ứng ngưng tụ aldol thông thường (C7H6O3
+ C3H6O = C10H12O2) bởi 1 H2O. Sẽ hợp lí khi giả định rằng việc tách nước
trong trường hợp ngưng tụ benzaldehyde được thực hiện bởi sự tạo
thành hệ liên hợp giữa vòng benzene và liên kết đôi C=C (sau đó là liên
kết đôi C=O). Do đó, cấu trúc của X là:

530 | Chuẩn bị kiến thức cho kì thi HSGQG Hóa học


Chủ đề 21: Ngưng tụ các hợp chất carbonyl
Trong phản ứng của benzaldehyde với diethyl malonic acid, hợp chất
A hoặc B được tạo thành tùy thuộc vào nhiệt độ và xúc tác được sử
dụng. A có chứa 67.7 % carbon và 6.5 % hydrogen, dễ bị hydrogen hóa
ở áp suất khí quyển tạo thành hợp chất C, chất này khi đun nóng có mặt
acid giải phóng khí và chuyển thành D. Hợp chất B không bị hydrogen
hóa trong điều kiện tương tự, và khi đun nóng có mặt acid, nó cũng
chuyển thành hợp chất E và có giải phóng khí.

Chuỗi chuyển hóa sau với hợp chất D đã được tiến hành:

Khi ngưng tụ F với benzaldehyde khi có mặt KOH, hợp chất K (87.3 %
carbon) được tạo thành. Khi đun nóng kéo dài (100 oC, 2 ngày) khi có
mặt base yếu (NaHCO3) thì bị dimer hóa tạo thành hợp chất L. Phổ 1H
NMR của L chứa các tín hiệu của 18 nguyên tử hydrogen thơm và các
tín hiệu trong vùng mạnh tương ứng với 1 nhóm CH2 và 4 nhóm CH.
Xác định công thức các chất A-L.

531 | Chuẩn bị kiến thức cho kì thi HSGQG Hóa học


Hướng dẫn

532 | Chuẩn bị kiến thức cho kì thi HSGQG Hóa học


533 | Chuẩn bị kiến thức cho kì thi HSGQG Hóa học
Chủ đề 22: Cơ chế phản ứng
1) Phản ứng enamine Stork và phản ứng ngưng tụ aldol nội phân tử có
thể được tiến hành liên tiếp để tổng hợp các cyclohexenone. Ví dụ,
phản ứng của pyrrolidine enamine của cyclohexanone với 3-buten-
2-one, sau đó thủy phân enamine và xử lí với base, thì tạo thành sản
phẩm như dưới đây. Trình bày các bước phản ứng và cơ chế mỗi
bước.

2) Quá trình sau đây gồm hai giai đoạn phản ứng Michael nội phân tử
liên tiếp. Hãy trình bày cơ chế phản ứng.

3) Ester Hagemann được điều chế bằng cách xử lí hỗn hợp của
formaldehyde và ethyl acetoacetate với base, sau đó decarboxyl
hóa xúc tác acid.

a) Giai đoạn đầu tiên là phản ứng ngưng tụ kiểu aldol giữa ethyl
acetoacetate và formaldehyde tạo thành sản phẩm không bão
hòa α,β. Giai đoạn thứ hai là phản ứng Michael giữa ethyl
formaldehyde và sản phẩm ở bước 1. Hãy trình bày cơ chế hai
giai đoạn.
b) Giai đoạn thứ ba và thứ tư của tổng hợp ester Hagemann lần
lượt là phản ứng vòng hóa aldol nội phân tử, tạo thành

534 | Chuẩn bị kiến thức cho kì thi HSGQG Hóa học


cyclohexenone có nhóm thế, rồi sau đó là phản ứng decarboxyl
hóa. Hãy trình bày cơ chế hai giai đoạn này.
4) Khi chuyển hóa 2-methylcyclohexanone thành enamine, chỉ có duy
nhất một sản phẩm được tạo thành, mặc dù ketone ban đầu không
đối xứng. Giải thích tại sao.

5)
a) Chuyển hóa sau bao gồm sự thủy phân, sau đó là phản ứng thế
acyl nucleophile nội phân tử. Hãy trình bày cơ chế phản ứng.

b) Chuyển hóa sau bao gồm phản ứng Michael nội phân tử, sau đó
là phản ứng aldol hóa nội phân tử. Hãy trình bày cơ chế phản
ứng.

535 | Chuẩn bị kiến thức cho kì thi HSGQG Hóa học


c) Chuyển hóa sau bao gồm phản ứng cộng liên hợp, sau đó là
ngưng tụ Claisen nội phân tử. Hãy trình bày cơ chế phản ứng.

d) Chuyển hóa sau bao gồm một phản ứng aldol nội phân tử, sau
đó là phản ứng kiểu aldol-ngược. Hãy trình bày cơ chế phản ứng.

e) Đề xuất cơ chế cho phản ứng đồng phân hóa xúc tác base sau:

f) Đề xuất cơ chế cho phản ứng sau:

536 | Chuẩn bị kiến thức cho kì thi HSGQG Hóa học


Hướng dẫn
1)

2)

537 | Chuẩn bị kiến thức cho kì thi HSGQG Hóa học


3)

(b)

538 | Chuẩn bị kiến thức cho kì thi HSGQG Hóa học


4) Bởi tương tác không gian giữa nhóm methyl và vòng pyrrolidine
khiến sự tạo thành enamine này không thuận lợi.

Tương tác này có thể được giảm thiểu bằng cách lật vòng, đặt nhóm
methyl vào vị trí trục và đây là enamine duy nhất được tạo thành.

4) a) Cơ chế này có thể chia thành hai phần. Ở phần đầu tiên, acetal bị
thủy phân tạo thành acetone và một dihydroxycarboxylic acid.

539 | Chuẩn bị kiến thức cho kì thi HSGQG Hóa học


Ở phần sau, dihydroxycarboxylic acid tạo thành ester vòng (lactone).

b)

540 | Chuẩn bị kiến thức cho kì thi HSGQG Hóa học


c)

d)

e)

541 | Chuẩn bị kiến thức cho kì thi HSGQG Hóa học


f)

542 | Chuẩn bị kiến thức cho kì thi HSGQG Hóa học


Chủ đề 23*: Chuyển vị Schmidt
Năm 1923, Schmidt nhận thấy rằng khi thêm sulfuric acid vào hỗn hợp
benzophenone và nitrous acid HN3 thì thu được benzanilide. Phản ứng
Schmidt được thực hiện với các ketone, aldehyde, carboxylic acid. Cơ
chế phản ứng của benzophenone được trình bày dưới đây:

Trong trường hợp các ketone đối xứng, phản ứng Schmidt diễn ra rất
rõ ràng, nhưng với với ketone không đối xứng thì hỗn hợp sản phẩm
được tạo thành.
1) Xác định các sản phẩm phản ứng của 3-methylcyclohexanone.
Chất khí tạo thành khi đun nóng 5.8 gam n-hexanoic acid với 1 đương
lượng HN3 khi có mặt sulfuric acid đã được thu lại, làm khô và đo thể
tích ở điều kiện tiêu chuẩn. Nếu sử dụng sulfuric acid đặc hoặc
phosphorus(V) oxide làm chất hút ẩm thì thể tích bằng 2.24 lít, còn nếu
dùng potassium hydroxide rắn thì thể tích giảm đi một nửa.
2) Xác định cấu trúc sản phẩm phản ứng sau khi trung hòa và tính
lượng chất (theo gam), giả sử rằng phản ứng diễn ra hoàn toàn,
không tạo thành sản phẩm phụ.
Phản ứng của cinnamic acid (C6H5CH=CHCOOH) với 1 đương lượng HN3
khi có mặt H2SO4 tạo thành sản phẩm Y chứa 80 % carbon và 6.67 %
hydrogen.
3) Viết cấu trúc hợp chất Y.
Trong phản ứng Schmidt, các aldehyde RCHO có thể tạo thành một
amide và một hợp chất không chứa oxygen. Chiều hướng phản ứng phụ
thuộc vào nhóm R và lượng sulfuric được thêm vào. Trong phản ứng
của benzaldehyde có lượng dư sulfuric acid, có 50 % amide và 5 % sản
phẩm phụ Z. Còn với lượng acid ít, lượng chất Z là 70 % và lượng chất Y
chỉ chiếm 13 %.
4) Viết cấu trúc các sản phẩm W, Z.

543 | Chuẩn bị kiến thức cho kì thi HSGQG Hóa học


Hướng dẫn
1) Trong phản ứng của ketone không đối xứng, cả hai nhóm chức đều
có thể chuyển vị. Do đó, các sản phẩm sau được tạo thành phản ứng
của 3-methylcyclohexanone.

2) 0.05 mol n-hexanoic acid đã tham gia vào phản ứng. Theo sơ đồ
phản ứng, một lượng mol nitrogen (1.12 L) sẽ được giải phóng. Thí
nghiệm cho thấy thể tích khí là gấp đôi. Trong đó, 1.12 lít khí nitrogen
và 1.12 lít còn lại là của acid oxide (bị hấp thụ bởi kiềm). Giả định hợp lí
nhất là khí CO2. Đối với phản ứng của ketone không đối xứng, về cơ bản
thì trong trường hợp của acid, sự chuyển vị 2 nhóm thế khác nhau gắn
với nhóm carbon của nhóm carbonyl là có thể. Nếu nhóm hydroxy được
chuyển đến nguyên tử nitrogen thì sản phẩm phản ứng là hydroxamic
acid không bay hơi. Nếu nhóm alkyl R bị chuyển đi thì sản phẩm chuyển
vị là carbamic acid thế N (carbon monoxide monoamide), chất này
trong H2SO4 nhanh chóng bị decarboxyl hóa tạo thành amine tương
ứng. Trong trường hợp này, giải phóng thêm 0.05 mol (1.12 lít CO2).

n-pentylamine được tạo thành từ hexanoic acid. Cứ 0.05 mol acid thì
0.05 mol n-pentylamine được tạo thành (4.35 gam).
3) Sản phẩm Y chứa số nguyên tử carbon và hydrogen bằng nhau .
Cinnamic acid chứa 9 nguyên tử carbon. Tuy nhiên, X = N hoặc X = O)
đều không phù hợp với công thức C9H9X. Đồng thời, nếu phản ứng của
cinnamic acid, giống như hexanoic acid, đi kèm với sự decarboxyl hóa
thì sản phẩm phải chứa 8 nguyên tử carbon (do đó là 8 nguyên tử
hydrogen). Khi đó, 13.3 % khối lượng còn lại tương ứng với 16 đơn vị khối

544 | Chuẩn bị kiến thức cho kì thi HSGQG Hóa học


lượng - nghĩa là oxygen. Sản phẩm có công thức C8H8O. Sự tạo thành
chất này có thể được giải thích như sau: khi nhóm C6H5CH=CH chuyển
vị, cinnamic acid bị chuyển thành 1-amino-2-phenylethylene (C8H9N),
chất này dưới các điều kiện phản ứng bị đồng phân hóa thành imine
C6H5CH2CH=NH, rồi thủy phân tạo thành phenylacetic aldehyde:

4) Khi sử dụng Schmidt aldehyde, hoặc nhóm R hoặc nguyên tử


hydrogen bị chuyển vị. Trong trường hợp đầu tiên, ta có một sản phẩm
“truyền thống”, nghĩa là amide. Trường hợp kia thì sự chuyển vị
hydrogen đi kèm với tách nước, và sản phẩm là nitrile (hợp chất này
không chứa oxygen), chất này bền trong điều kiện của phản ứng. Tách
nước, cách duy nhất để loại nguyên tử oxygen khỏi phân tử, không thể
xảy ra trong sự chuyển vị nhóm R, do đó trong trường hợp này thì
isonitrile RNC đã được tạo thành. Isonitrile chứa 1 nguyên tử carbon
kiểu carbene, do đó có hoạt tính cao với các tác nhân nucleophile, và
dĩ nhiên là dễ dàng phản ứng với nước.

Do đó, W là N-phenylformamide (PhNHCHO), và Z là benzonitrile


(PhCN).

545 | Chuẩn bị kiến thức cho kì thi HSGQG Hóa học


Chủ đề 24*: Chuyển vị Schmidt
Chuyển vị Schmidt là giai đoạn then chốt của một trong các phương
pháp tổng hợp LY-134046, là một chất ức chế -methyltransferase
enzyme. Nhược điểm của phương pháp này là sự tạo thành hai sản
phẩm đồng phân, do đó một phương pháp thay thế để điều chế LY-
134046 đã được phát triển (xem sơ đồ):

1) Hoàn thành sơ đồ chuyển hóa và xác định cấu trúc các hợp chất A-
H.
Chuyển hóa của A thành B, C có thể được hoàn thành theo cách khác:

2) Viết cấu trúc của sản phẩm trung gian I và các phương trình phản
ứng tương ứng. Chỉ ra tác nhân và điều kiện phản ứng phù hợp.

546 | Chuẩn bị kiến thức cho kì thi HSGQG Hóa học


Hướng dẫn
1) Cách đầu tiên để tổng hợp LY-134046 là một ví dụ về phản ứng
Schmidt của ketone không đối xứng. Hai sản phẩm B và C là kết quả
của việc đưa mảnh NH vào 2 liên kết C-C(O). Cấu trúc của sản phẩm
LY-134046 chỉ xác định duy nhất cấu trúc amide V. Do đó, sản phẩm
thứ hai có thể là một amide đồng phân.

Cách thứ hai bắt đầu với chuyển hóa của hợp chất có công thức
C8H4Cl2O3 thành chất D có công thức C10H5Cl2NO2. Trong tiến trình
chuyển hóa, thay vì 1 nguyên tử oxygen thì mảnh C2HN đã được đưa vào.
Mảnh này có mặt trong tác nhân ở dạng NC-CH. Sự thế 1 nguyên tử
oxygen bởi 1 nguyên tử carbon 2 nhóm thế là một biến thể của phản
ứng Wittig. Ta biết rằng các ketone và aldehyde tham gia vào phản ứng
này. Thoạt tiên, nguyên liệu đầu không phải là aldehyde và ketone. Tuy
nhiên, nó được đặc trưng bởi hiện tượng hỗ biến mạch vòng, tương tự
như những gì quan sát được trong trường hợp glucose và các đường
khác. Do đó, chất đầu có thể biểu diễn ở dạng 2,3-dichloro-6-
formylbenzoic acid. Nhóm aldehyde phản ứng với diethyl ester của
cyanomethylphosphonic acid tạo thành nitrile không no D. Chuyển
hóa của D thành E là sự cộng hợp 2 nguyên tử hydrogen. Phương án
thực tế duy nhất là khử liên kết dôi HC=CH thành CH2-CH2. Sẽ có cảm
giác hơi bất thường khi phản ứng khử được sử dụng. Vấn đề là bởi trong
nitrile D không chỉ có nhóm C≡N bị phân cực mà còn cả liên kết C=C
liên hợp với nó. Nhóm nitrile không bị khử bởi sodium borohydride (nếu
không thì không chỉ 2 mà sẽ có đến 4 nguyên tử H được bổ sung), nhưng
nitrile không no sẽ phản ứng với NaBH4 bằng cách gắn hydride ion có
tính nucleophile vào liên kết đôi C=C để tạo thành enolate ion, mà sự
proton hóa chất này tạo thành nitrile E no (C10H7Cl2NO2). Đây là một

547 | Chuẩn bị kiến thức cho kì thi HSGQG Hóa học


kiểu phản ứng Michael, mặc dù sản phẩm tạo thành là kết quả của sự
khử C=C. Nhóm carboxyl trong E bị methyl hóa tạo thành ester F.
Boran khử chọn lọc nhóm nitrile trong hợp chất F mà không ảnh hưởng
đến nhóm ester. Kết luận này được xác thực bởi thực tế rằng sản phẩm
LY-134046 được tạo thành trong phản ứng khử hợp chất H, chất này rõ
ràng là aminde đồng phân B dựa vào vị trí của nhóm carbonyl. Amide
này được tạo thành như là kết quả của sự tấn công nội phân tử của
nhóm amino vào mảnh ester của G.

2) Phương pháp này tạo thành ketone oxime (I) (xảy ra chuyển vị
Beckmann):

548 | Chuẩn bị kiến thức cho kì thi HSGQG Hóa học


Chủ đề 25*: Chuyển vị Claisen
Năm 1912, Ludwig Claisen phát hiện rằng khi đun nóng các o-
allylphenol dẫn đến sự chuyển vị thành 2-allylphenol. Sự chuyển vị này,
về sau được đặt theo tên ông, là ví dụ đầu tiên về nhóm chuyển vị [3,3]-
sigmatropic.

Ví dụ, nó được dùng để tổng hợp tác nhân kháng khuẩn lancheolatin B:

Hoàn thành sơ đồ trên (xác định cấu trúc các chất A-D.)

549 | Chuẩn bị kiến thức cho kì thi HSGQG Hóa học


Hướng dẫn
Công thức phân tử của chất đầu: C8H8O3, công thức của chất C: C10H8O3.
Do đó, trong 3 giai đoạn đầu tiên, đã có thêm 2 nguyên tử carbon được
đưa vào phân tử. Giai đoạn đầu tiên là phản ứng alkyl hóa phenol với
allyl bromide chứa 3 nguyên tử carbon. Mặc dù phân tử ban đầu chứa 2
phenolic oxygen nhưng sự alkyl hóa trên cả 2 nguyên tử đã không xảy
ra: không rõ làm thế nào để loại bỏ ngay 4 nguyên tử carbon. Có nghĩa
là chỉ xảy ra sự monoalkyl hóa. Bây giờ chún gta cần xác định nhóm
phenol nào phản ứng. Một nhóm phenol ở vị trí ortho so với nhóm acetyl
tham gia tạo thành liên kết hydrogen nội phân tử, còn nhóm ở vị trí para
thì không có khả năng tạo tương tác như vậy. Điều này dẫn đến việc
giảm hoạt tính của nhóm phenol ở vị trí ortho và tăng tính bền của sản
phẩm alkyl hóa trên nhóm OH ở vị trí para. Giai đoạn thứ hai (A → B) là
chuyển vị Claisen. Nó cũng dẫn tới sự tạo thành các sản phẩm đồng
phân, nhưng hướng “chính xác” của phản ứng được xác định bởi cấu
trúc của sản phẩm cuối (lancheolatin B). Công thức phân tử B là
C11H12O3. Do đó, ở giai đoạn chuyển B thành C, chúng ta phải loại bỏ 1
nguyên tử carbon và 4 nguyên tử oxygen. Để làm điều này, hai chất
oxid hóa đã được sử dụng đồng thời. OsO4 thực hiện dihydroxyl hóa vào
liên kết đôi C=C (cộng thêm vào mỗi carbon của liên kết đôi 1 nhóm OH)
và potassium periodate ngay lập tức phân cắt diol tạo thành. Điều này
cho phép loại bỏ nguyên tử carbon. Sản phẩm của phản ứng oxid hóa
như vậy có công thức C10H10O4. Có nghĩa là trong giai đoạn thứ hai của
chuyển hóa B thì 1 phân tử nước đã bị tách ra. Trong quá trình xúc tác
acid, nhóm hydroxy hoạt động hơn ở vị trí para so với nhóm thể acetyl
được gắn vào nhóm chức carbonyl tạo thành, sau đó tách nước xúc tác
acid của sản phẩm cộng tạo thành hệ thơm furan.

550 | Chuẩn bị kiến thức cho kì thi HSGQG Hóa học


Trong phản ứng với benzaldehyde khi có mặt base, đã xảy ra phản ứng
ngưng tụ aldol-croton giữa C và benzaldehye, tạo thành D có công thức
C17H12O3. Công thức phân tử của lancheolatin B là C17H10O3. Do đó phản
ứng iodine hóa là một quá trình oxid hóa-vòng hóa:

551 | Chuẩn bị kiến thức cho kì thi HSGQG Hóa học


Chủ đề 26*: Chuyển vị Schmidt
Các biến thể của chuyển vị Claisen đã được phát triển, trong đó mảnh
allyl vinyl ether được điều chế in situ (tại chỗ, ngay trong quá trình). Các
allyl ester của carboxylic acid đã bị chuyển vị bởi một chuỗi xử lí với
lithium diisopropylamide và trimethylchlorosilane, sau đó đun nóng:

Gần đây, phản ứng chuyển vị này đã được sử dụng để tổng hợp racemic
của muscone (мускон). Các giai đoạn cuối của tổng hợp này được trình
bày trong sơ đồ:

1) Xác định cấu trúc các chất E-I.


Một loại chuyển vị Claisen khác diễn ra khi các allylic alcohol được xử lí
với các orthoester khi có mặt xúc tác acid.
2) Xác định cấu trúc sản phẩm tạo thành khi xử lí triethyl orthoacetate
và 3-hydroxy-2,6-dimethylhepta-1,6-diene.

552 | Chuẩn bị kiến thức cho kì thi HSGQG Hóa học


Hướng dẫn
1) Giai đoạn đầu tiên: ester bị chuyển thành một hệ chứa 1 mảnh allyl
vinyl ester (E). Khi đun nóng, xảy ra chuyển vị Claisen (biến thể này
được gọi là chuyển vị Ireland-Claisen), tạo thành trimethylsilyl
ether của 3-methylcyclopentadecene-3-carboxylic acid, phản ứng
thủy phân chất này tạo thành acid F. Hydrogen hóa liên kết đôi tạo
thành vòng no. Sản phẩm tạo này tham gia vào phản ứng Borodine-
Hunsdiecker, chuyển hóa nhóm carboxyl, tạo ra
methylcyclopentadecyl bromide (C16H31Br) đi kèm sự tách CO2.
Phản ứng của bromide với kiềm tạo thành alcohol, chất này bị oxid
hóa ở giai đoạn cuối tạo thành ketone mong muốn.

2) Để trả lời câu hỏi này, cần hiểu làm sao mà chất đầu có thể tạo
thành 1 mảnh allyl vinyl ether. Hệ C=C-C-O đã có sẵn trong 2,6-
dimethylhepta-1,6-dien-3-ol ban đầu. Cần tạo thành 1 mảnh C=C-
O. Acid proton hóa nguyên tử oxygen của nhóm alkoxy trong
orthoester. Cation này tách phân tử ethanol tạo thành bis (alkoxy)
alkyl cation X tương đối bền. Nó phản ứng với 2,6-dimethylhepta-
1,6-dien-3-ol, xảy ra sự trao đổi các nhóm alcohol trong orthoester.
Orthoester hỗn hợp Y tạo thành bị proton hóa, hoặc ở nguyên tử
oxygen trong hệ O-C-C=C hoặc bởi oxyge trong nhóm OC2H5. Trong
trường hợp đầu tiên, sự tách phân tử ethyl alcohol thứ hai tạo thành
một cation Z mới tương đối bền. Cation này được bền hóa bởi sự
tách proton, tạo thành hệ allyl vinyl ester mong muốn trong hợp
chất trung gian W. Chuyển vị Claisen (trong trường hợp này gọi là

553 | Chuẩn bị kiến thức cho kì thi HSGQG Hóa học


chuyển vị Johnson-Claisen) bây giờ có thể diễn ra tạo thành ester
J.

554 | Chuẩn bị kiến thức cho kì thi HSGQG Hóa học


Phụ lục: Một số bài tập giải thích
cơ chế phản ứng
Nhắc lại quy tắc sử dụng mũi tên cơ chế

Nguyên tử có tính nucleophile Nguyên tử có tính electrophile


Nguồn electron Orbital trống
MO bị chiếm cao nhất (HOMO) MO không bị chiếm thấp nhất (LUMO)
Cặp electron Nguyên tử có tính electrophile
Liên kết π Nguyên tử có tính electrophile
Liên kết σ Nguyên tử có tính electrophile
Liên kết π Liên kết σ

Ví dụ 1 – Các mũi tên cong mô tả sự chuyển 2-electron

Ví dụ 2 – Bromine hóa olefin

Cơ chế trên sai vì:


1. Không được cân bằng điện tích.
2. Mũi tên cơ chế chứa cả phần chạy thẳng lẫn quay vòng.

555 | Chuẩn bị kiến thức cho kì thi HSGQG Hóa học


Các cơ chế thay thế hợp lí:

Ví dụ 3 – Oxi hóa sulfide thành sulfone

Cơ chế ở trên cũng sai, lí do tương tự như ví dụ 2. Dưới đây là cơ chế


thay thế hợp lí.

Ví dụ 4 – Epoxi hóa olefin.


Cơ chế sai:

Cơ chế thay thế:

556 | Chuẩn bị kiến thức cho kì thi HSGQG Hóa học


Ví dụ 5 – Amide hóa ester.

Cơ chế thay thế hợp lí:

Ví dụ 6 – Tạo thành isocyanate.

Cơ chế trên hợp lí bởi các mũi tên a và b không tạo thành một vòng tròn
(đầu của mũi tên a không đi đến đuôi của mũi tên b). Vậy nên, sơ đồ cơ
chế này đúng về mặt kĩ thuật, dù rằng không rõ ràng lắm. Bạn có thể
viết lại như sau cho rõ ràng hơn:

Ví dụ 7 – Các mũi tên cong mô tả sự chuyển 1-electron.

557 | Chuẩn bị kiến thức cho kì thi HSGQG Hóa học


Ví dụ 8 – Epoxi hóa olefin, sử dụng tác nhân gốc nitroxyl.

Ví dụ 9 – Epoxi hóa olefin, sử dụng tác nhân peroxide.

558 | Chuẩn bị kiến thức cho kì thi HSGQG Hóa học


Chủ đề 1: Chuyển hóa liên kế (tandem) tạo thành
naphthalene thế tetra-1,2,3,4
Năm 1995, các nhà Hóa học Mĩ đã chứng minh rằng có thể nhận được
dẫn xuất thế naphthalene 3 với hiệu suất cao bởi chuỗi chuyển hóa liên
kế (tandem) Pummerer-Diels-Alder bao gồm sự đun nóng sulfoxide 1,
acetic anhydride, và maleic anhydride ở 120 oC để tạo thành sản phẩm
2 (hiệu suất 87%), sau đó cho 2 phản ứng với p-toluenesulfonic acid
(PTSA) trong tetrahydrofuran (THF) ở 25 oC. Điều thú vị là khi sử dụng
methyl propiolate thay cho maleic anhydride thì các tác giả đã cô lập
được tetralone 4 (hiệu suất 51%) thay vì sản phẩm cộng 2. Đề xuất cơ
chế để giải thích các chuyển hóa này.

559 | Chuẩn bị kiến thức cho kì thi HSGQG Hóa học


Hướng dẫn
Cơ chế 1 → 2
Cơ chế được đề xuất bởi các tác giả để giải thích sự tạo thành
naphthalene thế 3 từ 1 bắt đầu với một chuyển vị Pummerer của
sulfoxide 1 khi xử lí với acetic anhydrid, tạo thành ion sulfonium B sau
khi deproton và tách acetic acid. Phản ứng vòng hóa của nhóm keton
của B tới ion sulfonium tạo thành C, chất này bị deproton hóa tạo
thành furan thế D. Lúc này, sự hiện diện của maleic anhydride đóng vai
trò như một dienophile cho phản ứng cộng vòng hetero-Diels-Alder [4
+ 2] giữa nó và diene D, tạo thành sản phẩm 2. Chúng ta chú ý rằng sản
phẩm 2 tồn tại ở dạng hỗn hợp xuyên lập thể phân do có thể có hai
hướng để dienophile tiếp cận với diene D trong phản ứng hetero-Diels-
Alder

Cơ chế 2 → 3
Xử lí 2 trong môi trường acid dẫn đến sự mất một phân tử nước để tái
tạo tính thơm của vòng cyclohexane trung tâm, dẫn đến sự tạo thành
sản phẩm naphthalene thế 3.

560 | Chuẩn bị kiến thức cho kì thi HSGQG Hóa học


Cơ chế D → 4
Chúng ta chú ý rằng trong giai đoạn này, khung bicyclic [2.2.1] của sản
phẩm 2 được chuyển hóa thành vòng benzene trung tâm. Khi sử dụng
methyl propiolate thay thế cho maleic anhydride thì các tác giả
Cochran và Padwa đã đề xuất một cơ chế khác để giải thích cho sự tạo
thành sản phẩm 4. Trong cơ chế này, vòng oxabicyclic của trung gian
H bị mở, tạo thành ion oxonium I, sau đó xảy ra phản ứng hetero-Diels-
Alder giữa D và methyl propiolate. Các tác giả đã lập luận rằng giai
đoạn H tạo thành I được thúc đẩy bởi cặp electron chưa liên kết của
nguyên tử lưu huỳnh. Sau đó, chuyển vị kiểu pinacol dẫn đến sự dịch
chuyển 1,2 nhóm phenyl của I, tạo thành sản phẩm 4. Các tác giả cũng
lưu ý rằng “tương tác FMO thuận lợi nhất là giữa HOMO của
isobenzofuran và LUMO của methyl propiolate. Hệ số nguyên tử ở vị trí
thế ethylthio trong vòng isobenzofuran lớn hơn so với hệ số ở vị trí
phenyl trong HOMO và điều này phù hợp với độ chọn lọc vị trí cao đã
thấy được.”

561 | Chuẩn bị kiến thức cho kì thi HSGQG Hóa học


Chủ đề 2: Vòng hóa gốc tạo ra các hydroazulene
Năm 1995, các nhà Hóa học Nhật Bản đã tiến hành phản ứng của
cyclopentanone thế 1 với triphenyltin hydride và AIBN
(azobis(isobutyronitrile)) trong toluene hồi lưu để thu được sản phẩm
hydroazulene 2 với hiệu suất 79%. Đề xuất cơ chế phản ứng cho
chuyển hóa này.

562 | Chuẩn bị kiến thức cho kì thi HSGQG Hóa học


Hướng dẫn
Cơ chế được đề xuất bởi nhóm Nishida cho sự tạo thành hydroazulene
2 từ cyclopentanone thế 1 bắt đầu với sự khơi mào gốc của gốc
triphenyltin. Quá trình này diễn ra qua sự tách nguyên tử hydrogen của
triphenyltinhydride bởi gốc isobutyronitrile được tạo thành từ AIBN.
Sau đó, gốc triphenyltin phản ứng với hợp phần alkyne của 1, tạo thành
gốc alkenyl A, tiểu phân này vòng hóa thành gốc alkoxyl B có bộ khung
bicyclic [4.3.0].
Quá trình phân mảnh đồng li của liên kết C-C chung của hệ vòng dung
hợp tạo thành gốc C, tiểu phân này lại đóng vòng 5-exo-trig tạo thành
gốc D có bộ khung bicyclic [4.3.0] khác với B. Tâm carbon gốc của D
chèn vào liên kết π của nhóm carbonyl, tạo thành gốc alkoxyl tricyclic
(ba vòng) E, tiểu phân này bị phân mảnh tạo thành sản phẩm
hydroazulene 2 khi tách gốc triphenyltin.

563 | Chuẩn bị kiến thức cho kì thi HSGQG Hóa học


Chủ đề 3: Chuyển hóa cinnoline thành indole
Năm 1995, các nhà hóa học Nhật Bản đã cho thấy rằng khi khuấy qua
đêm dẫn xuất dihydrocinnoline 1 với bột potassium cyanide trong
dung dịch nước dimethylformamide ở nhiệt độ thường, sau đó xử lí
nước rồi kết tinh lại thì thu được sản phẩm 2-acetyl-3-cyanoindole 2
với hiệu suất 54%. Đề xuất cơ chế cho chuyển hóa này.

564 | Chuẩn bị kiến thức cho kì thi HSGQG Hóa học


Hướng dẫn
Cơ chế do nhóm tác giả đề xuất cho chuyển hóa 1 thành 2 bắt đầu với
phản ứng cộng Michael của anion cyanide vào nhóm alkene phân cực
của enone 1, dẫn đến sự phân cắt vòng dicarboximide, tạo thành trung
gian A. Sau đó, bộ khung bicyclic [4.3.0] của trung gian A chuyển vị
thành bộ khung bicyclic [6.3.0], tạo thành trung gian B sau khi phân
cắt liên kết N-N. Ion amide trung gian B lại chuyển vị tạo thành enolate
tricyclic (ba vòng) C, tiểu phân này trải qua quá trình chuyển proton nội
phân tử, tạo thành carbanion D. Chú ý rằng carbanion này được bền
hóa bởi nhóm cyano hút electron.
Tiếp theo, trung gian D tự tách nhóm isocyanate (HN=C-OH) và phenyl
isocyanate để tạo thành trung gian E, tiểu phân này bị proton hóa khi
work-up tạo thành sản phẩm cuối indole 2. Các sản phẩm phụ
isocyanate và phenyl isocyanate được dự đoán có phản ứng với nước
trong phase work-up tạo thành hai đương lượng khí carbon dioxide,
một đương lượng ammonia và một đương lượng aniline (các sản phẩm
phụ cuối cùng). Chúng ta cũng có thể dự đoán rằng sự thủy phân
isocyanate diễn ra nhanh hơn của phenyl isocyanate.

Một cơ chế thay thế cũng có thể được đề xuất như sơ đồ dưới đây.
Trong cơ chế này, giai đoạn bắt đầu là sự tác kích nucleophile của
anion cyanide vào 1, gây ra sự phân mảnh của liên kết N-N tạo thành

565 | Chuẩn bị kiến thức cho kì thi HSGQG Hóa học


trung gian F. Trung gian này trải qua sự phân mảnh tiếp tạo thành G,
tiểu phân này tách phenylisocyanate tạo thành ion imide H. Sự proton
hóa H và đồng thời nhóm hydroxide tấn công vào nhóm isocyanate tạo
thành amide enolate I, tiểu phân này vòng hóa tạo thành ion amide J.
Ion amide này trải qua chuyển dịch protic, sau đó decarboxyl hóa, tạo
thành ion L. L tách ammonide tạo thành sản phẩm cuối 2 sau chuyển
vị 1,3-protic. Động lực cho hướng chuyển hóa này là việc tránh được
các trung gian ba và bốn vòng có năng lượng cao như B, C, D trong cơ
chế ban đầu được đề xuất.

566 | Chuẩn bị kiến thức cho kì thi HSGQG Hóa học


Chủ đề 4: Chuyển vị trong quá trình đo NMR
Năm 1995, khi nghiên cứu về các chất chuyển hóa được phân lập từ quá
trình phát triển quá mức của tảo ở vùng Okinawa, các nhà Hóa học Mỹ
đã quan sát thấy có sự chuyển vị của nakienone 1 thành hemiacetal 2
trong quá trình đo cộng hưởng từ hạt nhân (dung dịch CDCl3) của 1. Hãy
đề xuất cơ chế phản ứng cho chuyển hóa này.

567 | Chuẩn bị kiến thức cho kì thi HSGQG Hóa học


Hướng dẫn
Cơ chế được Nagle và Gerwick đề xuất để giải thích sự chuyển vị trong
quá trình đo NMR của nakienone 1 thành hemiacetal 2 gồm một quá
trình xúc tác acid diễn ra qua sự vòng hoa 1 tạo thành trung gian
bicyclic furan A. Trung gian này trải qua một quá trình vòng hóa khác
với sự mở vòng furan tạo thành B, tiểu phân này sau chuyển vị proton
và mất nước thì tạo thành tiểu phân cationic D. Chúng ta đưa ra một số
dạng cộng hưởng của D và chú ý rằng dạng thứ ba (D”’) có thể trải qua
chuyển vị [1,2] proton tạo thành ion oxonium E. Sau đó, ion này bị nước
tác kích, tạo thành F. F bị deproton hóa tạo thành G. Chuyển vị hydride
[1,5] liên phân tử của G tạo thành sản phẩm cuối 2.

Do các tác giả không cung cấp thêm các bằng chứng thực nghiệm
hoặc lí thuyết để củng cố cho cơ chế này, nên chúng ta cũng nên chú ý
đến một cơ chế thay thế như dưới đây. Trong trường hợp này, cơ chế
bắt đầu với sự proton hóa và tách nước khỏi 1 tạo thành tiểu phân
cationic I, tiểu phân này trải qua sự vòng hóa tạo thành trung gian

568 | Chuẩn bị kiến thức cho kì thi HSGQG Hóa học


bicyclic K sau khi deproton hóa. Trung gian K trải qua chuyển vị [1,7]
proton liên phân tử tạo thành L. Sau đó, K bị proton hóa tạo thành ion
oxonium M. Nước tấn công vào M, sau đó deproton hóa, tạo thành sản
phẩm cuối 2 – tồn tại trong một cân bằng với aldehyde mở vòng O.

569 | Chuẩn bị kiến thức cho kì thi HSGQG Hóa học


Chủ đề 5: Tổng hợp các dị vòng trifluoromethyl
Năm 1995, các nhà Hóa học Pháp sử dụng thành công các trung gian
trifluoroacetylketene (tạo ra từ phản ứng của acid chloride 1 và
trifluoroacetic anhydride trong pyridine) trong các phản ứng với
những olefin giàu điện tử như N,N-dimethylcyanamide để tạo thành
các sản phẩm như 1,3-oxazin-4-one 2 với hiệu suất cao (78%). Lưu ý
rằng sản phẩm này được tạo thành chủ yếu từ một phản ứng cộng vòng
[4+2] hình thức. Ngoài ra, trong các trường hợp mà sản phẩm 2 không
được phân lập và lượng dư trifluoroacetic anhydride không được loại
bỏ trước khi thêm cyanamide vào thì khi làm xử lí hỗn hợp phản ứng với
methanol thì tạo thành dihydro-1,3-oxazin-4-one 3, chất này chuyển
hóa thành 4 khi đun nóng trong ethanol khoảng 2 ngày. Hãy đề xuất cơ
chế để giải thích các chuyển hóa trên.

570 | Chuẩn bị kiến thức cho kì thi HSGQG Hóa học


Hướng dẫn
Cơ chế 1 → 2
Quá trình tổng hợp sản phẩm [1,3]oxazin-4-one 2 bắt đầu với sự
deproton hóa acid chloride 1 bởi pyridine, tạo thành carbanion A, tiểu
phân này sau đó tấn công nucleophile vào trifluoroacetic anhydride
tạo thành acid chloride B. Tiếp đó B lại bị deproton hóa bởi đương
lượng pyridine thứ hai, tạo thành acetylketene C, tiểu phân này sau đó
cộng hợp vào N,N-dimethylcyanamide qua phản ứng cộng vòng
heteroDiels-Alder [4+2] tạo thành sản phẩm 2.

571 | Chuẩn bị kiến thức cho kì thi HSGQG Hóa học


Cơ chế 2 → 3

Trong trường hợp 2 phản ứng với lượng dư trifluoroacetic anhydride thì
chúng ta có sự tạo thành ion iminium D/D’. Ion này phản ứng với ion
trifluoroacetate tạo thành sản phẩm cộng vòng E. Sản phẩm cộng này
trải qua quá trình phân mảnh, tạo thành trung gian lưỡng cực không
vòng F. Sau đó, sự vòng hóa lại của F tạo thành G, tiểu phân này tách
cation dimethylisocyanate và anion trifluoroacetate tạo thành trung
gian quan trọng 1,3-oxazin-4-one H. Trung gian này phản ứng với
methanol tạo thành 3. Sản phẩm methoxy 3 có thể được chuyển hóa
thành sản phẩm ethoxy 4 qua trung gian H. Tuy nhiên, các tác giả chưa
đưa ra được thêm các bằng chứng thực nghiệm hoặc lí thuyết để củng
cố cho cơ chế đã đề xuất.

572 | Chuẩn bị kiến thức cho kì thi HSGQG Hóa học


Cơ chế 3 → 4

573 | Chuẩn bị kiến thức cho kì thi HSGQG Hóa học


Chủ đề 6: Tổng hợp các chromone thế vị trí 3 từ
các pyranobenzopyran
Năm 1995, các nhà hóa học Ấn Độ đã xử lí pyranobenzopyran 1 với một
lượng dư ketone RCOCH3 có thể enol hóa khi có mặt dung dịch acid,
tạo thành sản phẩm chromone 2. Đề xuất cơ chế giải thích chuyển hóa
này và phác thảo quy trình tổng hợp chất 1.

574 | Chuẩn bị kiến thức cho kì thi HSGQG Hóa học


Hướng dẫn

Cơ chế được nhóm của Uddin đề xuất cho chuyển hóa 1 thành 2 bắt đầu
với sự tách ethanol từ 1 và tạo thành ion 3,4-dihydro-pyranylium A, tiểu
phân này phản ứng với dạng enol của ketone tạo thành B. Với proton
dễ bị enol hóa ở vị trí β, B sẽ bị mở vòng γ-pyrone bởi xúc tác acid tạo
thành C. Tiểu phân này bị quay liên kết tạo thành C’ rồi vòng hóa trở lại
thành D, sau đó mất nước tạo thành sản phẩm cuối chromone 2.
Đề xuất quy trình tổng hợp 1 từ các chất sẵn có thương mại:

575 | Chuẩn bị kiến thức cho kì thi HSGQG Hóa học


Chủ đề 7: Chuyển vị xúc tác acid của các 1-
arylindole
Năm 1995, các nhà hóa học Nga đã quan sát thấy rằng khi đun nóng 1-
arylindole 1 với polyphosphoric acid (PPA) ở 75-115 oC trong 25-150 giờ
thì tạo thành các 5H-dibenz[b,f]azepine 2 với hiệu suất trong khoảng
8-65%. Các tác giả cũng nhận thấy rằng hiệu suất cao nhất xuất hiện
khi nhóm R là 4-Me, trong khi với R là 4-NO2 hoặc 3-CF3 thì không xảy
ra chuyển vị 1 thành 2. Hãy đề xuất cơ chế cho chuyển hóa này, phù hợp
với các quan sát thực nghiệm ở trên.

576 | Chuẩn bị kiến thức cho kì thi HSGQG Hóa học


Hướng dẫn
Cơ chế được đề xuất bởi Tokmakov và Grandberg để giải thích chuyển
hóa của indole 1 thành azepine 2 bắt đầu với sự proton hóa 1 bởi
polyphosphoric acid, tạo thành ion indolium A. Ion này dễ bị tấn công
nucleophile bởi base liên hợp của polyphosphoric acid, phản ứng diễn
ra ở vị trí 2 tạo thành indoline B. Tiếp theo, trung gian B bị proton hóa
ở nguyên tử nitrogen để tạo điều kiện cho sự mở vòng qua sự phân cắt
liên kết C2-N tạo thành carbocation D. Sau đó, trung gian D được cho
là trải qua phản ứng thế electrophile nhân thơm ở vị trí ortho của nhóm
aryl (nghĩa là vị trí ortho so với nhóm amino), gây ra sự vòng hóa tạo
thành ion iminium E, tiểu phân này thơm hóa lại qua quá trình deproton
hóa bởi base liên hợp của PPA, tạo thành 10,11-dihydro5H-
dibenzo[b,f]azepine F. Tiếp đó là quá trình tách anion của PPA để tạo
ra G và quá trình tách thứ hai, tái tạo xúc tác và tạo thành sản phẩm
cuối azepine 2. Các tác giả cũng giải thích rằng sự hoạt hóa
electrophile của nhóm aryl ở vị trí ortho là rất quan trọng vì các nhóm
thế hút electron ở các vị trí 3 và 4 như 4-NO2 (nghĩa là R2) hoặc 3-CF3
(nghĩa là R1) đều không làm xảy ra chuyển vị. Do đó, có thể dự đoán rằng
một nhóm nhường electron ở vị trí 3 (R1) sẽ làm tăng hiệu suất phản
ứng. Đó là trong trường hợp khi R1 = Me, dẫn đến hiệu suất 2 là 65%.
Tuy nhiên, khi R2 là methoxy (OMe) thì hiệu suất của 2 chỉ là 8%. Điều
thú vị là khi R2 = Me thì hiệu suất 2 là 43%, và khi R2 = Cl thì hiệu suất
của 2 là 25%. Cuối cùng, khi R1 = CF3 thì không xảy ra chuyển vị.

577 | Chuẩn bị kiến thức cho kì thi HSGQG Hóa học


578 | Chuẩn bị kiến thức cho kì thi HSGQG Hóa học
Chủ đề 8: Loại nhóm bảo vệ bất thường của aryl
ester
Năm 1995, một nhóm nhà Hóa học Mỹ đã chứng minh rằng khi xử lí
2’,4’,6’- trihydroxyacetophenone 1 với 3.3 đương lượng methyl
chloroformate và triethylamine trong THF ở 0 oC thì dẫn đến sự tạo
thành tricarbonate 2 với hiệu suất 87% (Sơ đồ 1). Sau đó, cho
tricarbonate 2 phản ứng với 4 đương lượng sodium borohydride trong
hỗn hợp THF/nước 1:1 ở 0 – 25 oC thì tạo thành phenol với hiệu suất
83%. Hãy đề xuất cơ chế cho chuyển hóa 2 → 3.

579 | Chuẩn bị kiến thức cho kì thi HSGQG Hóa học


Hướng dẫn

Sơ đồ 1
Cơ chế được các tác giả đề xuất để giải thích cho chuyển hóa 2 → 3 bắt
đầu với sự khử bởi sodium borohydride của hợp phần ketone trong 2,
tạo thành alkoxide A, tiểu phân này trải qua sự chuyển nhóm
carbonate giữa oxygen mang điện tích âm và ester ở vị trí ortho qua
trung gian vòng B. Quá trình này tạo thành trung gian phenoxide C mà
sau đó sẽ trải qua quá trình decarboxyl hóa và tách loại anion
methoxide, tạo thành trung gian quinone methide D. Sử khử hóa D khôi
phục tính thơm của vòng và tạo ra sản phẩm phenol cuối 3 sau khi
proton hóa.
Chú ý rằng một hướng thay thế, không được các tác giả đề cập đến
trong báo cáo, cũng có thể được đề xuất (xem Sơ đồ 2), trong đó sự
phân mảnh của trung gian vòng B dẫn đến sự tách loại anion
methoxide và tạo thành 4H-[1,3]dioxin-2-one E. Tiếp đó, sự khử trung
gian này bởi sodium borohydride ở nguyên tử carbon benzylic dẫn đến
quá trình decarboxyl hóa và tạo thành phenoxide F – tiểu phân này bị
proton hóa khi work up, tạo thành sản phẩm cuối, phenol 3. Điều thú
vị trong cơ chế này là không tạo thành trung gian quinone methide.

580 | Chuẩn bị kiến thức cho kì thi HSGQG Hóa học


Sơ đồ 2
Cùng cần chú ý rằng các tác giả không cung cấp bằng chứng thực
nghiệm về trung gian quinone methide D mà họ đã đề xuất. Tuy nhiên,
chúng tôi đề xuất một thí nghiệm “bẫy” khả thi, theo đó, phản ứng khử
được thực hiện khi có mặt các dienophile Diels-Alder như các cấu trúc
tổng quát trong Sơ đồ 3 dưới đây. Nếu quinone methide D thực sự được
tạo thành ở dạng trung gian thì nó có thể bị “giữ lại” bởi các alkyne hoặc
alkene, dẫn đến sự tạo thành các sản phẩm chromene hoặc chroman
tương ứng. Nếu không quan sát được sự tạo thành sản phẩm nào, thì
có thể loại bỏ ý tưởng về “trung gian quinone methide D”.

Sơ đồ 3

581 | Chuẩn bị kiến thức cho kì thi HSGQG Hóa học


Chủ đề 9: Sự hình thành 3,1-benzoxathiin
Năm 1997, một nhóm nghiên cứu Nhật Bản khi đun nóng sulfoxide 1 với
1.2 đương lượng p-toluenesulfonic acid trong xylene trong 50 phút đã
nhận được 1,3-oxathiane 2 với hiệu suất 53%. Hãy đề xuất cơ chế giải
thích chuyển hóa này.

582 | Chuẩn bị kiến thức cho kì thi HSGQG Hóa học


Hướng dẫn
Abe và các cộng sự đã đề xuất rằng một quá trình chuyển vị Pummerer
nội phân tử của hợp chất sulfinyl không no γ,δ chịu trách nhiệm cho sự
tạo thành sản phẩm cuối 1,3-benzoxathiin 2 từ sulfoxide 1. Theo đó, cơ
chế đầu tiên được họ đề xuất bắt đầu với sự tạo thành ion vòng năm-
cạnh 3H-benzo[c][1,2]oxathiol-1-ium A qua quá trình proton hóa hợp
phần olefin của 1 và vòng hóa nhóm sulfoxide trên carbocation được
tạo ra đồng thời. Sau đó, trung gian A phân mảnh thành ion sulfonium
B, tiểu phân này vòng hóa thành ion oxonium sáu-cạnh C. Sự deproton
hóa C tạo ra sản phẩm 4H-benzo[d][1,3]oxathiine 2.
Cơ chế 1:

Cũng trong báo cáo này, các tác giả đã mô tả một hướng chuyển hóa
khác, liên quan đến chuyển vị Pummerer nội phân tử thông thường
hơn. Theo cơ chế này, sulfoxide 1 bị proton hóa tạo thành ion
hydroxysulfonium D, tiểu phân này trải qua sự tách nước, tạo thành ion
sulfonium E. Sự tái hợp nước của E ở nguyên tử ethylidene có tính
electrophile, sau đó proton hóa hợp phần olefin, tạo thành ion
oxathiin-3-ium C như cơ chế trước. Tuy nhiên, Abe và các cộng sự
không đi tới kết luận rằng cơ chế nào được diễn ra cho chuyển hóa này
và đã đề xuất rằng các nghiên cứu sâu hơn đối với phản ứng này là
không cần thiết.
Cơ chế 2:

583 | Chuẩn bị kiến thức cho kì thi HSGQG Hóa học


584 | Chuẩn bị kiến thức cho kì thi HSGQG Hóa học
Chủ đề 10: Tổng hợp Arylacetone bởi chuyển vị
kiểu-Carroll
Năm 1995, các nhà Hóa học Mỹ đã thực hiện phản ứng của p-quinol 1
với diketene 2, sử dụng một lượng xúc tác DMAP (4-N,N’-dimethyl
aminopyridine) ở nhiệt độ thường, tạo thành hỗn hợp sản phẩm chứa
arylacetone 3 với hiệu suất 72% và benzofuran 4 với hiệu suất 3%. Hãy
đề xuất cơ chế phản ứng để giải thích sự tạo thành các sản phẩm này.

585 | Chuẩn bị kiến thức cho kì thi HSGQG Hóa học


Hướng dẫn

Cơ chế do Sorgi và cộng sự đề xuất bắt đầu với sự tác kích nucleophile
ở nhóm hydroxyl của quinol 1 lên diketene 2, tạo thành enol B sau sự
chuyển proton nội phân tử trong A. Quá trình ketone hóa B sau đó dẫn
đến sự tạo thành trung gian 1,3-diketo C, tiểu phân này trải qua sự enol
hóa và xoay liên kết, tạo thành trung gian E’. Trung gian này trải qua
chuyển vị Carroll sigmatropic [3,3] tạo thành F, rồi sau đó tiểu phân
này thơm hóa qua sự enol hóa, tạo thành trung gian G. Trung gian này
trải qua quá trình decarboxyl hóa, xúc tác bởi DMAP (4-N,N’-
dimethylaminopyridine), tạo thành carboxylate H, chất này ketone
hóa, tái sinh DMAP, tạo thành sản phẩm, cuối phenol 3. Ở thời điểm
này, một lượng nhỏ 3 có thể tham gia vào phản ứng thứ cấp bắt đầu bởi

586 | Chuẩn bị kiến thức cho kì thi HSGQG Hóa học


quá trình nội phân tử tạo thành ketal I sau khi đóng vòng và chuyển
proton. Quá trình tách nước I được trợ lực bởi DMAP tạo thành sản
phẩm furan 4 cùng với sản phẩm phụ là nước. Lưu ý rằng kiểu tạo vòng
để hình thành benzofuran 4 là [3 + 2].
Chuyển hóa 3 → 4:

587 | Chuẩn bị kiến thức cho kì thi HSGQG Hóa học


Chủ đề 11: Thử nghiệm phản ứng Knoevenagel lại
tạo thành các sản phẩm kiểu Mannich!
Năm 1995, một nhóm nghiên cứu ở Bulgaria đã cố gắng thực hiện phản
ứng ngưng tụ Knoevenagel của coumarin aldehyde 1 (thường phản ứng
với các hợp chất malonate), trong đó piperidine đóng vai trò xúc tác
base, nhằm tạo thành các sản phẩm như 4. Tuy nhiên, họ đã quan sát
thấy một phản ứng Mannich ngoài dự đoán giữa piperidine và 1, tạo
thành sản phẩm 2 với hiệu suất 80%. Họ cũng chứng minh được rằng
khi xử lí 4-amino-2H-chromen-2-one 3 với formaldehyde và muối
piperidine hydrochloride cũng tạo thành 2 với hiệu suất 82%. Hãy đề
xuất cơ chế cho hai chuyển hóa 1 → 2 và 3 → 2.

588 | Chuẩn bị kiến thức cho kì thi HSGQG Hóa học


Hướng dẫn
Cơ chế được các tác giả đề xuất để giải thích cho chuyển hóa 1 → 2 bắt
đầu với phản ứng của piperidine và nhóm aldehyde của 1 để tạo thành,
sau khi chuyển proton, trung gian hemiaminal B, tiểu phân này tách
anion hydroxide tạo thành trung gian iminium hydroxide C. Trung gian
này được cho là trải qua sự chuyển hydride nội phân tử, tạo thành trung
gian iminium hydroxide D. Ở thời điểm này, anion hydroxide tác kích
nguyên tử carbon iminium có tính electrophile vừa được tạo thành,
sinh ra trung gian E, tiểu phân này bị phân mảnh thành sản phẩm cuối
E và sản phẩm phụ propanal sau quá trình chuyển proton.
Chuyển hóa 1 → 2:

Chúng tôi nhận thấy rằng các tác giả cũng thực hiện phản ứng kiểu
Mannich đi từ chất nền 3 và xử lí nó với formaldehyde và muối
hydrochloride của piperidine để tạo thành cùng sản phẩm 2 với hiệu
suất cao. Cơ chế được đề xuất cho chuyển hóa này bắt đầu với phản
ứng của 3 với formaldehyde, tạo thành trung gian imine G sau khi
chuyển proton. Sau đó, imine G trải qua sự hỗ biến tạo thành enamine
H, tiểu phân này có nhóm hydroxyl được proton hóa bởi piperidinium
hydrochloride tạo thành I. Sau đấy là sự tác kích nucleophile của
piperidine vào I, tách đồng thời nước và hydrochloric acid, tạo thành

589 | Chuẩn bị kiến thức cho kì thi HSGQG Hóa học


sản phẩm cuối 2. Các nhà nghiên cứu không thảo luận về cơ chế này
trong báo cáo của họ và họ cũng không cung cấp thêm các dữ kiện
thực nghiệm và bằng chứng quan trọng để củng cố cho cơ chế được đề
xuất cho chuyển hóa 1 → 2.
Chuyển hóa 1 → 3:

590 | Chuẩn bị kiến thức cho kì thi HSGQG Hóa học


Chủ đề 12: Các chuyển vị của muối pyridinium
Năm 1990, một nhóm nghiên cứu ở Latvia đã quan sát thấy rằng khi xử
lí các muối pyridinum 1 hoặc 3 với ethanolic sodium hydroxide ở nhiệt
độ thường trong 1 giờ thì thu được các sản phẩm tương ứng là 2 hoặc 4
với hiệu suất cao. Hãy đề xuất cơ chế giải thích hai chuyển hóa này.

591 | Chuẩn bị kiến thức cho kì thi HSGQG Hóa học


Hướng dẫn
Cơ chế 1 → 2:

592 | Chuẩn bị kiến thức cho kì thi HSGQG Hóa học


Cơ chế 3 → 4:

593 | Chuẩn bị kiến thức cho kì thi HSGQG Hóa học


Phần C: Hóa Lí – Hóa
Phân tích
Chương 7. Động hóa học
Chủ đề 1: Bậc phản ứng
1) Xét phản ứng diễn ra trong dung dịch nước: A + B → C
- Khi tăng gấp đôi nồng độ [B] và giữ nguyên nồng độ [A] thì
tốc độ phản ứng v tăng gấp đôi.
- Khi tăng gấp đôi nồng độ [A] và [B] thì v tăng 8 lần.
Hãy xác định các giá trị bậc riêng phần p, q lần lượt tương ứng với
các tác nhân A và B:
A. p = 1 và q = 1.
B. p = 2 và q = 1.
C. p = 1 và q = 2.
D. p = 2 và q = 2.
2) Hằng số tốc độ k được biểu diễn theo
A. mol-3 L-3 s-1.
B. mol-2 L2 s-1.
C. mol2 L-2 s.
D. mol3 L-3 s.
3) Để giảm thành quy luật động học bậc hai, hỗn hợp phản ứng ban
đầu phải được sử dụng sao cho
A. tác nhân B rất dư.
B. tác nhân A rất dư.
C. tác nhân A và B được lấy với các lượng bằng nhau.
D. tác nhân A và B được lấy theo các lượng hợp thức.
4) Từ đây, xét phản ứng với hỗn hợp ban đầu có tác nhân A lấy rất dư
so với tác nhân B. Hãy thiết lập mối liên hệ giữa hằng số tốc độ biểu
kiến k’ với k và nồng độ ban đầu của các tác nhân A, B.
A. k' = k A 
ini

B. k ' = k B
ini
2
C. k' = k A 
ini

594 | Chuẩn bị kiến thức cho kì thi HSGQG Hóa học


2
D. k' = k B
ini

5) Dựa vào đề bài đã cho ở ý trước, hãy xác định hàm số theo thời gian
có dạng đường thẳng (tuyến tính)
A. B = f ( t )
1
B. = f ( t)
B
C. ln B = f ( t )
1
D. 2
= f ( t)
B
6) Hệ số góc của đường này là
A. 2k’
B. -2k’
C. k’
D. -k’

595 | Chuẩn bị kiến thức cho kì thi HSGQG Hóa học


Hướng dẫn
1) Tốc độ của phản ứng A + B → C được viết là:
p q
v = k  A   B (1)

Khi tăng gấp đôi nồng độ [B] nhưng giữ nguyên nồng độ [A] thì tốc độ
phản ứng tăng gấp đôi, do đó:

( )
p q
2v = k  A   2 B (2)

Còn khi tăng gấp đôi nồng độ [A] và [B] thì tốc độ phản ứng tăng gấp 8
lần, do đó:

( ) ( )
p q
8v = k  2 A   2 B (3)

Từ các thương số
(2 ) và ( 3) - ta lập được hệ phương trình:
(1) (1)
2 = 2q p = 2
 
q = 1
p+q
8 = 2

Chọn phương án B.
2) Theo kết quả ở ý trước, tốc độ phản ứng đang xét là:
2 v
v = k  A   B  k = 2
A   B

Tiến hành phân tích thứ nguyên:

mol L−1  s−1 


k  3
  mol−2 L2  s−1 
 
mol L 
−1
 
Chọn phương án B.
3) Bậc toàn phần của phản ứng bằng 3. Để chuyển thành động học bậc
2 thì nồng độ của [B] phải là hằng số. Cách duy nhất để đạt được kết

596 | Chuẩn bị kiến thức cho kì thi HSGQG Hóa học


quả này là phải sử dụng hỗn hợp phản ứng mà B rất dư so với A. Trong
trường hợp này, nó tác động như sau:
2 2
B = B = const  v = k  B  A  = kapp  A 
ini ini
kapp

kapp là hằng số tốc độ biểu kiến.


Chọn phương án A.
4) Hỗn hợp ban đầu lấy tác nhân A rất dư so với B:
2
A  = A  = const  v = k  A   B = k' B
ini ini
k'
2
k' = k  A 
ini

Chọn phương án C.
5) Theo điều kiện đã cho, động học phản ứng bị chuyển về bậc 1:
v = k ' B

d B
Ngoài ra, theo định nghĩa tốc độ phản ứng: v = −  
dt

d B d B
Vậy: −   = k' B    = −k'dt
dt B

Lấy tích phân giữa các thởi điểm t = 0 và t, ta có:

B d B t
 = −k'  dt
t

B
ini B 0

B t
ln B   t = −k' t
    Bini  0

ln B − ln B = −k't


t ini

ln B = ln B − k't


t ini

Vậy đồ thị ln[B] có dạng tuyến tính. Chọn phương án C.

597 | Chuẩn bị kiến thức cho kì thi HSGQG Hóa học


6) Độ dốc của đồ thị này là −k’. Chọn phương án D.

598 | Chuẩn bị kiến thức cho kì thi HSGQG Hóa học


Chủ đề 2: Chu kì bán hủy
Nghiên cứu về sự suy giảm lượng chất phản ứng A trong quá trình phản
ứng diễn ra trong dung dịch nước: A → B + C. Thời gian bán phản ứng
(chu kì bán hủy) của nó không phụ thuộc vào nồng độ chất phản ứng
ban đầu và bằng: 1/2 = 1110 min

1)
A. Phản ứng tuân theo quy luật động học bậc 0.
B. Phản ứng tuân theo quy luật động học bậc 1.
C. Phản ứng tuân theo quy luật động học bậc 2.
D. Phản ứng tuân theo quy luật động học bậc 3.
2) Tính hằng số tốc độ phản ứng k:
−3 −1
A. k = 1.3  10 min
B. k = 2.71 10−4 min−1
C. k = 6.24  10−4 min−1
−4 −1
D. k = 9.01 10 min

Nồng độ chất phản ứng ban đầu là: A  = 2.4 mol L−1 .
0

3) Hãy tính nồng độ chất phản ứng sau 3330 phút (min):
A. A  = 0.8 mol L−1
3330

B. A  = 0.72 mol L−1


3330

C. A  = 0.4 mol L−1


3330

D. A  = 0.3 mol L−1


3330

4) Tính thời gian t1 để tại thời điểm đó có 25% chất phản ứng đã tiêu
hao:
A. t1 = 319 min
B. t1 = 2220 min
C. t1 = 555 min
D. t1 = 461 min

5) Tính thời gian t2 để tại thời điểm đó chỉ còn lại 10% chất phản ứng:

599 | Chuẩn bị kiến thức cho kì thi HSGQG Hóa học


A. t2 = 2560 min
B. t2 = 8490 min
C. t2 = 2000 min
D. t2 = 3690 min

600 | Chuẩn bị kiến thức cho kì thi HSGQG Hóa học


Hướng dẫn
Chu kì bán hủy
Xét phương trình phản ứng trong dung dịch:
A +B → C + D

Giả sử tốc độ phản ứng chỉ phụ thuộc vào nồng độ chất A:

1 d A  p
v=− = k  A 
 dt

Chu kì bán hủy 1/2 là thời gian cần để tiêu hao một nửa lượng chất
phản ứng ban đầu. Biểu thức của nó phụ thuộc vào giá trị của p:
ln2
- Nếu p = 1: 1/2 = - tức là 1/2 không phụ thuộc vào nồng độ chất
 k
phản ứng ban đầu.
2p−1 − 1
- Nếu p  1: 1/2 =
 k  (p − 1)  A 
p −1

1) A là chất phản ứng duy nhất và chu kì bán hủy không phụ thuộc vào
nồng độ chất phản ứng ban đầu. Vậy p = 1 và phản ứng tuân theo quy
luật động học bậc 1. Chọn phương án B.
ln2
2) Do p = 1 nên: 1/2 =
 k

Trong trường hợp này, hệ số tỉ lượng  = 1 nên ta có:


ln2 ln2
k= = = 6.24  10−4 min−1
1/2 1110

Chọn phương án C.

601 | Chuẩn bị kiến thức cho kì thi HSGQG Hóa học


3)

- Sau một chu kì bán hủy 1/2 thì lượng chất phản ứng A giảm đi 2 lần.
- Sau khoảng thời gian bằng n1/2 thì lượng chất phản ứng A giảm đi
2n lần. Ở đây, n = 3, nên:
A  2.4
A  = 30 = = 0.3 mol L−1
3330
2 8

Phương án D.
4) Biểu thức tốc độ phản ứng:

d A  d A 
v = −   = k  A     = −k  dt
dt dt
Lấy tích phân giữa khoảng thời gian t = 0 và t, ta có:
A 
t
d A  t
A  t
 = −k  dt  ln A   = −k t 
t

A 
A  0
A 
0
0
0

cuối cùng nhận được:  A  =  A   e−kt


t 0

Sau khoảng thời gian t1, 25% chất phản ứng đã biến mất:

A  = 0.75  A  =  A   e−kt1


t1 0 0

−kt1 3 kt 4 4
e = 0.75 =  e 1 =  kt1 = ln  
4 3 3

4 4 4


ln   ln   ln  
3 3 3
Xác định được: t1 =   =   1/2 =    1110 = 461 min
k ln2 ln2
Phương án D.

602 | Chuẩn bị kiến thức cho kì thi HSGQG Hóa học


5) Sau khoảng thời gian t2, còn lại 10% lượng chất phản ứng:

A  = 0.10  A  = A   e−kt2


t2 0 0
−kt2 kt2
e = 0.10  e = 10  kt2 = ln10

ln10 ln10 ln10


Xác định được t2 = = 1/2 =  1110 = 3690 min
k ln2 ln2

Phương án D.

603 | Chuẩn bị kiến thức cho kì thi HSGQG Hóa học


Chủ đề 3: Phản ứng bậc 2
2-bromooctane C8H17Br phản ứng với ion hydroxide tạo thành octane-
2-ol và ion bromide theo phương trình:

C8H17Br + HO− → C8H17OH + Br −

Ban đầu, hai chất phản ứng có cùng nồng độ a = 0.5 mol·L-1. Ion
bromdie được xác định bằng phương pháp chuẩn độ bạc. Ở 25 oC, nhận
được các kết quả như sau:
t(s) 1000 2000 3000 4000 5000
x = [Br-] (mol·L-1) 0.202 0.288 0.336 0.366 0.436
1) Chúng ta giả định bậc phản ứng toàn phần là bậc 2 và kí hiệu k là
hằng số tốc độ. Biểu thức tốc độ phản ứng v là:
A. v = kx2
( )
2
B. v = k a − x
dx
C. v = −
dt
d( a − x )
D. v = −
dt
2) Xác định biểu thức của sự diễn tiến ở dạng hàm của thời gian –
nhận được sau khi lấy tích phân biểu thức động học vi phân:
1 1
A. = − kt
a−x a
1 1
B. = + kt
a−x a
1 1
C. = + kt
x −a a
1 1
D. = − kt
x −a a
3) Xác định hàm số có đồ thị là một đường thẳng sao cho giả thuyết
về động học toàn phần bằng 2 được thỏa mãn:
1
A. = f ( t)
a−x
1
B. a − x = f  
t

604 | Chuẩn bị kiến thức cho kì thi HSGQG Hóa học


C. ln ( a − x ) = f ( t )
D. ln ( a − x ) = f (lnt )
4) Từ đồ thị ở ý 3, hãy suy ra giá trị k1 của k ở nhiệt độ 25 oC:
A. k1 = 1.36  10−3 mol−1 L  s−1
B. k1 = 1.56  10−3 mol−1 L  s−1
C. k1 = 2.75  10−4 mol−1 L  s−1
D. k1 = 1.54  10−4 mol−1 L  s−1
5) Ở 50 oC, hằng số tốc độ có giá trị là k2 = 1.86  10−2 mol L−1  s−1 . Sử
dụng định luật Arrhenius để xác định biểu thức năng lượng hoạt
hóa của phản ứng:

A. Ea =
(
R T2 − T1 k1
ln
)
TT
1 2
k2
R ( T2 − T1 ) k2
B. Ea = ln
TT
1 2
k1
RTT k
C. Ea = 1 2
ln 2
T2 − T1 k1
RTT
D. Ea = 1 2
ln (k1k2 )
T2 − T1
6) Cho biết hằng số khí lí tưởng R = 8.31 J·K-1·mol-1. Hãy tính giá trị
năng lượng hoạt hóa:
A. Ea = 83.7 kJmol−1
B. Ea = 2.11 kJmol−1
C. Ea = 79.3 kJmol−1
D. Ea = 1090 kJmol−1

605 | Chuẩn bị kiến thức cho kì thi HSGQG Hóa học


Hướng dẫn
1)
Tốc độ phản ứng theo thể tích
Cho phương trình phản ứng diễn ra trong dung dịch:
A +B → C + D

Tốc độ phản ứng theo thể tích có thể được xác định theo từng chất
phản ứng hoặc sản phẩm:

1 d A  1 d B 1 d C 1 d D


v=− =− =− =−
 dt  dt  dt  dt

v được biểu diễn theo mol·L-1·s-1.


Hãy lập bảng diễn tiến của phản ứng theo nồng độ:
Chất C8H17Br HO- C8H17OH Br-
Nồng độ ở t = 0 (mol·L-1) a a 0 0
Nồng độ ở t (mol·L )
-1
a–x a–x x x
Do tất cả các hệ số tỉ lượng đều bằng 1 nên:

d( a − x ) dx
v=− =
dt dt
Vậy phương án D đúng còn C thì sai.
Ngoài ra, phản ứng có bậc toàn phần bằng 2 nên:
2

v = k  C8H17Br   HO−  = k ( a − x ) = k (a − x )
p q p+ q 2

Vậy phương án B đúng còn A thì sai.


Chọn phương án B và D.
2) Bằng cách xác định được hai biểu thức của v thiết lập được ở ý
trước, ta nhận được phương trình vi phân:

d( a − x )
= k (a − x )
2
v=−
dt

606 | Chuẩn bị kiến thức cho kì thi HSGQG Hóa học


Ta có:

d( a − x )
= −kdt
(a − x )
2

d ( a − x ) = dx

dx
Phương trình vi phân được đơn giản hóa thành: = kdt
(a − x )
2

Ta lấy tích phân giữa t = 0 và t:

Chọn phương án B.
3) Để giả thuyết về bậc phản ứng toàn phần bằng 2 được thỏa mãn,
biểu thức được thiết lập ở ý trước phải được xác thực:
1 1
= + kt
a−x a

1
nói cách khác, đồ thị = f ( t ) phải là đường thẳng.
a−x

Chọn phương án A.
1
4) Hãy tính ở các thời điểm khác nhau:
a−x

Thời gian (s) 1000 2000 3000 4000 10000

607 | Chuẩn bị kiến thức cho kì thi HSGQG Hóa học


x (mol·L-1) 0.202 0.288 0.336 0.366 0.436

1
(mol-1·L) 3.36 4.72 6.10 7.46 15.6
a−x
1
Hãy dựng đồ thị = f ( t) :
a−x

Sự sắp xếp hoàn hảo của các điểm trên đồ thị khiến cho giả thuyết về
động học toàn phần bậc 2 được thỏa mãn. Phương trình đường thẳng
nhận được là:
1 1
= + kt
a−x a

Để tính độ dốc k của đường này, có thể sử dụng các điểm trong bảng
dữ kiện, do hầu như không có điểm nào lệch khỏi đường thẳng (nếu
ngược lại, có thể sẽ phải sử dụng các điểm mới).
Ở 25°C

 1   1 
  − 
 a − x 10000  a − x 1000 15.3 − 3.36
k1 = = = 1.36  10−3 mol L−1  s−1
10000 − 1000 9000
Phương án A.
5)
Định luật Arrhenius

608 | Chuẩn bị kiến thức cho kì thi HSGQG Hóa học


Hằng số tốc độ k của một phản ứng hóa học biến đổi theo nhiệt độ theo
dlnK Ea
quy luật: =
dT RT2
trong đó:
- R = 8.31 J·K-1·mol-1 là kí hiệu của hằng số khí lí tưởng;
- T là kí hiệu của nhiệt độ tuyệt đối, biểu diễn theo Kelvin (K);
- Ea là năng lượng hoạt hóa của phản ứng (J·mol-1) – đại diện cho hàng
rào năng lượng mà các chất phản ứng phải vượt qua để tổ hợp thành
các sản phẩm.
Để xác định Ea, cần lấy tích phân giữa các nhiệt độ T1 và T2.
Phương trình vi phân Arrhenius là:

Chọn phương án C.

8.31 298  323  1.86  10−2 


6) Ea = ln  −3 
323 − 298  1.36  10 

Ea = 83700 Jmol−1 = 83.7 kJmol−1

Chọn phương án A.

609 | Chuẩn bị kiến thức cho kì thi HSGQG Hóa học


Chủ đề 4: Loại bỏ các ion hypochlorite
Các ion hypochlorite ClO- có thể bị loại bỏ theo phương trình:
1 2
ClO− → ClO3− + Cl−
3 3

1) Tốc độ tiêu hao v của ion ClO- tuân theo quy luật động học bậc hai.
Hãy iểu diễn v ở dạng hàm của nồng độ ion chloride Cl-:
d Cl− 
A. v =
dt
2 d Cl 

B. v =
3 dt
3 d Cl 

C. v =
2 dt
1 d Cl 

D. v = −
2 dt
2) Đặt k là hằng số tốc độ của phản ứng. Hãy biểu diễn nồng độ ion
ClO- tại thời điểm t ở dạng hàm của nồng độ ion ClO- ban đầu:
ClO− 
A. ClO  = 
− 0
  t 1 + kt  ClO− 
 0
ClO  −

B. ClO−  =  0
  t 1 − kt  ClO− 
 0
(
C. ClO−  = ClO−   1 − kt
t 0
)
D. ClO−  = ClO−   e−kt
t 0

3) Ở thời điểm t = 0, nồng độ ion ClO- là c = 0.10 mol·L-1. Phản ứng diễn
ra ở nhiệt độ T1 = 343 K với hằng số tốc độ là k1 = 3.1·10-3 mol-1·L·s-1.
Xác định thời điểm t1 mà tại đó 30% ion ClO- đã tiêu hao:
A. t1 = 2 h 5 m.
B. t1 = 1 m 30 s.
C. t1 = 23 m.
D. t1 = 6 m 30 s.

610 | Chuẩn bị kiến thức cho kì thi HSGQG Hóa học


4) Năng lượng hoạt hóa của phản ứng Ea = 47 kJ·mol-1. Hằng số khí lí
tưởng R = 8.31 J·K-1·mol-1. Hãy tính giá trị hằng số tốc độ phản ứng
k2 ở nhiệt độ T2 = 363 K.
A. k2 = 7.7  10−3 mol−1 L  s−1
B. k2 = 9.3  10−3 mol−1 L  s−1
C. k2 = 1.0  10−3 mol−1 L  s−1
D. k2 = 1.25  10−3 mol−1 L  s−1
5) Xác định thời điểm t2 ở nhiệt độ T2 = 363 K mà tại đó phản ứng có tỉ
lệ diễn tiến tương tự (30%) từ cùng dung dịch ban đầu:
A. t2 = 7 m 40 s.
B. t2 = 50 m 30 s.
C. t2 = 41 m 50 s.
D. t2 = 9 m 20 s.

611 | Chuẩn bị kiến thức cho kì thi HSGQG Hóa học


Hướng dẫn
1) Tốc độ phản ứng đang xét được biểu diễn như sau:

d ClO−  d ClO3−  3 d Cl 


v=− =3 =
dt dt 2 dt
Chọn phương án C.
2) Tốc độ phản ứng tuân theo quy luật động học bậc 2:

d ClO−  2 d ClO− 
v=− = k ClO   −

= kdt
dt ClO− 
2

 
Lấy tích phân giữa các thời điểm t = 0 và t:

Vậy:

Chọn phương án A.

612 | Chuẩn bị kiến thức cho kì thi HSGQG Hóa học


3) Ở thời điểm t1: ClO−  = 0.7 ClO− 
t1 0

ClO−  1
 0
0.7 ClO  =−
 1 + k1t1 ClO−  =
0
1 + k1t1 ClO 
− 0 0.7
0

1  1 
t1 =  − 1
k1 ClO   0.7 

0

Thay số vào:

1  1 
t1 =  − 1  = 1380 s = 23 m
3.1 10  0.10  0.7 
−3

Chọn phương án C.
4) Để xác định k2, cần lấy tích phân giữa các nhiệt độ T1 và T2 của
phương trình vi phân Arrhenius:

Ta có:

47000( 363−343)
−3
Thay số vào: k2 = 3.1 10  e 8.31343363
= 7.7  10−3 mol−1 L  s−1

613 | Chuẩn bị kiến thức cho kì thi HSGQG Hóa học


Phương án A.

dlnk Ea
Hãy nhắc lại biểu thức Arrhenius: =
dt RT2
Năng lượng hoạt hóa Ea dương, theo đó hằng số tốc độ phản ứng tăng
theo nhiệt độ. Kết quả này, có thể được tổng quát hóa cho mọi phương
trình phản ứng, tương đối dễ hiểu: tốc độ phản ứng tỉ lệ thuận với tần
số va chạm giữa các vi hạt chất phản ứng. Xác suất gặp nhau của hai
vi hạt phản ứng tăng khi nồng độ tăng nhưng cũng tăng theo nhiệt độ.
Do đó, các câu trả lời C và D, đề xuất cho giá trị k2 nhỏ hơn k1, đều sai,
bởi T2 > T1.

1  1 
5) Tương tự ý 3, ta có: t2 =  − 1 
k2 ClO−   0.7 
0

1  1 
Thay số vào: t2 =  − 1  = 560 s = 9m 20 s
7.7  10  0.10  0.7 
−3

Chọn phương án D.

614 | Chuẩn bị kiến thức cho kì thi HSGQG Hóa học


Chủ đề 5: Phương pháp tốc độ đầu
Hydrogen chloride B cộng hợp vào cyclohexene A tạo thành
chlorocyclohexane C theo phương trình phản ứng:
C6H10 + HCl → C6H11Cl hoặc viết ở dạng đơn giản: A + B → C .

Phương pháp sắc kí khí giúp xác định tỉ lệ tương đối của A và C ở mọi
thời điểm, và dựa vào đó để nghiên cứu động học phản ứng. Bảng dưới
đây cung cấp các giá trị khác nhau của tốc độ đầu v0 theo các nồng độ
đầu a0 và b0 tương ứng với các chất phản ứng A và B. Các thí nghiệm
được tiến hành ở nhiệt độ 25 oC.
Thí nghiệm a0 (mol·L-1) b0 (mol·L-1) 109·v0 (mol·L-1·s-1)
1 0.587 0.294 30.8
2 0.587 0.336 40.2
3 0.587 0.410 59.8
4 0.587 0.560 111.6
5 0.391 0.560 74.3
6 0.196 0.560 37.2
Ta kí hiệu p và q lần lượt là bậc riêng phần của phản ứng theo các chất
phản ứng A và B, còn k là hằng số tốc độ. Biểu thức tốc độ phản ứng:
p q
v = k A   B

1) Để xác định bậc riêng phần p


A. chỉ cần dựa vào duy nhất một trong các thí nghiệm 1 – 6.
B. cần dựa vào các thí nghiệm 1 – 6.
C. cần dựa vào các thí nghiệm 1 – 4.
D. cần dựa vào các thí nghiệm 4 – 6.
2) Cần vẽ đồ thị nào để xác định bậc riêng phần p?
( )
A. v0 = f ln a0 

B. ln ( v ) = f ln ( a ) 
0 0

C. v0 = f (b0 )

( )
D. v0 = f ln b0 
3) Xác định độ dốc của đồ thị trên:
A. -p.

615 | Chuẩn bị kiến thức cho kì thi HSGQG Hóa học


B. p.
1
C. .
p
D. ln(p).
4) Xác định điểm chặn của đồ thị trên:
( ) q
A. ln k b0
B. k  ap0
C. ln (k )
D. q  ln (b0 )
5) Xác định các bậc riêng phần p và q:
A. p = 2 và q = 0.
B. p = 1 và q = 1.
C. p = 2 và q = 1.
D. p = 1 và q = 2.
6) Tính giá trị số của k:
A. k = 7.3  10−4 mol−1 L  s −1
B. k = 2.3  10−6 mol−1 L  s −1
C. k = 6.1 10−7 mol−2 L2  s −1
D. k = 1.4  10−5 mol−2 L2  s −1
7) Cho một hỗn hợp đẳng mol của A và B, với nồng độ ban đầu đều là
a0. Hãy thiết lập biểu thức tính chu kì bán hủy 1/2 :
1
A. 1/2 =
k  a0
2
B. 1/2 =
k  a0
3
C. 1/2 =
2k  a02
2
D. 1/2 =
k  a02

Hướng dẫn

616 | Chuẩn bị kiến thức cho kì thi HSGQG Hóa học


1) Trong trường hợp phản ứng cộng được xét, rõ ràng phương pháp tốc
độ đầu phải được sử dụng để phân tích dữ liệu. Biểu thức tốc độ phản
p q
ứng là: v = k A   B

và tốc độ đầu: v0 = k  ap0 b0q

v0 phụ thuộc vào hai biến a0 và b0: một trong đó phải được cố định. Để
xác định bậc riêng phần p thì cần biến đổi a0 nhưng không biến đổi b0,
do đó cần dựa vào dữ kiện của các thí nghiệm 4 – 6. Chọn phương án D.
2) Hãy nhắc lại biểu thức tốc độ đầu ở ý 1: v0 = k  ap0 b0q

Nếu ta biến đổi a0 nhưng cố định b0 thì v0 được viết lại ở dạng hàm của
biến duy nhất a0:

v0 ( a0 ) = k b0q  ap0 = kapp  ap0


kapp

Sẽ rất khó để xử lí số liệu trên các đường không-thẳng. Và việc lần lượt
đưa ra giả thuyết về các giá trị khả dĩ của p cũng tương đối… tẻ nhạt!
Vậy nên, một giải pháp phù hợp là biến đổi biểu thức trên bằng cách
lấy logarithm tự nhiên (hoặc thập phân) của hai vế, nhằm biểu diễn p ở
dạng hệ số hướng của một đường thẳng:

( )
ln ( v0 ) = ln kapp + p ln ( a0 )

Trong đó kapp = k  b0q là hằng số tốc độ biểu kiến. Do đó, để xác định bậc

riêng phần p, cần vẽ đồ thị của hàm số: ln v0 = f ln a0  - chọn ( ) ( )


phương án B.

( ) ( ) ( )
3) Đồ thị ln v0 = ln kapp + p ln a0 có độ dốc là p.

Chọn phương án B.

( ) ( q
)
4) ln kapp = ln k b0 . Chọn phương án A.

617 | Chuẩn bị kiến thức cho kì thi HSGQG Hóa học


( )
5) Xác định p: Xét đồ thị ln v0 = f ln a0  ( )
Chúng ta sử dụng các thí nghiệm 4 – 6, với b0 được cố định và bằng:

b0 = 0.560 mol L−1

Hãy tính các giá trị ln(a0) và ln(v0) khác nhau:


Thí nghiệm a0 (mol·L-1) ln(a0) ln(v0)
4 0.587 -0.533 -16
5 0.391 -0.939 -16.4
6 0.196 -1.63 -17.1
Sau đó, ta dựng được đồ thị như dưới đây:

p đại diện cho độ dốc của đường thẳng:

ln ( v0 ) − ln ( v0 ) −17.1 − ( −16.0) −1.1


p= 6 4
= = 1
ln ( a0 ) − ln ( a0 ) −1.63 − ( −0.533) −1.097
6 4

Xác định q: Khi a0 cố định, v0 có thể được viết lại như sau:

v0 (b0 ) = k  ap0 b0q = k' app b0q


k'app

( ) (
Lấy logarithm tự nhiên của hai vế: ln v0 = ln k' app + q ln b0 ) ( )
( ) ( )
Xét đồ thị ln v0 = f ln b0 

Chúng ta sử dụng các thí nghiệm 1 – 4, với a0 được cố định và bằng:

618 | Chuẩn bị kiến thức cho kì thi HSGQG Hóa học


a0 = 0.587 mol L−1

Hãy tính các giá trị ln(b0) và ln(v0) khác nhau:


Thí nghiệm b0 (mol·L-1) ln(b0) ln(v0)
1 0.294 -1.22 -17.3
2 0.336 -1.09 -17.0
3 0.410 -0.892 -16.6
4 0.560 -0.580 -16.0
Sau đó, ta dựng được đồ thị như dưới đây:

p đại diện cho độ dốc của đường thẳng:

ln ( v0 ) − ln ( v0 ) −16.0 − ( −17.3 ) 1.3


q= 4 1
= = 2
ln (b0 ) − ln (b0 ) −0.580 − ( −1.22 ) 0.64
4 1

Chọn phương án D.
6) Tốc độ đầu được viết là: v0 = k  ap0 b0q = k  a0 b02

Hãy sử dụng thí nghiệm 1 để xác định giá trị số của k:

v0 30.8  10−9
k= = = 6.1 10−7 mol−2 L2  s−1
0.587  (0.294)
2 2
a0 b 0

Chọn phương án C.

619 | Chuẩn bị kiến thức cho kì thi HSGQG Hóa học


Để chắc chắn, hãy thử kiểm tra lại với một thí nghiệm khác, ví dụ như
thí nghiệm 6:

v0 37.2  10−9
k= = = 6.1 10−7 mol−2 L2  s−1
0.196  (0.560)
2 2
a0 b 0

7) Trong hỗn hợp đẳng mol của A và B, ở mỗi thời điểm, nồng độ a và b
của các chất phản ứng A và B đều bằng nhau:
a = b, t

Tốc độ phản ứng được viết lại là: v = k  a b2 = k  a3


da
Mặt khác: v = −
dt

da da
Do đó phương trình vi phân: − = k  a3  − 3 = kdt
dt a

Lấy tích phân giữa các thời điểm t = 0 và 1/2 :


a0 a0

2
da 1/2
 1 2 1 1 3
a a3 0 dt   2a2  = k1/2   2 − 2a2 = k1/2  2a2 = k1/2
− = k
a0 a 0 0
2 0 
0

 2 
3
1/2 =
2k  a02

Chọn phương án C.

620 | Chuẩn bị kiến thức cho kì thi HSGQG Hóa học


Chương 8. Cân bằng hóa học
Chủ đề 1: Nguyên lí Le Chatelier
1) Khái niệm “cân bằng” được sử dụng để mô tả trạng thái trong các
hệ hóa học được nghiên cứu ở cấp độ vĩ mô (quy mô “của chúng ta”),
bằng cách lập luận về các lượng vật chất có thể đo được. Liệu khái
niệm này có thể đo được ở cấp độ vi mô, ví dụ như với phản ứng diễn
ra theo hai chiều và được thực hiện từ một số ít phân tử, hoặc thậm
chí từ một phân tử chất phản ứng riêng lẻ?
2) Sau khi để phản ứng H2 ( g ) + I2 ( g ) 2HI ( g ) đạt tới cân bằng, sao
cho thành phần hỗn hợp ba chất khí không còn biến đổi theo thời
gian, thì khí deuteium D2 được đưa vào hệ (deuterium D là đồng vị
hydrogen 2H và D2 có tính chất hóa học gần như giống hệt H2). Sau
một thời gian, hỗn hợp khí cũng có chứa deuterium iodide DI và các
phân tử hỗn tạp HD. Hãy mô tả về các cân bằng trong hệ mới.
3) Các cân bằng sau có bị chuyển dịch không nếu thể tích của hệ được
tăng gấp đôi. Nếu có thì chiều chuyển dịch sẽ là trái sang phải
(chiều thuận) hay phải sang trái (chiều nghịch)?
a) Br2 (l) + Cl2 ( g ) 2BrCl ( g )
b) CO ( g ) + Cl2 ( g ) COCl2 ( g )
c) 2H2O ( g ) + 2Cl2 ( g ) 4HCl ( g ) + O2 ( g )
d) 2COF2 ( g ) CO2 ( g ) + CF4 (g )
e) H2O (l) + CO2 ( g ) H2CO3 ( aq)
f) H2S ( g ) + I2 ( s ) 2HI ( g ) + S ( s )

621 | Chuẩn bị kiến thức cho kì thi HSGQG Hóa học


1
4) Cân bằng HgO( s ) Hg (l) + O2 ( g ) được thiết lập trong một bình
2
phản ứng với dung tích có thể biến đổi. Hãy xác định hệ quả của các
tác động sau (tất cả đều được thực hiện ở nhiệt độ không đổi):
a) Thêm vào một lượng nhỏ mercury oxide HgO.
b) Tách một phần thủy ngân.
c) Thêm oxygen vào ở áp suất không đổi.
d) Tách một phần oxygen ra ở áp suất không đổi (bằng cách biến
đổi thể tích sao cho áp suất giữ nguyên).
5) Việc giảm nhiệt độ có gây ra sự dịch chuyển các cân bằng sau đây
không. Nếu có, thì theo chiều nào?
a) BaCO3 ( s ) BaO ( s ) + CO2 ( g ) o
rHm = +269.3 kJ  mol−1
b) CO ( g ) + 3H2 ( g ) CH4 ( g ) + H2O (g ) o
rHm = −230 kJ mol−1
c) CH3COOH( aq) + CH3OH( aq) CH3COOCH3 ( aq) + H2O (l )

rHmo  0 kJmol−1

6) Cân bằng PCl5 ( g ) PCl3 ( g ) + Cl2 (g ) được tạo thành từ 1 mol PCl5
trong bình phản ứng với dung tích có thể thay đổi ở nhiệt độ 500 oC.
Sau đó, đưa argon (chất khí trơ, không phản ứng với bất kì cấu tử
nào trong hệ) vào và vẫn duy trì nhiệt độ ở 500 oC. Việc bổ sung khí
trơ có gây chuyển dịch cân bằng không
a) thể tích của bình phản ứng được giữ không đổi.
b) áp suất tổng được giữ không đổi.
7) Xác định biểu thức thương số phản ứng cho các cân bằng sau:
a) 2NO ( g ) + Cl2 ( g ) 2NOCl (g )
b) MgSO3 ( s ) MgO ( s ) + SO2 ( g )
c) HNO2 ( aq) + H2O (l) H3O+ ( aq) + NO2− ( aq)
d) 2NaHCO3 ( s ) Na2CO3 ( s ) + H2O ( g ) + CO2 ( g )
e) 3Fe ( s ) + 4H2O ( g ) Fe3O4 ( s ) + 4H2 ( g )
f) Zn ( s ) + 2Ag+ ( aq) Zn2+ ( aq) + 2Ag ( s )

622 | Chuẩn bị kiến thức cho kì thi HSGQG Hóa học


Hướng dẫn

1) Hãy lấy một ví dụ: H2 + Cl2 2HCl. Phản ứng giữa một phân tử H2 và
một phân tử Cl2 không thể dẫn đến trạng thái đặc trưng của một cân
bằng hóa học: nếu có diễn ra, hai phân tử HCl cần được tạo thành đồng
thời và chẳng còn lại phân tử H2 hoặc Cl2 nào. Khi đó, phản ứng ngược
lại diễn ra, hai phân tử HCl biến mất để tạo thành một phân tử H2 và
một phân tử Cl2; phản ứng không thể diễn ra một phần.Trạng thái cân
bằng chỉ có thể được xác định về mặt thống kê, với một hệ cấu thành
bởi số lượng lớn các phân tử. Các đại lượng mà nó phụ thuộc (H, S, G, p,
T, …) có bản chất vĩ mô và chỉ có giá trị được xác định với số lượng lớn
các phân tử (ví dụ, hãy hình dung về áp suất hoặc nhiệt độ, vốn liên
quan đến động năng trung bình của các phân tử chất khí). Do đó, khái
niệm “cân bằng” không có ý nghĩa ở cấp độ vi mô.
2) Trong bình phản ứng ban đầu, ba chất khí (H2, I2, HI) tồn tại đồng thời
trong cân bằng:
H2 ( g ) + I2 ( g ) 2HI ( g )

Khi thêm deuterium vào, nó phản ứng với I2 tương tự như H2. Có một
cân bằng khác:
D2 ( g ) + I2 ( g ) 2DI ( g )

Chúng ta có thể hình dung rằng các phân tử tạo thành cũng có thể
phản ứng với nhau:
HI( g ) + DI( g ) HD ( g ) + I2 ( g )

Nồng độ tất cả các tiểu phân trong phase khí sẽ thay đổi bởi phản ứng
xảy ra giữa HI và DI. Hai cân bằng đầu tiên sẽ dịch chuyển và cho phép
một cân bằng mới đạt được giữa HI, DI, HD và I2.
3) Nguyên lí Le Chatelier giúp dự đoán định tính ảnh hưởng của các
biến đổi tác động lên hệ hóa học. Sự thay đổi thể tích dẫn đến sự biến
đổi áp suất khí và có thể gây ra sự chuyển dịch cân bằng. Hệ hóa học
có thể biến đổi nếu sự dịch chuyển cân bằng làm cho áp suất tổng biến
đổi theo chiều ngược lại với chiều của tác động được áp đặt vào.

623 | Chuẩn bị kiến thức cho kì thi HSGQG Hóa học


Khi thể tích tăng thì áp suất giảm và cân bằng sẽ chuyển dịch theo
chiều tạo thành nhiều phân tử khí hơn (để làm áp suất tăng lên). Trong
trường hợp hai vế của cân bằng có tổng số phân tử khí bằng nhau thì
cân bằng không chuyển dịch theo chiều nào cả.
- Chuyển dịch theo chiều thuận: a, c, f.
- Chuyển dịch theo chiều nghịch: b, e.
- Không chuyển dịch: d
4)
2
tương đối của một chất rắn hoặc chất lỏng tinh khiết, cấu thành một
3
phase riêng biệt trong hệ, luôn luôn bằng 1 – bất kể lượng chất cỡ nào.
Hoạt độ tương đối của một chất khí bằng tỉ lệ áp suất của nó (áp suất
tổng nếu chỉ có một mình nó, hoặc áp suất riêng phần nếu trong hỗn
hợp chất khí) với áp suất tham chiếu p0.
Tác động liên quan đến lượng chất lỏng hoặc chất rắn không ảnh hưởng
đến chuyển dịch cân bằng của phase khí.
Các tác động a và b không dẫn đến bất kì sự dịch chuyển cân bằng nào
(thông tin “một lượng nhỏ” giúp ta đưa ra giả định rằng thể tích bị chiếm
bởi các chất khí hầu như không bị ảnh hưởng).
Tác động c: Việc đưa thêm oxygen vào ở thể tích không đổi sẽ làm tăng
thể tích của phase khí. Điều này dẫn đến sự dịch chuyển cân bằng sang
trái (để chống lại sự tăng áp suất).
Tác động d: Do áp suất không thay đổi nên không có sự chuyển dịch
cân bằng.
5) Tương tự như áp suất, sự biến đổi nhiệt độ từ bên ngoài cũng có thể
dẫn đến sự chuyển dịch cân bằng nếu như phản ứng có thể gây ra tác
động – điều này có nghĩa là phản ứng phải thu nhiệt (làm hạ nhiệt độ,
trong một hệ cô lập) hoặc tỏa nhiệt (làm tăng nhiệt độ). Với phản ứng c
thì sự thay đổi nhiệt độ không gây ra chuyển dịch cân bằng (vì phản ứng
không thu nhiệt mà cũng chẳng tỏa nhiệt).
Tuy nhiên, rất hiếm có phản ứng có enthalpy bằng 0 như vậy – nên gần
như trong mọi trường hợp, sự biến đổi nhiệt độ từ bên ngoài sẽ luôn là
yếu tốc làm chuyển dịch cân bằng. Theo nguyên lí Le Chatelier, nếu

624 | Chuẩn bị kiến thức cho kì thi HSGQG Hóa học


nhiệt độ bị hạ thấp, thì phản ứng sẽ chuyển dịch theo chiều làm nhiệt
o
(
độ tăng lên – nghĩa là chiều mà nó tỏa nhiệt rHm  0 )
- Chuyển dịch theo chiều thuận: b
- Chuyển dịch theo chiều nghịch: a
- Không chuyển dịch: c
6) a) Rõ ràng, trong trường hợp này, áp suất tổng tăng lên. Nếu nó tăng
lên bởi sự giảm thể tích sẵn có của hỗn hợp khí thì có khả năng xảy ra
sự chuyển dịch cân bằng, như dự đoán của nguyên lí Le Chatelier.
Nhưng, do một chất khí khác được đưa vào hệ, nên áp suất tổng không
phải biến số được xét và trường hợp này phải được giải thích dựa trên
quan điểm về các áp suất riêng phần.
Liệu áp suất riêng phần của PCl5, PCl3 và Cl2 có biến đổi không? Nếu
vậy, sẽ cần xác định liệu sự biến đổi của chúng làm tăng hay giảm
thương số phản ứng Q (còn hằng số cân bằng K thì vẫn giữ nguyên, bởi
nhiệt độ không thay đổi). Việc thêm khí trơ vào hệ ở thể tích không đổi
không làm biến đổi áp suất riêng phần của ba chất khí đã hiện diện
trước đó (luôn có cùng lượng chất khí trong cùng thể tích như vậy). Do
đó, thương số phản ứng Q không đổi và không có sự chuyển dịch cân
bằng.
b) Áp suất riêng phần của chất khí tham gia vào cân bằng giảm (coi như
thể tích đã tăng nhiều mà không có argon được đưa vào). Thương số
phản ứng có biểu thức là:

 pPCl  pCl 
 3  2 
 p0   p0 
Q=   
 pPCl 
 5
 p0 
 
Tử số có bậc 2, còn mẫu số bậc 1 nên Q giảm và có sự chuyển dịch cân
bằng theo chiều thuận (từ trái sang phải).

625 | Chuẩn bị kiến thức cho kì thi HSGQG Hóa học


7) Xét phản ứng tổng quát:
aA + bB + ... mM + nN + ...

Thương số phản ứng, có giá trị khi cân bằng chính là hằng số cân bằng
K, có biểu thức như sau:
aMm  aNn ...
Q=
aaA  aBb ...

Trong đó a là hoạt độ tương đối của mỗi thành phần trong hệ, có thể
được biểu diễn theo:
- Với chất khí: là tỉ lệ áp suất riêng phần của nó với áp suất tham
chiếu p0 (p0 = 1 bar).
- Với tiểu phân trong dung dịch: là tỉ lệ nồng độ mol của nó với
nồng độ tham chiếu c0 (c0 = 1 mol·L-1).
- Với chất rắn hoặc chất lỏng tinh khiết: hoạt độ tương đối được
định nghĩa bằng 1, nên chúng không xuất hiện trong biểu thức
thương số phản ứng. Dung môi của một dung dịch loãng thường
được xem như là một chất lỏng tinh khiết.
- Với hỗn hợp chất lỏng đồng thể (ví dụ như hỗn hợp các chất hữu
ni
cơ) thì hoạt độ là tỉ lệ phần mol i = (ni là số mol thành
nliq
phần đang xét; n liq
là tổng số mol chất lỏng trong hỗn hợp).
2
 pNOCl 
 
 p0 
a) 2
 pNO   pCl2 
   
 0   p0
p 

pSO
b) 2

p0
H3O+  NO2 
  
c)
HNO2 

626 | Chuẩn bị kiến thức cho kì thi HSGQG Hóa học


 pH O   pCO 
d)  2
 2 
 p0   p0 
   
4
 pH 
 02 
p 
e)  
4
 pH O 
 0 
2

 p 
 
Zn2+ 
 
f) 2
Ag  +
 

627 | Chuẩn bị kiến thức cho kì thi HSGQG Hóa học


Chủ đề 2: Hằng số cân bằng
1) Hằng số cân bằng K1 của phản ứng
1
N2O ( g ) + O2 ( g ) 2NO (g ) (1) có giá trị 0.97 ở 298 K.
2

a) Giá trị K2 tương ứng với phương trình có hệ số tỉ lượng như dưới
đây có khác với K1 không?
2N2O ( g ) + O2 ( g ) 4NO ( g ) (2 )
b) Tính (theo K1) giá trị hằng số cân bằng K3 liên hệ với phương trình
có hệ số tỉ lượng như dưới đây:
4NO ( g ) 2N2O ( g ) + O2 ( g ) (3 )
2) Biết các cân bằng
5 3
NH3 ( g ) + O2 ( g ) NO ( g ) + H2O ( g ) ( 1)
4 2
1
và NO2 ( g ) NO ( g ) + O2 ( g ) (2 )
2
lần lượt có hằng số cân bằng là K1 và K2. Hãy xác định biểu thức của
hằng số cân bằng K3 cho cân bằng dưới đây, ở dạng hàm của K1 và K2:
7
2NH3 ( g ) + (g)O2 2NO2 ( g ) + 3H2O ( g ) (3)
2

3) Trộn một hỗn hợp đẳng mol của ethanol C2H5OH và ethanoic acid
CH3COOH rồi đun nóng với xúc tác phù hợp, sẽ dẫn tới sự tạo thành
ester và nước theo phương trình phản ứng:
C2H5OH(l ) + CH3COOH(l ) CH3COOC2H5 (l ) + H2O (l )

2 2
Thành phần khi cân bằng của hệ có mol ester và mol nước. Hãy
3 3
tính thương số phản ứng ban đầu (Qr)I và hằng số cân bằng K.
4) Nguồn quặng chính của kẽm là zinc sulfide ZnS – thường gọi là
blende. Việc điều chế (luyện) kẽm được tiến hành theo hai giai

628 | Chuẩn bị kiến thức cho kì thi HSGQG Hóa học


đoạn. Trong bài này, chúng ta sẽ tìm hiểu về giai đoạn đầu tiên:
điều chế zinc oxide ZnO bằng cách nung blende. Phản ứng nung:
3
ZnS ( s ) + O2 (g ) ZnO (s ) + SO2 (g )
2
có hằng số phản ứng K0(T)
a) Biểu diễn K0(T) của phản ứng nung theo hoạt độ của các cấu
tử.
b) Biểu diễn K0(T) ở dạng hàm của các thông số sau: các áp suất
riêng phần khi cân bằng pO , pSO và áp suất tham chiếu p0.
2 2

c) Biểu diễn K (T) ở dạng hàm của các thông số sau: các lượng
0

chất (số mol) khi cân bằng nO ,nSO ; p0; áp suất tổng pT và tổng
2 2

lượng chất ở trạng thái khí nT.

629 | Chuẩn bị kiến thức cho kì thi HSGQG Hóa học


Hướng dẫn

1) G là đại lượng có thuộc tính khuếch độ, do đó rGmo của phản ứng (2)
( )
với hệ số tỉ lượng  NO2 = 2 sẽ gấp đôi rGmo của phản ứng (1).

o
 rGmo 
Do  G = −nRTlnK  hay lnK = − 2
 nên lnK2 = 2lnK1  K2 = K1
r m  RT 

Nói một cách tổng quát, nếu hệ số tỉ lượng trong phương trình phản
ứng nhân lên n lần thì hằng số cân bằng tăng lên theo số mũ n.
b) Với phương trình (3), ngược chiều với phương trình (2) nên
1 1
o
rGm3 o
= −rGm2 - tức là lnK3 = − lnK2  K3 = = 2
K2 K1

2) Để nhận được phương trình tỉ lượng (3) thì bạn cần phải:

- Nhân phương trình tỉ lượng (1) với 2 ( rGmo cũng được nhân 2,còn
K1 thì trở thành K12).
- Viết phương trình tỉ lượng (2) theo chiều ngược lại rồi nhân với 2
1
( rGmo nhân với -2 và K2 trở thành 2 ).
K2

5
2NH3 ( g ) + O2 ( g ) 2NO (g ) + 3H2O (g ) K12
2
1
2NO ( g ) + O2 ( g ) 2NO2 (g )
K22
2
7 K 
2NH3 ( g ) + ( g )O2 2NO2 (g ) + 3H2O (g ) K 3 =  1 
2  K2 

3) Tất cả các thành phần của hệ tạo thành một phase lỏng đồng thể,
do đó hoạt độ của mỗi thành phần đều bằng tỉ lệ mol i của nó trong
hỗn hợp. Ở mọi thời điểm và với mọi thành phần của hệ, ta đều có:
ester H O nester  nH O
Qr = 2
= 2

acid alcohol nacid  nalcohol

630 | Chuẩn bị kiến thức cho kì thi HSGQG Hóa học


Ở thời điểm ban đầu (t = 0), nester = nH O = 0  ( Qr ) = 0
i 2

(n )  (n ) 2 2

Khi cân bằng: ( Q ) = K =
ester eq H2O
eq
= 3 3 = 4.
r eq
(n )  (n ) acid eq alcohol eq
1 1

3 3
Do ( Qr )  K nên phản ứng diễn ra tự phát theo chiều thuận.
i

Chú ý rằng nước không đóng vai trò dung môi trong phản ứng này; do đó
cần phải xét đến hoạt độ tương đối của nó.

( a ( ) )  (a ( ) )
4) a) Hằng số cân bằng: K ( T ) =
ZnO s SO2 g
0 eq
3

(a ( ) )  (a ( ) ) ZnS s
eq
O2 g
2

eq

Các cấu tử rắn chỉ có trong phase tương ứng của chúng, do đó:

( aZnO(s) ) (
= 1 và aZnS(s) ) = 1  K0 ( T ) =
(a ( ) )
SO2 g
3

(a ( ) )
eq eq
2
O2 g
eq

b) Hoạt độ của một chất khí (lí tưởng) trong hỗn hợp khí là tỉ lệ áp suất
riêng phần của nó trong hỗn hợp với áp suất tham chiếu (áp suất
chuẩn) p0. Do đó, ta có:
pSO
2

p0 pSO 1

K0 ( T ) = 3
 K0 ( T ) =
pO
2
 p0( ) 2

 pO  2 2

 2 
 p0 
 
c) Trong một hỗn hợp khí, áp suất riêng phần của một chất khí là
nO
pi = i pT . Với cân bằng đang xét, pO = O  pT với O = 2 và
2
nT 2 2

631 | Chuẩn bị kiến thức cho kì thi HSGQG Hóa học


nSO
pSO = SO  pT với SO = . Thế các biểu thức này vào K0(T) ở trên, ta
2

2 2 2
nT
có được:
1
nSO 1
p  2
K0 ( T ) = 2
 (nT )   0 
2

 pT 
3
n2
O2

632 | Chuẩn bị kiến thức cho kì thi HSGQG Hóa học


Chủ đề 3: Thành phần cân bằng
Quá trình trùng chỉnh (reforming) methane được tiến hành bởi phản ứng
với hơi nước, tạo thành hydrogen và carbon monoxide, là những tác
nhân được sử dụng nhiều trong tổng hợp hữu cơ. Trong bài này, chúng
ta nghiên cứu về phản ứng trong phase khí:
CH4 (g + H2O ( g ) CO ( g ) + 3H2 (g )

a) Biểu diễn hằng số Kp của phản ứng theo các áp suất riêng phần.
b) Giả sử rằng hỗn hợp ban đầu có 1 mol methane với 1 mol hơi
nước. Gọi neq là số mol CO khi cân bằng. Hãy biểu diễn phần mol
của mỗi cấu tử trong hỗn hợp ở dạng hàm của neq.
c) Viết biểu thức cho Kp ở dạng hàm của neq và áp suất tổng.
d) Tiến hành phản ứng ở 800 oC dưới áp suất tổng ptot = 1 bar. Giá
trị thực nghiệm của neq = 0.913. Hãy tính giá trị của Kp.
Hướng dẫn

( )
3
pCO  pH
a) Kp =
2

pCH pH O
4 2

b) Lập bảng diễn tiến của phản ứng:


CH4(g) + H2O(g) CO(g) + 3H2(g)
Số mol
1 1 0 0
ban đầu
Số mol Tổng số mol
khi cân
bằng
1 – neq 1 – neq neq 3neq
n g
= 2 + 2neq
c) Áp suất riêng phần pi của một chất khí trong hỗn hợp khí là
pi = i ptot . Do đó, có thể biểu diễn Kp như sau:

( )  (p ) ( )  (p )
3 4 3
CO  H tot
CO  H 2
Kp = 2
= 2

(p )
tot
CH H O 2
CH H2O
4 2 tot 4

633 | Chuẩn bị kiến thức cho kì thi HSGQG Hóa học


3
neq  3n 
  eq 
 ng   ng  ( ptot )
2
4
27neq
 (ptot )  Kp =
2
nên: Kp = 
 1 − neq   1 − neq 
( ) ( )
2 2

  
1 − neq  n g
 n   n 
 g   g 

Do n g ( )
= 2 + 2neq = 2 1 + neq nên nhận được:

4
27neq
 (ptot )
2
Kp =
( ) ( )
2 2
4 1 − neq  1 − neq

d) Thay neq và ptot vào biểu thức trên, nhận được: Kp = 1.69  102 bar 2 .

634 | Chuẩn bị kiến thức cho kì thi HSGQG Hóa học


Chủ đề 4: Sự phụ thuộc của hằng số cân bằng vào
nhiệt độ
Nghiên cứu cân bằng:
1
CuBr2 ( s ) CuBr ( s ) + Br2 ( g )
2

Dibromien Br2 được xem như là chất khí lí tưởng. Enthalpy chuẩn của
phản ứng ở 298 K: rH0 = 47.35 kJmol−1. Đo áp suất dibromine khi cân
bằng và nhận được p1 = 6.71·10-3 bar ở T1 = 450 K.

a) Xác định giá trị hằng số cân bằng K01 ở nhiệt độ T1.
b) Cho biết ảnh hưởng của nhiệt độ (ở áp suất không đổi) vào cân bằng
đang xét.
c) Tính giá trị K02 ở nhiệt độ T2 = 550 K (có thể xác định giá trị gần đúng)
và áp suất tương đối p2 của dibromine.
d) Trong một bình hút chân không (rỗng) dung tích V = 2.0 L, được duy
trì ở 550 K, có 0.20 mol copper(II) bromide được đưa vào. Hãy tính
số mol các tiểu phân hóa học hiện diện khi cân bằng được thiết lập.
Biết hằng số khí lí tưởng R = 8.31 J·K-1·mol-1.

635 | Chuẩn bị kiến thức cho kì thi HSGQG Hóa học


Hướng dẫn
a) Biểu thức hằng số cân bằng tại nhiệt độ T1:
1

( a ( ) )  (a ( ) )
2
CuBr s Br2 g
0 eq eq
K =
1
( a( ( ) ) )CuBr2 s
eq

Các cấu tử rắn chỉ có trong phase tương ứng của chúng, do đó:

1
(p ( ) ) 1

( ) ( ) p  p 2
Br2 g
2 eq
K01 = aBr (g) mà aBr (g) = = 1  K01 =  1  = 8.19 10−2
2 eq 2 eq p0 p0  p0 
b) Nếu hệ tồn tại ở cân bằng tại nhiệt độ T1 thì khi tăng nhiệt độ đến giá
trị T2 thì hệ sẽ diễn tiến theo chiều thu nhiệt (hấp thụ nhiệt): nguyên lí
Le Chatelier được thỏa mãn. Vậy, sự tăng nhiệt độ sẽ thuận lợi về mặt
nhiệt động học cho phản ứng thuận.

c) Lấy tích phân biểu thức Van’t Hoff để tính K0 với điều kiện rằng  rH0
không phụ thuộc vào nhiệt độ trong khoảng 298 K đến 550 K (đây là
giả định rất quan trọng!). Ta có:
T
lnK02 rH0 T2 dT rH0  1  2
lnK01 dlnK = R T1 T2  lnK2 − lnK1 = R  − T 
0 0 0

T 1

rH  1 1 
0
 lnK02 = lnK01 +  − 
R  T1 T2 

Thay các giá trị số vào, tìm được:

47.35  103  1 1 
(
lnK02 = ln 8.19  10−2 + ) 8.31
 −  = −0.200
 450 550 
 K02 = 8.19  10−1

636 | Chuẩn bị kiến thức cho kì thi HSGQG Hóa học


1
 p 2
K02 =  2  nên áp suất khi cân bằng của dibromine ở T2 là:
 p0 

( ) p
2
p2 = K02 0
= 6.70  10−1 bar.

d) Ở trạng thái ban đầu, ( Qr ) = 0 do đó hệ tự diễn tiến theo chiều thuận


i

(tạo thành CuBr và Br2).

637 | Chuẩn bị kiến thức cho kì thi HSGQG Hóa học


1
CuBr2(s) CuBr(s) + Br ( g )
2 2
Số mol ban đầu 0.20 0 0
xeq
Số mol khi cân bằng 0.20 – xeq xeq
2
Có thể sử dụng phương trình trạng thái khí lí tưởng để tính xeq:

xeq p2V
(n ) Br2
eq
= =
2 RT2

2p2 V 2  6.70  10−1  105  2  10−3


 xeq = = = 5.87  10−2 mol
RT2 8.31 550

Thành phần của hệ khi cân bằng:

nCuBr (s) = 1.41 10−1 mol


2

nCuBr(s) = 5.87  10−2 mol


nBr (g) = 2.93  10−2 mol
2

638 | Chuẩn bị kiến thức cho kì thi HSGQG Hóa học


Chương 9. Cân bằng acid-base
Chủ đề 1: pH và lực acid
1) Dung dịch (1) được điều chế bằng cách hòa tan 5 mmol một acid
mạnh vào 50 mL nước tinh khiết. Xác định pH của (1).
A. pH = 0. B. pH = 1. C. pH = 2. D. pH = 3.
2) Dung dịch (2) được điều chế bằng cách hòa tan 5 mmol một base
mạnh vào 50 mL nước tinh khiết. Xác định pH của (2).
A. pH = 5.5. B. pH = 12. C. pH = 13. D. pH = 14.
3) Tính pH dung dịch tạo thành khi trộn lẫn dung dịch (1) và (2).
A. pH = 1.3. B. pH = 6. C. pH = 7. D. pH = 8.
Trong dung dịch của acid yếu HA (tương ứng với base liên hợp A-), khi
A − 
  eq
biết hằng số phân li acid KA và tỉ lệ các nồng độ khi cân bằng
HA 
eq

thì pH của dung dịch có thể được tính theo phương trình Henderson–
 A −  
  eq 
Hasselbalch: pH = pKA + log 
 HA  
   eq 
4) Dung dịch (3) được điều chế bằng cách hòa tan 5 mmol một acid
yếu có pKa = 5.2 vào 50 mL nước tinh khiết. Xác định pH của (3).
A. pH = 1. B. pH = 2. C. pH = 2.7. D. pH = 3.1.
5) Dung dịch (4) được điều chế bằng cách hòa tan 5 mmol một base
yếu có pKa = 9.2 vào 50 mL nước tinh khiết. Xác định pH của (3).
A. pH = 11.1. B. pH = 11.3. C. pH = 11.9. D. pH = 13.
6) Tính pH dung dịch tạo thành khi trộn lẫn dung dịch (1) và (2).
A. pH = 6.8. B. pH = 7. C. pH = 7.2. D. pH = 7.4.

639 | Chuẩn bị kiến thức cho kì thi HSGQG Hóa học


Hướng dẫn
1) Hãy kí hiệu acid mạnh là HA và AH/A- là cặp acid-base liên hợp tương
ứng. Sự phân li của acid mạnh trong nước diễn ra hoàn toàn. Phương
trình phản ứng được viết như sau:

AH + H2O → A− + H3O+

Trong nước tinh khiết, với pH = 7, nồng độ ban đầu của ion hydronium
là: H3O  = 10 = 10 mol L - tương ứng với số mol ban đầu:
+ −pH −7 −1
i

nH O+ ,i = H3O+   V = 10−7  50  10−3 = 5  10−9 mol


3 i

Lượng chất này khá nhỏ so với số mol ion hydronium được tạo thành
bởi sự phân li của acid. Ta có thể vẽ bảng diễn tiến phản ứng5 như bên
dưới theo số mol các chất:
Chất AH H2O A- H3O+
Số mol ban đầu 5·10-3 dư 0 5·10-9
Số mol cuối 0 dư 5·10-3  5·10-3
(bỏ qua lượng H+ ban đầu)

nH O+ ,f 5  10−3
 +

Ở trạng thái cuối: H3O  = 3 = = 10−1 mol L−1
f V 50  10 −3

5
Giai đoạn then chốt trong việc giải các dạng bài tập hóa học như thế này là tạo một
bảng diễn tiến phản ứng. Tùy vào trường hợp, mà bảng này được thiết lập theo nồng
độ hoặc số mol.

Trường hợp 1: Nếu phản ứng diễn ra mà không có sự biến đổi về thể tích và nếu đã
biết nồng độ ban đầu của mỗi tiểu phân (chất phản ứng và sản phẩm) thì sẽ việc xử lí
trực tiếp với nồng độ sẽ nhanh hơn, bởi các thao tác tính toán thường liên quan về
nồng độ (ví dụ như tính pH).

Trường hợp 2: Nếu phản ứng diễn ra với sự thay đổi thể tích (ví dụ như tạo hỗn hợp
của hai dung dịch), hoặc nếu một trong các chất phản ứng không thuộc dung dịch
(sự hòa tan một chất rắn ion, một chất lỏng tinh khiết hoặc một chất khí, vào nước)
thì bắt buộc phải xử lí theo số mol. Ngoài ra, hãy lưu ý sử dụng thể tích tổng khi xác
định nồng độ cuối.

Bài tập này tương ứng với trường hợp thứ hai.

640 | Chuẩn bị kiến thức cho kì thi HSGQG Hóa học


pH của dung dịch là: pH = − log H3O  = − log 10 = 1
+
f
−1
( )
Chọn phương án B.
Bài tập này tương đối đơn giản, nhưng KEM vẫn trình bày chi tiết để hỗ
trợ cho các độc giả mới làm quen với kiến thức Hóa học chuyên. Nếu bạn
đã quen với các thao tác tính toán thì có thể giản lược quá trình lập bảng
diễn tiến.
2) Trước tiên, có thể nhận ra ngay là nếu bỏ qua sự biến đổi thể tích thì
việc thêm một base mạnh vào nước tinh khiết (pH ban đầu bằng 7) sẽ
gây ra hiệu ứng làm tăng pH (tăng độ kiềm). Vậy nên câu trả lời A (pH =
5.5) chắc chắn là phương án sai – có thể loại bỏ luôn.
Hãy kí hiệu base mạnh là A- và cặp acid/base liên hợp tương ứng là
AH/A-. Sự phân li của base mạnh trong nước diễn ra hoàn toàn. Phương
trình phản ứng được viết như sau:

A− + H2O → AH + HO−

Trong nước tinh khiết, với pH = 7, nồng độ ban đầu của ion hydronium
là: H3O  = 10 = 10 mol L - vậy nồng độ ban đầu của ion
+ −pH −7 −1
i

Kw 10−14
hydroxide là: OH−  = = = 10−7 mol L−1
i H3O  10
+ −7
 i
tương ứng với số mol ban đầu:

nOH− ,i = OH−   V = 10−7  50  10−3 = 5  10−9 mol


i

Lượng chất này khá nhỏ so với số mol ion hydroxide được tạo thành bởi
sự phân li của acid. Ta có thể vẽ bảng diễn tiến phản ứng như bên dưới
theo số mol các chất:
Chất A- H2O AH OH-
Số mol ban đầu 5·10 -3
dư 0 5·10-9
Số mol cuối 0 dư 5·10-3  5·10-3
n − 5  10−3
Ở trạng thái cuối: OH−  = OH ,f = = 10−1 mol−1 L−1
f V 50  10 −3

641 | Chuẩn bị kiến thức cho kì thi HSGQG Hóa học


Từ đây:
Kw 10−14
H3O+  = −13 −1
  f OH−  = 10−1 = 10 mol L
 f
Vậy pH của dung dịch là:

( )
pH = − log H3O+  = −log 10−13 = 13
f

Chọn phương án C.
3) Chúng ta trộn lẫn hai dung dịch có cùng thể tích V = 50 mL, dẫn đến
sự thay đổi về thể tích và các thao tác biến đổi qua lại nên dựa vào số
mol. Phản ứng acid-base chủ yếu diễn ra giữa acid mạnh nhất, H3O+,
với base mạnh nhất, HO-. Phương trình phản ứng trung hòa này được
viết như sau:

H3O+ + HO− 2H2O

Dung dịch (1) cung cấp 5·10-3 mol ion H3O+ và


Kw 10−14
HO−   V = =  50  10−3 = 5  10−15 mol ion HO−
  f,1 H3O+  10−1
  f,1

Dung dịch (2) cung cấp 5·10-3 mol ion HO- và


Kw
H3O+   V = = 10−13  50  10−3 = 5  10−15 mol ion H3O+
  f,2 HO− 
  f,2

Dễ thấy rằng lượng ion HO- từ (1) và H3O+ từ (2) quá nhỏ (so với lượng HO-
từ (2) và H3O+ từ (1)) – do đó, có thể loại bỏ những lượng chất này để đơn
giản hóa bài toán.
Ta lập bảng diễn tiến phản ứng như dưới đây:
Chất H3O+ HO- 2H2O
Số mol ban đầu 5·10 -3
5·10 -3

Số mol cuối X X dư
Trạng thái cuối là trạng thái cân bằng, theo đó tích số ion của nước:

642 | Chuẩn bị kiến thức cho kì thi HSGQG Hóa học


2
 x 
KW = H3O+   HO−  = 10−14    = 10−14
f f
 2V 
x
 = H3O+  = 10−14 = 10−7 mol L−1
2V f

+
f
( )
Vậy: pH = − log H3O  = − log 10 = 7 (phương án C).
−7

Hãy lưu ý rằng trong phản ứng giữa acid và base mạnh thì hỗn hợp cuối
có môi trường trung tính chỉ khi lượng chất (số mol) ban đầu của ion
hydronium và ion hydroxide bằng nhau.
4) Kí hiệu acid yếu là AH và cặp acid-base liên hợp tương ứng sẽ là
AH/A-. Phương trình phản ứng phân li acid được viết như sau:

AH + H2O A − + H3O+

Đây là một quá trình cân bằng thuận nghịch, với hằng số tương ứng là
hằng số acid KA của cặp AH/A-:
A −   H3O+ 
  eq  
KA = = 10−5.2 = 6.31 10−6
AH
eq

Chúng ta sẽ bỏ qua sự tự phân li của nước (tương tự các ý 1, 2):


nH O+  0
3

Sau đó, ta rút ra được bảng diễn tiến phản ứng theo số mol các chất:
Chất AH H2O A- H3O+
Số mol ban đầu 5·10-3 dư 0 0
Số mol cuối 5·10-3 – x dư x x

643 | Chuẩn bị kiến thức cho kì thi HSGQG Hóa học


Theo biểu thức KA:
2
x
 
V
KA =  −3 = 6.31 10−6
5  10 − x
V
x2
 KA  V = −3
5.10 − x
( )( )
= 6.31 10−6  50  10−3 = 3.15  10−7

Giải phương trình bậc hai theo x:


x2 + 3.15  10−7 x − 1.58  10−9 = 0

 x = 3.96  10−5 mol (đây là nghiệm dương duy nhất).

x2 3.96  10−5
Vậy: H3O+  = = = 7.91 10−4 mol L−1
eq V 50  10 −3

( )
 pH = − log H3O+  = −log 7.91 10−4 = 3.1 (phương án D).
eq

5) Kí hiệu base yếu là A- và cặp acid/base tương ứng là AH/A_. Phương


trình phản ứng phân li base được viết như sau:

A− + H2O AH + HO−

Biểu thức hằng số cân bằng của phản ứng này là:

AH  HO− 
AH eq
K=
eq 

A 

  eq
 eq
=
A   H3O 

  eq 
+
 eq
(
 H3O+   HO− 
eq eq
)
KW 10−14
K = = −9.2 = 10−4.8 = 1.585  10−5
KA 10

Ta bỏ qua quá trình tự phân li của nước ( nHO− ,i  0 ).

644 | Chuẩn bị kiến thức cho kì thi HSGQG Hóa học


Ta rút ra được bảng diễn tiến phản ứng theo số mol các chất:
Chất A- H2O AH HO-
Số mol ban đầu 5·10-3 dư 0 0
Số mol cuối 5·10-3 – x dư x x

Theo biểu thức K:


2
x
 
V
K =  −3 = 1.585  10−5
5  10 − x
V
x2
KV = −3
5  10 − x
( )( )
= 1.585  10−5  50  10−3 = 7.92  10−7

Giải phương trình bậc hai theo x:


x2 + 7.92  10−7 x − 3.96  10−9 = 0

 x = 6.26  10−5 mol (đây là nghiệm dương duy nhất).

x2 6.26  10−5
 −

Vậy: HO  = = = 1.25  10−3 mol L−1
V 50  10 −3

KW 10−14
 +

 H3O  = = = 7.99  10−12 mol L−1
HO 

1.25  10−4
  eq

( )
Ta có: pH = − log H3O+  = − log 7.99  10−12 = 11.1 (phương án A).
eq

Ta cũng có thể sử dụng phương trình Henderson–Hasselbalch để tính


nhanh giá trị pH:

 A −    nA− ,eq 
   eq 
pH = pKA + log = pKA + log  
 HA    nAH,eq 
   eq   
 5  10−3 − 6.26  10−5 
 pH = 9.2 + log   = 11.1
 6.26  10−5 

645 | Chuẩn bị kiến thức cho kì thi HSGQG Hóa học


6) Chúng ta trộn lẫn hai dung dịch có cùng thể tích V = 50 mL, dẫn đến
sự thay đổi về thể tích và các thao tác biến đổi qua lại nên dựa vào số
mol. Để thuận tiện, chúng ta sẽ dùng số chỉ bên dưới để phân biệt các
tiểu phân đến từ dung dịch nào và viết phương trình phản ứng khi trộn
lẫn như sau:

A(3)H + A(−4) A(−3) + A(4)H

Hằng số cân bằng tương ứng:

A(3)H + A(−4) A(−3) + A( 4)H

A −   A H   −   +   A H 
 (3)  eq  (4)  eq  A( 3)  eq  H3O  eq    (4)  eq 
K= =   
A H  A −   A H   A −   H O+  
 ( 3 )  eq  ( 4 )  eq   ( 3 )  eq    ( 4 )  eq  3  eq 
KA
( 3) 10−5.2
K = = −9.2 = 104 1
K 10
Giá trị hằng số cân bằng rất lớn nên có thể xem rằng phản ứng diễn ra
hoàn toàn. Theo các ý 4, 5 thì:

- Dung dịch (3) cung cấp 3.96·10-5 mol ion A( 3) và

5  10−3 − 3.96  10−5 = 4.96  10−3 mol acid A(3)H


- Dung dịch (4) cung cấp 6.26·10-5 mol A (4)H và

5  10−3 − 6.26  10−5 = 4.94  10−3 mol ion A(−4)

Ta lập được bảng diễn tiến phản ứng theo số mol các chất:
Chất A(3)H A(−4) A(−3) A(4)H

Số mol ban đầu 4.96·10-3 4.94·10-3 3.96·10-5 6.26·10-5


Số mol cuối  0  0 5·10-3 5·10-3

646 | Chuẩn bị kiến thức cho kì thi HSGQG Hóa học


2
 5  10−3   
2
 
 2V   5  10−3  5  10−3
Ta có: K = =   = 104
  = = 5  10−5 mol
  
2 2
   10
   2V 
 2V 
Sử dụng phương trình Henderson–Hasselbalch để tính giá trị pH theo

cặp acid-basel iên hợp A (3)H/A(3) :

 A −    nA− 
  (3)  eq   (3),eq 
pH = pKA + log   = pK A(3 + log 
( 3)
 A(3)H   nA(3)H,eq 
  eq   
 5  10−3 
 pH = 5.2 + log  −5 
= 5.2 + 2 = 7.2
 5  10 
Chọn phương án C.
Bạn cũng có thể thực hiện tính toán tương tự với cặp acid-base liên

hợp A (4)H/A(4) để kiểm chứng lại kết quả:

 A −    nA− 
  (4)  eq   (4),eq 
pH = pKA + log   = pKA(4) + log 
( 4)
 A(4)H   nA(4)H,eq 
  eq   
 5  10−5 
 pH = 9.2 + log  −3 
= 9.2 − 2 = 7.2
 5  10 

647 | Chuẩn bị kiến thức cho kì thi HSGQG Hóa học


Chủ đề 2: Định lượng acid yếu bởi base mạnh
Trong phản ứng định lượng acid một nấc yếu bởi base một nấc mạnh,
dung dịch được chuẩn độ là dung dịch acid, với nồng độ ban đầu cA và
thể tích ban đầu VA; dung dịch chuẩn độ là dung dịch base, với nồng độ
cB. Tiến trình phản ứng được đo bởi sự biến diễn của tỉ lệ phản ứng x,
số mol base thêm vào
được định nghĩa là x = số mol acid được chuẩn độ

1) Khi thể tích base thêm vào là VB thì nồng độ mới trong bình phản
ứng của acid là c’A và base là c’B, lần lượt là
VA + VB V +V
A. c' A = cA và c'B = cB A B
VA VB

VA VB
B. c' A = cA và c'B = cB
VA + VB VA + VB

VA V
C. c' A = cA và c'B = cB A
VB VB

VA V
D. c' A = cA và c'B = cB B
VB VA

2) Kí hiệu KA là hằng số acid của acid một nấc yếu. Khi x = 0, pH trong
bình phản ứng khi cân bằng là

pKA + pcA
A. pH = B. pH = pKA + pcA
2
1
C. pH = pcA D. pH = pcA
2

3) Cho biết acid yếu này có hằng số phân li KA 10−14 . Khi 0 < x < 1 thì
pH trong bình phản ứng khi cân bằng là

1  1   1 
A. pH = pKA + log   B. pH = pKA + log  
2  1− x   1− x 

648 | Chuẩn bị kiến thức cho kì thi HSGQG Hóa học


1  x   x 
C. pH = pKA + log   D. pH = pKA + log  
2  1− x   1− x 

4) Khi x = 1 thì pH trong bình phản ứng khi cân bằng là

pKA + pKW
A. pH = pKA B. pH =
2
pKA + pc'B pKA + pKW − pc'B
C. pH = D. pH =
2 2
5) Khi x > 1 thì pH trong bình phản ứng khi cân bằng là
 x −1   x 
A. pH = pKA + pc'B + log   B. pH = pKA + pc'B + log  
 x   x −1 

 x −1   x −1 
C. pH = pKW − pc'B + log   D. pH = pKW − pc'B + log  
 x   x 

6)
1
A. Ở điểm bán tương đương: pH = pK A
2

B. Ở điểm bán tương đương: pH = pKA

C. Ở điểm tương đương, đường cong chuẩn độ pH = f(x) đi qua một


cực tiểu.
D. Ở điểm tương đương, đường cong chuẩn độ pH = f(x) đi qua một
điểm uốn.

649 | Chuẩn bị kiến thức cho kì thi HSGQG Hóa học


Hướng dẫn
cA VA VA cBVA VA
1) c' A = = cA c'B = = cB
VA + VB VA + VB VA + VB VA + VB

Chọn phương án B.
2) Kí hiệu acid yếu là AH, cặp acid-base liên hợp tương ứng là AH/A-.
Khi chưa thêm base vào, acid phản ứng với nước theo phương trình
sau:

AH + H2O A − + H3O+

Hằng số cân bằng của phản ứng này chính là hằng số acid KA của cặp
acid-base liên hợp AH/A-:
A −   H3O+ 
  eq   eq
KA =
AH
eq

Đây là acid yếu, nên KA 1 và phản ứng gần như không diễn ra (acid chỉ
bị phân li rất ít), cho phép ta đưa ra các giả định gần đúng trong bảng
diễn tiến phản ứng được thiết lập theo nồng độ6 như sau:
Chất AH H2O A- H3O+
Nồng độ ban đầu cA dư 0 0
Nồng độ cân bằng cA dư  0  0
2

Theo đó: KA =  H3O+  =  = KA  cA
cA eq

Khi x = 0, pH trong bình phản ứng khi cân bằng là


 1
 1 1
pH = − log H3O+  = − log (K A  cA ) 2  = − log (K A  cA ) = ( − logK A − logcA )
eq
  2 2
pK + pcA
 pH = A
2

6
Trường hợp này thể tích không thay đổi nên có thể xét theo nồng độ. Các trường
hợp sau, khi thêm base vào thì thể tích sẽ thay đổi nên phải xét theo số mol.

650 | Chuẩn bị kiến thức cho kì thi HSGQG Hóa học


(Lưu ý về kí hiệu tổng thể: pX = −logX)
Chọn phương án A.
3) Khi 0 < x < 1 thì acid phản ứng với ion hydroxide từ base mạnh, theo
phương trình phản ứng chuẩn độ dưới đây:

AH + HO− A − + H2O

Hằng số cân bằng của phản ứng này là:

A −   A −   H O+    
  eq    eq  3  eq   1 
K= = 
AH  HO−   AH   H O+   HO−  
eq   eq  eq    3  eq   eq 
K
K = A
Kw

Đây là acid yếu nhưng có KA 10−14  K 1 và phản ứng có thể diễn ra


gần như hoàn toàn.
số mol base thêm vào 𝑐𝐵 𝑉𝐵
x= =
số mol acid được chuẩn độ 𝑐𝐴 𝑉𝐴
Nhưng 0 < x < 1 nên tác nhân giới hạn là base. Ta nhận được bảng diễn
tiến phản ứng được thiết lập theo số mol như sau:
Chất AH HO- A- H2O
Nồng độ ban đầu cAVA cBVB 0 dư
Nồng độ cân bằng cAVA - cBVB   0 cBVB dư
cBVB cBVB
K VA + VB cA VA x
K= A = = =
KW  cA VA − cBVB 
 
  HO−

 cBVB 
1 −  HO−
 ( 1 − x )  HO− 
 eq  c V  
eq
V + V  eq
 A B   A A 

K x  H3O 
+
eq
K = A =
K W ( 1 − x ) K w
 1− x 
 H3O+  = KA   
eq
 x 

651 | Chuẩn bị kiến thức cho kì thi HSGQG Hóa học


Khi 0 < x < 1 thì pH trong bình phản ứng khi cân bằng là
 1− x   x 
pH = − log H3O+  = − logK A − log   = pK A + log  
eq
 x   1− x 

Chọn phương án D

4) Khi x = 1 thì cBVB = cA VA

Phương trình phản ứng chuẩn độ vẫn giống như ở ý 3:

AH + HO− A− + H2O

A −  K
  eq
Hằng số cân bằng: K = = A 1
AH  HO 

Kw
eq   eq

Phản ứng vẫn diễn ra gần như hoàn toàn, nhưng lần này hai chất phản
ứng có tỉ lệ hợp thức và sẽ đồng thời phản ứng hết. Trong trường hợp
này, có thể lập bảng diễn tiến phản ứng theo số mol:
Chất AH HO- A- H2O
Số mol ban đầu cAVA cBVB = cAVA 0 dư
Số mol cân bằng   0  0 cAVA dư
cA VA
( ) =K
2
A −  VA + VB c' A c' A  H3O+ 
  eq eq
K= = = = A

AH  HO− 
( ) ( HO  ) Kw
2 2 2
  K
eq   eq − −
HO 
e
eq eq

K A K w
 H3O+  =
eq c' A

Khi x = 1 thì pH trong bình phản ứng khi cân bằng là

 1

  K  K  2
 1
pH = − log H3O  = − log    = ( − logK A − logK w + logc' A )
+ A w
eq c' A  2
 
pK + pKw − pc' A
 pH = A
2

652 | Chuẩn bị kiến thức cho kì thi HSGQG Hóa học


pKA + pKw − pc'B
Do: c' A = c'B  pH =
2
Chọn phương án D.
5) Khi x > 1 thì phản ứng vẫn diễn ra gần như hoàn toàn, nhưng lúc này
acid là tác nhân giới hjan. Bảng diễn tiến phản ứng được cho dưới đây:
Chất AH HO- A- H2O
Số mol ban đầu cAVA cBVB 0 dư
Số mol cân bằng   0 cBVB - cAVA cAVA dư
Biểu thức tích số ion của nước:
cBVB − cA VA
KW = H3O+   OH−  = H3O+  
eq eq eq VA + VB
cBVB  cA VA 
 Kw = H3O+    1− 
eq VA + VB  cBVB 
 1  x −1 
 Kw = H3O+   c'B   1 −  = H3O+   c'B   
eq
 x eq
 x 
Kw  x
 H3O+  =
eq c' x − 1
B ( )
Khi x > 1 thì pH trong bình phản ứng khi cân bằng là

( )
 K x   x 
pH = − log H3O+  = − log  w
 = − logK w + logc'B − log  
eq  c'B ( x − 1)   x −1 
 
x −1
 pH = pKW − pc'B + log
x
Chọn phương án C.
Để xác thực các kết quả ở ý 2 – 5, ta có thể vẽ đường cong pH = f(x) bằng
cách xét ví dụ: pKA = 4.8 và cA = cB = 0.1 mol·L-1. Bảng giá trị dưới đây được
xét ở một số điểm cụ thể:
x 0 0.5 1 2 
pH 2.9 4.8 8.75 12.5 13

653 | Chuẩn bị kiến thức cho kì thi HSGQG Hóa học


Ta thấy được dạng đặc trưng của đường cong chuẩn độ acid yếu bởi base
một nấc mạnh:

6) Ở điểm bán tương đương, tức x = 0.5, thì theo ý 3:


 x   0.5 
pH = pKA + log   = pK A + log   = pK A + log1  pH = pK A
 1− x   1 − 0.5 

Ở điểm tương đương, x = 1 và đường cong chuẩn độ pH = f(x) (được biểu


diễn như ý trước) đi qua một điểm uốn.
Vậy chọn hai phương án là B và D.
Chú ý không nhầm lẫn giữa trạng thái cân bằng và điểm tương đương.
- Mỗi điểm trên đường cong chuẩn độ đều là một trạng thái cân bằng.
Điều này có nghĩa là sau khi thêm các lượng nhỏ base mạnh và các
dung dịch được trộn lẫn làm một (đồng thể hóa) thì thành phần hỗn
hợp trong bình phản ứng không thay đổi nữa.
- Điểm tương đương là trạng thái cân bằng đặc biệt mà tại đó x = 1.

654 | Chuẩn bị kiến thức cho kì thi HSGQG Hóa học


Chủ đề 3: Phosphoric acid
1) Phosphoric acid, có công thức H3PO4, là acid ba nấc, được đặc
trưng bởi các giá trị pKA1 = 2.2, pKA2 = 7.2, pKA3 = 12.2.
A. H2PO4- là acid hai nấc. B. H2O là acid mạnh hơn PO42-.
C. HPO42- là chất lưỡng tính. D. PO43- là base mạnh.
2) Một cốc mỏ có chứa dung dịch phosphoric acid. Khi cân bằng, giá
trị pH của dung dịch là
 H PO  
A. pH = pKA1 + log   3 4  
 H2PO4−  
 

 HPO2−  
B. pH = pKA2 + log  
4 

 H2PO4−  
 

 HPO2−  
C. pH = pKA3 − log   4 

 PO4  
3−
   

pKA1 + pK A2 + pK A3  H PO  
D. pH = + log   3−  
3 4

3  PO4  
   

3) Khi pH = 5, dạng chiếm ưu thế trong dung dịch là


A. H3PO4 B. H2PO4- C. HPO42- D. PO43-
4) Khi pH = 7.2 thì
A. H2PO4- là dạng chiếm ưu thế trong dung dịch.
B. HPO42- là dạng chiếm ưu thế trong dung dịch.

C. H3PO4  = H2PO4  = HPO4  = PO4 


− 2− 3−

D. H2PO4  = HPO4  và H3PO4  = PO4 


− 2− 3−

655 | Chuẩn bị kiến thức cho kì thi HSGQG Hóa học


5) Khi pH = 8.2 thì
HPO24− 
A. H2PO  = 

4

10

B. H2PO4  = 10  HPO4 
− 2−

C. H2PO4  = 10  H3PO4 

D. H2PO4  = 10  H3PO4 
− 5

656 | Chuẩn bị kiến thức cho kì thi HSGQG Hóa học


Hướng dẫn
1) Có ba cặp acid-base tương ứng với phosphoric acid:
- Cặp H3PO4 /H2PO4− , pKA1 = 2.2
- Cặp H2PO4− /HPO24− ,pKA2 = 7.2
- Cặp HPO24− /PO43− ,pKA3 = 12.2

H2PO4− là acid hai nấc do nó có thể giải phóng hai proton để lần lượt tạo
thành HPO24− và PO34− . Do đó phương án A đúng.

Để đánh giá phương án B và D, ta cần hiểu qua mối liên hệ giữa pKA với
độ mạnh/yếu của một acid và base liên hợp.
Các acid có 0 < pKA < 14 được xem là “yếu”. Base liên hợp của nó cũng
yếu.
- H3O+, acid của cặp H3O+/H2O, có pKA = 0, được xem là “mạnh nhất
trong số các acid yếu”.
- HO-, base của cặp H2O/HO-, có pKA = 14, được xem là “mạnh nhất
trong số các base yếu”.
- pKA càng nhỏ thì acid yếu càng mạnh, và base liên hợp của nó càng
yếu. Ngược lại, pKA càng lớn thì acid yếu càng yếu, và base liên hợp
của nó càng mạnh.
Theo luận điểm thứ hai ở trên thì, H2O là acid yếu “yếu nhất”, yếu hơn cả
HPO24− - vậy phương án B sai. Ngoài ra, PO43-, base của cặp HPO24− /PO34−
với pKA < 14, là một base yếu – nên phương án D cũng sai.

Còn phương án C thì sao? HPO24− là acid của cặp HPO24− /PO34− và là base
của cặp H2PO24− /HPO24− - vậy nên tiểu phân này có tính lưỡng tính và C
chính xác.
Vậy chọn A và C.
2)
Hằng số acid của một cặp acid-base liên hợp

657 | Chuẩn bị kiến thức cho kì thi HSGQG Hóa học


Kí hiệu acid yếu là AH, và cặp acid-base liên hợp tương ứng là AH/A-.
AH phản ứng với nước theo phương trình: AH + H2O A − + H3O+

Hằng số cân bằng của phương trình này, gọi là hằng số acid KA, của cặp
A −   H3O+ 
  eq   eq
AH/A- bằng: K A = .
AH eq

Bằng cách lấy logarithm thập phân của hai vế rồi sắp xếp lại, ta sẽ nhận
 A −  
được biểu thức: pH = pK A + log     . Đây chính là phương trình
 AH 
 
Henderson–Hasselbalch đã đề cập ở phần trước.
 H PO−  
Với cặp H3PO4 /H2PO thì pH = pKA1 + log  
4

4
2
 (A sai)
 H3PO4  
 

 HPO2−  
thì pH = pKA2 + log  
4 

Với cặp H2PO /HPO
4
2−
4
 (B đúng)
 H2PO4−  
 

Với cặp HPO24− /PO34− thì

 PO3−    HPO2-  
pH = pKA3 + log   4 
 = pK A3 - log  
4 
 (C đúng)
 HPO4  
2−
 PO4  
3-
     

Phương án D sai. Vậy chọn hai phương án B và C.


 H PO2−  
3) Với cặp H3PO4 /H2PO : pH = 2.2 + log  
4 
2−
4
2

 H3PO4  
 

 H PO−  
Nếu pH  2.2: log  
4
-
2
  0  H PO   H PO−  - tức dạng
 H3PO4    3 4  2 4
 
acid chiếm ưu thế hơn dạng base.

658 | Chuẩn bị kiến thức cho kì thi HSGQG Hóa học


 H PO−  
- Nếu pH  2.2: log   2 4    0  H3PO4   H2PO4−  - tức dạng
 H3PO4    
 
base chiếm ưu thế hơn dạng acid.
Ta tiến hành lập luận tương tự với hai cặp còn lại, và từ đó có thể vẽ
được ba giản đồ chiếm ưu thế riêng biệt:

Dựa vào giản đồ này, chúng ta có thể xác định được dạng tồn tại chiếm
ưu thế ở mỗi khoảng pH nhất định. Bạn cũng có thể biểu diễn ở dạng
giản đồ tổng thể bằng cách xếp chồng các giản đồ với nhau:

Dễ thấy rằng ở pH = 5, tiểu phân chiếm ưu thế là H2PO4− (phương án B).

Cần lưu ý rằng với mỗi giá trị pH thì giản đồ chỉ cho biết dạng chiếm ưu
thế, tức là chiếm tỉ lệ lớn nhất – chứ không phải là dạng tồn tại duy nhất.
Tức là, ngoài dạng chiếm ưu thế thì cũng có các dạng khác, với tỉ lệ nhỏ
hơn.

659 | Chuẩn bị kiến thức cho kì thi HSGQG Hóa học


4) Ở pH = 7.2

- Với cặp H3PO4 /H2PO4− :

 H PO−    H PO−  
7.2 = 2.2 + log   2 4
  log  
2 4
 = 5 ( 1)
 H3PO4    H3PO4  
   

- Với cặp H2PO4− /HPO24− :

 HPO2−  
7.2 = 7.2 + log  
4 

  H PO-  = HPO2- 
 H2PO4    2 4   4  (2 )
   

Lúc này, trạng thái cân bằng tương ứng nằm ở biên giới giữa hai
vùng chiếm ưu thế, như đã chỉ ra bởi đường nét đứt ở giản đồ tổng
thể (xem ý 3). Đây là trường hợp đồng ưu thế (vậy các phương án A
và B đều sai).
- Với cặp HPO24− /PO34− :

 PO3−    HPO2−  
7.2 = 12.2 + log   4 
  log  
4 
 = 5 (3)
 HPO4  
2−
 PO4  
3−
     

Bằng cách so sánh (1), (2), (3), ta nhận được: H3PO4  = PO4  (phương
3-

án D đúng).
5) Khi pH = 8.2
- Với cặp H3PO4 /H2PO4− :

 H PO−    H PO−   H2PO4− 


8.2 = 2.2 + log   2 4
  log   2 4
 =6   = 106
 H3PO4    H3PO4   H3PO4 
   

 H2PO4−  = 106  H3PO4  (các phương án C và D đều sai)

- Với cặp H2PO4− /HPO24− :

660 | Chuẩn bị kiến thức cho kì thi HSGQG Hóa học


 HPO2−    H PO−   H2PO4− 
8.2 = 7.2 + log   4 
  log   2 4
 = −1    = 10−1
 H2PO4−    HPO24−   HPO24− 
     

HPO2- 
 H2PO  = 
4 
-
4
(phương án A đúng).
10

661 | Chuẩn bị kiến thức cho kì thi HSGQG Hóa học


Chủ đề 4: Sự diễn tiến của một hệ acid-base
Hỗn hợp của methanoic acid HCOOH, ion methanoate HCOO-, nitrous
acid HNO2 và ion nitrite NO2- có khả năng diễn ra phản ứng theo phương
trình như sau:
(1)
⎯⎯
→NO− + HCOOH cân bằng (E)
HNO2 + HCOO− ⎯
⎯ (2)
2

Cho biết:

- ( )
pKA HCOOH/HCOO− = pKA1 = 3.8

- pK (HNO /NO ) = pK
A 2

2 A2
= 3.2
- Tích số ion của nước ở 25 oC: KW = 10−14
1) Hằng số cân bằng của (E) được viết ở dạng
Ke
A. K = KA1 KA2 B. K =
K A1 K A2

K A2 K A1
C. K = D. K =
K A1 K A2

2) Tính giá trị số của K:


A. K = 0.25. B. K = 4.0. C. K = 12. D. K = 107.
3) Nồng độ ban đầu của các tiểu phân, trước khi xảy ra phản ứng,
trong hỗn hợp là
HNO2  = HCOO−  = 2  NO2−  = 2  HCOOH = 3.00  10−2 mol L−1
i  i  i i

Hãy tính thương số phản ứng ban đầu, Qr,i :

A. Qr,i = 0.25. B. Qr,i = 0.50. C. Qr,i = 1.0. D. Qr,i = 4.0.

4)
A. Hệ phản ứng sẽ diễn tiến theo chiều (1).
B. Hệ phản ứng sẽ diễn tiến theo chiều (2).
C. Phản ứng diễn ra hoàn toàn.

662 | Chuẩn bị kiến thức cho kì thi HSGQG Hóa học


D. Phản ứng không diễn ra.
5) Tính giá trị pH khi cân bằng
A. pH = 1.8. B. pH = 4.1. C. pH = 2.9. D. pH = 3.5.
6) Xét một trường hợp khác, khi đó ồng độ ban đầu của các tiểu phân,
trước khi xảy ra phản ứng, trong hỗn hợp là
HNO2  = HCOO−  = 0.1 NO2−  = 0.1 HCOOH = 3.00  10−2 mol L−1
i  i  i i

A. Hệ phản ứng sẽ diễn tiến theo chiều (1).


B. Hệ phản ứng sẽ diễn tiến theo chiều (2).
C. Phản ứng diễn ra hoàn toàn.
D. Phản ứng không diễn ra.

663 | Chuẩn bị kiến thức cho kì thi HSGQG Hóa học


Hướng dẫn
1) Phương trình phản ứng cân bằng:
(1)
⎯⎯
→NO− + HCOOH
HNO2 + HCOO− ⎯
⎯ 2
(2)

NO2−   HCOOH   −  +   
  eq   eq  NO2  eq  H3O  eq   HCOOH eq 
K= = 
HNO2   HCOO 
−  HNO2   
  H3O  eq  HCOO  
+ −
eq   eq  eq  eq 

K
 K = A2
KA1

Chọn phương án C.
KA2 10−3.2
2) Thay số vào: K = = = 100.6 = 4.0 - chọn phương án B.
KA1 10−3.8

3) Trước tiên, ta cần nhắc lại khái niệm về thương số phản ứng và mối
liên hệ với hằng số cân bằng:
▪ Thương số phản ứng Qr có công thức tính tương tự như hằng số cân
bằng K, nhưng có thể được tính ở mọi thời điểm khác (ngoài trạng
thái cân bằng như K), ví dụ như trạng thái ban đầu hoặc sau thời
gian t bất kì, …
▪ Chúng ta kí hiệu Qr,i cho giá trị thương số phản ứng ở thời điểm ban
đầu. Việc so sánh Qr,I với K giúp dự đoán chiều hướng diễn ra phản
ứng.
▪ Độ lớn của K cũng giúp dự đoán thành phần nào – các chất phản
ứng hay sản phẩm – sẽ chiếm ưu thế ở thời điểm cân bằng.
NO2−   HCOOH
 i  i
Theo đó: Qr,i =
HNO2   HCOO 

i  i

1.50  10−2  1.50  10−2


Thay các giá trị số vào, ta có: Qr,i = = 0.25 - chọn
3.00  10−2  3.00  10−2
phương án A.

664 | Chuẩn bị kiến thức cho kì thi HSGQG Hóa học


4)
Giá trị thương số phản ứng ban đầu và chiều hướng diễn tiến của một
hệ phản ứng:
- Nếu Qr,i  K: thì hệ diễn ra theo chiều thuận, tức là chiều tạo thành
các sản phẩm.
- Nếu Qr,i  K: thì hệ diễn ra theo chiều nghịch, tức là chiều tạo thành
các chất phản ứng.
- Nếu Qr,i = K: hệ không diễn tiến, tức là phản ứng không xảy ra.

Trong trường hợp này: Qr,i = 0.25 và K = 0.4  Qr,i  K - tức là phản ứng
diễn ra theo chiều (1). Vậy phương án A đúng, còn các phương án B và
D sai.
Mối liên hệ giữa độ lớn của K và thành phần của hệ khi cân bằng
- Nếu K 1 (thường là K > 103): sản phẩm chiếm đa số thành phần cân
bằng. Nếu hệ ban đầu chỉ có các chất phản ứng thì có thể xem rằng
phản ứng đã diễn ra hoàn toàn.
- Nếu K 1 (thường là K < 10-3): chất phản ứng chiếm đa số thành
phần cân bằng. Nếu hệ ban đầu chỉ có các chất phản ứng thì có thể
xem rằng phản ứng không diễn ra.
Trong trường hợp này, K không quá lớn (so với giá trị tham chiếu là 1),
nên phản ứng diễn ra nhưng không hoàn toàn – do đó phương án C cũng
sai.
5) Phản ứng diễn ra không đi kèm sự biến đổi thể tích, và nồng độ ban
đầu của mỗi tiểu phân trong dung dịch đều đã biết. Nên có thể xây
dựng bảng diễn tiến phản ứng theo nồng độ:
Chất HNO2 HCOO- NO2- HCOOH
Nồng độ
ban đầu 3.00·10-2 3.00·10-2 1.50·10-2 1.50·10-2
(mol·L )
-1

Nồng độ
3.00·10-2 - x 3.00·10-2 - x 1.50·10-2 + x 1.50·10-2 + x
cân bằng
Thế vào biểu thức K, ta có:

665 | Chuẩn bị kiến thức cho kì thi HSGQG Hóa học


( )
2
NO2−   HCOOH 1.50  10−2 + x
  eq   eq
K= = = 4.0
HNO2   HCOO−  ( )
2
−2
eq   eq 3.00  10 − x

Giải phương trình bậc hai theo x, chỉ có duy nhất một nghiệm dương
thỏa mãn là x = 1.50  10−2 mol L−1 - các giá trị nồng độ khi cân bằng là:

HNO2  = 1.50  10−2 mol L−1


HCOO−  = 1.50  10−2 mol L−1
 
NO2−  = 3.00  10−2mol L−1
 
HCOOH = 3.00  10−2mol L−1

Có thể tính được pH khi cân bằng dựa theo một trong các cặp acid-
base liên hợp trong hệ, ví dụ cặp HNO2/NO2-:

 HCOO−  
  eq   3.00  10−2 
pH = 3.2 + log  = 3.2 + log  −2 
= 3.5
 HCOOH   1.50  10 
  eq 

Chọn phương án D.
NO2−   HCOOH −1 −1
 i  i 3.00  10  3.00  10
6) Qr,i = = = 102  K
HNO2   HCOO  3.00  10  3.00  10
− −2 −2
i  i
Trong trường hợp này thì hệ phản ứng diễn ra theo chiều (2). Chọn
phương án B.

666 | Chuẩn bị kiến thức cho kì thi HSGQG Hóa học


Chương 10. Cân bằng tạo phức
Chủ đề 1: Các hằng số cân bằng
1) Cation kim loại M có khả năng liên kết với một hoặc nhiều phối tử
đơn càng L để tạo thành phức chất. Hằng số tạo thành phức chất
(MLn) tổng thể được viết như sau:
n
M  L  MLn 
eq eq eq
A. n = B. n =
MLn  M  L 
n
eq eq eq

n
MLn  MLn 
eq
D. n =
eq
C. n =
MLn−1   L  M  L 
eq eq eq eq

2)
A.  không có đơn vị.

B.  được biểu diễn theo đơn vị moln L−n .

C.  được biểu diễn theo đơn vị mol−n L−n .

D.  được biểu diễn theo đơn vị mol−1 L .

3) Hằng số phân li từng nấc của phức MLi được viết ở dạng

MLi   L  eq MLi 


eq
B. Kd,i =
eq
A. Kd,i = i
MLi+1  M  L 
eq eq eq

MLi−1   L  MLi 


eq eq eq
C. Kd = D. Kd =
MLi  MLi−1   L 
eq eq eq

4) Chúng ta kí hiệu Kf,I là hằng số tạo thành từng nấc của phức MLi:
i=n
i−1
A. n = Kf,i B. K f,i =
i =1 i

667 | Chuẩn bị kiến thức cho kì thi HSGQG Hóa học


i=n
C. pKd,i = logi−1 − logi D. logn = pKd,i
i =1

5) Kí hiệu pL = -log[L]. Hãy biểu diễn pL theo hằng số cân bằng:


 MLi  
  eq 
A. pL = Kf,i + log 
 MLi−1  
 eq 

 MLi−1   L  
  eq   eq 
B. pL = pKd,i + log 
 MLi  
 eq 

 MLi  
  eq 
C. pL = pKd,i + log 
 MLi−1  
 eq 

 MLi−1  
  eq 
D. pL = pKd,i + log 
 MLi  
 eq 

668 | Chuẩn bị kiến thức cho kì thi HSGQG Hóa học


Hướng dẫn
1) Như thường lệ, trước khi bắt đầu giải đáp câu hỏi, chúng ta hãy nhắc
lại một chút về các kiến thức liên quan đến cân bằng tạo phức:
▪ Phương trình phản ứng tạo thành tổng thể của phức MLn:

M + nL MLn

MLn 
eq
Tương ứng với hằng số tạo thành tổng thể: n = n
M  L 
eq eq

▪ Phương trình phản ứng tạo thành từng nấc của phức MLi:

MLi−1 + L MLi

MLi 
eq
Tương ứng với hằng số tạo thành từng nấc: K f,i =
MLi−1   L 
eq eq

▪ Phương trình phản ứng phân li từng nấc của phức MLi:

MLi MLi−1 + L

1 MLi−1  eq  L  eq
Tương ứng với hằng số phân li từng nấc: Kd,i = =
K f,i MLi 
eq

Trong trường hợp này, phương án chính xác là B.

669 | Chuẩn bị kiến thức cho kì thi HSGQG Hóa học


2)
Định luật tác dụng khối lượng (định luật Guldberg - Waage)
Xét phương trình phản ứng:
A + B C + D

Trong đó , , ,  là các hệ số tỉ lượng. Khi phản ứng đạt tới trạng


thái cân bằng, hằng số cân bằng được viết như sau:

(a )  (a )
 
C D
K=
(a ) (a )
 
A B

trong đó a kí hiệu hoạt độ của tiểu phân hóa học được xét đến. Cụ thể:
- Với dung môi (nước): a = 1
c
- Với chất tan: a = - trong đó c là nồng độ chất tan, tính theo đơn
c0
vị mol·L-1, còn c0 = 1 mol·L-1.
p
- Với chất khí: a = - trong đó p là áp suất riêng phần của chất khí,
p0
tính theo đơn vị bar, và p0 = 1 bar.
Biểu thức ở trên chính là biểu thức của định luật tác dụng khối lượng,
biểu diễn mối liên quan giữa hoạt độ của các chất phản ứng và của sản
phẩm phản ứng ở trạng thái cân bằng. Biểu thức này được áp dụng cho
mọi loại cân bằng (acid-base, tạo phức, tạo kết tủa, oxid hóa-khử hóa),
bất kể nó diễn ra trong phase lỏng (dung dịch) hoặc phase khí.

Nói một cách chính xác thì n phải được biểu diễn là:

MLn 
eq
MLn   ( c0 ) mol L−1 
n n+1
c0
n = = eq
= 
 M   L  
n

M
  
L 
 mol L 
−1
n+1

   eq      eq   
eq eq

 c0   c0 
  

670 | Chuẩn bị kiến thức cho kì thi HSGQG Hóa học


cách viết này không làm thay đổi giá trị số của n bởi c0 = 1 mol·L-1 nhưng
sẽ làm thay đổi đơn vị: n không có đơn vị (chọn A).

Hoạt độ là đại lượng không đơn vị, nên các hằng số cân bằng luôn không
có đơn vị.
3) Hằng số phân li từng nấc của phức MLi:
MLi−1   L 
eq eq
Kd,i = (chọn phương án C).
MLi 
eq

4) Theo ý 1:

MLn  MLi 
eq eq
n = và K f,i =
M  L 
n
MLi−1   L 
eq eq eq eq

 MLn    MLn−1    MLi    ML  


 eq     eq   eq     eq 
n =   
 MLn−1   L    MLn−2   L    MLi−1   L    M  L  
 eq eq   eq   eq eq   eq eq 

Kf,n Kf,n−1 Kf,i Kf,1


i=n
n = Kf,i
i =1

Phương án A đúng.
Bằng cách lấy logarithm thập phân cả hai vế của biểu thức trên, ta
nhận được:

 i=n  i=n i=n  1  i=n i=n


logn = log  Kf,i  = logKf,i = log   =  ( − logKd,i )  logn = pKd,i
 K  i =1
 i =1  i =1 i =1  d,i  i =1

Vậy phương án D cũng đúng.

671 | Chuẩn bị kiến thức cho kì thi HSGQG Hóa học


Mặt khác:

MLi   ML    i −1

eq   i  eq   M eq  L  eq 
Kf,i = = 
i   
MLi−1   L   M  L    MLi−1  eq 
eq eq
 eq eq   
i 1
i−1

i
 K f,i =
i−1

Phương án B sai.
Bằng cách lấy logarithm thập phân cả hai vế của biểu thức trên, ta
nhận được:

    1 
logKf,i = log  i   log  = logi − log i−1
  K 
 i −1   d,i 
 − logKd,i = pKd,i = logi − logi−1

Phương án C cũng sai.


MLi 
eq
5) Hằng số tạo thành từng nấc của phức MLi: K f,i =
MLi−1   L 
eq eq

Bằng cách lấy logarithm thập phân cả hai vế của biểu thức trên, ta
nhận được:

 MLi    ML  
logKf,i = log  eq  = log   i  eq  − log L 
 MLi−1   L    MLi−1     eq
pKd,i  eq eq   eq  −pL

 MLi    MLi−1  
  eq    eq 
 pKd,i = log  + pL  pL = pKd,i + log 
 MLi−1    MLi  
 eq   eq 

Chọn phương án D.

672 | Chuẩn bị kiến thức cho kì thi HSGQG Hóa học


Chủ đề 2: Phức amine của copper(II)
Ion Cu2+ tạo thành bốn phức chất với ammonia NH3. Chúng ta kí hiệu n

( )
2+
là hằng số tạo thành tổng thể của phức chất Cu NH3 n  . Cho biết:

log1 = 4.1 log2 = 7.6 log3 = 10.5 log4 = 12.6

1) Hãy tính các giá trị pKd khác nhau.


A. pKd,1 = 4.1; pKd,2 = 7.6; pKd,3 = 10.5; pKd,4 = 12.6

B. pKd,1 = 10−4.1; pKd,2 = 10−7.6 ; pKd,3 = 10−10.5 ; pKd,4 = 10−12.6

C. pKd,1 = 4.1; pKd,2 = 3.5; pKd,3 = 2.9; pKd,4 = 2.1

D. pKd,1 = 1; pKd,2 = 1.85; pKd,3 = 1.38; pKd,4 = 1.2

2) Chỉ ra giản đồ chiếm ưu thế tổng thể cho các tiểu phân chứa đồng

( )
2+
(Cu2+ và phức chất dạng Cu NH3 n  ) theo nồng độ NH3 (được xác
định theo pNH3):

3) Một dung dịch nước chứa ion Cu2+ và các phức chất của nó với
ammonia.

673 | Chuẩn bị kiến thức cho kì thi HSGQG Hóa học


( )
2+
A. Nồng độ ammonia càng đậm đặc thì nồng độ Cu NH3 4  càng
loãng.

( )
2+
B. Nồng độ ammonia càng đậm đặc thì nồng độ Cu NH3 4  càng
đậm đặc.

C. Khi NH3 eq  10 mol L thì ion Cu2+ là tiểu phân chứa đồng
−4 −1

chiếm ưu thế trong dung dịch.

D. Khi NH3 eq  10 mol L thì đồng chỉ tồn tại ở dạng Cu2+ trong
−4 −1

dung dịch.
4) Ion Ag+ tạo thành hai phức chất với ammonia. Cho biết:
pKd,1 = 3.3, pKd,2 = 3.9. Chỉ ra giản đồ chiếm ưu thế tổng thể cho các

( )
+
tiểu phân chứa bạc (Ag+ và phức chất dạng Ag NH3 n  ) theo nồng
độ NH3 (được xác định theo pNH3):

674 | Chuẩn bị kiến thức cho kì thi HSGQG Hóa học


Hướng dẫn
 Cu NH  2+ 
  ( 3 )   eq
1) Kf,1 = = 1  pKd,1 = − logKd,1 = logK f,1 = log1 = 4.1
Cu2+   NH3 
  eq   eq

Ngoài ra, với i  2 thì:

       i −1 
 Cu(NH3 )i     Cu(NH3 )i     Cu2+   NH3  
2+ 2+

  eq     eq
= 
eq eq
Kf,1 = 
   i
    
 Cu(NH3 )i−1    NH3  eq    eq    Cu (NH3 )i−1   
2+ 2 +
 Cu   NH3 
2+
 eq 
eq     eq 
i 1
i−1

i
 K f,i =
i−1

  
Lấy logarithm thập phân hai vế, ta nhận được: logKf,i = log  i 
 i−1 

 1 
log  = logi − logi−1  pKd,i = logi − logi−1
 K 
 d,i 
pKd,i

Thay số vào, ta tìm được:

pKd,2 = log2 − log1 = 7.6 − 4.1 = 3.5


pKd,3 = log3 − log2 = 10.5 − 7.6 = 2.9
pKd,4 = log4 − log3 = 12.6 − 10.5 = 2.1

Chọn phương án C.

675 | Chuẩn bị kiến thức cho kì thi HSGQG Hóa học


( )
2+
2) Hằng số tạo thành từng nấc của phức Cu NH3 i  là:

  
( )
2+

 Cu NH3 i 
Kf,i = 
  
 Cu(NH3 )i−1    NH3  eq
2+

Lấy logarithm thập phân hai vế, ta nhận được:

       
( ) ( )
2+ 2+

 Cu NH     Cu NH  
  3 i 
 eq
3 i 
logKf,i = log   = log    eq 
− log NH3 
       
  Cu(NH3 )i−1    NH3  eq    Cu(NH3 )i−1   
2+ 2+
pKd,i
   −pNH3
 eq   eq 

   
( )
2+

  Cu NH  
3 i −1  
  eq 
 pNH3 = pKd,i + log 
    
  Cu(NH3 )i   
2+

 eq 

( ) ( )
2+ 2+
Với cặp Cu NH3 i  / Cu NH3  :
i −1 

- Nếu pNH3 < pKd,i:


   
( )
2+

  Cu NH  
3 i −1  
  eq       Cu (NH )  
( )
2+ 2+
log   0  Cu NH
    3 i 
 eq  
3 i −1  
( )     eq
2+

  Cu NH 3 i  
  eq 

( ) ( )
2+ 2+
Vậy dạng Cu NH3 i  chiếm ưu thế hơn dạng Cu NH3  .
i −1 

( )
2+
- Nếu pNH3 > pKd,I thì ngược lại, dạng Cu NH3  chiếm ưu thế hơn
i −1 

( )
2+
dạng Cu NH3 i  .

676 | Chuẩn bị kiến thức cho kì thi HSGQG Hóa học


Chúng ta thực hiện cách lập luận này cho cả bốn cặp, từ đó có thể vẽ
ra các giản đồ chiếm ưu thế riêng biệt và có thể xác định được tiểu
phân chiếm ưu thế ở mỗi vùng pNH3:

Bạn cũng có thể biểu diễn ở dạng giản đồ tổng thể bằng cách xếp
chồng các giản đồ với nhau:

Chọn phương án D.
3) Nồng độ ammonia càng đậm đặc thì pNH3 càng thấp, và nồng độ
 Cu NH  2+  càng đậm đặc – phù hợp với giản đồ chiếm ưu thế đã
  ( 3 )  4 
thiết lập ở ý 2. Vậy phương án B đúng, còn A sai.
Có một lập luận hoàn toàn sai, nhưng đáng tiếc lại rất phổ biến trong
hóa học – đặc biệt là các vấn đề liên quan đến dung dịch – bắt nguồn
từ những hiểu lầm về lí thuyết cân bằng. Lập luận này cho rằng “theo
định luật bảo toàn vật chất, dung dịch ammonia càng đậm đặc (NH3 tự
do càng nhiều) thì càng ít ammonia tạo phức”. Thực tế diễn ra hoàn
toàn ngược lại, ví dụ, hãy thử phân tích thương số phản ứng Kf,1:

677 | Chuẩn bị kiến thức cho kì thi HSGQG Hóa học


 Cu NH 2+  
  ( 3 )   eq
Kf,1 =
Cu2+   NH3 
  eq   eq

Do Kf,1 không đổi, [NH3] ở mẫu số càng lớn thì để “bù trừ”7:
- Cu2+ ở mẫu số phải càng nhỏ.
 Cu NH  2+  ở tử số phải càng lớn.
  ( 3 )  
-

Khi NH3  eq  10 mol L  pNH3  4 và theo giản đồ chiếm ưu thế đã


−4 −1

thiết lập ở ý 2, ion Cu2+ không phải dạng chiếm ưu thế trong dung dịch.
Các phương án C và D đều sai8.
4) Tương tự như ý 2, sử dụng quan hệ:
   
( )
+

  Ag NH  
3 i −1  
   eq 
pNH3 = pKd,i + log
    
( )
+

   Ag NH
 eq 
3 i 

Ta sẽ nhận được hai giản đồ chiếm ưu thế như sau cho mỗi vùng pNH3:

7
Nói cách khác, nếu ta so sánh hai cân bằng khác nhau có cùng hằng số K thì cân
bằng nào có nồng độ các sản phẩm lớn nhất thì cũng có nồng độ các chất phản ứng
lớn nhất.
8
Thực ra phương án D có thể loại bỏ đầu tiên. Bởi như đã trình bày trước đây, trong
các cân bằng hóa học, sự tồn tại cả một tiểu phân duy nhất là không thể, mà đồng
thời sẽ có nhiều loại tiểu phân cùng tồn tại – dù có thể tỉ lệ rất nhỏ.

678 | Chuẩn bị kiến thức cho kì thi HSGQG Hóa học


Hai giản đồ chiếm ưu thế này chứa những thông tin thể hiện sự mâu
thuẫn. Theo đó, tiểu phân Ag (NH3 )  có hai vùng chiếm ưu thế riêng
+

biệt (được tô màu đậm) và bởi vậy nên nó không bền. Tiểu phân này bị
chuyển thành hai dạng khác, theo phương trình như sau:

2 Ag (NH3 )  Ag+ + Ag (NH3 ) 


+ +

 2

Vậy nên, Ag (NH3 )  không bao giờ là dạng chiếm ưu thế, và không
+

xuất hiện trong giản đồ chiếm ưu thế tổng thể. Các đường biên pNH3 =
3.3 và pNH3 = 3.9 được thay thế bởi một đường biên phân tách hai vùng

( )
+
chiếm ưu thế riêng biệt của Ag+ và Ag NH3 2  . Để xác định đường
biên này, cần thiết lập mối liên hệ trực tiếp giữa các nồng độ
Ag+  và  Ag (NH3 )   .
+

  eq   2 
 eq

Ta có:

  
  Ag+  eq 
pNH3 = pKd,1 + log 

 
  Ag NH  +  
 ( 1)
    ( 3 )   
  eq 

  
Ag (NH3 )   
+

      eq
pNH3 = pKd,2 + log 
  


(2 )
Ag (NH ) 
+

    3 2 
 eq 


Lấy tổng (1) + (2), ta có:

679 | Chuẩn bị kiến thức cho kì thi HSGQG Hóa học


 
 Ag+  
  eq
2pNH3 = pKd,1 + pKd,2 + log  
   
  Ag (NH3 )2   
+

 eq 

 
pKd,1 + pKd,2  Ag+  
1    eq 
 pNH3 = + log
2 2    
  Ag (NH3 )2   
+

 eq 

 
 Ag+  
1   eq
 pNH3 = 3.6 + log  
2    
  Ag (NH3 )2   
+

 eq 

Vậy đường biên mới được thiết lập tại pNH3 = 3.6. Vùng chiếm ưu thế
của Ag+, có nồng độ xuất hiện ở phần tử số trong biểu thức trên, được
đặt ở bên phải đường biên này. Giản đồ chiếm ưu thế tổng thể:

Chọn phương án C.

680 | Chuẩn bị kiến thức cho kì thi HSGQG Hóa học


Chủ đề 3: Cân bằng cạnh tranh giữa hai cation
1) Ion thiocyanate SCN- tạo với
ion Cu2+ ion phức Cu( SCN)  có hằng số tạo thành Kf,a = 50.
+
-

ion Fe3+ ion phức Fe ( SCN)  có hằng số tạo thành Kf,b = 103.
2+
-

A. Cu( SCN)  bền hơn Fe ( SCN)  .


+ 2+

B. Fe ( SCN)  bền hơn Cu( SCN)  .


2+ +

C. Ái lực của ion SCN- với Cu2+ lớn hơn với Fe3+.
D. Ái lực của ion SCN- với Fe3+ lớn hơn với Cu2+.
2) Trộn lẫn V1 = 50.0 mL dung dịch copper(II) sulfate Cu2+ + SO42- có
nồng độ c1 = 1.00·10-2 mol·L-1 với V2 = 50.0 mL dung dịch potassium
thiocynate K+ + SCN- có nồng độ c2 = 1.00·10-2 mol·L-1. Hãy tính giá
trị pSCN = −log SCN  khi cân bằng.

A. pSCN = 2.0. B. pSCN = 2.1.

C. pSCN = 2.4. D. pSCN = 3.3.

3) Thêm V3 = 100.00 mL dung dịch iron(II) sulfate Fe3+ + SO42- có nồng


độ c3 = 1.00·10-2 mol·L-1 vào dung dịch tạo thành ở ý 2. Diễn ra phản
ứng theo phương trình sau đây:

Cu( SCN)  + Fe3+ Fe ( SCN)  + Cu2+


+ 2+

   

Tính hằng số K của cân bằng này.


A. K = 0.05. B. K = 20.
C. K = 5  104. D. K = 2  10−5.

4) Xác định nồng độ ion Fe ( SCN)  khi cân bằng:


2+

A. Fe ( SCN)  = 4.3  10−4 mol L−1.


2+

681 | Chuẩn bị kiến thức cho kì thi HSGQG Hóa học


B. Fe ( SCN)  = 3.3  10−3 mol L−1.
2+

C. Fe ( SCN)  = 5.0  10−3 mol L−1.


2+

D. Fe ( SCN)  = 1.010−2 mol L−1.


2+

5) Xác định pSCN khi cân bằng:


A. pSCN = 4.0. B. pSCN = 5.0.

C. pSCN = 2.5. D. pSCN = 3.0.

682 | Chuẩn bị kiến thức cho kì thi HSGQG Hóa học


Hướng dẫn
1) Ion thiocyanate SCN- phản ứng:

Với ion Cu2+ tạo thành phức Cu( SCN)  theo phương trình:
+
-

Cu ( SCN) 
+
Cu2+ + SCN−  

 Cu SCN  + 
  ( ) 
Hằng số cân bằng: Kf,a = eq
= 50
Cu   SCN 
2+ −
  eq   eq

Với ion Fe3+ tạo thành phức Fe ( SCN) 


2+
- theo phương trình:

Fe ( SCN) 
2+
Fe3+ + SCN−  

 Fe SCN  2+ 
  ( ) 
Hằng số cân bằng: Kf,b = eq
= 103
Fe   SCN 
3+ −
  eq   eq

Với cùng nồng độ đầu SCN  và các nồng độ đầu Cu  = Fe 
− 2+ 3+
ini ini ini

   
( ) ( )
2+ +
thì với Kf,b  Kf,a   Fe SCN     Cu SCN   - do đó, có nhiều
  eq  eq
 
( )
+
Fe ( SCN) 
2+

 
được tạo thành hơn  Cu SCN   . Điều này nghĩa là ái
  eq
lực của ion SCN- với Fe3+ lớn hơn với Cu2+ và rằng Fe ( SCN) 
2+
bền hơn

Cu( SCN)  .
+

 

Các phương án B và D đúng.


2) Phương trình phản ứng được xét:

Cu ( SCN) 
+
Cu2+ + SCN−  

Tính các nồng độ ban đầu sau khi trộn và trước khi phản ứng:

683 | Chuẩn bị kiến thức cho kì thi HSGQG Hóa học


V1 50.0  10−3
c'1 = c1 = 1.00  10−2 
V1 + V2 50.0  10−3 + 50.0  10−3
= 5.00  10−3 mol L−1 và c'2 = c' 1

Thiết lập bảng diễn tiến phản ứng theo nồng độ như sau:
Chất Cu2+ SCN- [Cu(SCN)]+
Nồng độ ban
5.00·10-3 5.00·10-3 0
đầu, mol·L-1
Nồng độ cân
5.00·10-3 - x 5.00·10-3 - x x
bằng
Thế vào biểu thức Kf,a, ta có:

 Cu SCN +  
  ( )  
eq x
Kf,a = = = 50
Cu   SCN 
( )
2+ − 2
−3
  eq   eq 5.00  10 − x
 x2 − 3.00  10−2 x + 2.50  10−5 = 0
Giải phương trình bậc hai trên, tìm được nghiệm nguyên dương duy
−4 −1
nhất thỏa mãn là x = 8.58  10 mol L .

Do đó, các nồng độ khi cân bằng là:


Chất Cu2+ SCN- [Cu(SCN)]+
Nồng độ cân
4.14·10-3 4.14·10-3 8.58·10-4
bằng, mol·L-1
Cuối cùng, xác định được giá trị pSCN khi cân bằng:

(
pSCN = − log SCN−  = − log 4.14  10−3 = 2.4 )
Chọn phương án C.
3) Phản ứng diễn ra theo phương trình:

Cu( SCN)  + Fe3+ Fe ( SCN)  + Cu2+


+ 2+

   

Hằng số cân bằng:

684 | Chuẩn bị kiến thức cho kì thi HSGQG Hóa học


 Fe SCN  2+   Cu2+ 
  ( )   eq
eq
K=
 Cu SCN    Fe3+ 
  ( )   eq
+

eq

    
  Fe ( SCN)     Cu2+   SCN−  
2+

K= eq
  eq eq

 Fe   SCN     Cu SCN   
 eq     ( )  
3+ − +

  eq 
   eq 

Kf,b 1
Kf,a

Kf,b 103
K = = = 20
Kf,a 50

Chọn phương án B.
4) Hãy tính các nồng độ ban đầu mới, sau khi trộn và trước khi phản
ứng:
- Xét cân bằng đã đánh giá ở ý 2:

 Cu SCN  +  =  Cu SNCN  +   V1 + V2


  ( )    (
int
)  V + V + V
eq,2 1 2 3

8.58  10−4
= = 4.29  10−4 mol L−1
2
V1 + V2 4.14  10−3
Cu2+  = Cu2+  = = 2.07  10−3 mol L−1
 ini   eq,2 V + V + V 2
1 2 3

V3 −2 100.0  10
−3
- Fe3+  = c3 
 ini = 1.00  10  = 5.00  10−3 mol L−1
V +V +V
1 2 3 200.0  10 −3

K = 20 nên có thể giả sử rằng phản ứng diễn ra hoàn toàn.


Thiết lập bảng diễn tiến phản ứng theo nồng độ như sau:
Chất [Cu(SCN)]+ Fe3+ [Fe(SCN)]2+ Cu2+
Nồng độ
ban đầu, 4.29·10-4 5.00·10-3 0 2.07·10-3
mol·L-1

685 | Chuẩn bị kiến thức cho kì thi HSGQG Hóa học


Nồng độ
 rất nhỏ 4.57·10-3 4.29·10-4 2.50·10-3
cân bằng

 Fe SCN  2+   Cu2+ 


  ( )   eq
eq
Thế vào biểu thức hằng số cân bằng: K =
 Cu SCN    Fe3+ 
  ( )   eq
+

eq

 Fe SCN  2+   Cu2+ 


    ( )   eq
Do đó: Cu( SCN) 
+
eq
    eq = = 1.17  10−5 mol L−1
 3+
K  Fe  
eq

  Fe ( SCN)    4.3  10−4 mol L−1


2+

 eq
Chọn phương án A.
5)
 
 Cu2+    2.50  10−3 
  eq
pSCN = pKd,a + log   = log (50 ) + log   = 4.0
  Cu SCN  +  
   ( )    1.17  10−5 
 eq 

Hãy xác thực kết quả này bằng cách sử dụng biểu thức trên cho cặp
thứ hai:
 
 Fe3+    4.57  10−3 
  eq
pSCN = pKd,b + log 
  Fe SCN  2+  
 ( )
3
= log 10 + log   = 4.0
   ( )    4.29  10−4 
 eq 

Chọn phương án A.

686 | Chuẩn bị kiến thức cho kì thi HSGQG Hóa học


Chương 11. Cân bằng kết tủa
Chủ đề 1: Các hằng số cân bằng
1) Ion carbonate CO32- phản ứng được với ion Pb2+ tạo thành kết tủa
lead carbonate PbCO3, theo phương trình phản ứng:

Pb2+ + CO23− PbCO3  pKs,1 = 13.5

Hằng số cân bằng của phản ứng tạo kết tủa là


1
A. K = B. K = Pb2+   CO23− 
Pb   CO 
2+ 2− eq eq
  eq   eq 3

C. K = 3.16  1013 D. K = 10−13.5


2) Độ tan của lead carbonate trong nước tinh khiết là

Ks1
A. S = B. S = K2S1
2
C. S = KS1 D. S = 2KS1

3) Ion iodide I- phản ứng với ion Pb2+ tạo thành kết tủa lead iodie. Cho
biết quá trình này tương ứng với pKs2 = 8.2. Tích số tan của PbI2 là
2 2
A. Ks2 = Pb   I  B. Ks2 = 2 Pb   I 
2+ − 2+ −
eq eq eq eq

2 1
C. Ks2 = 4 Pb   I 
2+ −
D. Ks2 = 2
eq eq
4 Pb2+   I− 
eq eq

4) Độ tan của lead iodide trong nước tinh khiết là

Ks2
A. S = Ks2 B. S = 3
4

Ks2 Ks2
C. S = D. S = 3
2 2

687 | Chuẩn bị kiến thức cho kì thi HSGQG Hóa học


5) Cho các giá trị khối lượng mol nguyên tử:
M(I) = 127 g  mol−1 M (Pb ) = 207 g mol−1

Hãy tính khối lượng lead iodide cực đại có thể được hòa tan trong 1
L nước tinh khiết:
A. m = 1.16 g B. m = 0.68 g

C. m = 26 mg D. m = 540 mg

6) Ion phosphate PO43- phản ứng được với ion Pb2+ tạo thành kết tủa
lead phosphate, có độ tan trong nước là S. Biểu thức tính tích số
tan của lead phosphate là

A. Ks3 = S2 B. Ks3 = 6S2

C. Ks3 = 72S5 D. Ks3 = 108S5

688 | Chuẩn bị kiến thức cho kì thi HSGQG Hóa học


Hướng dẫn
1
1) K =
Pb   CO23− 
2+
  eq   eq

1 1
K= = −pK = 1013.5 = 3.16  1013
Ks1 10 s1

Chọn phương án A và C.
2) Cùng tìm hiểu qua một chút về dung dịch bão hòa – độ tan:
Một dung dịch được xem là bão hòa nếu nó chứa một chất rắn ion kém
tan CxAy tồn tại trong cân bằng với các ion Cp+ và Aq- theo phương trình
sau:
hoa tan
⎯⎯⎯
Cx Ay ⎯⎯
⎯→ xCp+ + yAq−
ket tua

Theo định luật bảo toàn điện tích thì: xp = yq

Hằng số cân bằng của phản ứng hòa tan được kí hiệu là Ks và gọi là tích
số tan:
x y
Ks = Cp+   Aq− 
eq eq

x y
Qr,t = Cp+    Aq−  là thương số phản ứng ở thời điểm t.
t t

- Nếu thương số phản ứng ban đầu Qr,ini  K thì chỉ có các ion Cp+ và
Aq- tồn tại trong dung dịch, và lượng kết quả quá nhỏ để được xem
là hiện diện. Hệ mất trạng thái cân bằng và dung dịch không bão
hòa.
- Nếu Qr,ini  K thì có kết tủa, cho đến khi hệ đạt tới trạng thái cân
bằng, tại đó Qr,eq = K thì dung dịch bão hòa.

Độ tan S của một chất rắn ion trong nước tinh khiết
Là số mol cực tiểu của chất rắn ion này phải đưa vào 1 L nước tinh khiết
để tạo thành dung dịch bão hòa.

689 | Chuẩn bị kiến thức cho kì thi HSGQG Hóa học


Xét cân bằng hòa tan lead carbonate:

PbCO3 Pb2+ + CO23−

Nếu đưa S mol PbCO3 vào 1 L nước tinh khiết, toàn bộ chất rắn sẽ
chuyển vào dung dịch. Trạng thái cuối cùng là trạng thái cân bằng bởi
dung dịch đạt tới giới hạn bão hòa: bất kì lượng chất rắn bổ sung nào
cũng sẽ tồn tại ở thể rắn. Ta có thể lập bảng diễn tiến phản ứng, thiết
lập theo nồng độ, như sau:
Chất PbCO3 Pb2+ CO32-
Nồng độ ban đầu chất rắn 0 0
Khi cân bằng 0 S S
Thế vào biểu thức Ks1, ta tính được:

Ks1 = Pb2+   CO23−  = S2  S = Ks1


eq eq

Chọn phương án C.
3) Xét cân bằng hòa tan lead iodide:

PbI2 Pb2+ + 2I−

Tích số tan:
2
Ks2 = Pb2+   I− 
eq eq

Chọn phương án A.
4) Tiến hành tương tự như ý 2:
Chất PbI2 Pb2+ I-
Nồng độ ban đầu chất rắn 0 0
Khi cân bằng 0 S 2S
K
Ks2 = Pb2+   I−  = S  ( 2S ) = 4S3  S = 3 s2
2 2

eq eq 4

Chọn phương án B.
5) MPbI = M(Pb ) + 2M(I) = 207 + 2  127 = 461 g mol−1
2

690 | Chuẩn bị kiến thức cho kì thi HSGQG Hóa học


Khối lượng lead iodide cực đại có thể được hòa tan trong 1 L nước tinh
K 10−8.2
khiết là: m = S MPbI = 3 s2 MPbI = 3  461 = 0.54 g = 540 mg
2
4 2
4

Chọn phương án D.
6) Cân bằng hòa tan lead phosphate:
Pb3 (PO4 ) 3Pb2+ + 2PO34−
2

3 2
Biểu thức tích số tan: Ks3 = Pb   PO4 
2+ 3−
eq eq

Bảng diễn tiến của phản ứng hòa tan tương ứng với giới hạn bão hòa:
Chất Pb3(PO4)2 Pb2+ PO43-
Nồng độ ban đầu chất rắn 0 0
Khi cân bằng 0 3S 2S
Thế vào biểu thức tích số tan, ta tìm được:

Ks3 = Pb2+   PO34−  = ( 3S )  (2S ) = 108S5


3 2 3 2

eq eq

Chọn phương án D.

691 | Chuẩn bị kiến thức cho kì thi HSGQG Hóa học


Chúng ta có thể tổng quát hóa biểu thức liên hệ giữa S với Ks cho chất
rắn ion bất kì có công thức CxAy. Phương trình của quá trình hòa tan
được viết như sau:
Cx A y xCp+ + yAq−

x y
Tích số tan: Ks = C   A 
p+ q−
eq eq

Bảng diễn tiến của phản ứng hòa tan tương ứng với giới hạn bão hòa:
Chất CxAy Cp+ Aq-
Nồng độ ban đầu chất rắn 0 0
Khi cân bằng 0 xS yS
Thế vào biểu thức tích số tan, ta tìm được:

K
Ks = Cp+   Aq−  = ( xS ) ( yS ) = x x y ySx +y  S = x +y x s y
x y x y

eq eq xy

692 | Chuẩn bị kiến thức cho kì thi HSGQG Hóa học


Chủ đề 2: Hợp chất tan nhiều nhất
1) Chúng ta nghiên cứu về độ tan trong nước tinh khiết của ba chất
rắn ion sau:
▪ Silver chloride AgCl: pKs1 = 9.7
▪ Silver chromate Ag2CrO4: pKs2 = 12
▪ Silver carbonate Ag2CO3: pKs3 = 11

A. AgCl tan nhiều hơn Ag2CrO4 bởi pKs1 < pKs2.


B. AgCl tan kém hơn Ag2CrO4 bởi pKs1 < pKs2.
C. Ag2CO3 tan nhiều hơn Ag2CrO4 bởi pKs3 < pKs2.
D. Ag2CO3 tan kém hơn Ag2CrO4 bởi pKs3 < pKs2.
2) Ta kí hiệu S1, S2 và S3 lần lượt là độ tan của silver chloride, silver
chromate và silver carbonate trong nước tinh khiết. Hãy sắp xếp
S1, S2, S3 theo trình tự giá trị tăng dần.

A. S1  S3  S2 B. S1  S2  S3

C. S2  S1  S3 D. S2  S3  S1

3) Tính S3:
A. S3 = 1.36  10−4 mol L−1 B. S3 = 1.71 10−4 mol L−1

C. S3 = 2.24  10−6 mol L−1 D. S3 = 10−11 mol L−1

4) Sodium chloride NaCl là chất rắn ion tan tốt trong nước. Khi thêm
sodium chloride rắn vào dung dịch bão hòa của silver chloride AgCl
thì
A. toàn bộ silver chloride kết tủa.
B. độ tan của AgCl không thay đổi.
C. độ tan của AgCl tăng lên.
D. độ tan của AgCl giảm xuống.

693 | Chuẩn bị kiến thức cho kì thi HSGQG Hóa học


5) Tính độ tan của AgCl ( S'1 ) trong dung dịch sodium chloride nồng
độ 5.00·10-2 mol·L-1.

A. S'1 = 0 B. S'1 = 7.1 10−7 mol L−1

C. S'1 = 1.4  10−5 mol L−1 D. S'1 = 4.0  10−9 mol L−1

694 | Chuẩn bị kiến thức cho kì thi HSGQG Hóa học


Hướng dẫn
1)
Cân bằng hòa tan Tích số tan
AgCl Ag+ + Cl− Ks1 = Ag+ eq  Cl− 
eq
2
Ag2CrO4 2Ag+ + CrO24− Ks2 = Ag+   CrO24− 
eq eq
2
Ag2CO3 2Ag+ + CO23− Ks3 = Ag+   CO23− 
eq eq

Silver chromate và silver carbonate, với cùng công thức kiểu C2A, có
biểu thức tích số tan tương tự nhau. Do đó, có thể so sánh trực tiếp
chúng. Chất nào có tích số tan cao hơn thì có pKs nhỏ hơn và do đó các
nồng độ ion cân bằng cao hơn – tức là tan nhiều hơn.
pKs3  pKs2  Ag2CO3 tan nhiều hơn Ag2CrO4.

Chọn phương án C.
Cần lưu ý rằng silver chloride có công thức kiểu CA – do đó, tích số tan
của nó không thể được so sánh với silver chromate hay silver
carbonate, vậy nên các phương án A và B đều sai.
2) Theo ý trước, Ag2CO3 tan nhiều hơn Ag2CrO4, do đó S2 < S3 và A sai.
Để xác định S1 trong mối liên hệ với S2 và S3, cần tính giá trị số của các
độ tan. Trước đây, chúng ta đã thiết lập được biểu thức tổng quát cho
độ tan của chất rắn ion có công thức CxAy:

Ks
S = x+y
xxyy

Với silver chloride AgCl:

x = y = 1  S1 = Ks1 = 10−9.7 = 1.41 10−5 mol L−1

Với silver chromate Ag2CrO4:

Ks2 3 10−12
x = 2 và y = 1  S2 = 3 = = 6.30  10−5 mol L−1
4 4

695 | Chuẩn bị kiến thức cho kì thi HSGQG Hóa học


S1  S2  S1  S2  S3  Chọn phương án B.

Ks
3) Nhắc lại biểu thức trên một lần nữa: S = x + y
xxyy

Với silver carbonate Ag2CO3:

Ks3 3 10−11
x = 2 và y = 1  S3 = 3 = = 1.36  10−4 mol L−1
4 4
() 1
⎯⎯
4) Phương trình cân bằng: AgCl ⎯ → Ag+ + Cl−
(2⎯
)

Hằng số cân bằng: Ks1 = Ag+   Cl−  = 10−9.7


eq eq

Khi thêm sodium chloride rắn vào dung dịch bão hòa silver chloride,
thì NaCl bị phân li hoàn toàn theo phương trình sau:
NaCl → Na+ + Cl−

Việc thêm muối vào làm tăng nồng độ [Cl-]eq và do Ks1 không đổi nên
[Ag+]eq giảm. Do đó, cân bằng dịch chuyển theo chiều tạo thành kết tủa
AgCl (chiều (2)). Vậy, độ tan AgCl giảm  Phương án D đúng.
5) Biểu thức tổng quát cho độ tan của chất rắn ion có công thức CxAy ở
trên chỉ hợp lệ với môi trường nước tinh khiết. Độ tan trong dung dịch
nào khác đều khác, như lập luận ở ý 4. Để tính giá trị độ tan mới, chỉ cần
đơn giản thiết lập bảng diễn tiến phản ứng. Trong trường hợp này, ta
thiết lập theo nồng độ:
Chất AgCl Ag+ Cl-
Nồng độ ban đầu chất rắn S1 S1 + 5.00·10-2
Khi cân bằng chất rắn S’1 S’1 + 5.00·10-2
Thế vào biểu thức tính tích số tan:

Ks1 = Ag+   Cl−  = 10−9.7 = 2.00  10−10


eq eq

( )
 Ks1 = S'1  S'1 + 5.00  10−2 = 2.00  10−10

696 | Chuẩn bị kiến thức cho kì thi HSGQG Hóa học


Từ ý trước, ta đã biết độ tan AgCl giảm, do đó S' 1  S1 . Tuy nhiên, theo
ý 2 thì S1 = 1.41 10−5 5.00  10−2  Ks1  5.00  10−2  S' 1  2.00  10−10

2.00  10−10
 S'1 = −2
= 4.0  10−9 mol L−1 - chọn phương án D.
5.00  10

(
Phép giả định này hoàn toàn hợp lí S'1  S1 )
5.00  10−2 và giúp ta không
phải mất thời gian giải phương trình bậc hai theo S’1.

697 | Chuẩn bị kiến thức cho kì thi HSGQG Hóa học


Chủ đề 3: Khối lượng kết tủa tạo thành
Trong bài này, chúng ta tìm hiểu về sự tạo thành kết tủa silver sulfate
Ag2SO4, pKs = 4.8. Cho biết giá trị khối lượng mol nguyên tử:
M(O) = 16 g mol−1 M ( S ) = 32 g mol−1 M ( Ag ) = 108 g mol−1

Thêm V2 = 1 mL dung dịch sodium sulfate 2Na+ + SO42- với nồng độ c2 =


10-1 mol·L-1 vào V1 = 100 mL dung dịch silver nitrate Ag+ + NO3- với nồng
độ c1 = 10-1 mol·L-1.
1) Hãy thiết lập biểu thức của thương số phản ứng ban đầu.

c21 c2V12V2
A. Qr,ini = c c2
B. Qr,ini =
(V + V )
1 2 3
1 2

c1c2VV c21 c2V12V2


C. Qr,ini = 1 2
D. Qr,ini = 4
(V + V ) (V + V )
2 3
1 2 1 2

2) Tính khối lượng silver sulfate tạo thành:


A. m = 31.2 mg B. m = 30.0 g
C. m = 23.4 g D. m = 0.
3) Thí nghiệm được lặp lại bằng cách tăng gấp đôi nồng độ dung dịch
silver nitrate. Hãy tính giá trị thương số phản ứng ban đầu.

A. Qr,ini = 4  10−3 B. Qr,ini = 1.96  10−4

C. Qr,ini = 3.88  10−5 D. Qr,ini = 1.55  10−4

4) Tính khối lượng silver sulfate tạo thành trong trường hợp này:
A. m = 0 B. m = 29 mg
C. m = 30 g D. m = 18.5 mg

698 | Chuẩn bị kiến thức cho kì thi HSGQG Hóa học


Hướng dẫn
1) Xét cân bằng kết tủa của silver sulfate:

2Ag+ + SO24− Ag2SO4


2
Biểu thức tích số tan: Ks = Ag   SO4 
+ 2−
eq eq

Biểu thức thương số phản ứng ban đầu liên hệ với Ks là:
2
Qr,ini = Ag+   SO24− 
ini ini

Khi xét đến sự pha loãng, thì các nồng độ đầu – sau khi trộn lẫn nhưng
trước khi phản ứng – là:
cV cV
Ag+  = 1 1 SO24−  = 2 2
 ini V + V  ini V + V
1 2 1 2

c21 c2V12V2
Thế vào biểu thức, ta nhận được: Qr,ini = - chọn phương án B.
(V + V )
3
1 2

c21 c2V12V2
2) Nhắc lại biểu thức ở ý 1: Qr,ini = và thay số vào:
(V + V )
3
1 2

(10 ) (10 )(100 10 )  (110 ) = 9.7110


2 2
−1 −1 −3 −3

Qr,ini = −6
< Ks = 10−4.8 = 1.58  10−5
(10110 )
3
−3

Vậy hệ chưa đạt tới cân bằng, m = 0. Chọn phương án D.


3) Hãy tính giá trị số mới của thương số phản ứng ban đầu:

(2 10 ) (10 )(100 10 )  (110 ) = 3.88 10


2 2
−1 −1 −3 −3
−5
Qr,ini = - phương án C.
(10110 )
3
−3

4) Qr,ini  Ks - vậy nên cân bằng đã được thiết lập và xảy ra quá trình kết
tủa theo phương trình phản ứng:

699 | Chuẩn bị kiến thức cho kì thi HSGQG Hóa học


2Ag+ + SO24− Ag2SO4

Bảng diễn tiến phản ứng, thiết lập theo số mol, được cho dưới đây:
Chất Ag+ SO42- Ag2SO4
Nồng độ ban đầu 2·10-2 10 -4
0
Khi cân bằng  2·10 −2
10−4 − nAg SO nAg SO
2 4 2 4

Thế vào biểu thức tích số tan, ta có:

(2 10 )  (10 − n ) = 1.5810


2
−2 −4
2 Ag2SO4
Ks = Ag   SO 
+ 2−
= −5

(10110 )
eq 4 eq 3
−3

 nAg SO = 5.92  10−5 mol


2 4

MAg SO = 312 g mol−1  mAg SO = 18.5 mg - chọn phương án D.


2 4 2 4

700 | Chuẩn bị kiến thức cho kì thi HSGQG Hóa học


Chủ đề 4: Sự kết tủa cạnh tranh
1) Đưa 1.5 g silver chloride, là chất rắn ion với hằng số tan pKs1 = 9.7,
vào cốc chứa V = 250 mL nước tinh khiết. Cho biết các giá trị khối
lượng mol nguyên tử: M(Cl) = 35.5 g mol−1; M( Ag ) = 108 g mol−1 .
Hãy xác định số mol Cl- khi cân bằng:
A. nCl = 1.41 10−5 mol

B. nCl = 3.53  10−6 mol

C. nCl = 2.00  10−10 mol



D. nCl = 4.99  10−11 mol

2) Thêm vào dung dịch trên 250 mL dung dịch potassium iodide K+ +
I- nồng độ c = 10-2 mol·L-1 thì xuất hiện kết tủa vàng silver iodide
AgI, là chất rắn ion với hằng số tan pKs2 = 16.2. Phương trình phản

ứng diễn ra là: AgCl + I AgI + Cl− . Tính hằng số cân bằng.
A. K = 3.16  10−7 B. K = 3.16  106
C. K = 1.26  10−26 D. K = 7.94  1025
3) Biết khối lượng mol nguyên tử của iodine là M(I) = 127 g mol−1. Hãy
xác định khối lượng kết tủa silver iodide tạo thành.
A. mAgI = 9.4 g B. mAgI = 1.5 g

C. mAgI = 2.35 g D. mAgI = 0.59 g

4) Xác định nồng độ cuối của ion Ag+:

A. Ag  = 2  10 mol B. Ag  = 7.1 10 mol


+ −8 + −6
fin fin

C. Ag  = 4  10 mol D. Ag  = 1.4  10 mol


+ −8 + −5
fin fin

5) Xác định nồng độ cuối của ion I-:

A. I  = 1.6  10 mol L B. I  = 7.9  10 mol L


− −9 −1 − −9 −1
fin fin

C. I  = 8.9  10 mol L D. I  = 7.9  10 mol L


− −12 −1 − −10 −1
fin fin

701 | Chuẩn bị kiến thức cho kì thi HSGQG Hóa học


Hướng dẫn
1) MAgCl = 143.5 g mol−1 - vậy số mol AgCl đưa vào cốc là

mAgCl,ini 1.5
nAgCl,ini = = = 1.045  10−2 mol
MAgI 143.5

Phương trình phản ứng hòa tan AgCl trong nước:

AgCl Ag+ + Cl−


Dưới đây là bảng diễn tiến phản ứng theo số mol:
Chất AgCl Ag+ Cl-
Nồng độ ban đầu 1.045·10-2 0 0
Khi cân bằng 1.045·10-2 – x x x
Thế vào biểu thức tích số tan, ta có:

x2
Ks1 = Ag+   Cl−  = = 10−9.7 = 2.00  10−10
(25010 )
eq eq 2
−3

Số mol Cl- khi cân bằng là nCl = x = 3.53  10−6 mol (phương án B). Ngoài

ra, số mol AgCl rắn khi cân bằng là:


nAgCl = 1.045  10−2 − 3.53  10−6  1.045  10−2 mol


2) AgCl + I AgI + Cl−
Hằng số cân bằng:

Cl−   +  −
  eq Ag  eq  Cl  eq Ks1 10−9.7
K= − = = = −16.2 = 106.5 = 3.16  106
I  Ag   I 
+ −
Ks2 10
 eq  eq   eq

 phương án B.
−2 −1
3) 250 mL dung dịch potassium iodide nồng độ c = 10 mol L chứa

nI− ,ini = cV = 10−2  250  10−3 = 2.5  10−3 mol ion I−

702 | Chuẩn bị kiến thức cho kì thi HSGQG Hóa học


Hãy lập một bảng liệt kê, số mol của mỗi tiểu phân hiện diện trong hỗn
hợp phản ứng ban đầu (ngoại trừ ion bù trừ điện tích, K+). Lưu ý đây
không phải bảng diễn tiến phản ứng.
Chất AgCl I- AgI Cl- Ag+
Số mol ban đầu 1.045·10-2 2.5·10-3 0 3.53·10-6 3.53·10-6
nAg+ ,ini
nI ,ini do đó, phản ứng chiếm ưu thế - từ quan điểm các lượng

chất tham gia – và sự hòa tan lại kết tủa AgCl để ưu tiên tạo kết tủa AgI
(ta gọi là “sự tạo kết tủa cạnh tranh), theo phương trình phản ứng sau:

AgCl + I− AgI + Cl−


Hằng số cân bằng, đã tính được ở ý trước, là K = 3.16  106 1 - vậy phản
ứng diễn ra gần như hoàn toàn (hiệu suất toàn lượng):
nAgI,eq = nCl− ,eq = 2.5  10−3 mol

MAgI = 235 g  mol−1  mAgI = 0.59 g. Chọn phương án D.

4) Theo ý 3: nCl −
,eq
= 2.5  10−3 mol

Khi xét rằng hỗn hợp được pha loãng 2 lần thì:
n − 2.5  10−3
Cl−  = Cl ,eq = = 5  10−3 mol L−1
  eq 2V 500  10−3
Xác định nồng độ cuối của ion Ag+:

Ks1 = Ag+   Cl−  = 10−9.7 = 2.00  10−10


eq eq

K 2.00  10−10
 Ag+  = −s1 = = 4  10−8 mol L−1
eq Cl  5  10 −3
  eq

Chọn phương án C.
5) Xác định nồng độ cuối của ion I-:

703 | Chuẩn bị kiến thức cho kì thi HSGQG Hóa học


Ks2 = Ag+   I−  = 10−16.2 = 6.31 10−17
eq eq

Ks2 6.31 10−17


 I−  = = = 1.6  10−9 mol L−1
eq Ag+  4  10−8
  eq

Hãy lập một bảng liệt kê, số mol của mỗi tiểu phân hiện diện trong hỗn
hợp phản ứng ban đầu (ngoại trừ ion bù trừ điện tích):
Chất AgCl I- AgI Cl- Ag+
Số mol ban đầu 7.95·10 7.9·10
-3 -10
2.5·10 2.5·10 2·10-8
-3 -3

Hãy sử dụng định luật bảo toàn khối lượng để kiêm tra lại:
- Với nguyên tố bạc:

nAgCl,fin + nAgI,fin + nAg+ ,fin = 7.95  10−3 + 2.5  10−3 + 2  10−8


= 1.045  10−2 mol = nAgCl,ini

- Với nguyên tố chlorine:


nAgCl,fin + nCl− ,fin = 7.95  10−3 + 2.5  10−3 = 1.045  10−2 mol = nAgCl,ini

- Với nguyên tố iodine:


nAgI,fin + nI− ,fin = 2.5  10−3 + 7.9  10−10 = 2.5  10−3 mol = nI− ,ini

Vậy sự bảo toàn khối lượng được thỏa mãn.

704 | Chuẩn bị kiến thức cho kì thi HSGQG Hóa học


Chương 12. Cân bằng oxid hóa-khử
Chủ đề 1: Phương trình Nernst
1) Thế oxid hóa-khử của cặp Fe2+/Fe được viết là
( ) ( )
A. E Fe2+ /Fe = Eo Fe2+ /Fe + 0.06log  Fe2+  
2

 
 1 
( ) (
B. E Fe2+ /Fe = Eo Fe2+ /Fe + 0.06log  )
 Fe2+  

 
( ) (
C. E Fe /Fe = E Fe /Fe + 0.03log Fe 
2+ o 2+
) 2+
( )
D. E (Fe 2+
/Fe) = E (Fe
o 2+
/Fe) + 0.03lng ( Fe  )
2+

2) Thế oxid hóa-khử của cặp ClO4− /Cl2 (g ) được viết là


 ClO−  2 
0.06
( ) (
A. E ClO4− /Cl2 (g ) = Eo ClO4− /Cl2 (g ) +
14
)
log   16
4 

 H3O  PCl 
+
  2 

 ClO−   H O+  16 
2
0.06
( ) (
B. E ClO4− /Cl2 ( g ) = Eo ClO4− /Cl2 (g ) +
14
)
log  
4

PCl
 3  

 
 2

 ClO−  2
0.06
( ) (
C. E ClO4− /Cl2 (g ) = Eo ClO4− /Cl2 (g ) +
14
)
log  
4 

 PCl2 
 
 ClO−  
2
0.06
( ) (
D. E ClO4− /Cl2 ( g ) = Eo ClO4− /Cl2 (g ) −
14
)
log  
4 
 − 0.069pH
 PCl2 
 
3) Thế oxid hóa-khử của cặp HgCl2(s)/Hg(l) được viết là
( ( ) ( )) ( ( ) ( ))
A. A. E HgCl2 s /Hg l = E0 HgCl2 s /Hg l − 0.06log Cl−  ( )
B. B. E (HgCl (s ) /Hg (l) ) = E (HgCl (s ) /Hg (l ) ) − 0.03log ( Cl  )
2
0
2

C. C. E (HgCl (s ) /Hg (l) ) = E (HgCl (s ) /Hg (l ) ) − 0.03log ( Cl  )


2
0
2

D. E (HgCl (s ) /Hg (l) ) = E (HgCl (s ) /Hg (l ) )


0
D. 2 2

705 | Chuẩn bị kiến thức cho kì thi HSGQG Hóa học


4) Ks là tích số tan của lead sulfate PbSO4. Khi cân bằng, thế oxid hóa-
khử của cặp PbSO4(s)/Pb được viết là
( ) ( )
A. E PbSO4 (s) /Pb = E0 Pb2+ /Pb − 0.03pKs + 0.03log Pb2+ (eq
)
B. E (PbSO (s) /Pb) = E (Pb
4
0 2+
/Pb ) + 0.03pK + 0.03log ( Pb
s
2+

eq )
( ) ( ) (
C. E PbSO4 (s) /Pb = E0 PbSO4 (s) /Pb − 0.03pKs + 0.03log Pb2+ 
eq
)
D. E (PbSO (s) /Pb ) = E (PbSO (s) /Pb ) + 0.03pK + 0.03log ( Pb  )
4
0
4 s
2+
eq

706 | Chuẩn bị kiến thức cho kì thi HSGQG Hóa học


Hướng dẫn
1) Thế oxid hóa-khử của một cặp oxid hóa-khử ở nhiệt độ T là:

RT  O 
E ( ox /red) = E0 ( ox /red) + ln  
nF  R 

Được gọi là phương trình Nernst.


- E và E0 được biểu diễn theo volt (V).
- E0(ox/red) là thế oxid hóa-khử chuẩn của cặp tiểu phân ở nhiệt
độ T.
- R là hằng số khí lí tưởng, R = 8.31 J·K-1·mol-1.
- T là nhiệt độ nhiệt động học (hay “nhiệt độ tuyệt đối”), biểu diễn
theo Kelvin (K).
- n là số electron trao đổi trong bán phản ứng điện hóa của cặp
tiểu phân.
- F là điện tích của 1 mol điện tích nguyên tố (“hằng số Faraday”):
F = NA  e = 6.02  1023  1.6  10−19 = 96500 C mol−1 = 1 Faraday
- O là kí hiệu tích các hoạt độ của tất cả các tiểu phân ở cùng phía
với chất oxid hóa (với số mũ bằng hệ số tỉ lượng); tương tự, R là
kí hiệu tích các hoạt độ (kèm số mũ) của các tiểu phân cùng phía
với chất khử.
- Hoạt độ của dung môi hoặc chất rắng được xem là bằng 1; của
chất khí bằng áp suất riêng phần (bar); còn của chất tan thì
bằng nồng độ.
Mối quan hệ giữa logarithm tự nhiên và logarithm thập phân cho phép
chúng ta viết lại biểu thức như sau:
RT O
E ( ox /red) = E0 ( ox /red) + ln10 log  
nF R

RT
Ở nhiệt độ 25 oC, tức là khoảng 298 K, thì: ln10  0.06 . Do đó, có
F
thể viết lại phương trình Nernst như sau:

0.06 O
E ( ox /red) = E0 ( ox /red) + log  
n R

Với cặp Fe2+/Fe thì bán phản ứng tương ứng là:

707 | Chuẩn bị kiến thức cho kì thi HSGQG Hóa học


Fe2+ + 2e Fe ( s )
0.06  aFe2+ 
( )
E Fe2+ /Fe = E0 Fe2+ /Fe + ( ) 2
log 
 a


 Fe 
( ) (
 E Fe2+ /Fe = E0 Fe2+ /Fe + 0.03log Fe2+  ) ( )
Chọn phương án C.
2) Bán phản ứng tương ứng:

2ClO4− + 14e− + 16H3O+ Cl2 ( g ) + 24H2O


 
( )( )
2 16
 
0.06  aClO−  aH O+ 
( ) (
E ClO4− /Cl2 ( g ) = E0 ClO4− /Cl2 (g ) + log ) 4 3
24 
14
 2
( )( )
 aCl (g)  aH O 
2

 =1 
 ClO−   H O+  16 
2
0.06
( ) (
 E ClO4− /Cl2 ( g ) = E0 ClO4− /Cl2 (g ) +
14
)
log  
4

PCl
 3  

 
 2

Chọn phương án B.
Chúng ta cũng có thể biểu diễn theo pH, bằng cách chuyển hóa
logarithm của một tích thành tổng các logarithm:

708 | Chuẩn bị kiến thức cho kì thi HSGQG Hóa học


2ClO4− + 14e− + 16H3O+ Cl2 ( g ) + 24H2O

 ClO−  2 
0.06 4   0.06
( ) (
E ClO4− /Cl2 ( g ) = E0 ClO4− /Cl2 (g ) + )log   log  H3O+  
16

 +
14  PCl2  14  
 
 ClO−  2 
( ) (
 E ClO4− /Cl2 ( g ) = E0 ClO4− /Cl2 (g ) +
0.06
14
)
log  
4   0.06  16

 PCl2 
 +
14
(
log H3O+  )
 
 2

0.06  ClO4  

( −
)
0 −
(
 E ClO4 /Cl2 ( g ) = E ClO4 /Cl2 (g ) +
14
)
log   − 0.069pH
 PCl2 
 
Phương án D không đúng.
3) Bán phản ứng tương ứng:

HgCl2 ( s ) + 2e− Hg (l) + 2Cl−


 
 =1 
 
 aHgCl (s) 
( ) ( )
E HgCl2 ( s ) /Hg (l) = E0 HgCl2 (s ) /Hg (l ) + 0.03log 

2




( )
 aCl−   aHg(l)  
2

  
  =1  
 
1 
( ) (
 E HgCl2 ( s ) /Hg (l ) = E HgCl2 (s ) /Hg (l ) + 0.03log 
0
)
2 
 Cl−  
  
( ) ( )
 E HgCl2 ( s ) /Hg (l ) = E0 HgCl2 (s ) /Hg (l ) − 0.03log  Cl−  
2

 
( ) ( ) (
 E HgCl2 ( s ) /Hg (l ) = E HgCl2 (s ) /Hg (l ) − 0.06log Cl 
0 −
)
Chọn phương án A.

709 | Chuẩn bị kiến thức cho kì thi HSGQG Hóa học


4) Bán phản ứng tương ứng là:

PbSO4 ( s ) + 2e Pb + SO24−
 =1

 
 aPbSO (s) 
( ) ( )
E PbSO4 ( s ) /Pb = E0 PbSO4 ( s ) /Pb + 0.03log 
 
4


  4
( )
  aPb   aSO2− 

  =1  
 
1
( ) (
 E PbSO4 ( s ) /Pb = E PbSO4 ( s ) /Pb + 0.03log
0
)
 SO2−  

  4  eq 
Để nhận được biểu thức thế oxid hóa-khử ở dạng hàm của pKs, ta có
thể biến đổi như sau:

 Pb2+  
  eq
( ) (
E PbSO4 ( s ) /Pb = E PbSO4 ( s ) /Pb + 0.03log
0
)

 Pb2+   SO2−  

  eq  4  eq 
 Pb2+  
  eq 
( ) (
 E PbSO4 ( s ) /Pb = E PbSO4 ( s ) /Pb + 0.03log 
0
)
 K 
s
 
( ) ( )
 E PbSO4 ( s ) /Pb = E0 PbSO4 ( s ) /Pb − 0.03logKs + 0.03log Pb2+ 
eq( )
 E (PbSO ( s ) /Pb ) = E (PbSO ( s ) /Pb ) + 0.03pK
4
0
4 s (
+ 0.03log Pb2+ 
eq
)
Chọn phương án D.

710 | Chuẩn bị kiến thức cho kì thi HSGQG Hóa học


Chủ đề 2: Thế oxid hóa-khử chuẩn
1) Biểu diễn thế oxid hóa-khử chuẩn của cặp Fe3+/Fe ở dạng hàm của
thế oxid hóa-khử chuẩn của các cặp Fe3+/Fe2+ và Fe2+/Fe:
(
E0 Fe3+ /Fe2+ + 2E0 Fe2+ /Fe) ( )
0
(
A. E Fe /Fe = 3+
) 3
0
2E Fe /Fe( 3+ 2+
) + E (Fe
0 2+
/Fe )
(
B. E0 Fe3+ /Fe = ) 3
0
(
E Fe /Fe 3+ 2+
) (
+ E0 Fe2+ /Fe )
(
C. E0 Fe3+ /Fe = ) 2
0
( 3+
D. E Fe /Fe = E Fe /Fe ) 0
( 3+ 2+
) − 2E (Fe 0 2+
/Fe )
2) Cho biết thế oxid hóa-khử chuẩn của cặp HOCl/Cl2(g) và Cl2(g)/Cl-:

( )
E0 HOCl/Cl2 (g) = 1.63 V; E0 Cl2 (g ) /Cl− = 1.36 V( )
Hãy tính thế oxid hóa-khử chuẩn của cặp HOCl/Cl-:
0
( −
)
A. E HOCl/Cl = 2.99 V

B. E (HOCl/Cl ) = 1.90 V
0 −

C. E (HOCl/Cl ) = 1.495 V
0 −

D. E (HOCl/Cl ) = 1.09 V
0 −

3) Biểu diễn thế oxid hóa-khử chuẩn của cặp H2O2/H2O ở dạng hàm
của thế oxid hóa-khử chuẩn của các cặp O2/H2O và O2/H2O2:
A. E0 (H2O2 /H2O) = −2E0 (O2 /H2O ) + E0 (O2 /H2O2 )
2E0 (O2 /H2O) + E0 (O2 /H2O2 )
B. E (H2O2 /H2O) =
0

3
C. E (H2O2 /H2O) = 2E (O2 /H2O ) − E0 (O2 /H2O2 )
0 0

E0 (O2 /H2O) + E0 (O2 /H2O2 )


D. E (H2O2 /H2O) =
0

2
4) Thế oxid hóa-khử chuẩn của các cặp In3+/In và In+/In là:

( )
E0 In3+ /In = −0.34 V; E0 In+ /In = −0.14 V ( )

711 | Chuẩn bị kiến thức cho kì thi HSGQG Hóa học


Hãy tính thế oxid hóa-khử chuẩn của cặp In3+/In+:
0
(
3+ +
)
A. E In /In = −0.04 V

B. E (In
0 3+
/In ) = −0.58 V
+

C. E (In
0 3+
/In ) = −0.44 V
+

D. E (In
0 3+
/In ) = −0.29 V
+

( )
2+
5) Biểu diễn thế oxid hóa-khử chuẩn của cặp Cu NH3 4  /Cu ở
dạng hàm của thế oxid hóa-khử chuẩn của cặp Cu2+/Cu và hằng số

( )
2+
tạo phức tổng thể 4 của phức Cu NH3 4  :

0
( ) 
( )
2+
A. E  Cu NH3 4  /Cu  = 2E Cu /Cu − 0.06log4
0 2+

 
0
( ) 
( )
2+
B. E  Cu NH3 4  /Cu  = 2E Cu /Cu + 0.06log4
0 2+

 
0
( )  0
( )
2+
C. E  Cu NH3 4  /Cu  = E Cu /Cu + 0.03log4
2+

 
0
( )  0
( )
2+
D. E  Cu NH3 4  /Cu  = E Cu /Cu − 0.03log4
2+

 

712 | Chuẩn bị kiến thức cho kì thi HSGQG Hóa học


Hướng dẫn
1) Bán phản ứng liên quan đến cặp oxid hóa-khử Fe3+/Fe2+ là:

Fe3+ + e Fe2+
Thế của cặp oxid hóa-khử Fe3+/Fe2+ được biểu diễn theo phương trình
Nernst:
 Fe3+  
(
E Fe /Fe 3+ 2+
) = E (Fe
0 3+ 2+
/Fe ) + 0.06log   2+  
 Fe  
 

Tương tự với cặp Fe2+/Fe:

Fe2+ + 2e Fe
( ) ( )
E Fe2+ /Fe = E0 Fe2+ /Fe + 0.03log Fe2+  ( )
và cặp Fe3+/Fe:

Fe3+ + 3e Fe
( ) ( )
E Fe3+ /Fe = E0 Fe3+ /Fe + 0.02log Fe3+  ( )
Thế oxid hóa-khử của một hệ khi cân bằng
Khi một hệ điện hóa (dung dịch hoặc pin) đạt cân bằng, nghĩa là khi
nồng độ của các tiểu phân khác nhau không còn biến đổi nữa, thì thế
oxid hóa-khử của tất cả các cặp oxid-hóa khử trong hệ đều bằng
nhau. Trường hợp cụ thể là khi pin bị ăn mòn: suất điện động của nó
bằng 0 và không còn cung cấp dòng điện được nữa.
Hãy dựa vào tính chất này để thiết lập mối liên hệ giữa thế oxid hóa-
khử chuẩn của ba cặp này. Khi cân bằng, ba thế oxid hóa-khử bằng
( 3+
) (
3+ 2+ 2+
nhau: E Fe /Fe = E Fe /Fe = E Fe /Fe ) ( )
3+
( 3+
) (
2+ 2+
Do đó, có thể viết là: 3E Fe /Fe = E Fe /Fe + 2E Fe /Fe ) ( )
Chúng ta hãy thay thế mỗi thế oxid hóa-khử bằng biểu thức đã được
thiết lập ở trên:

713 | Chuẩn bị kiến thức cho kì thi HSGQG Hóa học


( )
3E0 Fe3+ /Fe + 0.06log Fe3+ 
eq ( )
 Fe3+  
0
(
= E Fe /Fe 3+ 2+
) + 0.06log   2+   + 2E0 Fe2+ /Fe + 0.06log Fe2+ 
eq

 Fe   eq
( ) ( )
  eq 

(
 3E0 Fe3+ /Fe + 0.06log Fe3+ 
eq
) ( )
 Fe3+   Fe2+  
  eq   eq 
0
(
= E Fe /Fe + 0.06log 
3+

2+

Fe 2 +
) 
+ 2E0 Fe2+ /Fe ( )
   eq 
0 3+
(
0 3+ 2+
 3E Fe /Fe = E Fe /Fe + 2E Fe /Fe )0 2+
( ) ( )
(
E0 Fe3+ /Fe2+ + 2E0 Fe2+ /Fe ) ( )
0
(
 E Fe /Fe = 3+
) 3
Chọn phương án A.
2) Với cặp HOCl/Cl2 ( g ) : 2HOCl + 2e + 2H3O+ Cl2 ( g ) + 4H2O

 HOCl 2  H O+  2 
( )
E HOCl/Cl2 ( g ) = E HOCl/Cl2 (g ) + 0.03log 
 0
(
  3  
PCl  )
 
 2

Còn với cặp Cl2 ( g ) /Cl− : Cl2 ( g ) + 2e 2Cl−

 P 
(
E Cl2 ( g ) /Cl −
) ( 
= E Cl2 (g ) /Cl + 0.03log 
0 Cl2 
2 

)
 Cl−  
  

Và cặp HOCl/Cl− : HOCl + 2e + H3O+ Cl− + 2H2O

 HOCl  H O+  
(
E HOCl/Cl −
) 0
(
= E HOCl/Cl + 0.03log 

   3 
Cl− 

)
   

Khi cân bằng, thế của ba cặp này bằng nhau:

714 | Chuẩn bị kiến thức cho kì thi HSGQG Hóa học


( ) ( ) (
E HOCl/Cl− = E HOCl/Cl2 (g ) = E Cl2 (g ) /Cl− )
 2E (HOCl/Cl ) − E (HOCl/Cl (g ) ) − E (Cl (g ) /Cl ) = 0

2 2

Biến đổi tương tự như ý trước, ta thiết lập được mối quan hệ giữa các
giá trị thế oxid hóa-khử chuẩn là:

( ) (
2E0 HOCl/Cl− − E0 HOCl/Cl2 (g ) − E0 Cl2 (g ) /Cl− = 0 ) ( )
( ) (
E0 HOCl/Cl2 ( g ) + E0 Cl2 ( g ) /Cl− ) = 1.63 + 1.36 = 1.495 V
0
(
E HOCl/Cl −
)= 2 2
Chọn phương án C.
3) E0 (H2O2 /H2O) = 2E0 (O2 /H2O ) − E0 (O2 /H2O2 )

Chọn phương án C.
4)
( )
3 E0 In3+ /In − E0 In+ /In ( ) = 3  ( −0.34) − ( −0.14) = −0.44 V
0
(
E In /In 3+ +
)= 2 2
Chọn phương án C.
2+
5) Với cặp oxid hóa-khử Cu2+ /Cu: Cu + 2e Cu

( ) (
E Cu2+ /Cu = E0 Cu2+ /Cu + 0.03log Cu2+  ) ( )
( ) ( )
2+ 2+
Với cặp Cu NH3 4  /Cu: Cu NH3 4  + 2e Cu + 4NH3

  2+ 

      
Cu ( NH ) 
3 4  
E  Cu(NH3 )  /Cu  = E0  Cu (NH3 )  /Cu  + 0.03log   
2+ 2+

  4
   4
 
4

NH3 
 
 

( )
2+
Ngoài ra, phương trình phản ứng tạo thành phức chất Cu NH3 4  là:

715 | Chuẩn bị kiến thức cho kì thi HSGQG Hóa học


Cu(NH ) 
2+
Cu2+ + 4NH3
 3 4

  
( )
2+

 Cu NH3 4 
 eq
Hằng số tạo thành tổng thể: 4 = 4
Cu2+   NH3 
  eq   eq

Khi cân bằng, thế của hai cặp bằng nhau:

 
E  Cu(NH3 )  /Cu  = E Cu2+ /Cu ( )
2+

  4

Thay biểu thức tính thế của mỗi cặp vào:
 
(
E  Cu(NH3 )  /Cu  = E Cu2+ /Cu )
2+

  4

    
( )
2+

  Cu NH3 4 
   eq 
 E0  Cu(NH3 )  /Cu  + 0.03log 
2+

  4
  4

NH3 
 eq 
 
( ) (
= E0 Cu2+ /Cu + 0.03log Cu2+ 
eq
)
 
 E0  Cu(NH3 )  /Cu  + 0.03log4 = E0 Cu2+ /Cu . ( )
2+

  4

0
( )  0
( )
2+
Cuối cùng ta có: E  Cu NH3 4  /Cu  = E Cu /Cu − 0.03log4
2+

 
Chọn phương án D.

716 | Chuẩn bị kiến thức cho kì thi HSGQG Hóa học


Chủ đề 3: Pin chì-acid
Một pin được tạo ra ở điều kiện 25 oC với:
- nửa pin (1) gồm điện cực chì Pb nhúng vào dung dịch ion Pb2+ nồng
độ 0.1 M;
- nửa pin (2) gồm một điện cực thiếc Sn nhúng vào dung dịch ion Sn2+
nồng độ 0.1 M;
- cầu muối để duy trì sự dẫn điện giữa hai nửa pin.
Thế oxid hóa-khử chuẩn của các cặp Pb2+/Pb và Sn2+/Sn lần lượt là:
( ) (
E0 Pb2+ /Pb = −0.13 V; E0 Sn2+ /Sn = −0.14 V )
1)
A. Điện cực chì xảy ra sự oxid hóa.
B. Điện cực chì là cathode.
C. Thế oxid hóa-khử ban đầu của hai điện cực bằng các giá trị thế
oxid hóa-khử chuẩn.
D. Suất điện động ban đầu của pin bằng 0.
2)
A. Điện cực chì là cực âm của pin.
B. Trong quá trình pin hoạt động, dung dịch (1) trở nên tích điện
âm.
C. Trong quá trình pin hoạt động, dung dịch (1) trở nên tích điện
dương.
D. Trong cầu muối, anion dịch chuyển đến dung dịch (2).
3) Từ ý này, chúng ta sẽ xét pin như trên nhưng với nồng độ dung dịch
ion Pb2+ ban đầu là 0.01 M. Hãy tính thế oxid hóa-khử ban đầu của
hai nửa pin:
A. E1,ini = −0.16 V và E2,ini = −0.17 V
B. E1,ini = −0.19 V và E2,ini = −0.17 V
C. E1,ini = −0.10 V và E2,ini = −0.17 V
D. E1,ini = −0.07 V và E2,ini = −0.17 V
4) Dẫn ra biểu thức tính hằng số cân bằng K của phản ứng oxid hóa-
khử diễn ra trong pin:
(
2 E02 −E01 )
A. K = 10 0.06

717 | Chuẩn bị kiến thức cho kì thi HSGQG Hóa học


(
2 E01 −E02 )
B. K = 10 0.06

(
2 E2,ini −E1,ini )
C. K = 10 0.06

(
2 E1,ini −E2,ini )
D. K = 10 0.06
5) Hai dung dịch có cùng thể tích V = 100 mL. Hãy tính nồng độ các ion
Pb2+ và Sn2+ khi không còn dòng điện chạy qua pin.
A. Pb  = 0.35 mol L và Sn  = 0.75 mol L
2+ −1 2+ −1
fin fin

B. Pb  = 0.035 mol L và Sn  = 0.075 mol L


2+ −1 2+ −1
fin fin

C. Pb  = 0.075 mol L và Sn  = 0.035 mol L


2+ −1 2+ −1
fin fin

D. Pb  = 0.75 mol L và Sn  = 0.35 mol L


2+ −1 2+ −1
fin fin
6) Cho biết: Số Avogadro NA = 6.02·1023 mol-1; giá trị điện tích nguyên
tố e = 1.6·10-19 C. Dòng điện từ pin đi qua một điện trở. Hãy tính điện
lượng Q (biểu diễn theo Coulomb) dẫn qua mạch trong quá trình pin
hoạt động:
A. Q = 240 C.
B. Q = 2400 C.
C. Q = 480 C.
D. Q = 4800 C.

718 | Chuẩn bị kiến thức cho kì thi HSGQG Hóa học


Hướng dẫn
2+
1) Bán phản ứng của cặp Pb2+/Pb: Pb + 2e Pb

( ) ( )
và thế điện hóa: E1 = E Pb2+ /Pb = E0 Pb2+ /Pb + 0.03log Pb2+  ( )
2+
Tương tự với cặp Sn2+/Sn: Sn + 2e Sn

( ) ( )
E2 = E Sn2+ /Sn = E0 Sn2+ /Sn + 0.03log Sn2+  ( )
Ở đây, các dung dịch ban đầu có nồng độ 0.1 M nên thế oxid hóa-khử ban
đầu không bằng thế oxid hóa-khử chuẩn. Do đó ý C sai.
Các giá trị thế oxid hóa-khử ban đầu là:

( ) ( )
E1,ini = E0 Pb2+ /Pb + 0.03log Pb2+  = −0.13 + 0.03log 10−1 = −0.16 V
ini
( )
E2,ini = E ( Sn
0 2+
/Sn ) + 0.03log ( Sn 2+

ini) = −0.14 + 0.03log(10 ) = −0.17 V
−1

Điện cực chì có thế cao nhất, nên nó là cathode – xảy ra sự khử. Vậy
phương án B đúng còn A sai.
Suất điện động: EMF = E1,ini − E2,ini = −0.16 − ( −0.17 ) = 0.01 V - tuy yếu,
nhưng vẫn khác 0 và do đó phương án D sai.
2) Như đã xác định ở ý trước, điện cực chì là cathode – tức là dương cực
của pin và phương án A sai. Nửa pin (1) xảy ra sự khử:

Pb2+ + 2e Pb
Do đó, dung dịch (1) suy giảm lượng ion Pb2+. Tuy nhiên, bất kì dung dịch
điện li nào cũng trung hòa điện. Do đó, sự suy giảm lượng cation được
bù trừ ở cầu nối bởi hai chiều dịch chuyển ion ngược nhau: cation dịch
chuyển về dung dịch (1), anion dịch chuyển về dung dịch (2). Phương án
D đúng, còn các phương án B và C đều sai.
3) Các giá trị thế oxid hóa-khử ban đầu là:

719 | Chuẩn bị kiến thức cho kì thi HSGQG Hóa học


( ) (
E1,ini = E0 Pb2+ /Pb + 0.03log Pb2+  = −0.13 + 0.03log 10−2 = −0.19 V
ini
) ( )
E2,ini = E ( Sn
0 2+
/Sn ) + 0.03log ( Sn 2+

ini ) = −0.14 + 0.03log(10 ) = −0.17 V
−1

Chọn phương án B.
4) So với ý 1, sự phân cực của pin đã thay đổi: điện cực thiếc có thế cao
hơn nên là cathode và xảy ra sự khử:

Sn2+ + 2e Sn
Điện cực chì là anode, xảy ra sự oxid hóa:

Pb Pb2+ + 2e
2+
Phản ứng oxid hóa-khử diễn ra là: Sn + Pb Sn + Pb2+
Phản ứng diễn ra giữa chất oxid hóa (2) và chất khử (1) và có sự trao đổi
(
2 E02 -E01 )
2 electron, do đó: K = 10 0.06
(chọn phương án A).
Biểu thức B là của hằng số cân bằng của phản ứng diễn ra theo chiều
ngược lại. Biểu thức C và D, là các thương số phản ứng của chiều thuận
và chiều nghịch.
5) Phương trình phản ứng oxid hóa-khử đang xét:

Sn2+ + Pb Sn + Pb2+

Pb2+ 
  eq
Hằng số cân bằng: K =
Sn2+ 
  eq

Theo ý trước, giá trị số của K là:


(
2 E02 −E01 ) 2  −0.14−( −0.13)

K = 10 0.06
= 10 0.06
= 10−0.333 = 0.464
K không lớn hơn so với 103, cũng không nhỏ hơn 10-3 nên phản ứng này
có diễn ra nhưng cũng chẳng hoàn toàn. Hãy thiết lập bảng diễn tiến
phản ứng theo nồng độ như sau:

720 | Chuẩn bị kiến thức cho kì thi HSGQG Hóa học


Tiểu phân Sn2+ Pb Sn Pb2+
Nồng độ ban đầu, mol·L-1 10-1 dư dư 10-2
Nồng độ cân bằng, mol·L-1 10-1 – x dư dư 10-2 + x
10−2 + x
Thế vào biểu thức K: K = −1
10 + x
= 0.464  10−2 + x = 0.464 10−1 − x ( )
 x = 2.5  10−2 mol L−1

Vậy nồng độ cân bằng, khi pin không còn cung cấp dòng điện là:
Pb2+  = 10−2 + 2.5  10−2 = 0.035 mol L−1
  fin
Sn  = 10−1 − 2.5  10−2 = 0.075 mol L−1
2+
  fin

Chọn phương án B.
6) Trong quá trình pin hoạt động, nó tạo ra:
xV = 2.5  10−2  100.10−3 = 2.5  10−3 mol ion Pb2+

Sự tạo thành 1 ion Pb2+ đi kèm với sự giải phóng 2 electron, tổng điện
lượng tạo thành là: Q = 2x  V NA  e
−3 23 −19
Thay số vào: Q = 2  2.5  10  6.02  10  1.6  10 = 480 C

721 | Chuẩn bị kiến thức cho kì thi HSGQG Hóa học


Chủ đề 4: Ảnh hưởng của acid
1) Biểu diễn thế oxid hóa-khử chuẩn của cặp MnO4-/Mn2+ ở dạng hàm
của các thế oxid hóa-khử chuẩn của các cặp MnO4-/MnO2 và
MnO2/Mn2+.
( )
2E0 MnO4− /MnO2 + 3E0 MnO2 /Mn2+ ( )
0
(
A. E MnO /Mn −
4
2+
)= 5
( )
E MnO /MnO2 + E0 MnO2 /Mn2+
0 −
( )
(
B. E0 MnO4− /Mn2+ = ) 4

2
(
5E MnO /MnO2 − 2E0 MnO2 /Mn2+
0 −
) ( )
0
(
C. E MnO /Mn −
4
2+
)= 3
4

(
3E MnO /MnO2 + 2E0 MnO2 /Mn2+
0 −
) ( )
(
D. E0 MnO4− /Mn2+ = ) 4

5
2) Trộn hai dung dịch có cùng thể tích V = 50.0 mL và cùng nồng độ c
= 0.100 mol·L-1 – một của manganese sulfate Mn2+ + SO42-, một của
potassium permanganate K+ + MnO4-. Phản ứng oxid hóa-khử diễn
ra như sau: 2MnO4− + 3Mn2+ + 6H2O 5MnO2 + 4H3O+ . Cho biết thế
oxid hóa-khử chuẩn của các cặp MnO4-/MnO2 và MnO2/Mn2+ lần lượt
là ( )
E0 MnO4− /MnO2 = 1.70 V; E0 MnO2 /Mn2+ = 1.23 V. Trong( ) quá
trình phản ứng, có sự trao đổi
A. 4 electron.
B. 5 electron.
C. 6 electron.
D. 8 electron.
3) Tính hằng số cân bằng K của phản ứng này:
A. K = 1047.
B. K = 106.
C. K = 1039.
D. K = 1031.
4) Khi hỗn hợp phản ứng đạt cân bằng, giả sử rằng pH = 1, hãy tính
nồng độ các ion MnO4- và Mn2+:
A. MnO4−  = 1.67  10−2 mol L−1; Mn2+  = 7.11 10−14 mol L−1
eq

B. MnO  = 1.67  10−2 mol L−1; Mn2+  = 1.53  10−16 mol L−1

4 eq

722 | Chuẩn bị kiến thức cho kì thi HSGQG Hóa học


C. MnO4−  = 3.33  10−2 mol L−1; Mn2+  = 9.65  10−17 mol L−1
eq

D. MnO4−  = 1.67  10−2 mol L−1; Mn2+  = 4.48  10−14 mol L−1
eq

5) Tính thế oxid hóa-khử của hỗn hợp khi cân bằng:
A. Eeq = 1.23 V.
B. Eeq = 1.58 V.
C. Eeq = 1.50 V.
D. Eeq = 1.70 V.

723 | Chuẩn bị kiến thức cho kì thi HSGQG Hóa học


Hướng dẫn
1) Bán phản ứng của cặp MnO4-/MnO2:

MnO4− + 3e + 4H3O+ MnO2 + 6H2O

Thế oxid hóa-khử của cặp MnO4-/MnO2 là:

( ) ( )
E MnO4− /MnO2 = E0 MnO4− /MnO2 + 0.02log  MnO4−   H3O+  
4

 

Bán phản ứng của cặp MnO2/Mn2+:

MnO2 + 2e + 4H3O+ Mn2+ + 6H2O

Thế oxid hóa-khử:


 H O+  4 
(
E MnO2 /Mn 2+
) = E (MnO /Mn )
0
2
2+
+ 0.03log   2+ 
3

 Mn  
 

Bán phản ứng của cặp MnO4-/Mn2+ là:

MnO4− + 5e + 8H3O+ Mn2+ + 12H2O

Thế oxid hóa-khử:


 MnO−   H O+  8 
( )
E MnO4− /Mn2+ = E0 MnO4− /Mn2+( ) + 0.012log  

4  3

Mn 2+

 

   
 

Khi cân bằng, thế của ba cặp bằng nhau:

( ) (
E MnO4− /Mn2+ = E MnO4− /MnO2 = E MnO2 /Mn2+ ) ( )
(
− 2+ −
) ( 2+
Ta có: 5E MnO4 /Mn − 3E MnO4 /MnO2 − 2E MnO2 /Mn = 0 ) ( )
( ) (
 5E0 MnO4− /Mn2+ − 3E0 MnO4− /MnO2 − 2E0 MnO2 /Mn2+ = 0 ) ( )
( )
3E0 MnO4− /MnO2 + 2E0 MnO2 /Mn2+ ( )
(
Vậy nên: E0 MnO4− /Mn2+ = ) 5

724 | Chuẩn bị kiến thức cho kì thi HSGQG Hóa học


Chọn phương án D.
2) Phản ứng được xét liên quan đến các cặp oxid hóa-khử MnO4-/MnO2
và MnO2/Mn2+ với các bán phản ứng lần lượt là:

MnO4− + 3e + 4H3O+ MnO2 + 6H2O (1)

và MnO2 + 2e + 4H3O+ Mn2+ + 6H2O (2)

Phương trình cân bằng:

2MnO4− + 3Mn2+ + 6H2O 5MnO2 + 4H3O+

Phản ứng có sự trao đổi của 6 electron. Phương án C.


3) Phương trình cân bằng:

2MnO4− + 3Mn2+ + 6H2O 5MnO2 + 4H3O+

liên quan đến sự trao đổi 6 electron và có hằng số cân bằng:


( ) ( )
6 E0 MnO4− /MnO2 −E0 MnO2 /Mn2+ 
  6(1.70−1.23)

K = 10 0.06
= 10 0.06
= 1047
Phương án A.
4) Giá trị K = 1047 rất cao, gợi ý rằng phản ứng diễn ra toàn lượng (hoàn
toàn). Các dung dịch gốc có cùng thể tích V, nồng độ ban đầu mới của
các ion MnO4- và Mn2+ (sau khi trộn và trước khi phản ứng) bị chia đôi:

MnO4−  = Mn2+  = 5.00  10−2 mol L−1


 ini  ini
Phương trình phản ứng: 2MnO4− + 3Mn2+ + 6H2O = 5MnO2 + 4H3O+

Ion Mn2+ tiêu hao nhanh hơn 1.5 lần so với ion MnO4-. Do đó, Mn2+ là tác
nhân giới hạn, nó sẽ phản ứng cho đến khi gần như biến mất hoàn toàn.
Do đó:
 2
MnO4−  =  1 −   5.00  10−2 = 1.67  10−2 mol L−1
  eq
 3

725 | Chuẩn bị kiến thức cho kì thi HSGQG Hóa học


Ngoài ra, hằng số cân bằng của phản ứng có biểu thức:
4
H3O+ 
K=  
2 3
MnO4−   Mn2+ 
  eq   eq

Do đó, nồng độ ion Mn2+ khi cân bằng là:


4
H3O+ 
  eq 10−4pH
Mn2+  = 3 =3
  eq 2 2
MnO4−  K MnO4−  K
  eq   eq

10−4
Thay số vào tìm được: Mn2+  = 3 = 1.53  10−16 mol L−1
  eq
(1.67 10 ) 10
2
−2 47

Phương án B.
Giá trị cực kì nhỏ của nồng độ [Mn2+]eq phù hợp với giả thuyết đã được đưa
ra về phản ứng toàn lượng. Việc kiểm chứng rất quan trọng!
5) Hãy nhắc lại các biểu thức thế oxid hóa-khử của ba cặp đã thiết lập
ở ý 1:
- Với cặp MnO4-/MnO2:

( ) ( )
E MnO4− /MnO2 = E0 MnO4− /MnO2 + 0.02log  MnO4−   H3O+  
4

 
( ) ( )
 E MnO4− /MnO2 = E0 MnO4− /MnO2 + 0.02log MnO4−  − 0.08pH ( )
- Với cặp MnO2/Mn2+:

 H O+  4 
( ) ( )
E MnO2 /Mn2+ = E0 MnO2 /Mn2+ + 0.03log   2+ 
3

 Mn  
 
( 0
) 2+
( 2+
)
 E MnO2 /Mn = E MnO2 /Mn − 0.03log Mn  − 0.12pH
2+
( )
- Với cặp MnO4-/Mn2+:

726 | Chuẩn bị kiến thức cho kì thi HSGQG Hóa học


 MnO−   H O+  8 
( ) (
E MnO4− /Mn2+ = E0 MnO4− /Mn2+ + 0.012log  
4  3

Mn 2+
 
)

   
 
 MnO−  
(− 2+ 0 −
)
2+
(
 E MnO4 /Mn = E MnO4 /Mn + 0.012log   2+   − 0.096pH
4

 Mn  
)
 

Ngoài ra, theo ý 1 thì:

( )
3E0 MnO4− /MnO2 + 2E0 MnO2 /Mn2+ ( )
0
(
E MnO /Mn −
4
2+
)= 5
3  1.70 + 2  1.23
= = 1.51 V
5
Khi cân bằng, thế oxid hóa-khử của ba cặp bằng nhau, trong trường
hợp này sẽ là một sự xác thực kết quả tuyệt vời!
- Với cặp MnO4-/MnO2:

( )
Eeq = E0 MnO4− /MnO2 + 0.02log MnO4−  − 0.08pH ( )
(
Eeq = 1.70 + 0.02log 1.67  10−2 − 0.08  1 = 1.58 V )
- Với cặp MnO2/Mn2+:

( )
Eeq = E0 MnO2 /Mn2+ − 0.03log Mn2+  − 0.12pH ( )
(
Eeq = 1.23 − 0.03log 1.53  10−16 − 0.12  1 = 1.58 V )
- Với cặp MnO4-/Mn2+:
  MnO−  
(
Eeq = E0 MnO4− /Mn2+ + 0.012log   ) 4 
  Mn   
2 +
− 0.096pH
 
 1.67  10 
−2
Eeq = 1.51 + 0.012log  −16 
− 0.096  1 = 1.58 V
 1.53  10 
Phương án B.

727 | Chuẩn bị kiến thức cho kì thi HSGQG Hóa học

You might also like